NCLEX Miscellaneous 6

Ace your homework & exams now with Quizwiz!

Chap 48. A client comes to the primary care provider's office with the complaints of urinating all the time, pain on urination, small amounts of urine being passed when voiding, and a foul smell to the urine. A urine specimen has been sent for analysis. Based on the signs and symptoms expressed by the client, which of the following health problems would be anticipated?

The noted signs and symptoms help to identify the problem of urinary tract infection. The signs and symptoms noted are not common with the other diseases listed.

Which statement about older adults in the United States is correct?

The number of older adults is rising because of the increase in the average life span and the aging of the baby boom generation

suctioning

place in semi-fowler's, open sterile suction cath, gloves as ordered, pick up sterile cath and connect to suction tubing, moisten cath with saline, insert through nostril with no suction, suction about 10 seconds, repeat

What is cor pulmonae?

right sided HF- look for JVD, edema

meds used for antipyretic, inflamm, and analgesia

salicylates, NSAIDs

a common pressure point for a patient in the supine position is the

scapula

An example of community based approach to health care delivery is

school health sites offering primary care services

What type of pt would you not give demerol?

sickle cell

what is a collaborating practice

speaking to the social worker about the patients insurance problems

What are other measures for a woman in distressing labor?

stop pit, give O2, inc LR

negligence is defined as

the failure of an individual to provide care that a reasonable person would ordinarily use in a similar circumstance

The nurse cares for a client who has had an above-knee amputation (AKA) with an immediate prosthetic fitting. It is MOST important for the nurse to take which of the following actions? 1. Assess drainage from Penrose drains. 2. Observe dressings for signs of excessive bleeding. 3. Elevate the stump for no less than 40 hours. 4. Provide cast care on the affected extremity.

(1) drains not usually used with amputations (2) rigid cast dressing frequently used to create a socket for prosthesis (3) elevation of extremity unnecessary; rigid cast dressing prevents swelling (4) correct—cast applied to provide uniform compression, prevent pain and contractures

What is the therapeutic drug level for dilantin?

10-20; tx for seizures NEVER MIX w/ANY OTHER DRUG or DEXTROSE

Sodium

135 -145 mEq/L

1 pt

16 oz

36. A Code Red (fire) has been announced on the hospital unit. What is the nurse's first response?</question> <question_image/> <choice_1>Remove clients in danger from the fire.</choice_1> <choice_2>Contain the fire.</choice_2> <choice_3>Report the fire to other staff.</choice_3> <choice_4>Extinguish the fire.</choice_4>

1</correct> <rationale>The primary responsibility of the nurse is client safety. Removing a client from danger should be the priority. Others can come to help contain or extinguish the fire.

Umbilical cord has

2 arteries and 1 vein

WBC

4500 - 11,000 cells/mm3

Thyroxine (T4)

5 - 12 mcg/dL

Normal serum protein range

6-8 g/dL

1 tsp

60 gtt

A nurse who normally uses touch when caring for clients might consider this inappropriate for which of the following clients?

A psychiatric client who is displaying suspicion and fear

What is the med of choice for CHF?

ACE

Which of the following persons can legally give consent to a procedure?

An appointed guardian

Chap 34. The nurse is asked to obtain an arterial blood sample from a client. The nurse knows which of the following sites can be used to obtain the blood sample?

Arterial blood samples, drawn by a specialty nurse, can be obtained from femoral, radial, and brachial arteries.

Which of these patients do you expect will need teaching regarding dietary sodium restriction? A) An 88-year-old with a fractured femur scheduled for surgery B) A 65-year-old recently diagnosed with heart failure C) A 50-year-old recently diagnosed with asthma and diabetes D) A 20-year-old with vomiting and diarrhea from gastroenteritis

B

What does apresoline tx>

HTN CHF

Prerenal Stage of ARF

Interference with renal perfusion Causative factors: hemorrhage, hypovolemia, decreased cardiac output, decreased renal profusion

Restore client to the state that existed before the development of an illness

Purpose of Tertiary Care

What is the highest priority nursing diagnosis for clients in any type of renal failure?

Risk for imbalanced fluid volume

Chap 38. During discharge planning, the nurse is teaching the client how to prevent sensory disturbances. Which of the following actions is correct?

The nurse should teach the client the following: Wear protective eye goggles when using power tools, riding motorcycles, spraying chemicals, and so on. Wear ear protectors when working in an environment with high noise levels or brief loud impulse noises (e.g., blasting). Wear dark glasses with UV protection to avoid damage from ultraviolet rays and never look directly into the sun. Have regular health examinations.

When caring for the older adult, it is important to:

Treat the client as an individual with a unique history of his or her own

inserting an NG tube

check order, set up tube feed equip/suction, positon in fowlers, place towel, measure tube length, wipe face, ask about diff breathing through one nostril, apply lub., flex head forward, insert gently until coughs, have swallow water, ch, check back of throat for coiling, check for placement, chart

the best definition of a vulnerable population

client who are unable to gain access to health care when needed

What type of vision does catarcts see?

cloudy, opaque vision

Insulin

done sub-q into fatty tissue, measured in Units, draw up air prior and insert into vial

ear meds/ 3 and under

down and back

What are sx for NMS?

hyperpyrexia (you get hot) droolings stiff muscle tone sweaty BP, HR, & RR inc oculargyric crisis

When to not take diamox?

if allergic to sulfa drugs

ted hose

if feet have been dependant, allow 15 minutes before applying, check q 4 hrs, rmv q 8 hrs to wash and dry legs

When is an S3 sound normal?

in CHF, not MI

yes

in the case of a TURP procedure is it true that urine can be expected to be light pink, but that bright red blood with increased viscosity and blood clots is indicative of arterial bleeding and that the MD should be notified immediately?

When to give mevacor (statins)?

in the evening if scheduled QD (per day)

What is hodgkins dz?

lymphatic cancer curable if caught early

the reason for lengthening hand hygiene time when hands are contaminated is

mechanically remove as many microorganisms as possible

Metabolic Alkalosis

pH: increased PCO2: normal HCO3: increased Retention of base or removal of acid from body fluids

opiate

pain relievers r/t opium

if a nurse suspects a coworker is abusing chemicals the nurse should

report the coworker to nursing administration

sardines, liver, herring, sweetbreads

the following foods are high in what purines

causes of hyperglycemia

too much food, ill, too little insulin, decreased activity, infection, stress

causes of hypoglycemia

too much insulin,no food, too much/intense exercise

removal of the organ

what is the treatment for hyperacute graft rejection?

allopurinol (Zyloprim)

what medication can be given to lower uric acid levels?

Based on the transtheoretical model of change, what is the most appropriate response to the following client statement: "Me, exercise? I haven't done that since Junior High gym class and I hated it then!"

"I understand. Can you think of one reason why being more active would be helpful for you?"

quickest absorption

#1 is abdomen, #2 is arm, #3 is buttock

Placement for bucks traction?

(skin traction) elevated foot of bed for counter traction

The nurse demonstrates the concept of "knowing the client" when he or she:

) Is able to detect changes in the client's condition based on shared information and bonding

Multiple Sclerosis

+ Babinski reflex

What are the classifications of the commonly prescribed eye drops for glaucoma?

-Parasympathomimetic-- pupillary constriction -Beta-Adrenergic Receptor -- blocking agents to inhibit formation of aqueous humor production -Carbonic anhydrase inhibitors -- to reduce aqueous humor production -Prostaglandin agonists -- increase aqueous humor outflow

What affect does theophylline have on other drugs?

1. inc r/f dig toxicity 2. dec dilantin & Li

What are pt edu with diaphragm?

1. keep in 6 hr post sex 2. refit if get fat or skinny

1 L

1000 ml

Hemoglobin - female adult

12 - 15 g/dL

1 ft

12 in.

Hematocrit - male adult

42 - 52 %

BUN (blood urea nitrogen)

8 - 25 mg/dL

Normal Clotting Time

8 to 15 minutes

Normal BUN concentration

8 to 25 mg/dL

Chloride

98 - 107 mEq/L

Troponin I

< 0.6 ng/ml >1.5 ng/ml indicates MI

ABG - SaO2

>95%

A flat macular hemorrhage is called a(n): A. purpura. B. ecchymosis. C. petechiae. D. hemangioma.

A

A patient who is comatose is admitted to the hospital with an unknown history. Respirations are deep and rapid. Arterial blood gas levels on admission are pH, 7.20; PaCO2, 21 mm Hg; PaO2, 92 mm Hg; and HCO3-, 8. You interpret these laboratory values to indicate: A) Metabolic acidosis B) Metabolic alkalosis C) Respiratory acidosis D) Respiratory alkalosis

A

Evidence-based practice is defined as:

A problem-solving approach to clinical practice based on best practices

When is an application of a warm compress indicated? (Select all that apply.) A) To relieve edema B) For a patient who is shivering C) To improve blood flow to an injured part D) To protect bony prominences from pressure ulcers

A, C

Warfarin Sodium Coumadin

Anticoagulant Blocks the formation of prothrombin from vitamin K Assess PTT, PT, Hgb, Hct, platelets; Assess stools for occult blood; Avoid IM injection; Given orally; Avoid sudden change in intake of foods high in vitamin K Antagonist: vitamin K

Chap 49. Which of the following actions is NOT appropriate for the nurse removing a fecal impaction?

Ask the client to assume a left side-lying position, with the knees flexed and the back toward the nurse. Place a bedpad under the client's buttocks and a bedpan nearby to receive stool. Drape the client for comfort and to avoid unnecessary exposure of the body. Gently insert the index finger into the rectum and move the finger along the length of the rectum.

A patient of any age can develop a contracture of a joint when: A) The adductors muscles are weakened as a result of immobility. B) The muscle fibers become shortened because of disuse. C) The calcium-to-phosphorus ratio becomes disrupted. D) There is a deficiency in vitamin D.

B

For a patient who has a muscle sprain, localized hemorrhage, or hematoma, which wound care product helps prevent edema formation, control bleeding, and anesthetize the body part? A) Binder B) Ice bag C) Elastic bandage D) Absorptive diaper

B

Functions of the skin include: A. production of vitamin C. B. temperature regulation. C. the production of new cells by melanocytes. D. the secretion of a drying substance called sebum.

B

Which is an outcome for a patient diagnosed with osteoporosis? A) Maintain serum level of calcium. B) Maintain independence with activities of daily living (ADLs). C) Reduce supplemental sources of vitamin D. D) Reverse bone loss through dietary manipulation.

B

You teach patients to replace sweat, vomiting, or diarrhea fluid losses with which type of fluid? A) Tap water or bottled water B) Fluid that has sodium (salt) in it C) Fluid that has K+ and HCO3- in it D) Coffee or tea, whichever they prefer

B

A home care nurse is preparing the home for a patient who is discharged to home following a left-sided stroke. The patient is cooperative and can ambulate with a quad-cane. Which of the following must be corrected or removed for the patient's safety? (Select all that apply.) A) The rubber mat in the walk-in shower B) The three-legged stool on wheels in the kitchen C) The braided throw rugs in the entry hallway and between the bedroom and bathroom D) The night-lights in the hallways, bedroom, and bathroom E) The cordless phone next to the patient's bed

B, C

1.What are the smallest infectious agents capable of causing an infection? a. Bacteria b. Viruses c. Molds d. Yeasts

B.

10. A nurse is caring for an 80-year-old patient who has become weak and fatigues easily. He is unable to wash his body and always asks the nurse to brush his teeth. Based on this information, which of the following is an appropriate nursing diagnosis for this patient? a. Risk for Impaired Skin Integrity related to immobility b. Bathing/Hygiene Self-Care Deficit related to decreased strength and endurance c. Social Isolation related to lack of visitors d. Impaired Oral Mucous Membrane related to inability to brush his teeth

B.

The nurse is aware that which characteristic of a skin lesion warrants further examination by a dermatologist or surgeon? A.1-mm ecchymotic area on the upper extremity B. Presence of one of the ABCD features C. Dark red color D. Round and raised appearance

B. A lesion with one or more of the ABCDE features should be evaluated by a dermatologist or a surgeon.

The older adult immobile client has "sunk" to the bottom of the bed. What will the nurse do first? A. Gently pull the client up. B. Get help and lift the client. C. Look for broken skin areas. D. Pad the bony prominences.

B. The client should be gently lifted with a sheet. Pulling or dragging the client should be avoided.

Which of the following statements concerning ethical issues is correct?

Beneficence is working to do good or acting in a positive manner for another.

Glasgow Coma Scale

Best Eye opening: 4 Best Motor Response: 6 Best Verbal Response: 5 Max: 15 Min: 3 A score of 7 or less indicates coma

An example of a primary lesion is a(n): A. erosion. B. ulcer. C. urticaria. D. port-wine stain.

C

Chap 34. The nurse is preparing a client for an MRI. Which of the following would be a significant concern? The client:

Clients with implanted metal devices cannot undergo an MRI because of the strong magnetic field. Tattoo pigments may contain metal substances that create an electric current that can cause redness and swelling similar to a first-degree burn at the site of the tattoo.

A nurse who has filled a position on the same unit for 2 years understands the unit's organization and the care of the clients on that nursing unit. Benner defines this nurse as able to anticipate nursing care and to formulate long-range goals; this nurse is given the title:

Competent nurse

Miotics prescriptions

Constrict the pupils

The health care provider's order is 1000 mL 0.9% NaCl with 20 mEq K+ intravenously over 8 hours. Which assessment finding causes you to clarify the order with the health care provider before hanging this fluid? A) Flat neck veins B) Tachycardia C) Hypotension D) Oliguria

D

The nurse is assessing a client and observes multiple small pits in all of the client's fingernails. The nurse suspects that the client may have which condition? A. Cystic fibrosis B. Iron deficiency anemia C. Isolated periods of severe malnutrition D. Psoriasis

D. Pitting of the nails may be associated with plate thickening and onycholysis and most often involves several or all of the fingernails; it is seen in clients with psoriasis and alopecia areata.

Causes of Metabolic Acidosis

DM, diabetic ketoacidosis, excessive ingestion of acetylsalicylic acid (aspirin), high fat diet, insufficient metabolism of carbohydrates, malnurition, renal insufficeincy, renal failure, severve diarrhea

Dysarthria

Difficulty articulating

Answer: C Evidence-based practice is a problem-solving approach to clinical practice that uses the best available evidence, along with the nurse's expertise and the client's preference and values, in making decisions about care. The other answers are incorrect.

Evidence-based practice is defined as: A) Nursing care based on tradition B) Scholarly inquiry embodied in the nursing and biomedical research literature C) A problem-solving approach to clinical practice based on best practices D) Quality nursing care provided in an efficient and economically sound manner

Hyperthyroidism Graves Disease Goiter

Excessive activity of the thyroid gland, resulting in an elevated level of circulating thyroid hormones Weight loss, increased appetite, diarrhea, heat intolerance, tachycardia, palpitations, increased BP, diaphoresis, nervousness, exophthalmos

Rule of 9's for burns

Head = 9% Arms = 18% (9% each) Back = 18%, Front/chest 18% Legs 36% (18% each) Genitalia = 1

A person's ideas, convictions, and attitudes about health and illness can be described as:

Health beliefs

Dysphasia

Impairment of speech and verbal comprehension

A client is wheezing and short of breath. The physician orders a medicated nebulizer treatment now and in 4 hours. The nurse is providing what aspect of care?

Implementation

Chap 34. A female client has polycythemia. This indicates that the hemoglobin and hematocrit findings would be:

In polycythemia, the hemoglobin and hematocrit readings are increased. A decreased reading may indicate hemorrhage or anemia.

Apraxia

Inability to perform purposeful movements in the absence of motor problems

What is a blue bloater?

Insufficient oxygenation occurs with chronic bronchitis and leads to generalized cyanosis and often right sided heart failure (cor pulmonale)

The health belief model addresses the relationship between a person's belief and behaviors, therefore:

It provides a way of understanding and predicting how clients will behave in relation to their health and how they will comply with health care regimens

Where is pain in diverticulitis?

LLQ

What are dx of SIADH?

LOC change, dec DTR, tachy, N/V/D

Normal Bilirubin level

Less than 1/5 mg/dL

Alexia

Loss of ability to read

Aphasia

Loss of ability to speak

Chap 43. If a client experiences the death of an aging pet, the client will go through a grieving process. This process is called:

Losses that occur in the process of normal development are called developmental losses.

What dz are contact precautions?

MRS. WEE M-MRSA R- RSV S- skin infections (VCHIPS- varicella zoster, cutaneous dip, herpes simplex, impetigo, pediculosis-lice, scabies) W-wound E-eye infection (pink eye) E-enteric (C diff)

-nium

Neuromuscular blocking agents

administer 2L O2 when

O2 sat is less than 93

Myoglobin MI; Cardiac Enzyme

Onset: 1-4 hours Peak: 4-8 hours Return to Normal: 24 hour

-ide

Oral hypoglycemics

Which of the following is not a normal physiological change associated with aging?

Osteoporosis

Chap 48. Micturition is another word for:

Other words to use for urination are micturition and voiding.

ABG is respiratory if an only if

PCO2 is elevate or decreased and if it is the opposite impact of pH, else look at HCO3 and it's metabolic

The nurse demonstrates listening skills by:

Paying attention to the tone of voice in addition to the client's words so the meaning is clear

Clients' perceptions are important because health care organizations are:

Placing greater emphasis on client satisfaction

Hyperglycemia Symptoms

Polydipsia, polyuria, polyphagia, blurred vision, weakness, weight loss, syncope Encourage water intake, assess for ketoacidosis

Answer: A A positive benefit of a professional nursing staff is a decreased length of stay. The diagnosis-related group has greater influence on the rate of readmission. The ancillary personnel need to remain so that registered nurses can spend the necessary time to assess and manage clients. Nosocomial infections decrease with a professional nursing staff.

Recent research provided evidence that a professional nursing staff affects health care financing. These results indicated that the positive benefit of a professional nursing staff is: A) Decreased length of stay B) Decreased rate of readmission C) Increased rate of nosocomial infections D) Decreased need to hire ancillary personnel

The ethics of care suggests that ethical dilemmas can best be solved by attention to which of the following?

Relationships

EKG ST Segment

Represents early ventricular repolarization Measured from end of S wave to beginning of T wave

Hypokalemia and Electrocardiographic Changes

ST depression Shallow, flat or inverted T wave Prominent U Wave

Hypercalcemia and Electrocardiographic Changes

Shortened ST segment Widened T wave

What is the most common risk factor associated with lung cancer?

Smoking

Chap 48. Which of the following statements indicates a need for further teaching of the home care client with a long-term indwelling catheter?

Soaking in a bathtub can increase the risk of exposure to bacteria. The bag should be below the level of the bladder to promote proper drainage. Intake of cranberry juice creates an environment not conducive to infection. Clean technique is appropriate for touching the exterior portions of the system.

Which of the following assures clients that they will receive quality care from a competent nurse?

Standards of care

-sone

Steriods (Glucocorticoid)

Chap 30. Students are taught to follow a head-to-toe assessment. This assists the student with:

Students are taught to follow a head-to-toe assessment. This assists the student with consistency in performing a systematic assessment.

Nursing practice in the twenty-first century is an art and science that is centered on:

The client

Answer: D The scientific method is the foundation of research and the most reliable and objective of all methods of gaining knowledge. Experience, critical thinking, and evidence are not the foundation of research.

The foundation of research is which of the following? A) Evidence B) Experience C) Critical thinking D) Scientific method

Answer: B The Nurse Practice Act regulates the license and practice of nursing; it describes the scope of practice and is the correct answer. The NCLEX-RN national licensure examination is administered in each state to test that candidates have the minimum knowledge level required for practice. Passage of an examination and requirements for certification signify additional knowledge and competence in a specific area. The ANA Congress for Nursing is an organization that addresses legal aspects of nursing practice.

The licensure and practice of nursing is regulated by: A) The NCLEX-RN B) The Nurse Practice Act C) The certification examination D) The ANA Congress for Nursing

Family members make the following comments about the nursing care being received. Which one should be investigated further?

The night nurse tells us to wait and ask the doctor the questions we have."

Chap 38. A client, who is unconscious for a long period of time, needs 24-hour care. The nurse understands the need to:

The nurse should introduce him/herself to the client and explain procedures.

Answer: C An open system is defined as a system that interacts with the environment, exchanging information between the system and the environment.

The nursing process is an example of an open system. An open system: A) Is universal and dynamic B) Represents a relationship between two concepts C) Interacts with the environment by exchanging information D) Is a process through which information is returned to the system

To meet the psychosocial needs of the older adult, the nurse may:

Use appropriate therapeutic touch

What does VEAL CHOP mean?

V/C= variable decels/cord compression E/H = early decels/head compression A/O = accelerations/OK L/O = late decel/placenta insufficiency

tachycardia (rem is a short acting beta 2 adrenergic agonist)

What s/e should the nurse monitor for when giving a pt albuterol (Proventil) for acute exacerbations of asthma? Tachycardia, candidasis, n/v.

Answer: C The federal government, which pays for the Medicare and Medicaid programs, is the biggest consumer of health care. The other options are incorrect.

Which of the following is the biggest consumer of health care? A) Hospitals B) Businesses C) Federal government D) Private insurance companies

Answer: B Developmental theories discuss human growth from conception to death. The other options are incorrect.

Which theories describe an orderly process beginning with conception and continuing through death? A) Systems theories B) Developmental theories C) Interdisciplinary theories D) Stress and adaptation theories

Name the three important dimensions to consistently measure to determine wound healing.

Width, length and depth

what describes health

a relative state of being

increase (r/t loss in dialysis)

at patient on peritoneal dialysis should increase/decrease protein intake

tetracycline

atb

broad spectrum

atbs that are effective against many organisms

narrow spectrum

atbs that are effective agaist a few mo's

What is the med of choice for asystole?

atropine

What receptors are destroyed in COPD pt?

baroreceptors- these detect CO2 levels

eye ointments

befor administering squeeze a small amount out and apply thin layer, blink

What pancreatic cells produce insulin?

beta

What are sx of hypervolemia?

bounding pulse, SOB, dyspnea, rales/crackles, peripheral edema, HTN, urnie specgrav< 1.010

wet to dry dressing

common for debriding

Hip Fracture

commonly hemorrhage

When teaching a patient with primary insomnia about sleep hygiene the nurse should emphasize A) the importance of daytime naps B) the need to exercise before beditme C) the need for long-term use of hypnotics D) avoidance of caffeine-containing beverages before beditme

d Rationale: Consumption of stimulants (e.g., caffeine, nicotine, methamphetamine, other drugs of abuse), especially before bedtime, results in insomnia

Cullens sign

ecchymosis in umbilical area seen in pancreatitis

Bilirubin, direct (conjugated)

execreted primarily by the intestinal tract 0 - 0.3 mg/dL

a nursing theory is

existing information

metabolic acidosis, hypocalcemia, hyperkalemia, hyperphosphatemia

expected findings of serum lytes/ABG's for CRF are

Whats proper admin for mannitol?

filter needle b/c mannitol crystallizes at room temp

What can't a pt have before occult stool test?

foods high in vit C (Cantalope)

Why would bethamethasone be given to a premies?

for surfactant development

a holistic nursing assessment of a patient is

formulating an effective nursing care plan

What is a common cause of autonomic dysreflexia?

full bladder

Scarlet Fever signs

high fever flushed cheeks HA enlarged lymph nodes red, fine sandpaper rash Schultz-Charlton rxt (rash blanches with pressure) Pastia sign (except in areas of deep creases & joint folds) sheet like sloughing white fuzzy - strawberry tongue

What are common sx of Addison's dz?

hyponatremia, hypotension, hypovolemia, hypoglycemia HYPERkalemia

When to give dopamine?

hypotension shock

S/Sx of spinal shock

hypotension, bladder and bowel distention, total paralysis, lack of sensation below lesion

What is the cardinal sign of ARDS?

hypoxemia (low levels of tissue O2)

when on steriods watch for

infection

phlebitis r/t IV

inflamm of the vessel: caused by irritaition of vessel by needle, cannula, meds. S/S are warnth, swelling, tenderness. IV must be dc'd, warm compress to decrease discomfort

what are NSAIDs mainly given for

inflammation and pain

what is an acute illness

influenza

which statement is NOT true regarding informed consent

informed consent is required for emergency life threatening procedures

pyelonephritis, acute chronic

is an inflammation of the renal or kidney pelvis due to bacteria infection. what are the two classifications?

True (also among pt who received ABO type differnt from their own and pt with multiple pregnancies)

is it T/F that hyperacute graft rejection is more likely to occur among clients who have received multiple blood transfusions

which statement is true about fraud

it results from a deliberate deception intended to produce unlawful gain

What is used for EtOH withdrawal?

librium

Normal ABG Values

pH: 7.35-7.45 PCO2: 35-45mm Hg PO2: 80 to 100 mm Hg HCO3: 21-28 mEq/L

retropublic, suprapublic, perineal

prostatectomy can be of 3 different varieties name them

What medication is given prophylactically to prevent uclers?

protonix

Glycosylated Hemoglobin (HbA1c)

relfects blood glucose last 3 - 4 months less than 7% - good control 7% - 8% - fair control >8% - poor control

What are signs of hypoxia?

restlessness, anxious, cyanotic, tachycardic, inc RR

bacteriostatic

retards bacteria growth

an unexpected patient care occurrence that results in death or serious injury to the patient

sentinel event

onset of hyperglycemia

several hrs or days

What does navane tx?

shizo assess for EPS

an unconscious patient begins vomiting in which position would the nurse place the patient

side lying

What position s/p LP or myelogram?

side-lying

What is dunlap traction used for>

skeletal or skin

ear meds/3 and over

straight back

the person who makes harsh high sounds upon inspiration has what kind of respiratory condition

stridor

What is mileu therapy?

taking care of pt and environment

s/sx of hyperglycemia

thirst, frequent urination, fatigue, blurred vision, ketoacidosis, vomiting, dry mouth, very weak, confused, unconscious

ACE inhibitors, 130/80

what cardiac medication may help decrease the loss of protein from urine and delay kidney damage by keeping Blood pressure below _____/_____ in cases like Nephrotic syndrome?

cloudy dialysate, fever, abdominal pain

what complications should the pt undergoing peritoneal dialysis look for and report

DKA

when being treated monitor very very closely for increase ICP and cerebral edema

1 lb

0.45 kg

1000mg

1 g

2.2lbs.

1 kg

nitroglycerin

1 tab with 5 min not exceeding 3 doses

120 gtts of water

1 tsp

What is cushings triad?

1. HTN 2. Bradycardia 3. Apneic epidsodes

1 kg

1000 g

1 ml

1cc

The divisions of the spinal vertebrae include: A. Cervical, thoracic, scaphoid, sacral, and clavicular. B. Scapular, clavicular, lumbar, scaphoid, and fasciculi. C. Cervical, thoracic, lumbar, sacral, and coccygeal. D. Cervical, lumbar, iliac, synovial, and capsular.

C

What is a sign of pna in elderly?

confusion r/t hypoxia

when nurses set priorities of patient needs according to Maslow's hierarchy they should

consider airway place self esteem before security place activity needs before belonging needs

Myasthenia Gravis signs and symptoms

Weakness & Fatique difficulty chewing dysphagia ptosis diplopia weak, hoarse voice difficulty breathing & diminished breath sounds Resp. paralysis and failure

vasoconstrictor

constrict blood vessels

ABG - HCO3

22 - 27 mEq/L

What type of exercises for knee replacement?

continuous passive ROM

Uric Acid - female adult

2.5 - 6.2 mg/dL

Phosphorus

2.7 - 4.5 mg/dL

What's indicative of epiglottitis?

3 D's: drooling dysphonia dysphagia

most insulins

given right befor meals to control blood sugar

6.Nursing education efforts that focus on prevention of firearm injuries are important for which of the following reasons? a. The elderly population is particularly at risk for firearm injuries. b. Most homicide victims, 10 to 24 years of age, are killed with a gun. c. The National Safety Council recommends a gun in every household to protect its members. d. They have contributed to a decrease in injuries and deaths from guns.

B.

ampule

glass/break

off

gloves, mask, gown

9. A patient is having dyspnea. What would the nurse do first? a. Remove pillows from under the head b. Elevate the head of the bed c. Elevate the foot of the bed d. Take the blood pressure

B.

Position after supratentorial surgery?

hairline surgery, HOB 30-45

anthelminthic enema

helps destroy intestinal parasites

What do flapping tremors suggest?

hepatic encephalopathy

If a neonate has poor sucking, what could be the cause?

heroin w/draw

Which of the following defining characteristics is consistent with fluid volume deficit? A) A 1-lb (0.5 kg) weight loss, pale yellow urine B) Engorged neck veins when upright, bradycardia C) Dry mucous membranes, thready pulse, tachycardia D) Bounding radial pulse, fl at neck veins when supine

C

What does ganciclovir (cytovene) tx?

herpes simplex and retinitis cause by CMG pt will need regular eye exams report confusion

What type of diet should hypoPTH be on?

high ca, low po4

6.The CDC standard precaution recommendations apply to which of the following? a. Only patients with diagnosed infections b. Only blood and body fluids with visible blood c. All body fluids including sweat d. All patients receiving care in hospitals

D.

What happens during teralogy of fallot?

DROP d- defect, septal r- right ventricular hypertrophy o- overriding aorta p- pulmonary stenosis

Potassium

3.5 - 5.1 mEq/L

Potassium Safe Range

3.5-5.0

How many mL in ounce?

30

HDL's

30 - 70 mg/dL

checking NG tube placement

30 ml of air and listen to stomach, aspirate stomach contents(should be cloudy, green, bloody,brown), ph of aspirate, x-ray

Therapeutic drug level Magnesium sulfate

4 - 7 mg/dL

15.When completing a safety event report, the nurse should do which of the following? a. Include suggestions on how to prevent the incident from recurring b. Provide minimal information about the incident c. Discuss the details with the patient before documenting them d. Objectively describe the incident in detail

D.

Lantus

onset:1-2hr., clear, no peak, duration:24 hrs., cannot be combined

NPH

onset:1-4hr., cloudy, peak:6-10hrs., duration:10-16, up to 24 hrs., cannot mix!!

6.The nurse is taking an apical pulse. What equipment will he take into the patient's room? a. Sphygmomanometer b. Electronic thermometer c. Stethoscope d. Doppler apparatus

C.

Chap 38. Which statement by a client with decreased hearing indicates a need for a sensory aid in the home?

A sensory aid will help the client compensate for hearing loss. "My eyesight is good if I wear my glasses" and "I tripped over that throw rug again" are unrelated to hearing deficit. "I can hear the radio if I turn it up high" is an example of compensation.

Alkaline phosphatase

4.5 - 13 King Armstrong units /dL

RBC - female adult

4.5 - 5 million/ul

RBC - male adult

4.5 - 6.2 million/ul

Urine pH

4.5 - 7.8

Uric Acid - male adult

4.5 - 8 mg/dL

What test is done prior to ABG draw?

Allens test to check for sufficient blood flow

-sartan

Angiotensin receptor blocker drug

Excess Fluid Symptoms

Dyspnea, tachycardia, jugular vein distention, peripheral edema, pulmonary edema, weight gain

McBurney's Point

pain in RLQ indicative of appendicitis

-pam

Anti-anxiety (Benzodiazepine)

The best way for nurses to avoid being liable for negligence might be to do all of the following except:

Choose not to report someone whose professional behavior the nurse questions.

hydronephrosis

Distension of the pelvis and calyces of the kidney caused by urine that cannot flow past an obstruction in a ureter

Which of the following represents utilitarian allocation of scarce resources?

Distribution of resources to achieve the greater good of the larger group

Dysphagia

Dysfunctional swallowing

The nurse is sure to implement strategies to reduce noise on the unit particularly on the ______ night of admission, when the client is especially sensitive to hospital noises.

First The client is most sensitive to noise in the hospital setting on the first night because everything is new. This represents sensory overload, which interferes with sleep and decreases rapid eye movement (REM) as well as total sleep time

Who acted to decrease mortality by improving sanitation in the battlefields, which resulted in a decline in illness and infection?

Florence Nightingale

The professional nurse responsible for increasing respect for the individual and awareness of cultural diversity was:

Mary Mahoney

Respiratory Isolation Technique

Mask is required for anyone entering room Private room with negative air pressure Client must wear mask when leaving room

Is unstable angina relieved by nitro?

NOOOOO

-done

Opiod Analgesics

-zine

Phenothiazine drug

Heparin antidote

Protamine sulfate

Because clients and nurses may differ in their perceptions of caring, it is important that the nurse:

Seek information regarding what is important to the client.

A client comes into the clinic for a complete physical examination. The nurse obtains a health history and determines that the client is at risk for heart disease. Which of the following would lead the nurse to conclude this?

The client's father died of a heart attack at age 40.

transdermal patch

sheck of proper placement, note last patchplacement, clean skin, inital/date/time

antidote for heparin

protamine sulfate

35 - 50%

reduced renal reserve is ___% to ____% of GFR

How to tx phobic d/o?

systemic dessensitization

What is the med of choice for status epilecticus?

valium, ativan dilantin

1 pinch

1/8 tsp

Therapeutic drug level Acetaminophen (Tylenol)

10 - 20 mcg/dL

Therapeutic drug level Gentamicin

5-10 mcg/ml

Iron - male adult

65 - 175 mcg/dL

Chap 30. After auscultating the abdomen, the nurse should report which of the following to the primary care provider?

A bruit suggests abnormal turbulence in the aorta, and the primary care provider must be notified. In order for absence of bowel sounds to be considered abnormal, the bowels must be silent for 3 to 5 minutes. Continuous bowel sounds are normally heard over the ileocecal valve following meals. Bowel sounds are more commonly irregular than they are regular.

Answer: D Implementation is the actual delivery of care. Assessment is data gathering. Then the information is developed into a diagnosis and the planning occurs with the diagnosis. Evaluation is the final step of the nursing process.

A client is wheezing and short of breath. The physician orders a medicated nebulizer treatment now and in 4 hours. The nurse is providing what aspect of care? A) Planning B) Evaluation C) Assessment D) Implementation

Which of the following activities can you delegate to nursing assistive personnel (NAP)? (Select all that apply.) A) Measuring oral intake and urine output B) Preparing intravenous (IV) tubing for routine change C) Reporting an IV container that is low in fluid D) Changing an IV fluid container

A, C

9.Providing perineal care to a patient requires which of the following? a. Using a clean portion of the washcloth for each stroke b. Moving from most contaminated to least contaminated area c. Using sterile gloves d. Leaving the foreskin undisturbed in an uncircumcised adult male

A.

The nurse conducts a physical examination of a client suspected to have bulimia. Which of the following observations by the nurse MOST likely indicates bulimia? 1. The client has edema of the lower extremities. 2. Physical exam of the client reveals the presence of lanugo. 3. The client has ulcerated mucous membranes of the mouth. 4. The client has dry, yellowish color of the skin.

(1) common with anorexia (2) seen with anorexia (3) correct—due to frequent vomiting (4) bulimics are normal in appearance

An adult woman has missed her menstrual period. The client's last menstrual period began May 8 and ended May 12. The nurse determines that the client's EDC (estimated date of confinement) is which of the following? 1. February 1. 2. February 15. 3. February 19. 4. March 14.

(1) should add 7 days (2) correct—when using the Naegele rule, add 7 days to first day of last menstrual period and subtract 3 months (3) incorrectly started with the last day of the menstrual cycle (4) incorrect

When would you not give bactrim?

allergy to sulfa drugs

Sleep is best described as a A) loosely organized state similar to coma B) state in which pain sensitivity decreases C) quiet state in which there is little brain activity D) state in which an individual lacks conscious awareness of the environment

d Rationale: Sleep is a state during which an individual lacks conscious awareness of environmental surroundings and from which the person can be easily aroused.

Turners sign

flank grayish blue- pancreatitis

Is tardive dyskinesia reversible?

no

Position for infant with cleft lip?

position on back or in infant seat to prevent pressure on surture line seat infant upright while feeding

What happens if a TPN pt has dec LOC?

position on right side to empty stomach and try to inc BG w/HOB elevated to prevent aspiration

What is imp with allopurinol admin?

push fluids to get uric acid out do not give Vit C

20 - 25%

renal failure is ____% to ___% of GRF

ear meds/adult

up and back

continuous ambulatory peritoneal dialysis

(CAPD) dialysis carried out manually by exchanging 1.5 to 3 L of peritoneal dialysate usually four times daily with dwell times of 4 to 10 hours; an indwelling catheter permits fluid to drain into and out of the peritoneal cavity by gravity.

Therapeutic drug level Phenytoin (Dilantin)

10 - 20 mcg/mL

1 lb

16 oz.

NG tube

16-18 F is standard adult size, children are standard 5-12F

FHR (fetal heart rate) first trimester

160 - 170 beats/min

-micin

Antibiotics/Anti-infectives

What are hypoparthyroidism sx?

CATS: CATS: convuslions arrhythmias tetany spasms

Conductive Hearing Loss

Hearing loss in which sound does not travel well to the sound organs of the inner ear. The volume of sound is less, but the sound remains clear.

Answer: C Pigmented skin lesions that are asymmetrical, have irregular borders, have variegated colors, and are larger than 6 mm in diameter are lesions that are suspect and should be reported to a medical provider.

The nurse teaches the client to inspect all skin surfaces and to report pigmented skin lesions that: A) Are symmetrical B) Are uniform in color C) Have irregular borders D) Are smaller than 6 mm in diameter

What can untx PVCs lead to?

VFIB

Antidote for Coumadin

Vitamin K

An elderly woman was admitted to the medical unit with dehydration. A clinical indication of this problem is A) weight loss B) full bounding pulse C) engorged neck veins D) Kussmaul respiration

a Rationale: Body weight loss, especially sudden change, is an excellent indicator of overall fluid volume loss.

Before transferring a patient from the bed to a stretcher, which assessment data does the nurse need to gather? (Select all that apply.) A) Patient's weight B) Patient's level of cooperation C) Patient's ability to assist D) Presence of medical equipment E) 24-hour calorie intake

All except E

Chap 42. A middle-aged male client is experiencing job-related stress associated with the fear of layoff, resulting in his accepting projects that require a great deal of travel. Which of the following would be the most important health promotion strategy for this client?

All of the four areas of health promotion strategies may be important, but for this client, sleep is likely to be the most adversely affected by travel, in which changing time zones and unfamiliar sleeping quarters are common. It is easier for clients to adapt to modifying exercise, nutrition, and time management during travel than it is to control sleep. Thus, it becomes the most important area requiring intervention to avoid worsening the existing stress.

Your patient who has diabetic ketoacidosis is breathing rapidly and deeply. Intravenous (IV) fluids and other treatments have just been started. What should you do about this patient's breathing? A) Notify her health care provider that she is hyperventilating B) Provide frequent oral care to keep her mucous membranes moist C) Ask her to breathe slower and help her to calm down and relax D) Assess her for pain and request an order for a sedative

B

11.An older patient with an unsteady gait requests a tub bath. Which of the following actions would be most appropriate? a. Add Alpha-Keri oil to the water to prevent dry skin. b. Allow the patient to lock the door to guarantee privacy. c. Assist the patient in and out of the tub to prevent falling. d. Keep the water temperature very warm because the patient chills easily.

C.

8.When caring for a patient with latex allergy, the nurse creates a latex-safe environment by doing which of the following? a. Carefully cleaning the wall-mounted blood pressure device before using it b. Donning latex gloves outside the room to limit powder dispersal c. Using a latex-free pharmacy protocol d. Placing the patient in a semi private room

C.

diagnostic tests for respiratory disorder

CBC, Pulmonary Function test, Chest x-ray, Lung Scan, bronchoscopy

What are sx of hyperkalemia?

MURDER: muscle weakness urine -oliguria/anuria respiratory depression dec cardiac contractility ECK changes reflexes

Chap 34. Nuclear Imaging studies are:

Nuclear scans study the "physiology or function" of an organ system in contrast to other studies (e.g., CT, MRI, x-ray) which visualize "anatomic" structures.

Chap 43. A nurse is evaluating a client who has just lost a spouse and the client states, "He used to like this time of year when he could plant his tomatoes in the garden." The nurse knows the client is experiencing which of the following stages of grieving?

The nurse assesses the grieving client or family members following a loss to determine the phase or stage of grieving. During resolving the loss, the person thinks over and talks about memories of the lost object.

Chap 30. Most health assessments are performed in a head-to-toe sequence. The nurse needs to be aware:

The nurse must be aware of physiological changes that occur with the aging process.

When many people share the same values, it may be possible to identify a philosophy of utilitarianism. Which of the following statements is a principle of utilitarianism?

The value of something is determined by its usefulness to society.

VEAL CHOP

Variable declarations = Cord Compression Early declerations = Head Compression Acclerations = OK Late declerations = uteroPlacental insufficiencies

Answer: D An oxygen saturation of 89% should be addressed first, because this indicates that a client needs oxygen. The high respiratory rate may be a result of hypoxemia and may decrease as the oxygen saturation climbs. The blood pressure is high, but this might be attributed to hypoxemia or anxiety. The heart rate and temperature are within normal limits.

Which of the following values for vital signs would the nurse address first? A) Heart rate = 72 beats per minute B) Respiration rate = 28 breaths per minute C) Blood pressure = 160/86 D) Oxygen saturation by pulse oximetry = 89% E) Temperature = 37.2° C (99° F), tympanic

considering the chain of infection, a vector might be

a tick carrying lyme disease

A patient with type 2 diabetes and cirrhosis asks the nurse if it would be okay to take silymarin (milk thistle) to help minimize liver damage. The nurse responds based on knowledge that a) Milk thistle may affect liver enzymes and thus alter drug metabolism. b) Milk thistle is generally safe in recommended doses for up to 10 years. c)There is unclear scientific evidence for the use of milk thistle in treating cirrhosis. d) Milk thistle may elevate the serum glucose levels and is thus contraindicated in diabetes.

a) milk thistle may affect liver enzymes and thus alter drug metabolism There is good scientific evidence for the use of milk thistle as an antioxidant to protect the liver cells from toxic damage in the treatment of cirrhosis. It is noted to be safe for up to 6 years, not 10 years, and it may lower, not elevate, blood glucose levels. It does affect liver enzymes and thus could alter drug metabolism. Therefore patients will need to be monitored for drug interactions.

When do the fontanells close?

anterior - 18 mo posterior 6-8 week

nitroglycerin

antianginal med used in heart attacks

What does atropine do?

drys stuff up

When looking at a pt's UOP, when would you not admin K+?

during low UOP b/c would irritate the kidneys

for a patient to successfully win a malpractice claim what elements move be proven

duty breach of duty harm or damage cause and effect relationship

What SE of apresoline (hydralazine) would you report?

flu-like sx

edema

fluid buildup in tissues

2000 to 30000

fluid volume of ____ to ____mL per day should be encouraged to help keep catheter clear of clots

What's the antedote for tylenonl?

mucomyst admin PO may also tx CF, pna, & emphysema

What reverses pancuronium (muscle relaxant)?

neostignmine/atropine

What is dangerous when your are pregars: German measles (rubella) or rubeloa?

nerver get preg w/a german = RUBELLA

catecholamine

neurotransmitters that play important role in body's stress response

Regular insulin

only insulin that can be given IV

deltoid muscle

only used for 1-2 ml of med b/c is smaller muscle

which document refers specifically to the client's right for an advance directive

patient's bill of rights

a basic premise of the American Nurses Association's nursing agenda for health care reform includes

primary health care services must play a very basic and prominent role in services delivery

Placement after cataract surgery?

pt sleep on unaffected side w/night shield for 1-4 weeks

when placing the elderly patient in Fowler's position you must

refrain from raising the knees ore than 15 degrees

Prebn's signs

relief from pain when the scrotom is lifted, indicating inflammation and positive for epididymis

25 - 35%

renal insufficiency is ___% to ____% of GFR

first sign of narcotic OD

resp. depression

What are nondairy sources of Ca?

rhubarb, sardines, collard greens

Normal Calcium Levels

8.6-10.0 mg/dL

39. Public health nurses have been activated to open a shelter due to an approaching hurricane. What most important items should families be encouraged to take to the emergency shelter?</question> <question_image/> <choice_1>Food and extra clothing</choice_1> <choice_2>Cats and small dogs</choice_2> <choice_3>Medication and vital records</choice_3> <choice_4>Radios, televisions, and small personal electronics</choice_4>

<correct>3</correct> <rationale>Client medications and vital records are needed for a short or extended stay at an emergency shelter. Space is very limited in a shelter. There is no provision for storing food, and animals are not allowed. Loud electronic devices such as radios or televisions may cause disturbance between families or individuals. Electricity may or may not be available.</rationale

To determine if a dark skinned patient is pale, the nurse should assess the color of the: A. conjunctivae. B. ear lobes. C. palms of the hands. D. skin in the antecubital space.

A

3

A patient experienced a myocardial infarction four weeks ago and is currently participating in the daily cardiac rehabilitation sessions at the local fitness center. In what level of prevention is the patient participating? 1. Primary Prevention 2. Secondary Prevention 3. Tertiary Prevention 4. Quaternary Prevention

What does APGAR mean?

A-appearance (color pink, pink and blue, blue [pale]) P-pulse (> 100, < 100, absent) G- grimace (cough, grimace, no response) A-acitivty (flexed, flaccid, limp) R-respiration (strong, weak, absent cry)

1. A nurse assesses an oral temperature for a patient as 38.5C (101.3F). What term would the nurse use to report this temperature? a. Fever b. Hypothermia c. Hypertension d. Afebrile

A.

2. A nurse is caring for a patient immediately following a transesophageal echocardiogram (TEE). Which of the following assessments are appropriate for this patient (select all that apply)? A. Assess for return of gag reflex. B. Assess groin for hematoma or bleeding. C. Monitor vital signs and oxygen saturation. D. Position patient supine with head of bed flat. E. Assess lower extremities for circulatory compromise.

A. Assess for return of gag reflex. C. Monitor vital signs and oxygen saturation.

The nurse should teach a patient who is taking which of the following drugs to avoid prolonged sun exposure? A. Tetracycline B. Ipratropium C. Morphine sulfate D. Oral contraceptives

A. Several antibiotics, including tetracycline, may cause photosensitivity. This is not the case with ipratropium, morphine, or oral contraceptives.

A nurse is explaining the concept of time-out in the perioperative area. The purpose of time out is:

4. To allow the surgical team a chance to verbally verify their agreement about the client's name, surgical procedure, and the site. The time-out occurs in the perioperative area after the client has been prepped and draped. The entire team must verbally verify their agreement regarding the client's name, the procedure to be performed, and the surgical site.

How many mL in 1 teaspoon?

5 mL

LDL (low density lipoproteins)

<130 mg/dL

96.The pregnant client has been started on an iron supplement. The nurse determines that the client understands possible side effects of therapy when the client states that the supplement may cause which of the following?</question> <question_image/> <choice_1>Red, raised rash</choice_1> <choice_2>Gastric upset</choice_2> <choice_3>Blood in the stool</choice_3> <choice_4>Headache</choice_4>

<correct>2</correct> <rationale>Iron supplementation can cause gastric distress, constipation, and diarrhea. It does not cause a red, raised rash (option 1), blood in the stool (option 3), or headache (option 4).</rationale>

86 A client has been admitted as an outpatient for a tubal ligation. Following the procedure, the client should be told to expect which of the following?</question> <question_image/> <choice_1>Hot flashes and other hormonally associated symptoms</choice_1> <choice_2>Heavier bleeding with menstruation</choice_2> <choice_3>Possible pain for several days</choice_3> <choice_4>Change in sexual function</choice_4>

<correct>3</correct> <rationale>Some clients report mild pain after the procedure, which is usually relieved with analgesics. Changes in menstruation, sexual function, or other hormonal symptoms are not typical.</rationale>

77.The client, a 16-year-old female, has come to the clinic to discuss contraception because she has recently become sexually active. The client states that many of her friends are using spermicides and asks the nurse about their advantages and disadvantages. The nurse's best response would be:</question> <question_image/> <choice_1>"If you want an effective method, you should choose something else."</choice_1> <choice_2>"It is a very convenient method and you will be able to insert the spermicide up to 4 hours before intercourse."</choice_2> <choice_3>"Spermicides cause very few problems, and they are almost 100 percent effective."</choice_3> <choice_4>"Spermicides may or may not be a good choice for you. They have a failure rate of about 21 percent and offer some protection against sexually transmitted infections."</choice_4>

<correct>4</correct> <rationale>Spermicides must be used within 30 minutes of intercourse, have a failure rate of 21%, and do offer some protection against sexually transmitted infections. Other key information needed is the sexual history of the client and her partner(s) to more accurately assess risk for STIs. Option 1 provides advice, which the nurse should not give. Options 2 and 3 are false statements.</rationale>

Crepitation is an audible sound that is produced by: A. roughened articular surfaces moving over each other. B. tendons or ligaments that slip over bones during motion. C. joints that are stretched when placed in hyperflexion or hyperextension. D. flexion and extension of an inflamed bursa.

A

Heberden and Bouchard nodes are hard and nontender and are associated with: A. osteoarthritis. B. rheumatoid arthritis. C. Dupuytren contracture. D. metacarpophalangeal bursitis.

A

In which of the following ethnic groups has the lowest incidence of osteoporosis? A. African Americans B. Whites C. Asians D. American Indians

A

Most important indicator of increased ICP?

A change in level of responsiveness

Chap 48. A urinary diversion is the surgical rerouting of urine from the kidneys to a site other than the bladder. Which type of client would benefit from this type of procedure?

A client with an injury to their bladder would need a diversion to route the urine to the outside of the body by a different route than via the bladder. A renal failure client and a client with kidney stones have problems at the kidney level where urine is produced. This is not a problem with the route the urine takes once it leave the kidney. Infections would not be treated with a urinary diversion. The bladder and urethra still function.

A nurse who uses critical thinking in the decision-making process to provide effective quality care to individuals is known as:

A clinical decision maker

Answer: B Assessing the learner's needs and readiness to learn are important to increase the success of the learning process. Options A and D are negative responses and would block the learning process. Repeating the old teaching plan is nonproductive and an inefficient application of the nursing process.

A home care nurse educator has repeatedly counseled a 33-year-old male diabetic client concerning the need for dietary compliance. In writing an effective teaching plan the nurse will first: A) Reprimand the client for noncompliant behavior B) Assess the client's learning needs and readiness to learn C) Repeat the old teaching plan to ensure the client's comprehension D) Provide a detailed description of complications associated with the disease process

Answer: A, B, C, D All are factors that will impact the client's potential to change.

A nurse at the community clinic nurse cares for a 40-year-old woman who takes insulin to manage diabetes. She is having increasing difficulty controlling the disease, and the nurse wants her to try a new insulin pump to help her manage her diabetes. Which of the following change factors increase the likelihood that she will accept this new insulin pump? (Select all that apply.) A) The innovation or change must be perceived as more advantageous than other alternatives. B) The innovation or change must be compatible with existing needs, values, and past experiences. C) The innovation must be tried on a limited basis. D) Simple innovations or changes are more readily adopted than those that are complex.

Answer: D The homeless person's lack of a storage site for medication and inability to obtain nutritious meals are factors that contribute to poor management of chronic disease. Homeless people are often stereotyped as having a lack of concern for their situations. Poor attire and lack of hygiene are not causes of chronic illness exacerbation. They are signs of the client's status as a member of an at-risk population. It is incorrect for the nurse to assume that the client lacks education and the ability to read.

A nurse is caring for a 64-year-old homeless woman with a chronic respiratory disease in the local community-based clinic. The nurse realizes that the client is at risk of experiencing exacerbation of the disease process related to: A) Poor attire and cleanliness practices B) The client's lack of education and ability to read C) The individual's lack of concern about the disease D) The client's lack of a storage site for medication and the inability to obtain nutritious meals

Answer: B Effective qualitative research can be carried out, because through narrative interviews the participants' perceptions can be compared and common characteristics can be discovered. It is difficult to collect data about perceptions or feelings without talking to those involved. Quantitative research involves precise measurements and would not be of use in this study of perceptions. Although obtaining suggestions for possible solutions could be useful, it does not help to identify the problems on this specific unit. The data must be collected first.

A nurse manager is researching the effects of staff shortages on job satisfaction among new graduates. It would be most effective for the nurse to gather data by: A) Directly observing the nursing behaviors on the unit B) Interviewing staff nurses on the unit regarding their perceptions C) Setting up an experimental group and a control group for the study D) Calling on other nurses in the facility to suggest ways of handling the problem

2

A patient comes into the local health clinic and states: "I've noticed how many people are out walking in my neighborhood. Is walking good for you?" What is the best response to help the patient through the stages of change for exercise? 1. "Walking is ok, running is better?" 2. "Yes, walking is great exercise. Do you think you could go for a five minute walk next week?" 3. "Yes, I want you to begin walking. Walk for 30 minutes everyday and start eating more fruits and vegetables." 4. "They probably aren't walking fast enough or far enough. You may need to spend more at least 45 minutes if you are going to do any good."

Assessment findings consistent with intravenous (IV) fluid infiltration include: (Select all that apply.) A) Edema and pain B) Streak formation C) Pain and erythema D) Pallor and coolness E) Numbness and pain

A, D

11. When a fire occurs in a patient's room, which of the following would be the nurse's priority? a. Rescue the patient b. Extinguish the fire c. Sound the alarm d. Run for help

A.

12.A patient has a blood pressure reading of 130/90 mm Hg when visiting a clinic. What would the nurse recommend to the patient? a. Follow-up measurements of blood pressure b. Immediate treatment by a physician c. Nothing, because the nurse considers this reading is due to anxiety d. A change in dietary intake

A.

14.It is important to have the appropriate cuff size when taking the blood pressure. What error may result from a cuff that is too large or too small? a. An incorrect reading b. Injury to the patient c. Prolonged pressure on the arm d. Loss of Korotkoff sounds

A.

5. A patient complains of severe abdominal pain. When assessing the vital signs, the nurse would not be surprised to find what assessment? a. An increase in the pulse rate b. A decrease in body temperature c. A decrease in blood pressure d. An increase in respiratory depth

A.

7. In addition to standard precautions, the nurse caring for a patient with rubella would plan to implement what type of precautions? a. Droplet precautions b. Airborne precautions c. Contact precautions d. Universal precautions

A.

7. Two nurses are taking an apical-radial pulse and note a difference in pulse rate of 8 beats per minute. The nurse would document this difference as which of the following? a. Pulse deficit b. Pulse amplitude c. Ventricular rhythm d. Heart arrhythmia

A.

When the nurse is assessing the skin of an older adult client, which of these data will be most important to report to the physician? A. A multicolored lesion is present on the client's thigh. B. Liver spots are present on both hands. C. Cherry hemangiomas are scattered on the client's back. D. The skin on the extremities is paper thin.

A. Color variation within a lesion is associated with skin cancer; the physician should be informed, so that the lesion can be further assessed.

Which of the following patients would be more likely to have the highest risk of developing malignant melanoma? A. A fair-skinned woman who uses a tanning booth regularly B. An African American patient with a family history of cancer C. A Hispanic male with a history of psoriasis and eczema that responded poorly to treatment D. An adult who required phototherapy as an infant for the treatment of hyperbilirubinemia

A. Risk factors for malignant melanoma include a fair complexion and exposure to ultraviolet light. Psoriasis, eczema, short-duration phototherapy and a family history of other cancers are less likely to be linked to malignant melanoma.

A client with bacteremia associated with a bacterial skin infection is receiving clindamycin (Cleocin) intravenously (IV). Which assessment finding indicates the need for immediate action by the nurse? A. Blood pressure is 88/40 mm Hg. B. White blood cell count is 15,000/mm3. C. Oral temperature is 101° F (38.3° C). D. Heart rate is 102 beats/min.

A. Too-rapid administration of clindamycin (Cleocin) can cause shock and cardiac arrest; the client's low blood pressure indicates a need to slow the rate and reassess the client.

Chap 49. The nurse assesses the intestinal tract function by:

Administering pain medications is not part of assessment. Food preferences may be related to many factors and may not provide the nurse with enough information about the client's intestinal tract function. A pain diary does not give the nurse direct information about the client's intestinal tract function. Completing a history, inspecting the client's stool, and auscultating the abdomen are appropriate ways of assessing the intestinal tract function. Reviewing serum (blood) lab values is not the most accurate method of assessing the client's intestinal tract function.

Answer: B APNs are generally the most independently functioning nurses. An APN can work in a primary, acute, or restorative care setting. The setting may be a private, public, or university facility. The APN may function as a clinician, educator, case manager, consultant, or researcher.

Advanced practice nurses (APNs) generally: A) Work in acute care settings B) Function independently C) Function as unit directors D) Work in the university setting

Immobilized patients are at risk for impaired skin integrity. Which of the following interventions would reduce this risk? (Select all that apply.) A) Repositioning patient every 1 to 2 hours while awake B) Using an objective, valid scale to assess patient's risk for pressure ulcer development C) Using a device to relieve pressure when patient is seated in chair D) Teaching patient how to shift weight at regular intervals while sitting in a chair E) A good rule is: the higher the risk for skin breakdown, the shorter the interval between position changes

All except A

Sexuality is recognized as a factor in the care of older adults, thus:

All older adults, whether healthy or frail, need to express sexual feelings

Chap 42. Which of the following clients is most likely to be experiencing a negative situational stress response?

An 18-year-old beginning college in a new state is experiencing a situational stress response. The other answers are examples of developmental stressors.

Answer: C The nurse is assessing the client. Diagnosis occurs after all assessments are completed. Then a plan is developed and implemented. The process is completed with evaluation.

An 18-year-old woman is in the emergency department with fever and cough. The physician asks the nurse to measure vital signs, auscultate lung sounds, listen to heart sounds, determine the level of comfort, and collect blood and sputum samples for analysis. The nurse is performing what aspect of practice? A) Diagnosis B) Evaluation C) Assessment D) Implementation

Chap 30. Auscultation is the:

Auscultation is the process of listening to sounds produced within the body. Inspection is the visual examination—that is, assessing by using the sense of sight. Palpation is the examination of the body using the sense of touch. Percussion is the act of striking the body surface to elicit sounds that can be heard or vibrations that can be felt.

What does the Braden Scale evaluate? A) Skin integrity at bony prominences, including any wounds B) Risk factors that place the patient at risk for skin breakdown C) The amount of repositioning that the patient can tolerate D) The factors that place the patient at risk for poor healing

B

A patient has her call bell on and looks frightened when you enter the room. She has been on bed rest for 3 days following a fractured femur. She says, "It hurts when I try to breathe, and I can't catch my breath." Your first action is to: A) Call the health care provider to report this change in condition. B) Give the patient a paper bag to breathe into to decrease her anxiety. C) Assess her vital signs, perform a respiratory assessment, and be prepared to start oxygen. D) Explain that this is normal after such trauma and administer the ordered pain medication.

C

9.While discussing home safety with the nurse, a patient admits that she always smokes a cigarette in bed before falling asleep at night. Which nursing diagnosis would be the priority for this patient? a. Impaired Gas Exchange related to cigarette smoking b. Anxiety related to inability to stop smoking c. Risk for Suffocation related to unfamiliarity with fire prevention guidelines d. Deficient knowledge related to lack of follow-through of recommendation to stop smoking

C.

The nurse is caring for a client prescribed linezolid (Zyvox) for treatment of methicillin-resistant Staphylococcus aureus (MRSA) infection. The nurse plans to monitor the client for which adverse effect of linezolid? A. Depression B. Hyperglycemia C. Hypertension D. Incontinence

C. Linezolid (Zyvox) constricts blood vessels and may trigger hypertensive crisis.

How to walk with CANE pt?

COAL: cane opposite affected leg

Calcium - food sources

Cheese, collard greens, milk including soy, rhubard, sardine, spinach, tofu, and yogurt

Chronic Bronchitis Pathophysiology Precipitating Factors Assessment Nursing Intervention

Chronic sputum with cough production on a daily basis for a minimum of 3 months per year; chronic hypoxemia, cor pulmonale; increase in mucus, cilia production; increase in bronchial wall thickness, reduced responsiveness of respiratory center to hypoxemic stimuli. Higher incidence in smokers Generalized cyanosis, blue boaters, right sided heart failure, distended neck veins, crackles, expiratory wheezes; Monitor for S/SX of fluid overload; Maintain PaO2 between 55 and 60; baseline ABGs; Teach pursed lip breathing and diaphragmatic breathing; Teach tripod position

Answer: A, B, C, D Each of the options is an example of a professional role or responsibility of the professional nurse.

Contemporary nursing requires that the nurse possess knowledge and skills to carry out a variety of professional roles and responsibilities. Examples include which of the following? (Select all that apply.) A) Autonomy and accountability B) Advocacy C) Provision of bedside care D) Health promotion and illness prevention

After the urinary catheter is removed in the TURP client, what are 3 priority nursing actions?

Continued strict I&O. Continued observations for hematuria. Inform client burning and frequency may last for a week.

Fluid-Deficient Symptoms

Decreased urine output, decreased skin turgor, dry mucous membranes, hypotension, tachycardia, weight loss

The nurse practices nursing in conformity with the code of ethics for professional registered nurses. This code:

Defines the principles by which nurses' provide care to their clients

An older adult recently diagnosed with a urinary tract infection displays a sudden onset of confusion. She most likely is experiencing

Delirium

Chap 42. Which of the following is an example of the defense mechanism of displacement?

Displacement is the transferring or discharging of emotional reactions from one object or person to another object or person. An example would be when a husband and wife are fighting, and the husband becomes so angry he hits a door instead of his wife. Denial is an attempt to screen or ignore unacceptable realities by refusing to acknowledge them. An example would be a woman who, though told her father has metastatic cancer, continues to plan a family reunion 18 months in advance. Projection is a process in which blame is attached to others or the environment for unacceptable desires, thoughts, shortcomings, and mistakes. An example would be a mother who is told that her child must repeat a grade in school, and she blames this on the teacher's poor instruction. Substitution is the replacement of a highly valued, unacceptable, or unavailable object by a less valuable, acceptable, or available object. An example would be a woman who wants to marry a man exactly like her dead father and settles for someone who looks a little bit like him.

Chap 48. Medications can affect the urinary process. The type of drug that increases urine output is:

Diuretics increase urine formation by preventing reabsorption of water and electrolytes from the tubules of the kidney into the bloodstream.

State four nursing interventions for assisting the client to cough productively

Encourage deep breathing; increase fluid intake to 3L/day; use humidity to loosen secretions; suction airway to stimulate coughing

In caring for a client, the nurse would describe learning about the client's family as:

Essential

Answer: D The result of theory-generating or theory-testing research is to increase the knowledge base of nursing. As these research activities continue, clients become the recipients of evidence-based nursing care.

Evidence-based nursing practice is the end result of: A) Prescriptive theory B) Use of practical knowledge C) Application of theoretical knowledge D) Theory-generating and theory-testing research

Chap 43. A client from another country has died in the United States, which is where the body will be buried. The nurse needs to:

Explore and respect the client's and family's ethnic, cultural, religious, and personal values in their expression of grief.

Chap 38. The nurse is assessing for sensory function. Match the assessment tool in column 1 to the specific sense it will be testing in column 2.

External stimuli are visual (sight - the snellen chart is a vision test), auditory (hearing - the tuning fork is a tool used to assess hearing), olfactory (smell - identifying aroma), tactile (touch - stereognosis), and gustatory (taste).

Advanced practice nurses (APNs) generally:

Function independently

The legal definition of death that facilitates organ donation is cessation of:

Functions of the entire brain

Answer: C Resonance is the low, hollow sound of normal lungs. Hyperresonance can be heard over emphysematous lungs as a booming sound. Tympany is the high-pitched, drumlike sound heard over a gastric air bubble. Dullness is the soft, thudlike sound that is heard over dense organ tissue.

In assessing the client's lungs, the nurse notes that the lungs are normal upon percussion. This means that the nurse detected: A) Dullness B) Tympany C) Resonance D) Hyperresonance

When assessing the older adult, the nurse should know which findings represent common physiological changes associated with aging and which are abnormal findings. A normal and common physiological change is:

Increase in the time it takes for the heart rate to return to baseline after exercise

What vital sign changes are indicative of increased ICP?

Increased BP, widening pulse pressure, increased or decreased pulse, respiratory irregularities, and temperature increase

Chap 52. Insensible fluid loss via exhaled air measures 300 to 400 mL per day. True or False

Insensible loss occurs from the water in exhaled air. In an adult, this is normally 300 to 400 mL per day.

Meningitis signs and symptoms intervention

Mild lethargy photophobia deterioriation of level of consciousness nuchal rigidity, + Kernig's sign + Brudzinski sign red macular rash with meningococcal Abdominal & chest pain with viral assess for signs of ICP, initiate seizure precautions, HOB 30 degree, avoid neck and hip flexion, rest of what you would expect

CK-2 MI; Cardiac Enzyme

Onset: 3-6 hour Peak: 12-24 hour Return to Normal: 2-5 days

Hypermagnesium & Electrocardiographic Changes

Prolonged PR interval Widened QRS complexes

Hyponatremia + normovolemic - assessment findings

Pulse - rapid BP - normal

Cushings Treatment

Restrict Sodium observe for signs of infection

Chronic Myelogenous Leukemia

Results from abnormal production of granulocytes. Poor prognosis

All of the following are considered internal variables that influence a client's health beliefs and practices except:

Socioeconomic factors

Which of the following statements is the World Health Organization's definition of health?

State of complete physical, mental, and social well-being, not merely the absence of disease"

Chap 48. Which of the following is an abnormal color or clarity of urine?

The normal color or clarity of urine is straw, amber, or transparent. Abnormal color or clarity of urine is dark amber, dark orange, red, dark brown, cloudy, mucous plugs, viscid, or thick.

1

The nurse is participating at a health fair at the local mall giving influenza vaccinations to senior citizens. What level of prevention is the nurse practicing? 1. Primary Prevention 2. Secondary Prevention 3. Tertiary Prevention 4. Quaternary Prevention

Chap 43. When asked to sign the permission form for surgical removal of a large but noncancerous lesion on the face, the client begins to cry. Which of the following is the most appropriate response?

The nurse needs to assess and explore the meaning of the client's crying. "You must be very glad to be having this lesion removed" and "Isn't it wonderful that the lesion is not cancer?" leap to assumptions about the meaning of the tears and ignore the possibility of the client's distress. "I cry when I am happy or relieved sometimes, too" suggests that the client has the same feelings as the nurse, which may not be correct.

The nurses anticipates that the pt with hyperphosphatemia secondary to renal failure will require A) Ca supplements B) K supplements C) Mg supplements D) fluid replacement therapy

a Rationale: The major condition that can lead to hyperphosphatemia is acute or chronic renal failure that results in an altered ability of the kidneys to excrete phosphate. For the patient with renal failure, measures to reduce serum phosphate levels include calcium supplements, phosphate-binding agents or gels, and dietary phosphate restrictions.

What is the med of choice for SVT?

adenosine

What's DOC for paroxysmal atrial tach?

adenosine

MDI

administer bronchodilators before other meds

ALS (Amytrophic Lateral Sclerosis)

aka Lou Gehrig's disease

constrict, increase, improves

alpha-blocking agents such as Cardura (doxazosin mesylate), Hytrin (terazosin) and Flomax (tamsulosin) are agents that dilate/constrict the prostate which increases/reduces urethral pressure and improves/impedes urine flow

when nursing was taken out into the community nursing education was essentially

an apprenticeship

when you are conducting a home care admission interview, time will be saved by

asking open-ended questions to obtain essential data

the person who has abnormal nonmusical sounds during inspiration

crackles

What does diamox tx?

glaucoma, motion sickness

What can rheumatic fever lead to?

heart valve probs

yes (if >100 in adult or >120 in infant consult MD)

is it true that prior to taking thyroid medication that the pt should assess pulse rate?

air embolus

may occur when changing bags, opening line

oliguric and diuretic

name two stages of ARF

When does pathological jaundice happen?

occurs before 24 hr and lasts 7 days

If the baby is in a posterior position, where are the heart sounds heard?

on the sides

catheter embolus

piece obstructing blood flow may have decreased bp, pain along vein, loss of conscoiusness

What would you given tigan for?

postop n/v nausea assoc with gastroenteritis

tube feeding

prep formula, check placement(aspirate stomach contents/measure residual), "make sure it does not exceed dr.s order, inject r10-20 ml of ai

medical asepsis differs from surgical asepsis in that medical sepsis is aimed at

preventing transmission of microorganisms

chronic renal failure

progressive and irreversible destruction of nephron mass, characterized by prolonged signs and symptoms of uremia. End result of all renal diseases. Glomerulonephritis is most common cause, others include diabetes mellitus, polycystic kidney disease, nephrosclerosis, pyelonephritis, interstitial nephritis Different stages include diminished renal reserve, renal insufficiency, overt renal failure, end stage renal disease

What's thoracentesis position?

pt with arms on pillow over bed table or lying on side no more than 1000 cc at a time

when mixing insulins

put air into cloudy first, then clear

nursing interventions for hypoglycemia

quick sugar source(glucose tabs, 4oz. juice, 6 oz. soda) wait 5-10 mins, recheck sugar, rrepreat if persists

IM meds

quickly absorbed, 90 degree angle, 19-23 G needle, 1-2" length, average adult:11/2, 22G, mandatory to aspirate, purpose is to deposit meds into deep muscle for quick absorption

What position during epidural puncture?

side lying

Where should the placenta be located?

the upper part of the uterus

What are FAS signs?

upturned nose flat nasal bridge thin upper lip SGA

ataxia, slurred speech, tremors, pruritus, skin eruptions

what are the s/s associated with uremia (occurs with renal failure)?

potassium

with renal insufficiency, despite cause, what dietary item is restricted when patient has oliguria?

when a negligence case is brought against a nurse, it is necessary to prove liability. which element is NOT essential for proof of liability

witnesses

What's dig level?

.5-2.0

Normal Sedimentation rate

0-30 mm/hr

A client who had abdominal surgery complains of feeling as though something gave way in the incisional site. The nurse removes the dressing and notes the presence of a loop of bowel protruding through the incision. Select all nursing interventions that nurse w/take.:

1. Notify RN 2. Document the client's complaint. 3. Instruct the client to remain quiet. 4. Prepare the client for wound closure.

A client arrives to the surgical nursing unit after surgery. The initial nursing action is to check the:

1. Patency of the airway. If the airway is not patent, immediate measures must be taken for the survival of the client. After checking the client's airway, the nurse would then check the client's vital signs, and this w/be followed by checking the dressings, tubes and drains.

A nurse is explaining the Joint Commission's universal protocol for preventing wrong-site, wrong-procedure, and wrong-person surgery to a group of nursing students. The nurse explains that site marking involves:

1. The surgeon marking the area of the operative procedure. The surgeon is responsible for verifying the operative site, and he or she must mark the operative site before the client is brought into the operating suite. The client will be asked to verify the site that requires surgery. The client may refuse to have the site marked and is asked about marking the site.

Therapeutic Drug level Theophylline

10 - 20 mcg/ml

Ammonia

10 - 80 mcg/dL elevated - hepatic dysfunction may lead to encephalopathy fast except for H20 no smoking 8 - 10 hrs before test

44.The nurse concludes that further explanation about standard precautions is needed when a family member of a client with acquired immunodeficiency syndrome (AIDS) states:</question> <question_image/> <choice_1>"I need to wear a mask when I visit."</choice_1> <choice_2>"We can still hug one another."</choice_2> <choice_3>"I don't need to use special dishes."</choice_3> <choice_4>"My children cannot catch AIDS if they play a board game together."</choice_4>

1</correct> <rationale>Standard precautions are used with all clients, regardless of the medical diagnosis. Clients with AIDS are not contagious, and family members are not required to wear protective equipment in a casual interaction.</rationale>

1 qt

2 pt

The nurse is conducting an integumentary assessment of an African American patient who has darkly pigmented skin and a history of chronic obstructive pulmonary disease (COPD). Which of the following locations should the nurse inspect for cyanosis (select all that apply)? 1. Patient's sclera 2. Patient's nail beds 3. Soles of the patient's feet 4. Palms of the patient's hands 5. Conjunctiva of the patient's eyes

2,5. In patients with darkly pigmented skin, the conjunctiva and nail beds are often examined to assess for cyanosis. Palms of the hands, soles of the feet, and the sclera are not the focus when assessing for cyanosis

When planning the care of a patient with dehydration, you would instruct the nursing assistive personnel (NAP) to report which of the following? 60 ml urine output in 90 minutes 1200 ml urine output in 24 hours 300 ml urine output per 8-hour shift 20 ml urine output for 2 consecutive hours

20 ml urine output for 2 consecutive hours The minimal urine output necessary to maintain kidney function is 30 ml/hr. If the output is less than this for 2 consecutive hours, the nurse should be notified so that additional fluid volume replacement therapy can be instituted.

30. The nurse would ask a client scheduled for thyroid scanning about allergy to which of the following before the procedure?</question> <question_image/> <choice_1>Peanuts</choice_1> <choice_2>Shellfish</choice_2> <choice_3>Eggs</choice_3> <choice_4>Meat tenderizer</choice_4>

2</correct> <rationale>Iodine is used in many radiological procedures. Shellfish allergies may be an indicator of iodine allergy. The other options do not address this concern.</rationale>

ABG - PCo2

35 - 45 mm Hg

Hematocrit - female adult

35 - 47 %

A male client is admitted with urinary tract problems. A prostate-specific antigen (PSA) and acid phosphatase test are to be done. The nurse knows that 1. these tests are valuable screening tests for prostatic cancer. 2. the level of PSA is decreased in clients with renal stones. 3. the tests reflect the level of renal involvement in acid-base problems. 4. the level of PSA is elevated in clients in early-stage renal failure.

(1) correct—PSA test has replaced acid phosphatase test in screening for prostatic cancer; test must be drawn before digital rectal exam, as manipulation of the prostate will abnormally increase PSA value (2) inaccurate information about a PSA (3) inaccurate information about a PSA (4) inaccurate information about a PSA

The nurse in the pediatrician's office observes a child in the waiting room. The nurse notes that the child can walk up and down steps, has a steady gait, can stand on one foot momentarily, and jumps with both feet. The nurse identifies the child's chronological age to be which of the following? 1. 1 year old. 2. 2 years old. 3. 3 years old. 4. 5 years old.

(1) unable to walk up and down stairs with hand held until 18 months (2) unable to jump until 30 months (3) correct—able to jump with both feet and stand on one foot momentarily at 30 months (4) behaviors are seen in younger child

Normal QRS interval time

0.04-0.10 seconds

Platelets

150,000 - 400,000 cells/mm3

Normal Platelet Count

150,000-450,000 cells/mm3 Bleeding precautions should be set when drops, including avoiding trauma such as rectal temps/injections

729. Intravenous heparin therapy is prescribed for a client. While implementing this prescription, a nurse ensures that which of the following medications is available on the nursing unit? 1. Protamine sulfate 2. Potassium chloride 3. Aminocaproic acid (Amicar) 4. Vitamin K (AquaMEPHYTON)

729. 1 Rationale: The antidote to heparin is protamine sulfate; it should be readily available for use if excessive bleeding or hemorrhage should occur. Vitamin K is an antidote for warfarin sodium. Aminocaproic acid is the antidote for thrombolytic therapy. Potassium chloride is administered for a potassium deficit. Test-Taking Strategy: Knowledge regarding the various antidotes is needed to answer this question. Remember the antidote to heparin is protamine sulfate. Learn these antidotes if you had difficulty with this question.

ABG - PO2

80 - 100 mm Hg

Triiodothyronine (T3)

80 -230 ng/dL

Unrelieved pain is? A) expected after major surgery B) expected in a person with cancer C) dangerous and can lead to many physical and psychologic complications D) an annoying sensation, but it is not as important as other physical care

: c Rationale: Consequences of untreated pain include unnecessary suffering, physical and psychosocial dysfunction, impaired recovery from acute illness and surgery, immunosuppression, and sleep disturbances. In the acutely ill patient, unrelieved pain can result in increased morbidity as a result of respiratory dysfunction, increased heart rate and cardiac workload, increased muscular contraction and spasm, decreased gastrointestinal motility and transit, and increased breakdown of body energy stores (i.e., catabolism).

The production of red blood cells in the bone marrow is called: A. hematopoiesis. B. hemolysis. C. hemoptysis. D. hemianopsia.

A

3. When assessing a temperature rectally, the nurse would use extreme care when inserting the thermometer to prevent which of the following? a. An increase in heart rate b. A decrease in heart rate c. A decrease in blood pressure d. An increase in respirations

B.

A client in renal failure asks why he is being given antacids. How should the nurse reply?

Calcium and aluminum antacids bind phosphates and help to keep phosphates from being absorbed into bloodstream, thereby preventing rising phosphate levels; must be taken with meals

Chap 52. A client with hypokalemia complains of an irregular heart rate. What are the most appropriate initial nursing interventions?

Cardiac dysrhythmias can be a result of hypokalemia and if it occurs it is an emergency situation. While notifying the primary care provider, monitor closely the heart rate and rhythm as well as the blood pressure. It is important to monitor the cardiovascular status, but encouraging fluids and starting an IV infusion are not appropriate immediate interventions. Trendelenberg's position is useful for clients in shock, not for dysrhythmias.

An APN is the most independently functioning of all professional nurses. All of the following are examples of a clinically focused APN except:

Care provider

nephrotic syndrome

Clinical state characterized by edema, proteinuria, hypoalbuminemia, hyperlipidemia & altered immunity. Is a group of symptoms - not a disease. Two classifications, 1) Primary (follows an infection), and 2) Secondary (follows multi-system disorder ie diabetes, lupus, cycle cell)

Which of the following sets forth ethical principles for professional nursing practice in a clinical setting?

Code of Ethics of the American Nurses Association (ANA)

Identify 2 types of hearing loss?

Conductive (transmission of sound to inner ear is blocked) Sensorineural (damage to 8th cranial nerve)

4.While taking an adult patient's pulse, a nurse finds the rate to be 140 beats/min. What should the nurse do next? a. Check the pulse again in 2 hours. b. Check the blood pressure. c. Record the information. d. Report the rate.

D.

Which intervention is appropriate to include on a care plan for improving sleep in the older adult?

Decrease fluids 2 to 4 hours before sleep By decreasing fluids 2 to 4 hours before sleep, it is less likely that the client will awaken because of a need to urinate. Limiting naps during the day will help improve nighttime sleep. The client should sleep until the same time each morning. Exercising in the evening can make falling asleep more difficult.

Chap 38. A client is at risk for sensory deprivation. Which of the following clinical signs are most likely to contribute?

Decreased attention span, excessive sleeping, crying and depression are clinical signs of sensory deprivation. Sleeplessness and irritability are clinical signs of sensory overload.

Chap 49. A client has the flu with gastrointestinal symptoms and has been incontinent. This is not uncommon if the client has:

Diarrhea refers to the passage of liquid feces and an increased frequency of defecation. Rapid passage of chyme reduces the time available for the large intestine to reabsorb water and electrolytes. The person with diarrhea finds it difficult or impossible to control the urge to defecate for very long. It is a threat of incontinence.

Mydriatics

Dilate the pupils

Answer: A Phenomena are defined as aspects of reality that can be consciously sensed or experienced.

Nursing theories focus on the phenomena of nursing and nursing care. Which of the following is true of phenomena? A) They are aspects of reality that can be consciously sensed or experienced. B) They convey the general meaning of concepts in a manner that fits the theory. C) They are statements that describe concepts or connect two concepts that are factual. D) They are mental formulations of an object or event that come from individual perceptual experience.

When analgesics are ordered for a client with obstructive sleep apnea (OSA) following surgery, the nurse is most concerned about:

Opioids Clients with obstructive sleep apnea are particularly sensitive to opioids. Thus the risk of respiratory depression is increased. The nurse must recognize that clients with OSA should start out receiving very low doses of opioids.

A client is fearful of upcoming surgery and a possible cancer diagnosis. The client has discussed a love of the Bible with the nurse, who then recommends a favorite Bible verse. The nurse is reprimanded and told that there is no place in nursing for spiritual caring. Which of the following would be an appropriate response?

Spiritual, mind, and body connections can affect health."

true

T/F impotence medication such as Viagra (sildenafil citrate) or Cialis (tadalifil) should not be take after a meal especially high fat meals because fat reduces the absorption and decreases peak levels?

A 72-year-old man diagnosed with chronic obstructive pulmonary disease 5 years ago has been participating for the last 2 years in a pulmonary rehabilitation exercise class offered by the local hospital at a fitness facility. This is what level of prevention?

Tertiary prevention

Answer: C Obtaining a sterile specimen requires insertion of a catheter, a procedure that must be performed by a licensed nurse. Therefore, this would not be an appropriate task to delegate to an assistive person. Assistive personnel would be able to ambulate a client, give a bed bath, and add to the I&O record.

Which task is it not appropriate for a professional nurse to delegate to assistive personnel? A) Ambulate a client B) Complete a bed bath C) Obtain a sterile urine specimen D) Complete the intake and output (I&O) record

Thyroxine free (FT4)

0.8 to 2.4 ng/dL

32.The nurse is administering an IM medication to a client known to have hepatitis B. Which precautions should the nurse use to protect against exposure? Select all that apply.</question> <question_image/> <choice_1>Hand washing</choice_1> <choice_2>Gloves</choice_2> <choice_3>Mask</choice_3> <choice_4>Face shield</choice_4> <choice_5>Gown</choice_5>

1, 2</correct> <rationale>Hand washing and gloves are the only precautions needed for administering an IM medication IV. Masks, face shields, and gowns are appropriate for procedures that may result in body fluids splashing.

17. A client has developed paroxysmal nocturnal dyspnea. Which of the following medications does a nurse anticipate will be prescribed by the physician? 1. Propranolol (Inderal) 2. Bumetanide (Bumex) 3. Lidocaine (Xylocaine) 4. Streptokinase (Streptase)

17. 2 Rationale: Bumetanide (Bumex) is a diuretic. The paroxysmal nocturnal dyspnea may be due to increased venous return when the client is lying in bed, and the client needs diuresis. Propranolol is a b-blocker, lidocaine is an antiarrhythmic, and streptokinase is a thrombolytic. Test-Taking Strategy: Use the process of elimination. Knowledge of each medication type and that a diuretic will increase urine output will direct you to option 2. Review the actions of the medications identified in the options, if you had difficulty with this question.

A patient had a left-sided cerebrovascular accident 3 days ago and is receiving 5000 units of heparin subcutaneously every 12 hours to prevent thrombophlebitis. The patient is receiving enteral feedings through a small-bore nasogastric (NG) tube because of dysphagia. Which of the following symptoms requires the nurse to call the health care provider immediately? A) Pale yellow urine B) Unilateral neglect C) Slight movement noted on the R side D) Coffee ground-like aspirate from the feeding tube

D

A scooped out, shallow depression in the skin is called a/an: A. ulcer. B. excoriation. C. fissure. D. erosion.

D

When taking the health history, the patient complains of pruritus. What is a common cause of this symptom? A. Excessive bruising B. Hyperpigmentation C. Cancer D. Drug reactions

D

side effects of barbituates

drowsiness, mood change, depression

Chap 34. As a nurse obtains a capillary glucose reading, the nurse knows that the meter:

As a nurse obtains a capillary glucose reading, the nurse knows that the meter must be calibrated or it will give an inaccurate reading.

What is the best indicator for dehydration?

weight

Uric Acid test consideration

Elevated amount deposit in joints =gout Client must fast for 8 hrs before Cause false elevations - Theophylline, caffeine, Vitamin C

Sensorineural Hearing Loss

Form of hearing loss in which sounds passes properly through the outer and middle but is distorted by a defect in the inner ear. Common causes: infection, ototoxic drugs, trauma, neuromas, noise, aging process

Chap 30. What are the methods used for physical examination?

Four primary techniques are used during the physical examination: inspection, palpation, percussion, and auscultation. Analysis is not a physical examination technique.

Lung Cancer Pathophysiology Precipitating Factors Assessment Nursing Intervention

Lung cancer is the leading cause of cancer related death in US; Cigarette smoking is responsible for 80-90% of cases. Exposure to occupational hazards such as asbestos and radioactive dust poses significant risk; lung cancer tends to appear years after exposure and is most commonly seen in persons 50-60 years old; lung cancer has a poor prognosis-- 5 year survival rate is 14% Dry hacking cough, hoarseness, dyspnea, hemopytsis, pain in chest, diminished breath sounds, abnormal chest radiograph, positive sputum for cytology and for pleural fluid Semi-Fowlers, teach purse lip breathing to improve gas exchange, teach relaxation techniques, administer O2 as indicated by pulse oximetry or ABGs, decrease pain to manageable level by administering analgesics as needed

Chap 38. An 85-year-old client has impaired hearing. When creating the care plan, which of the following should have the highest priority?

The amplified telephone helps with hearing and provides a means for communicating with others. "Teaching the importance of changing his position" refers to a tactile impairment. "Providing reading material with large print" relates to a visual impairment, and "Checking expiration dates on food packages" is an olfactory impairment.

Answer: C Healthy People 2010, a federal document, outlines goals for the public. Notes on Nursing set forth Nightingale's first nursing philosophy. The Last Acts Campaign has developed standards and policies for end-of-life care. Nursing Principles and Practice 2010—current readings in journals are necessary for all nurses in practice.

The document that developed goals and objectives to meet the health of the public is known as: A) Notes on Nursing B) Last Acts Campaign C) Healthy People 2010 D) Nursing Principles and Practice 2010

epinephrine

adrenergic - stimulates alpha/beta receptors - SE: tremulouness, headache, tachycardia, vomiting. nursing care: teach client to read label on OTC meds

When does physiologic jaundice happen?

after 24 hr

cromolyn sodium (Intal)

ant-inflammatory agent used for maintenance therapy for Asthma

he nurse recognizes that the majority of patients' caloric needs should come from which of the following sources? a) Fats b) Proteins c) Polysaccharides d) Monosaccharides

c) polysaccharides Carbohydrates should constitute between 45% and 65% of caloric needs, compared with 20% to 35% from fats and 10% to 35% from proteins. Polysaccharides are the complex carbohydrates that are contained in the breads and grains that form the base of the food pyramid.

Position after infratentorial surgery?

incision at nape of neck place pt flat and lateral on either side

What is bucks traction used for?

knee immobility

Position for prolapsed cord?

knee-chest position or trendelenburg

What is the tx for scabies & pediculosis?

kwell

What reverses ammonia?

lactulose

What are un-reassuring signs of a woman in labor?

late decel dec variability fetal brady

smaller to larger

left

How to position someone with PE/air embolism?

left lower side

If a woman is in distress during labor, what position do you place her in?

left side

What type of diet for hyperPTH?

low ca, high po4

Placement after thyroidectomy?

low or semi-fowlers, spt head, neck, and shoulders

Why might a cancer pt have epitaxis?

low platelets

The primary function of the skin is: A. insulation B. protection C. sensation D. absorption

B. The primary function of the skin is to protect the underlying tissues of the body by serving as a surface barrier to the external environment.

Cabbage, tomatoes, potatoes, and strawberries are high in ______________

Vitamin C

a way to promote trust with a patient is to

follow through when you say you will do something

what is the prophylactic drug of choice for household TB contacts

isoniazid (INH)

What's the DOC for bipolar?

lithium

What to give for inc ICP?

mannitol

What can longterm use amphogel lead to?

well it binds the PO4 and inc Ca robbing the bones of their Ca which can lead to OA

Profile for gallbladder disease

Fair, fat, forty, five pregnancies, flatulent actually gallbladder disease can occur in all ages and both sexes

Causes of Respiratory Alkalosis

Fever, hyperventilation, hypoxia, hysteria, overventilation by mechanical ventilators, pain

If a nurse assesses a client for pain and then offers a plan to manage the pain, which principle encourages the nurse to monitor the client's response to the pain management plan?

Fidelity

Chap 52. Which of the following is a route of fluid output?

Fluid losses from the body counterbalance the adult's 2,500 mL average daily intake of fluid. There are four routes of fluid output: urine, insensible loss through the skin as perspiration and through the lungs as water vapor in the expired air, noticeable loss through the skin, and loss through the intestines in feces.

When planning care for a patient with dehydration related to nausea and vomiting, the nurse would anticipate which of the following fluid shifts to occur because of the fluid volume deficit? Fluid movement from the blood vessels into the cells Fluid movement from the interstitial spaces into the cells Fluid movement from the blood vessels into interstitial spaces Fluid movement from the interstitial space into the blood vessels

Fluid movement from the interstitial space into the blood vessels In dehydration, fluid is lost first from the blood vessels. To compensate, fluid moves out of the interstitial spaces into the blood vessels to restore circulating volume in that compartment. As the interstitial spaces then become volume depleted, fluid moves out of the cells into the interstitial spaces.

Fab 9

Folic acid = B9 B stands for decreases the incidence of neural tube defects should take 3 months prior to becoming pregnant

Contractions (frequency, duration & Intensity)

Frequency - 2 - 5 mins apart Normal duration < 90 secs Intensity <100 mm hg

Older adults experience a change in sexual activity. Which best explains this change?

Frequency and opportunities for sexual activity may decline.

125ml per minute

Glomerular filtration rate is the Amount of filtrate formed each minute Important gauge of renal function. normal values is

Sickle Cell Treatment

HHOP Heat Hydration Oxygen Pain Medications

Hypomagnesium - assessment finding

HR - Tachycardia BP - Hypertension Resp - Shallow Neuro - twiches, + Trousseau & Chvostek's, Hyperreflexia, tetany, seizures CNS - irritability, confusion

Hypermagnesium - assessment finding

HR - bradycardia, dysrhythmias BP - hypotension Resp - insuff. when sketal muscles are involved Neuro - diminished to absent deep tendon reflexes, sketal muscle weakness CNS - drowsiness & lethargy, progress to coma

Hypocalemia - assessment finding

HR - decreased, diminished peripheral pulses BP - hypotension Resp - not directly impacted but resp failure or arrest can result from decreased resp movement Neuro - muscle (twitch,cramp, tetany, seizures), painful muscle spasm (calf), positive Trousseau's & Chvostek's, hyperactive deep tendon reflexes, anxiety & irritablility Renal - urniary output varies, depending on cause GI - hyperactive bowel sounds, cramping, diarrhea

When the nurse signs a form as a witness, the nurse's signature shows that the client:

Has signed that form and the witness saw it being done

A 33-year-old patient has recently been diagnosed with stage II cervical cancer. The nurse would understand that the patient's cancer Is in situ. Has metastasized. Has spread locally. Has spread extensively.

Has spread locally Stage II cancer is associated with local spread. Stage 0 denotes cancer in situ; stage III denotes extensive regional spread, and stage V denotes metastasis.

Nurses have the responsibility to dispel myths and replace stereotypes of older adults with accurate information. The nurse knows that most older adults:

Have a reduced ability to respond to stress

Older adults are at increased risk for drug toxicity because they:

Have reduced kidney functioning

A nurse notes that the health care unit keeps a listing of clients' names at the front desk in clear view so that health care providers can more efficiently locate clients. The nurse knows that this action is a violation of which act?

Health Insurance Portability and Accountability Act

The nurse is complying with the provisions of which act when the nurse requests that the client give permission to copy the client's medical reports for the insurance company?

Health Insurance Portability and Accountability Act (HIPAA)

Clients maintain health or enhance their health by routine exercise and proper nutrition. This is known as:

Health promotion

The document that developed goals and objectives to meet the health of the public is known as:

Healthy People 2010

-dine

Histamine H2 blocker/antagonists

A nurse routinely asks clients if they take any vitamins or herbal medications, encourages family members to bring in music that clients like to help them relax, and frequently prays with clients if that is important to them. The nurse is using which model of care?

Holistic

Chap 42. While performing discharge planning for a client recovering from a stroke, the nurse may use which of the following factors as an indication of the client's potential for effective coping?

How the client and family understand stressors, and the family's ability to provide a supportive environment, are important factors for the nurse to consider. Effective coping is not dependent upon one's education level. Confidence is an important element of coping, but may not be based on the reality of the client's current condition. Insurance may help the client afford community resources not covered by Medicare, but this alone does not indicate a client's or family's ability to cope.

S/Sx of autonomic dysreflexia

Hypertension, bladder and bowel distention, exaggerated autonomic responses, headache, sweating, goose bumps, and bradycardia

Chap 52. The nurse administers an IV solution of D5 1/2 NS to a postoperative client. This is classified as a __________ intravenous solution.

Hypertonic Solutions: 5% dextrose in 0.45% NaCl (D5 1/2NS) Hypertonic solutions draw fluid out of the intracellular and interstitial compartments into the vascular compartment, expanding vascular volume. Do not administer to clients with kidney or heart disease or clients who are dehydrated. Watch for signs of hypervolemia.

What does Bentyl tx?

IBS, its an anticholinergic (dec bowel movements)

What route for Fe inj?

IM so as not not leak into sub Q

Different attitudes about illness cause people to react in different ways when illness does occur. Medical sociologists call the reaction to illness:

Illness behavior

Addison'sTreatment

Increase Sodium intake Medications include cortison preparation

Chap 42. A client is having difficulty exploring and using resources available to deal with the stress of raising a child. Which of the following is the most appropriate nursing diagnosis?

Ineffective Coping is the inability to form a valid appraisal of stressors, inadequate choices of responses, and inability to use available resources. Post-Trauma Syndrome is a sustained maladaptive response to a traumatic event. Decisional Conflict is uncertainty about the course of action among competing actions that involve loss or challenge to life values. Defensive Coping is repeated projection of falsely positive self-evaluation to defend against a perceived threat to self-esteem.

prerenal ARF

Interference with renal perfusion Causative factors: hemorrhage, hypovolemia, decreased cardiac output, decreased renal profusion

Chap 52. A client is diagnosed with a GI bleed. The nurse has an order to administer two units of packed red blood cells (PRBCs). What is the purpose of this order?

Intravenous fluids can be effective in restoring intravascular (blood) volume; however, they do not affect the oxygen-carrying capacity of the blood. A blood transfusion is the introduction of whole blood or blood components into the venous circulation. When red and white blood cells, platelets, or blood proteins are lost because of hemorrhage or disease, it may be necessary to replace these components to restore the blood's ability to transport oxygen and carbon dioxide, to clot, to fight infection, and to keep extracellular fluid within the intravascular compartment.

Acute Myelogenous Leukemia

Involves inability of leukocytes to mature, those that do are abnormal Poor prognosis, cause of death is usually infection

Chronic Lymphocytic Leukemia

Involves increased production of leukocytes and lymphocytes and proliferation of cells within the bone marrow, spleen, and liver

Chap 34. A client reports an iodine allergy. This information is most significant if the client is scheduled for which exam?

Iodine is present in the dye used in some imaging x-rays and scans including intravenous pyelogram. Iodine is not used in lung scan, computed tomography, or magnetic resonance imaging.

Chap 48. Which of the following behaviors indicates that the client on a bladder training program has met the expected outcomes?

It is important for the client to inhibit the urge-to-void sensation when a premature urge is experienced by performing slow, deep breathing and pelvic muscle exercises . Some clients may need diapers; however, this is not the best indicator of a successful program. Citrus juices may irritate the bladder. Carbonated beverages increase diuresis, increasing the risk of incontinence.

Chap 43. In assessing a client who will be undergoing a lung resection, the nurse understands the significance of losing a body part. Which of the following questions is most appropriate to ask during the assessment?

It is important to ask the client about what changes the surgery will cause in order to find the meaning of this current loss of a body part. Asking what the client will do after the surgery is appropriate to assess the client's current grieving reaction. Questioning about what the nurse can do to help is too general and is not an appropriate assessment question. Asking about losses in the past assesses previous experiences.

Chap 43. What is a therapeutic strategy for communicating with a client who is dying?

It is therapeutic to acknowledge that clients who are dying may be having some struggles or concerns. Encouraging silence is denying the client the chance to speak about concerns. Talking about pleasant events is ignoring the fact that clients may have some issues with which to deal. Reassurance is appropriate unless it is false reassurance, but sympathy is nontherapeutic. Clients who are dying do not need pity.

Chap 42. Immediately after the parents of a hospitalized child are informed that the child has leukemia, the father responds by continuing his usual work schedule, rarely visiting, and asking when the child can return to school. Of the following, which is the least likely to be an appropriate nursing diagnosis at this time?

It is too soon for Caregiver Role Strain to be an appropriate nursing diagnosis—especially since the child is not at home. Ineffective Denial and Fear are common reactions to this type of threat. The father demonstrates Compromised Family Coping by his difficulty in being supportive.

A client needs a liver transplant but has been out of work for several months and does not have insurance or enough cash. A discussion about the ethics of this situation would predominantly involve which principle?

Justice

Chap 38. A client has impaired vision. An intervention to best adapt the environment to this loss includes:

Keeping the room pathways free of clutter is the best intervention to adapt the environment for the client with impaired vision. Interventions will focus on compensating for the loss of vision in order to optimize client independence. Initially, the nurse may need to assist the client with ambulation. Later, the nurse will need to evaluate whether this is still necessary. Clients unfamiliar with side rails may find them confusing and actually fall trying to climb over them. Regular schedules help clients orient to time, but are less likely to benefit those with vision loss.

Stroke Left Hemisphere

Language: aphasia, agraphia Memory: no deficit Vision: unable to discriminate words and letters, reading problems, deficits in right field. Behavior: slow, cautious, anxious when attempting new task, depression or catastrophic response to illness, sense of guilt, feelings of worthlessness, worries over future, quick anger and frustration Hearing: no deficit

Chap 49. The elderly population is known to use laxatives with regularity. In advising an older adult practicing this habit, the nurse would NOT identify which of the following?

Laxatives decrease the absorption of vitamins. The remaining answer choices are true.

Chap 38. A client's lifestyle affects the:

Lifestyle influences the quality and quantity of stimulation to which an individual is accustomed.

Thyroid Storm

Lifethreatening event that occurs with uncontrolled hyperthyroidism due to Graves disease. S/Sx: fever, tachycardia, agitation, anxiety, and hypertension. Propylthiouracil and methimazole are antithyroid drugs.

The nurse finds a client sleepwalking down the unit hallway. An appropriate intervention the nurse implements is:

Lightly tapping the client on the shoulder and leading him or her back to bed. The nurse should not startle the client but should gently awaken the client and lead him or her back to bed. Sleepwalkers are unaware of their surroundings. Asking them what they are doing is not helpful. The nurse may or may not need assistance. Startling the client may result in injury. Blocking the walkway with chairs may result in injury.

Chap 49. Which of the following interventions is appropriate for a client with flatulence?

Limit carbonated fluids and the use of straws and chewing gum to avoid ingestion of air. Cauliflower and onions are gas producing and should be avoided. Excessively hot or cold fluids stimulate peristalsis and should be avoided by clients with diarrhea. Increasing daily fluid intake is recommended for clients with constipation.

-caine

Local Anesthetics

Agraphia

Loss of ability to write

Bilirubin, total (general info and test considerations)

Lower than 1.5mg/dL Increases with any type of jaundice avoid yellow foods (carrots, yams, yellow beans, pumpkin) for 3-4 days Fast for 4 hours Elevated - alcohol, morphone sulfate, theophylline, ascorbic acid (Vitamin C), acetylsalicylic acid (aspirin)

What is dz has descending muscle weakness?

MG

What dz are airborn precautions?

MTV M-measles T-TB V-varicella/herpes zoster (shingles)

List five nursing interventions after chest tube insertion

Maintain a dry occlusive dressing on chest tube; keep all tubing connections tight and taped; monitor client's clinical status; encourage the client to breathe deeply periodically; monitor the fluid drainage; mark the time of measurement and the fluid level.

Chap 49. A nurse is evaluating a client's understanding of healthy defecation. Which of the following statements indicates a need for further teaching?

Maintaining fluid intake of 2000 to 3000 mL a day promotes healthy defecation. Other client teachings for healthy defecation are: Include high-fiber foods, such as vegetables, fruits, and whole grains, in the diet; allow time to defecate, preferably at the same time each day; avoid over-the-counter medications to treat constipation and diarrhea.

Hodgkin Disease

Malignancy of lymphoid system Generalized painless lymphadenopathy

Leukemia

Malignant neoplasm of blood forming organs. Leukemia is characterized by an abnormal overproduction of immature forms of any of the leukocytes. There is an interference with normal blood production that results in decreased numbers of erythrocytes and platelets. Dx by biopsy, bone marrow aspiration, lumbar puncture, and frequent blood counts. Tx: antineoplastic chemo

Chap 43. The ability of an individual to cope with death is dependent upon a number of factors. Which person is likely to have the most difficulty coping with a death?

Many factors affect the grieving experience. These include age, significance of the loss, culture, spiritual beliefs, gender, socioeconomic status, social support systems, and the cause of the death. In our culture, the death of an older person is accepted more easily than that of a younger person. The death is more easily accepted if it is anticipated, and if the person who died did not contribute to the death. Usually, the closer the individual is to the person who died, the more difficult it is to cope with the death.

During mechanical ventilation, what are three major nursing interventions?

Monitor client's respiratory status and secure connections; establish a communication mechanism with the client; keep airway clear by coughing and suctioning

Which statement made by a mother being discharged to home with her newborn infant indicates a need for further teaching?

My grandmother told me that babies sleep better on their stomachs." Babies should sleep on their backs, not their stomachs, as a SIDS preventative. Babies should not be put to bed with a bottle. Due to nighttime feedings, new moms should be encouraged to temporarily place a cradle near where they sleep and know that they will have to get up during the night to feed the baby.

Oxygen Administration

Nasal Cannula: low O2 flow for low O2 concentrations Simple face mask: low flow, but effectively delivers high O2 concentrations; cannot deliver <40% O2 Nonrebreather mask: low flow, but delivers high O2 concentrations (60%-90%) Partial rebreather mask: low flow O2 reservoir bad attached; can deliver high O2 concentrations Venturi mask: high flow system; can deliver exact O2 concentration

A client who is confused is left alone in bed with the side rails down and the bed in a high position, and the client falls and breaks a hip. In legal terms, what has occurred?

Negligence

A child's immunization may cause discomfort during administration, but the benefits of protection from disease outweigh the temporary discomforts. Which principle applies to this situation?

Nonmaleficence

Chap 48. A nurse is testing urine for specific gravity. Which of the following would be considered a normal result range?

Normal results for a urine specific gravity should be in the range of 1.010 to 1.025. All other results not in this range are considered abnormal.

Which of the following is an example of a violation of the provisions of the Health Insurance Portability and Accountability Act (HIPAA)?

Nurses discuss a client over dinner at the hospital cafeteria.

Chap 30. Nurses perform physical assessments for different reasons. One of those reasons is:

Nurses perform physical assessments for different reasons. One of those reasons is to obtain data about the client's functional abilities.

Answer: A A nurse who is new to practice has not developed the experience required for research but can begin at the less complicated level of data collection. An experienced researcher is more qualified to identify problems for formal research, although input from all levels of nursing is valuable. Nurses with doctoral-level training are typically prepared for obtaining financial backing. An American Nurses Association position paper cites a master's degree as qualification for implementing research-based change in nursing practice.

Nurses who are new to practice can best contribute to nursing research by: A) Assisting with data collection B) Identifying clinical problems in nursing C) Obtaining financial backing and public interest D) Implementing research-based change in nursing practice

Chap 43. When caring for a dying client, the nurse observes for which of the following signs of impending clinical death? (Select all that apply.) Note: Credit will be given only if all correct choices and no incorrect choices are selected.

Nursing care and support for the dying client and family include making an accurate assessment of the physiologic signs of approaching death. In addition to signs related to the client's specific disease, certain other physical signs are indicative of impending death. The four main characteristic changes are loss of muscle tone which makes it difficult for the client to speak, slowing of the circulation, changes in respiration, and sensory impairment.

Chap 43. Which of the following actions is NOT appropriate for the nurse providing postmortem care?

Nursing personnel may be responsible for care of a body after death. Normally, the body is placed in a supine position with the arms either at the sides (palms down) or across the abdomen. Dentures are usually inserted to help give the face a natural appearance. The mouth is then closed. One pillow is placed under the head and shoulders to prevent blood from settling into the face and discoloring it. The eyelids are closed and held in place for a few seconds so that they remain closed. All jewelry is removed, except a wedding band in some instances, which is taped to the finger.

Answer: A The client is the correct choice. The health care facility is where the client goes to receive treatment. The nursing process is how nurses proceed to plan care for the client. Cultural diversity is not the correct choice.

Nursing practice in the twenty-first century is an art and science that is centered on: A) The client B) The nursing process C) Cultural diversity D) The health care facility

Answer: D Nursing's paradigm includes four linkages: the person, health, environment/situation, and nursing.

Nursing's paradigm includes: A) Health, person, environment, and theory B) Concepts, theory, health, and environment C) Nurses, physicians, models, and client needs D) The person, health, environment/situation, and nursing

Postrenal Stage of ARF

Obstruction in the urinary tract anywhere from the tubules to the urethral meatus Causative Factors: calculi, prostatic hypertrophy, tumors

Chap 49. The most appropriate goal for clients with diarrhea related to ingestion of an antibiotic for an upper respiratory infection is which of the following?

Once the cause of diarrhea has been identified and corrected, interventions should focus on getting clients to return to their previous elimination patterns. This is not an example of an allergy to the antibiotic, but a common consequence of overgrowth of bowel organisms not killed by the drug. Antidiarrheal medications are usually prescribed according to the number of stools, not routinely around the clock. Increasing intake of soluble fiber, such as oatmeal or potatoes, may help absorb excess liquid and decrease the diarrhea, but insoluble fiber will not.

Chap 43. A nurse must tell a family of a client's death. The nurse knows the family will go through the Kübler-Ross stages of grieving, which include which of the following?

One of the most well-known descriptions of the stages or phases of grieving is the one by Kübler-Ross. It describes five stages of grief: denial, anger, bargaining, depression, and acceptance.

What does artane tx?

PD

What does cogentin tx?

PD watch out for EPS

What drugs are given to prevent a thyroid storm?

PTU and tapazole

5 P's of fracture and compartment syndrome

Pain Pallor Pulselessness Paesthesia Polar (Cold)

Chap 30. The nurse documented that a brown-skinned client has pallor. What is the specific observation for this client?

Pallor is most evident in body areas with the least pigmentation, such as the nail beds, oral mucus membranes, palms of the hand, and soles of the feet. Pallor in brown-skinned individuals appears as a yellow- brown tinge, and in black-skinned individuals it appears as ashen gray.

You are caring for a patient admitted with a diagnosis of COPD who has the following arterial blood gas results: pH 7.33, PaO2 47 mm Hg, PaCO2 60 mm Hg, HCO3 32 mEq/L, and O2 saturation of 92%. Which of the following is the correct interpretation of these results? Fully compensated respiratory alkalosis Partially compensated respiratory acidosis Normal acid-base balance with hypoxemia Normal acid-base balance with hypercapnia

Partially compensated respiratory acidosis A low pH (normal 7.35-7.45) indicates acidosis. In the patient with respiratory disease such as COPD, the patient retains carbon dioxide (normal 38-48 mm Hg), which acts as an acid in the body. For this reason, the patient has respiratory acidosis. The elevated HCO3 indicates a partial compensation for the elevated CO2.

Chap 42. Selye's description of general adaptation syndrome includes a physical response to the body. The body system most affected by the stress response is:

Parts of the body particularly affected by stress are the gastrointestinal tract, the adrenal glands, and the lymphatic structures.

The nurse demonstrates caring behavior when he or she:

Pats the client's arm when approaching the bed

Which of the following models of health or illness defines health as a positive, dynamic state, not merely the absence of disease?

Pender's health promotion model

You are caring for a patient receiving calcium carbonate for the treatment of osteopenia. Which of the following serum laboratory results would you identify as an adverse effect related to this therapy? Sodium falling to 138 mEq/L Potassium rising to 4.1 mEq/L Magnesium rising to 2.9 mg/dl Phosphorus falling to 2.1 mg/dl

Phosphorus falling 2.1 mg/dl Calcium has an inverse relationship with phosphorus in the body. When phosphorus levels fall, calcium rises, and vice versa. Since hypercalcemia rarely occurs as a result of calcium intake, the patient's phosphorus falling to 2.1 mg/dl (normal 2.4-4.4 mg/dl), may be a result of the phosphate-binding effect of calcium carbonate.

What immediate action should the nurse take when a chest tube becomes disconnected from a bottle or suction apparatus? What should the nurse do if a chest tube is accidentally removed from the client?

Place the end of the tube in a sterile water container at a 2 cm level. Apply an occlusive dressing, and notify the health care provider stat.

The Health Insurance Portability and Accountability Act (HIPAA) of 1996 provides clients with basic rights pertaining to their medical records. Which of the following is a violation of HIPAA provisions?

Posting daily nursing care information along with the medical condition of clients on a message board in the client's room

Bananas, Orange juice, cantaloupe, strawberries, avocados, spinach, and fish all contain high __________?

Potassium

A nurse teaches the importance of folic acid intake to a group of pregnant women. This is considered which level of preventive care?

Primary prevention

Sodium - food sources

Processed foods, white & whole wheat bread, bacon, butter, canned foods, cheese, hot dogs, ketchuip, lunch meat, milk, mustard, snack foods, soy sauce, table salt,

Chronic Renal Failure (CRF) End Stage Renal Disease (ESRD) Pathophysiology Precipitating Factors Assessment Nursing Intervention

Progressive, irreversible damage to the nephrons and glomeruli, resulting in uremia. Causes of chronic renal failure are multitudinous. As renal function diminishes dialysis becomes necessary. Transplantation is an alternative to dialysis for some clients. Increased blood pressure, edema, pulmonary edema, neurologic impairment, decreasing urinary function, hematuria, proteinuria, cloudy urine, oliguric (100 to 400mL/day), anuric (<100 mL/day), jaundice, GI upset, metallic tast in mouth, ammonia breath, lab info: azotemia, increased creatinine and BUN, decreased calcium, elevated phosphorus and magnesium. Monitor serum electrolyte levels, weigh daily, monitor strict I/O, check for jugular distention and other signs of fluid overload, monitor for edema and pulmonary edema, provide low protein, low sodium, low potassium, low phosphate diet, administer aluminum hydroxide anatacids to bind phosphates because client is unable to excrete phosphates, encourage client's protein intake to be of high biologic value (eggs, milk, meat), because the client is on a low protein diet, alternate periods of rest with periods of activity, encourage strict adherence to medication regimen; teach client to obtain health care provider's permission before taking any over the counter medications

The examination for the registered nurse (RN) licensure is exactly the same in every state in the United States. This examination

Provides a minimal standard of knowledge for practice

Hyponatremia + Hypovolemic - assessment findings

Pulse - thready, weak rapid pulse rate BP - hypotension Neck veins - flat CVP - normal to low

Hypokalemia - assesment findings

Pulse - thready, weak(including peripheral), irregular Orthostatic hypotension Resp - dimished breath sounds,shallow, ineffective Neuro - anxiety, lethagy, confusion, coma, muscle weakness, paresthesia, deep tendon hyporeflexia GI: constipation, abd. distention, paralytic ileus, nausea, vomiting, decreased motility, hypoative to absent bowel sound

Chap 34. A primary care provider is going to perform a thoracentesis. The nurse's role will include which of the following?

Puncture site is usually on the posterior chest. Leaning forward separates the ribs, allowing for exposure of the site. The site would not be exposed when supine in the Trendelenburg position. Vital sign changes and pain are not commonly associated with this procedure. Administering a narcotic analgesic is normally not indicated with this procedure.

To validate the suspicion that a married male client has sleep apnea, the nurse first:

Questions the spouse if she is awakened by her husband's snoring. Asking the spouse would be a starting place to determine if a client has sleep apnea. This may lead to determining whether more tests are needed. The client would not know if he experiences sleep apnea. CPAP is a treatment for sleep apnea. Although this is a diagnostic tool, the first thing the nurse would do is question the spouse.

The client reports vivid dreaming to the nurse. Through understanding of the sleep cycle, the nurse recognizes that vivid dreaming occurs in which sleep phase?

REM sleep The dreams of REM sleep are vivid and elaborate. The other answers are incorrect.

Spider bites and bleeding treatment

RICE Rest Ice Compression Elevate extremity

Where is the pain for appendicitus?

RLQ w/rebound tenderness

Emphysema Pathophysiology Precipitating Factors Assessment Nursing Intervention

Reduced gas exchange surface area, increased air trapping, decreased capillary network, increased work, increased O2 consumption Cigarette smoking, environmental and or occupational exposure, genetic Pink puffers, barrel chest, pursed lip breathing, distant, quiet breath sounds, wheezes, pulmonary blebs on radiograph Monitor S/SX of fluid overload, maintain PaO2 between 55 and 60, Baseline ABGs, Teach purse lip breathing and diaphragmatic breathing, teach tripod position

EKG T Wave

Represents repolarization of the ventricular muscle Follows the QRS complex Usually is slightly rounded, without peaking or notching.

EKG PR Interval

Represents time required for the impulse to travel from the atria (SA node) through the atrioventricular (AV node) to the Purkinje fibers in the ventricles Represents AV nodal function Normal 0.12-0.20 seconds

Chap 30. Which of the following indicates a normal finding on auscultation of the lungs?

Resonance is a normal sound over the lung. Tympany would be heard over the stomach (air filled); hyperresonance is never a normal finding; and dullness would be heard below (not above) the 10th intercostal space.

What dz are droplet precautions?

SPIDERMAN S-sepsis, scarlet fever, strep pharyngitis P-pertussis, parvovirus B19, pne I- influenza D-diptheria E- epiglottis R-rubella M- mumps, meningits, An-adenovirus

The nurse in a diabetic clinic conducts monthly seminars for diabetic clients. During these seminars, the importance of taking insulin as directed to prevent diabetic complications is emphasized. This is considered which level of preventive care?

Secondary prevention

Chap 34. The nurse needs to collect a sputum specimen to identify the presence of TB. Which of the following is indicated for this type of specimen?

Sending the specimen immediately to the lab, offering mouth care before and after collection of the sputum specimen, and collecting a specimen for 3 consecutive days are all indicated. A sputum specimen is often collected in the morning. "Spit" is usually saliva-the client needs to cough up or expectorate mucus or sputum.

Chap 38. Clients admitted into the emergency department may experience behavior changes due to:

Sensory overload generally occurs when a person is unable to process or manage the amount of time or intensity or sensory stimuli. Clients may be experiencing more quantity or quality of internal stimuli such as pain, syspnea, or anxiety as well as external stimuli such as noise, intrusive diagnostic studies, and contact with many strangers. Sensoristasis is time of optimum arousal, not too much or too little. Sensory reception is the process of receiving internal and external data.. Stereognosis is the awareness of an object's size, shape, and texture.

Chap 38. Which client is most likely to experience sensory deprivation?

Sensory stimulation comes from our senses, environment, and presence of meaningful data. The deaf client with +4 edema who lives in an upstairs apartment does not have easy access to sensory stimulation and has limited potential for socialization. Although the 93-year old client has no sight and is unable to get out of bed, she is still capable and likely to receive sensory stimulation. She may converse with staff and other residents, feel the touch of bathing, and taste a variety of foods. There is a potential for sensory deprivation related to abandonment and the presence of anomalies. Since the child is being cared for in a special needs foster home, and attends preschool, one can reasonably assume that the child receives some stimulation. Premature infants in neonatal intensive care units often suffer from sensory overload.

In caring for a patient admitted with poorly controlled hypertension, the nurse would understand that which of the following laboratory test results would indicate the presence of target organ damage secondary to the primary diagnosis? BUN of 15 mg/dl Serum uric acid of 3.8 mg/dl Serum creatinine of 2.6 mg/dl Serum potassium of 3.5 mEq/L

Serum Creatinine of 2.6 mg/dl The normal serum creatinine level is 0.6-1.3 mg/dl. This elevated level indicates target organ damage to the kidneys.

Chap 42. A bystander at an automobile crash is excited and alarmed. The bystander also feels nauseated and dizzy, has difficulty focusing, and has an elevated pulse. What level of anxiety is the bystander feeling?

Severe anxiety symptoms include nausea and dizziness, difficulty focusing, and elevated pulse. Mild anxiety symptoms include increased arousal, few if any gastric symptoms, and minor if any respiratory or circulatory changes. Moderate anxiety symptoms include a narrowed focus of attention, selective inattentiveness, slightly increased heart and respiratory rate, and "butterflies in the stomach." Panic symptoms include agitation, unpredictable responses, distorted perception, dyspnea, palpitations, and feelings of impending doom.

Chap 49. After having a transverse colostomy constructed for colon cancer, discharge planning for home care would include teaching about the ostomy appliance. Information appropriate for this intervention would include which of the following?

Signs and symptoms for monitoring infection at the ostomy site are a priority evaluation for clients with new ostomies. The remaining actions are not appropriate. There are supplies available for clients to help control odor that may be incurred because of the ostomy. Although ostomy supplies require a prescription, they can be ordered from any medical supplier. Dependent on the location and trainability of the ostomy, appliances are almost always worn throughout the day and when traveling.

Chap 34. A client is admitted with gastrointestinal bleeding. One of the earliest and most important blood tests completed will be:

Since bleeding is the priority concern, lab values that provide information about the amount of blood loss will be the ones with the most value. Complete Blood Count is the important test to be completed. The electrolyte panel, arterial blood gases and the liver panel do not provide information related to the amount of blood loss.

Answer: A Lying in the supine position with the ipsilateral arm behind the head helps the breast tissue to flatten evenly against the chest wall. The other options do not allow the tissue to spread on the chest wall.

So that breast tissue will be spread evenly over the chest wall during an examination, the nurse asks the client to lie supine with: A) The ipsilateral arm behind the head B) Hands clasped just above the umbilicus C) Both arms overhead with palms upward D) The dominant arm straight alongside the body

You receive a physician's order to change a patient's IV from D5½ NS with 40 mEq KCl/L to D5NS with 20 mEq KCl/L. Which of the following serum laboratory values, documented on this same patient, best supports the rationale for this IV order change? Sodium 136 mEq/L, potassium 4.5 mEq/L Sodium 145 mEq/L, potassium 4.8 mEq/L Sodium 135 mEq/L, potassium 3.6 mEq/L Sodium 144 mEq/L, potassium 3.7 mEq/L

Sodium 136 mEq/L, potassium 4.5 mEq/L The normal range for serum sodium is 135 to 145 mEq/L, whereas the normal range for potassium is 3.5 to 5.0 mEq/L. The change in the IV order decreases the amount of potassium and increases the amount of sodium. Therefore for this order to be appropriate, the potassium level must be near the high end and the sodium level near the low end of their respective ranges.

To enhance effectiveness in teaching the older adult, the nurse should:

Speak in clear, low-pitched tones

Nurses are bound by a variety of laws. Which of the following descriptions of types of law is correct?

Statutory law is created by elected legislatures, such as the state legislature that defines the Nurse Practice Act (NPA).

Chap 38. Stereognosis is:

Stereognosis is the ability to perceive and understand an object through touch by its size, shape, and texture. Sensory reception is the process of receiving stimuli or data. Sensory perception involves the conscious organization and translation of the data or stimuli into meaningful information. Sensoristasis is the term used to describe when a person is in optimal arousal.

Chap 30. A nurse asks a client to close her eyes, and then places a paper clip in the client's palm. The client correctly identifies the object. What test did the nurse perform?

Stereognosis is the act of recognizing objects by touching and manipulating them. Extinction is failure to perceive touch on one side of the body when both sides are touched simultaneously. One- and two-point discrimination entail the ability to sense if one or two areas of the skin, respectively, are being stimulated by pressure. Paresthesia is an abnormal sensation, such as burning or pain.

Chap 34. The nurse needs to obtain a throat culture from a child client. Which of following techniques is NOT correct?

Sterile gloves are not necessary for obtaining a throat culture and is considered an unnecessary expense. To obtain a throat culture specimen, the nurse puts on clean gloves, then inserts the swab into the oropharynx and runs the swab along the tonsils and areas on the pharynx that are reddened or contain exudate. The gag reflex, active in some clients, may be decreased by having the client sit upright if health permits, open the mouth, extend the tongue, and say "ah," and by taking the specimen quickly.

Chap 48. The nurse is counseling a young mother who complains of having stress incontinence continuing for three months after her pregnancy. It has been recommended that she practice pelvic muscle exercises to strengthen her bladder muscles. What action would the nurse recommend to this client in order to perform this activity correctly?

Stopping the flow of urination midstream focuses on the muscle used to control this activity. The remaining answers do not affect this muscle in the same manner.

What are the most important nursing interventions for clients with possible renal calculi?

Straining all urine is the most important intervention. Other interventions include accurate intake and output documentation and administering analgesics as needed.

Answer: C Substance abusers avoid health care for fear of judgmental attitudes by health care providers and concern about being turned in to the criminal authorities. Options 1, 2, and 4 are not primary concerns that result in avoidance of health care.

Substance abusers frequently avoid health care providers because of: A) Fear of the cost of health care B) Fear of institutions and people C) Fear of being turned in to the criminal authorities D) Fear of being without the recreational drug of choice

Stroke Brain Attack CVA

Sudden loss of brain function resulting from a disruption in the blood supply to a part of the brain; classified as thrombotic or hemorrhagic

postrenal ARF

Sudden obstruction of urine flow r/t enlarged prostate, kidney stones, bladder, tumor, or injury

true

T/F percutaneous nephrostomy should not be clamped or irrigated and if both kidneys are involved output should be recorded for both

true

T/F suprapubic prostatectomy nursing considerations include monitoring output for both suprapubic and urethral cathethers

List four common symptoms of pneumonia the nurse might note on physical examination

Tachypnea, fever with chills, productive cough, bronchial breath sounds

Hypomagnesium & Electrocardiographic Changes

Tall T waves Depressed ST segment

Hyperkalemia and Electrocardiographic Changes

Tall peaked T waves Flat P waves Widened QRS complex Prolonged PR interval

Chap 49. Which of the following guidelines should the nurse follow when administering enemas?

Tap water enemas should not be repeated because of the danger of circulatory overload when the water moves from the interstitial space to the circulatory system. Air instilled into the rectum can cause unnecessary distention. The client should be in the left lateral position to facilitate the flow of the solution into the sigmoid and descending colon which are on the left side. Enema solution should be administered slowly to avoid cramps and spasm.

Chap 43. When a nurse is taking care of a client who knows that death is imminent, the nurse must be aware that the client:

The Dying Person's Bill of Rights assists the client in maintaining dignity at the time of death.

Chap 43. A nurse is planning a seminar on the Dying Person's Bill of Rights. Which of the following statements is NOT part of the Dying Person's Bill of Rights?

The Dying Person's Bill of Rights includes: I have the right not to die alone. I have the right to express my feelings and emotions about my approaching death in my own way. I have the right to expect continuing medical and nursing attention even though cure goals must be changed to comfort goals. I have the right to be free from pain.

When a nurse stops to help in an emergency at the scene of an accident, if the injured party files suit and the insurance of the nurse's employing institution does not cover the nurse, the nurse would probably be covered by:

The Good Samaritan laws, which grant immunity from suit if there is no gross negligence

The licensure and practice of nursing is regulated by:

The Nurse Practice Act

To distinguish an ethical problem from other types of problems encountered, the nurse should be aware that an ethical problem is one for which:

The answer is not determined by logical deduction.

Answer: D Therapies such as tepid water or alcohol sponge baths should be avoided because they lead to shivering, which stimulates body heat. Antipyretics, not analgesics, are the medications that lower body temperature.

The client has an oral temperature of 39.2° C (102.6° F). What are the most appropriate nursing interventions? A) Provide an alcohol sponge bath and monitor laboratory results. B) Remove excess clothing, provide a tepid sponge bath, and administer an analgesic. C) Provide fluids and nutrition, keep the client's room warm, and administer an analgesic. D) Reduce external coverings and keep clothing and bed linens dry; administer antipyretics as ordered.

Chap 30. The nurse explains to the client that the client is in the hospital following a car accident. Several minutes later, the nurse asks the client, "Where are you now?" The nurse is assessing the client's:

The client is asked to identify where the client is, which assesses orientation to place.

Chap 42. A client comes into the clinic with vocal tremors and pitch changes. The client is also experiencing facial twitches, shakiness, and slightly elevated respiratory and heart rates. At the end of the assessment the client tells the nurse, "I feel like I have butterflies in my stomach." Which level of anxiety is this client experiencing?

The client is experiencing moderate anxiety as evidenced by voice tremors and pitch changes, facial twitches, shakiness, and slightly elevated respiratory and heart rates, and states, "I feel like I have butterflies in my stomach." Mild anxiety would be characterized by mild restlessness, sleeplessness, increased verbalization, feelings of increased arousal and alertness, and no changes in respiratory and heart rates. Severe anxiety is characterized by communication difficulties; increased motor activity; inability to relax, focus, and concentrate; ease of distractibility; tachycardia; and hyperventilation. Panic anxiety is characterized by increased motor activity, agitation, unpredictable responses, distorted or exaggerated perception, dyspnea, palpitations, choking, chest pain, and a feeling of impending doom.

Chap 48. The client with a neurogenic bladder:

The client with a neurogenic bladder does not perceive bladder fullness. The client with urgency has a sudden strong desire to void. The client with urinary frequency is voiding at frequent intervals, more than 4-6 times a day. The client with dysuria has painful or difficult voiding.

Chap 48. The client with the NANDA diagnosis of Stress Urinary Incontinence has:

The client with stress incontinence has sudden leakage of urine occurring with activities that increase abdominal pressure. The client with functional urinary incontinence has an inability to reach toilet in time to avoid unintentional loss of urine. The client with reflex urinary incontinence has an involuntary loss of urine at somewhat predictable intervals when a specific bladder volume is reached. The client with total urinary incontinence has continuous and unpredictable passage of urine.

Answer: C This is the description of an MCO. In a PPO, choice of care providers is limited to those listed in the group. Medicare is a federally funded national health insurance program. Private insurance is a traditional fee-for-service plan.

The client's health insurance changed, and instead of having a limited number of physicians from whom to choose, the client is voluntarily enrolled in a plan in which medical care is provided by a special group of caregivers. This arrangement is known as: A) Medicare B) Private insurance C) Managed care organization (MCO) D) Preferred provider organization (PPO)

Answer: B Rhonchi are loud, low-pitched, rumbling, coarse sounds heard most often during inspiration or expiration. Crackles are moist sounds heard during inspiration that are not cleared with coughing. Normal lungs produce no sounds such as that described. Wheezes are high-pitched continuous muscles sounds such as a squeak heard continuously during inspiration and expiration.

The client's respiratory assessment reveals loud, low-pitched, rumbling, coarse sounds heard during inspiration and expiration. The nurse interprets these sounds as: A) Normal B) Rhonchi C) Crackles D) Wheezes

Chap 49. A client has complained of diarrhea for the last 3 days. The client is passing mucus, but no stool. This indicates to the nurse that the part of the anatomy that is affected is:

The colon serves a protective function in that it secretes mucus. This mucus contains large amounts of bicarbonate ions. The mucus secretion is stimulated by excitation of parasympathetic nerves. During extreme stimulation—for example, as a result of emotions—large amounts of mucus are secreted with little or no feces.

Chap 52. Edema that forms in clients with kidney disease is due to:

The edema is due to low levels of plasma proteins that exist with this disease, altering the oncotic pressure that helps regulate fluid movement in the vascular space moving into interstitial area. Increased capillary hydrostatic pressure is the cause. Capillaries have increased permeability when edema formation is possible. Obstructed lymph flow impairs the movement of fluid from interstitial tissues back into the vascular compartment, resulting in edema.

Answer: D The RN licensure examination provides a minimum standard of knowledge for nurses. The examination cannot guarantee or ensure care for clients.

The examination for the registered nurse (RN) licensure is exactly the same in every state in the United States. This examination: A) Guarantees safe nursing care for all clients B) Ensures standard nursing care for all clients C) Ensures that honest and ethical care is provided D) Provides a minimal standard of knowledge for practice

Answer: C Option 3 defines the focus of community health nursing. Community health nursing focuses on the individual, family, and community. Educational requirements for community-based nurses are not as clearly defined as those for public health nurses. An advanced degree is not always required.

The focus of community health nursing differs from that of public health nursing because the nursing care: A) Is directed at the individual client only B) Is provided by nurses with a graduate degree in community health nursing C) Provides direct care to subpopulations who make up the community as a whole D) Is administered to a collection of individuals who have in common one or more personal or environmental characteristics

Chap 52. A nurse is planning a seminar on wellness care and promoting fluid and electrolyte balance. Which of the following statements is correct?

The following pertain to wellness care and promoting fluid and electrolyte balance: Consume six to eight glasses of water daily; avoid excess amounts of foods or fluids high in salt, sugar, and caffeine; limit alcohol intake because it has a diuretic effect; increase fluid intake before, during, and after strenuous exercise, particularly when the environmental temperature is high; and replace lost electrolytes from excessive perspiration as needed with commercial electrolyte solutions.

Chap 48. Urine production is a result of the process of which of the following areas of the urinary tract system?

The functional units of the kidneys, the nephrons, filter the blood and remove metabolic wastes.

Answer: A Secondary intervention includes disease prevention after a health issue has been identified. Primary intervention is prevention of a health problem that has not yet occurred in the community. Tertiary intervention occurs after a problem has occurred and aims at preventing long-term negative impacts or recurrences in a population.

The local health department received information from the Centers for Disease Control and Prevention that the flu was expected to be very contagious this season. The nurse is asked to set up flu vaccine clinics in local churches and senior citizen centers. This activity is an example of which level of prevention? A) Primary intervention B) Tertiary intervention C) Nursing intervention D) Secondary intervention

Chap 52. A client is admitted to the hospital for hypocalcemia. Nursing interventions relating to which system would have the highest priority?

The major clinical signs and symptoms of hypocalcemia are due to increased neuromuscular activity.

Chap 49. A primary care provider orders an examination of stool for signs of intestinal infection. What color stool would the nurse expect to see?

The normal color of stool is brown. An intestinal infection would result in green or orange stool. Red stool would indicate bleeding from the lower gastrointestinal tract (e.g., rectum); some foods, such as beets, may also cause red stool. Black or tarry stool indicates bleeding from the upper gastrointestinal tract (e.g., stomach, small intestine); a diet high in red meat and dark green vegetables (e.g., spinach) may also cause black, tarry school. White or clay-colored stool is due to absence of bile pigment (bile obstruction), or the result of a diagnostic study using barium.

Most litigation involving hospital care is related to situations in which:

The nurse abandons the clients when going to lunch.

Answer: D This common adage embodies an abstract idea, and explaining it indicates the client's ability to perform abstract reasoning. Judgment involves comparison and evaluation of facts and ideas to understand their relationships and to form appropriate conclusions. Knowledge is understanding or awareness of information gained through learning or experience. Association involves finding similarities between concepts.

The nurse asks the client to interpret the saying, "Don't count your chickens before they're hatched." The client's response provides information about the client's: A) Judgment B) Knowledge C) Association D) Abstract reasoning

Answer: D The movement of the head and shoulders is controlled by cranial nerve XI, the spinal accessory nerve. The facial nerve innervates the face. The hypoglossal nerve innervates portions of the tongue. The trigeminal nerve is a sensory and motor nerve enervating the side of the face and jaw.

The nurse asks the client to shrug the shoulders and turn the head side to side against the resistance of the examiner's hand. These actions allow the nurse to evaluate which cranial nerve? A) VII—Facial B) V—Trigeminal C) XII—Hypoglossal D) XI—Spinal accessory

2,3,5,6

The nurse assesses the following risk factors for coronary artery disease (CAD) in a male patient. Which factors are classified as genetic and physiological? (Select all that apply) 1. Sedentary lifestyle 2. Father died from CAD at 50 3. History of HTN 4. Eats diet high in sodium 5. Elevated cholesterol level 6. Age: 44

Answer: C A critical pathway is a multidisciplinary treatment plan with interventions prescribed within a structured framework. A discharge plan includes an assessment and anticipation of the client's needs. Medicare is a federal health insurance plan for those 65 years of age and older. Standard nursing care is the minimum care to be given to a client.

The nurse completes the standard orders on a client's first day postoperatively. The instrument that is used to coordinate the client's care is: A) A Medicare plan B) A discharge plan C) A critical pathway D) Standard nursing care

Answer: A The general survey focuses on general appearance and behavior, including gender and race, age, signs of distress, body type, posture, gait, hygiene and grooming, dress, affect, mood, and speech. The other actions are carried out in different parts of the assessment.

The nurse conducts a general survey of an adult client, which includes: A) Checking appearance and behavior B) Measuring vital signs C) Observing specific body systems D) Conducting a detailed health history

Answer: B The apical pulse gives the nurse the most information and accuracy when assessing irregular cardiac rhythm. The carotid or femoral pulses are usually used to assess a client in shock. The radial pulse is adequate for determining routine postoperative vital signs and for checking changes in orthostatic heart rate.

The nurse decides to take an apical pulse instead of a radial pulse. Which of the following client conditions influenced the nurse's decision? A) The client is in shock. B) The client has an arrhythmia. C) The client underwent surgery 18 hours earlier. D) The client showed a response to orthostatic changes.

Chap 38. Sensory perceptual deficts can be the etiology for other nursing diagnoses. An example of such a diagnosis is:

The nurse determines what effects the sensory defict will have on the client. Swallowing is not a sensory/perceptual deficit. Fluid overload is related to excess intake relative to output, or organ failure such as heart failure, not sensory deficit. Sensory perceptual deficts would not relate to a person's overeating.

Answer: D All questionable blood pressure readings should be rechecked. Ensuring the client's safety is a necessary safeguard, because low blood pressure is generally accompanied by weakness. For the majority of people, low blood pressure (systolic pressure of 90 mm Hg or below) is an abnormal finding and should be reported. Giving a client orange juice may raise blood glucose level but is not recommended to elevate blood pressure. Ambulating a client with hypotension would not be following safety precautions.

The nurse finds that the systolic blood pressure of an adult client is 88 mm Hg. What are the appropriate nursing interventions? A) Check other vital signs. B) Recheck the blood pressure and give the client orange juice. C) Recheck the blood pressure after ambulating the client safely. D) Recheck the blood pressure, make sure the client is safe, and report the findings.

Answer: D Evidence-based practice draws on both research and clinical experience. Competencies are evidence that skills have been demonstrated. Critical thinking is the questioning thought process that nurses need to use in practice. Primary care is health care provided in the community by one caregiver who takes responsibility for managing a client's care.

The nurse found that using tympanic thermometers was quick, easy, and yielded temperatures as reliable as those obtained using oral thermometers. This finding represents: A) Primary care B) Critical thinking C) Competency testing D) Evidence-based practice

Answer: B Critical thinking involves analyzing the data, learning, and problem solving to come up with a course of action. Tradition limits the ability to learn new ways and overlooks what research has to offer. The advice of experienced practitioners may limit research because experience may mean doing things the same way they have been done for years. Using personal opinion overlooks the objective data that are available.

The nurse involved in scientific research effectively analyzes the information collected and determines a course of nursing action by: A) Depending on tradition B) Using critical thinking C) Seeking the advice of experienced practitioners D) Relying on personal perspective or opinion

Answer: C An advocate helps speak for the client, communicating the client's concerns and wishes to family and other caregivers. A caregiver assists in meeting all health care needs of the client, including taking measures to restore emotional, spiritual, and social well-being. A manager coordinates all the activities of the members of the nursing staff in delivering nursing care and has personnel, policy, and budgetary responsibilities for a specific nursing unit or agency. An educator explains concepts and facts about health, demonstrates procedures such as self-care activities, reinforces learning or client behavior, and evaluates the client's progress in learning.

The nurse is caring for a client with end-stage lung disease. The client wants to go home on oxygen therapy and be comfortable. The family wants the client to undergo a new surgical procedure. The nurse explains the risk and benefits of the surgery to the family and discusses the client's wishes with the family. The nurse is acting as the client's: A) Manager B) Educator C) Advocate D) Caregiver

2

The nurse is conducting a home visit with an older adult couple. She assesses that the lighting in the home is poor and that there are several throw rugs throughout the home and a low footstool in the living room. She suggests that the rugs and the footstool be removed and improving the lighting in the home. The nurse is addressing which need according to Maslow? 1. Physiological 2. Safety and security 3. Love and belonging 4. Self-actualization

Answer: C Health promotion includes dietary counseling. Blood glucose monitoring at the pharmacy is an example of illness prevention. Restorative care is care of a client who, for instance, is recovering from complications of diabetes. Any diagnostic procedure or tests completed in the hospital would be examples of such care.

The nurse is giving discharge instructions to a client with newly diagnosed diabetes. The nurse discusses with the client what the dietary intake should be. This is an example of which health care service? A) Tertiary care B) Restorative care C) Health promotion D) Illness prevention

Chap 34. While preparing the client for a colonoscopy, the nurse's responsibilities include:

The nurse is responsible for instructing the client about the bowel preparation prior to the test. Explaining the risks and benefits of the exam, instructing the client about medication that will be used to sedate the client, and explaining the results of the exam are the physician's responsibility.

Answer: B Painless, pea-sized nodules should be checked by a health care provider. Testicular self-examination should be performed monthly and should be done after a bath or shower. The testes feel smooth, rubbery, and free of nodules.

The nurse is teaching a client how to perform a testicular self-examination. The nurse tells the client which of the following? A) "The testes are normally round, moveable, and have a lumpy consistency." B) "Contact your health care provider if you feel a painless pea-sized nodule." C) "The best time to do a testicular self-examination is before your bath or shower." D) "Perform a testicular self-examination weekly to detect signs of testicular cancer."

Answer: D Postponing this assessment is definitely a judgment call by the nurse. Postponing is appropriate unless the assessment of respiration is a critical aspect of the test and the client is leaving for the test immediately. Otherwise, it is probably not necessary to invade the client's privacy and disrupt the visitation. Agency policy will dictate whether the respiration rate should be documented as "deferred" or whether documentation can wait until the rate is obtained. Respirations should be counted when the client is "at rest"; therefore, counting respirations during the visitation would not be appropriate. Waiting at the bedside until the visitor leaves is an invasion of privacy for the client and a waste of the nurse's time.

The nurse is to measure vital signs as part of the preparation for a test. The client is talking with a visiting pastor. How should the nurse handle measuring the rate of respiration? A) Count respirations during the time the client is not talking to the visitor. B) Wait at the client's bedside until the visit is over and then count respirations. C) Tell the client it is very important to end the conversation so the nurse can count respirations. D) Document the respiration rate as "deferred" and measure the rate later, since the talking client is obviously not in respiratory distress.

Answer: D Cheyne-Stokes respiration is an irregular respiratory rate and depth with alternating periods of apnea and hyperventilation; it begins with slow breaths and climaxes in apnea before respiration resumes.

The nurse observes that a client's breathing pattern represents Cheyne-Stokes respiration. Which statement best describes the Cheyne-Stokes pattern? A) Respirations cease for several seconds. B) Respirations are abnormally shallow for two to three breaths followed by irregular periods of apnea. C) Respirations are labored, with an increase in depth and rate (more than 20 breaths per minute); the condition occurs normally during exercise. D) Respiration rate and depth are irregular, with alternating periods of apnea and hyperventilation; the cycle begins with slow breaths and climaxes in apnea.

Answer: C The conduct of research must meet ethical standards in which the rights of human subjects are protected. The research participants must be told about the study's purpose and procedure, and their roles in the study. The researcher is always legally responsible for his or her actions. Control of variables is related to the study design, not to informed consent. Confidentiality is part of the ethical nature of research but is not the focus of informed consent.

The nurse researcher obtains informed consent from participants in a study primarily to: A) Release the researcher from legal liability. B) Control variables that might affect the study. C) Ensure that the study subjects understand their roles in the study. D) Maintain the confidentiality of the researcher and the participants.

Answer: B Publication of research results provides other nurses with the scientific background of the study before they apply its findings in practice. Study subjects and setting should be similar to duplicate a study. Nurses should not change from accepted to unproven ways of providing care without careful research and collaboration with colleagues. Experimenting with new nursing measures is inappropriate and may place a client at risk.

The nurse researcher who gains new knowledge regarding a procedure can most effectively share the information with the nursing profession by: A) Duplicating the study using different clients in different settings B) Communicating the research findings in a professional journal C) Recruiting clients who are willing to demonstrate the new technique D) Asking individual nurses to report their experiences related to the new procedure

Chap 38. The nurse is providing care to an unconscious client. Which of the following actions by the nurse is correct?

The nurse should also provide mouth care, perform range-of-motion exercises, and provide aromatic stimuli. Nose care is not an appropriate action. Too much environmental stimuli can be very distressing to a client. The nurse should inform the client beforehand of the care to be provided, not during the care.

Answer: B Sitting upright provides full expansion of the lungs and provides better visualization of the symmetry of upper body parts. The lateral recumbent position aids in detecting heart murmurs. The dorsal recumbent position is used for abdominal assessment because it allows relaxation of abdominal muscles. The supine position provides easy access to pulse sites.

The nurse should assist the client to a sitting position to provide the best position to examine which of the following? A) Heart B) Lungs C) Abdomen D) Pulse sites

Chap 30. In what sites should the nurse auscultate the heart?

The nurse should auscultate the heart in all four anatomic sites: aortic, pulmonic, tricuspid, and apical (mitral). Lateral, anterior, and posterior do not refer to auscultation sites. Carotid, aortic, jugular and coronary refer to arteries and veins near the heart. Apical and mitral refer to the same site. Sternal and costal refer to bony landmarks.

Answer: D Aortic, pulmonic, tricuspid, mitral areas are the sites for auscultation assessment of cardiac function. Auscultation of the carotid arteries is not the same as auscultation of the heart itself. The sternal region is not an appropriate site. There are no costal sites where heart sounds can be heard well. Anterior, posterior, and lateral are too vague.

The nurse should use which anatomical sites for the auscultatory assessment of cardiac function? A) Inner costal, outer costal, and sternal B) Aortic, carotid, coronary, and jugular C) Apical, lateral, anterior, and posterior D) Aortic, pulmonic, tricuspid, and mitral

Answer: A The "Do" step consists of selecting an intervention based on a data review and implementing the change, plus studying the results of the change. The "Plan" step includes reviewing the available data to understand existing practice conditions or problems to identify the need for change. The results of the change are evaluated in the "Study" step. The "Act" step is the incorporation of the findings into current practice.

The nurses on a medical unit have seen an increase in the number of pressure ulcers developing in their clients. The nurses decide to initiate a quality improvement project using the PDSA (plan, do, study, act) model. Which of the following is an example of the "Do" step of that model? A) Implement a new skin care protocol on all medical units. B) Review the data collected on clients cared for using the new protocol. C) Review the quality improvement reports on the six clients who developed ulcers over the previous 3 months. D) Based on findings from clients who developed ulcers, implement an evidence-based skin care protocol.

Answer: B Identified linkages of a nursing paradigm are the person, health, environment/situation, and nursing itself. Concepts, definitions, relationship, and assumptions are components of a theory. The individuals, groups, situations, and interests specific to nursing are potential subjects for middle-range theories. Description, explanation, prediction, and prescription of an interrelationship of nursing are purposes of nursing theory.

The nursing paradigm identifies four linkages of interest to the nursing profession. These four linkages are: A) Concepts, definitions, relationships, and assumptions B) The person, health, environment/situation, and nursing C) The individual, groups, situations, and interests specific to nursing D) Description, explanation, prediction, and prescription of an interrelationship of nursing

Answer: B The question describes the nursing theory developed by Fay Abdellah and others. Rogers' theory considered the individual as an energy field existing within the universe. Henderson's theory defines nursing as "assisting the individual, sick, or well, in the performance of those activities that will contribute to health, recovery, or a peaceful death." Nightingale viewed nursing as providing fresh air, light, warmth, cleanliness, quiet, and adequate nutrition.

The nursing theory that emphasizes the delivery of nursing care for the whole person to meet the physical, emotional, intellectual, social, and spiritual needs of the client and family is: A) Rogers' theory B) Abdellah's theory C) Henderson's theory D) Nightingale's theory

4

The patient states that she joined a fitness club and attends the aerobics class three nights a week. The patient is in what stage of behavioral change? 1. Precontemplation 2. Contemplation 3. Preparation 4. Action

Chap 48. During assessment of the client with urinary incontinence, the nurse is most likely to assess which of the following?

The perineum may become irritated by the frequent contact with urine. Normal fluid intake is at least 1,500 mL/day and clients often decrease their intake to try to minimize urine leakage. Urinary tract infections can contribute to incontinence. Antihistamines can cause urinary retention rather than incontinence.

Even though the nurse may obtain the client's signature on a form, obtaining informed consent is the responsibility of:

The physician

Answer: B Mary Mahoney, the first African American professional nurse, worked to bring respect to individuals regardless of race, color, background, or religion. Tubman assisted slaves to freedom during the Civil War. Hampton founded the Nurses Associated Alumnae of the United States and Canada, which later became the American Nurses Association (ANA). Nutting was instrumental in the affiliation of nursing education with universities.

The professional nurse responsible for increasing respect for the individual and awareness of cultural diversity was: A) Harriet Tubman B) Mary Mahoney C) Isabel Hampton D) Mary Adelaide Nutting

Chap 52. Which individual would least likely have a disturbance in fluid volume, electrolyte, or acid-base balance?

The proportion of body water decreases with aging. Tissue trauma, such as burns, causes fluids and electrolytes to be lost from the damaged cells, and the breakdown in the continuity of the tissue. In type I pressure ulcer, the skin remains intact, and any shifting of fluids is due to the inflammatory process and internally maintained within the body. Vomiting and diarrhea can cause significant fluid losses. Age, sex, and body fat affect total body water. Infants have the highest proportion of water; it accounts for 70 to 80% of their body weight. Decreased blood flow to the kidneys as caused by impaired cardiac function stimulates the renin-angiotensin-aldosterone system, causing sodium and water retention. Clients who are confused or unable to communicate are at risk for inadequate fluid intake. Age does not play a significant factor here.

Answer: D The utilization review committee reviews admissions, diagnostic procedures, and treatments ordered by physicians. Review of the quality, quantity, and cost of care is more similar to the functions of a professional standards review organization. Review of reimbursement fees and appropriation of funds involves review of diagnosis-related groups. Reviewing the utilization of the payment mechanism is similar to capitation.

The purpose of a utilization review committee is to: A) Review quality, quantity, and cost of care B) Review the utilization of the payment mechanism C) Review reimbursement fees and appropriation of funds D) Review admissions, diagnostic tests, and treatments ordered by physicians

Chap 52. When a client is on a fluid restriction, it means the client cannot:

The restriction of fluids can be difficult for some clients, particularly if they are experiencing thirst. Provide frequent mouth care and rinses to reduce the thirst sensation. Periodically offer the client ice chips as an alternative to water because ice chips when melted are approximately half of the frozen volume.

Chap 42. A client's husband died suddenly this week. She has two young children and is staying strong for them emotionally. It has been 4 days and the funeral is over and family has gone. Which of the following stages of the general adaptation syndrome is the client in?

The stage of resistance is when the body's adaptation takes place. In other words, the body attempts to cope with the stressor and to limit the stressor to the smallest area of the body that can deal with it.

Chap 49. Which of the following statements provides evidence that an older adult who is prone to constipation is in need of further teaching?

The standard of practice in assisting elders to maintain normal function of the gastrointestinal tract is regular ingestion of a well-balanced diet, adequate fluid intake, and regular exercise. If the bowel pattern is not regular with these activities, this abnormality should be reported. Stimulant laxatives can be very irritating and are not the preferred treatment for occasional constipation in older adults. In addition, a normal stool pattern for an elder may not be daily elimination.

Chap 43. The shift changed while the nursing staff was waiting for the adult children of a deceased client to arrive. The oncoming nurse has never met the family. Which of the following greetings is most appropriate?

The statement, "I'm very sorry for your loss" acknowledges the family's grief simply. Avoid statements that may be interpreted as overly impersonal, false support, or harsh. Other responses may also be correct as the meeting progresses.

Chap 48. The process of urination is stimulated by which of the following?

The stretch receptors transmit impulses to the spinal cord, specifically to the voiding reflex center located at the level of the 2nd and 4th sacral vertebrae, causing the internal sphincter to relax and stimulating the urge to void.

Answer: C Prescriptive theory addresses nursing interventions and predicts the consequence of a specific nursing intervention. Middle-range theories are limited in scope, less abstract than grand theories, address specific phenomena or concepts, and reflect practice. Descriptive theories describe phenomena, speculate as to why the phenomena occur, and describe the consequences of phenomena. Grand theories are broad and complex.

The type of theory that tests the validity and predictability of nursing interventions is: A) A grand theory B) A descriptive theory C) A prescriptive theory D) A middle-range theory

Answer: A Grand theories are described as broad and complex. Middle-range theories are limited in scope, less abstract, address specific phenomena or concepts, and reflect practice. Descriptive theories describe phenomena, speculate as to why the phenomena occur, and describe the consequences of phenomena. Prescriptive theories address nursing interventions and predict the consequence of a specific intervention.

Theories that are broad and complex are: A) Grand theories B) Descriptive theories C) Middle-range theories D) Prescriptive theories

Chap 49. The nurse is presenting information at the community health fair about normal defecation patterns across the lifespan. Which of the following factors would NOT be part of the discussion?

There is no relationship noted between gender and defecation pattern. Diet, fluids, and medications all can affect the amount, consistency, or pattern of defecation.

What is true about the theories of aging?

There is no single theory that explains aging.

Chap 48. The nurse is requested to perform teaching to a client in the emergency department related to the diagnosis of a urinary tract infection. An intervention to be followed by the client includes which of the following?

Tight-fitting clothing creates irritation to the urethra and prevents ventilation of the perineal area. It is recommended that eight glasses of water be drunk to flush out the urinary system. Avoid harsh soaps, bubble bath, powders, and sprays in the perineal area, because they can have an irritating effect on the urethra, encouraging inflammation and a bacterial infection. Practice frequent voiding (q 2—3 hours) to flush bacteria out of the urethra and prevent organisms from ascending into the bladder.

Answer: D Superficial lymph nodes are gently palpated using the pads of the index and middle fingers. The lymph nodes are small, and any other method would not be helpful.

To assess a client's superficial lymph nodes, the nurse: A) Deeply palpates using the entire hand B) Deeply palpates using a bimanual technique C) Lightly palpates using a bimanual technique D) Gently palpates using the pads of the index and middle fingers

Answer: D The dorsalis pedis pulse is felt on the top of the foot in line with the groove between the extensor tendons of the great toe and the first toe. The popliteal pulse is felt behind the knee. The posterior tibial pulse is felt in the groove behind the medial malleolus. Typically pulses are not palpated behind the lateral malleolus.

To assess the client's dorsalis pedis pulse, the nurse palpates: A) Behind the knee B) Over the lateral malleolus C) In the groove behind the medial malleolus D) Lateral to the extensor tendon of the great toe

Answer: A The sounds of lung fields on one side of the body are compared with the sounds of the same fields on the opposite side of the body. The other answers would provide incorrect comparisons.

To auscultate the client's lung fields, the nurse uses a systematic pattern comparing: A) Side to side B) Top to bottom C) Anterior to posterior D) Interspace to interspace

Chap 34. In collecting a sputum specimen, which of the following guidelines must be followed?

To collect a sputum specimen, the nurse follows these steps: Offer mouth care so that the specimen will not be contaminated with microorganisms from the mouth. Ask the client to breathe deeply and then cough up 1 to 2 tablespoons, or 15 to 30mL (4 to 8 fluid drams), of sputum. Wear gloves and personal protective equipment to avoid direct contact with the sputum. Follow special precautions if tuberculosis is suspected, obtaining the specimen in a room equipped with a special airflow system or ultraviolet light, or outdoors. If these options are not available, wear a mask capable of filtering droplet nuclei. Ask the client to expectorate (spit out) the sputum into the specimen container. Make sure the sputum does not contact the outside of the container (Figure 34-8). If the outside of the container does become contaminated, wash it with a disinfectant. Following sputum collection, offer mouthwash to remove any unpleasant taste. Label and transport the specimen to the laboratory. Ensure that the specimen label and the laboratory requisition contain the correct information. Arrange for the specimen to be sent to the laboratory immediately or refrigerated. Bacterial cultures must be started immediately before any contaminating organisms can grow, multiply, and produce false results. Document the collection of the sputum specimen on the client's chart. Include the amount, color, consistency (thick, tenacious, watery), presence of hemoptysis (blood in the sputum), odor of the sputum, any measures needed to obtain the specimen (e.g., postural drainage), and any discomfort experienced by the client.

Answer: C The nurse accurately assesses temperature by palpating the skin with the dorsum or back of the hand, because this area of the hand is more sensitive to temperature than is the base of the hands, the fingertips, or the palmar surface.

To correctly palpate the client's skin for temperature, the nurse uses which of the following? A) Base of the hands B) Fingertips of the hands C) Dorsal surface of the hands D) Palmar surface of the hands

Chap 34. A nurse is instructing a female client on the procedure to collect a clean-catch specimen. What specifically needs to be emphasized to decrease the risk of contamination?

To decrease the risk of contamination, the nurse explains to the client to clean the perineal area from front to back and use an antiseptic towelette to wipe the urinary meatus. Cleaning with just water and soap reduces the risk of contamination, but the urinary meatus also needs to be cleansed. Washing the perineal area from back to front is cleaning from the area of most contamination to the least. Using a wet face cloth does not decrease the risk of contamination.

Chap 42. A nurse is planning a seminar on minimizing stress and anxiety. Which of the following statements is NOT correct?

To minimize stress and anxiety, the nurse should communicate in short, clear sentences; provide an atmosphere of warmth and trust; convey a sense of caring and empathy; listen attentively; try to understand the client's perspective on the situation; and control the environment to minimize additional stressors, such as by reducing noise, limiting the number of persons in the room, and providing care by the same nurse as much as possible.

Answer: C Interdisciplinary theories provide a systematic view of a phenomenon. Developmental theories, health and wellness theories, and systems theories are examples of other types of theories.

To practice in today's health care environment, nurses need a strong scientific knowledge base in nursing and other disciplines, such as the physical, social, and behavioral sciences. This relates to which of the following? A) Systems theories B) Developmental theories C) Interdisciplinary theories D) Health and wellness model

Chap 34. A medical technician has just finished drawing blood for arterial gases. What is the most appropriate nursing action after the removal of the aspirating needle?

To prevent hemorrhaging, the nurse needs to apply pressure to the puncture site for 5-10 minutes after the removal of the needle. This is important because of the relatively great pressure of the blood in these arteries. It is not necessary to be monitoring for vital signs or intake and output after the collection of arterial blood gases. There is no incision site. Ice packs may decrease bleeding, but the application of pressure on the injection site is most appropriate.

Answer: A Although early morning temperatures are routinely low, the best practice is for the nurse to check the client's previous temperatures. Clients may routinely have a low temperature. Depending on the client's temperature history, the nurse may retake the temperature with another thermometer to check for a malfunction. If everything seems satisfactory, the nurse should chart the temperature and check the client for signs of hypothermia.

Using an oral electronic thermometer, the nurse checks the early morning temperature of a client. The client's temperature is 36.1° C (97° F). The client's remaining vital signs are in the normally acceptable range. What should the nurse do next? A) Check the client's temperature history. B) Document the results; temperature is normal. C) Recheck the temperature every 15 minutes until it is normal. D) Get another thermometer; the temperature is obviously an error.

Answer: D A normal tympanic membrane is translucent, shiny, and pearly gray. Dark yellow and sticky describes normal moist cerumen (earwax) in front of the tympanic membrane. A white color indicates pus behind the membrane. A pink or red bulging membrane is an indication of inflammation.

Using an otoscope, the nurse can inspect the tympanic membrane. A normal tympanic membrane appears: A) Round and white B) Pink and bulging C) Dark yellow and sticky D) Translucent, shiny, and pearly gray

PT (prothrombin time) and INR (international normalized ratio)

Usually to monitor Coumadin (warfarin sodium) PT 9.6 - 11.8 seconds (male adult) 9.5 - 11.3 seconds (female adult) 1.5 - 2 times normal >30 secs =risk for bleeding INR 2 - 3 for standard, 3 - 4.5 high dose warfarin therapy Direct pressure 3-5 mins

aPTT (activated partial prothromboplastin time)

Usually to monitor Heparin Therapy 20 - 36 secs s/b between 1.5 - 2.5 times normal >90 seconds = risk of bleeding direct pressure 3-5 mins doesn't screen for factor VII and XIII

EKG QRS Complex

Ventricular Systole Represents depolarization of the ventricular muscle Normally follows P wave Is measured from beginning of the QRS to end of QRS. Normal < 0.11 seconds

Chap 30. Which of the following is an expected finding during assessment of the older adult?

Visual acuity often lessens with age. Facial hair is likely to become coarser, not finer. The sense of smell becomes less, rather than more, acute. The respiratory rate and rhythm is regular at rest. However, both may change quickly with activity and be slow to return to the resting level.

When caring for a patient with a biliary obstruction, the nurse will anticipate administering which of the following vitamin supplements (select all that apply)? Vitamin A Vitamin D Vitamin E Vitamin K Vitamin B

Vitamin A, D, E, K Biliary obstruction prevents bile from entering the small intestine and thus prevents the absorption of fat-soluble vitamins. Vitamins A, D, E, and K are all fat soluble and thus would need to be supplemented in a patient with biliary obstruction.

Answer: A Vulnerable populations are defined as specific populations with unique health care problems. Vulnerable populations are not limited to the very young or older adults. Such individuals are those living in poverty, homeless persons, abused clients, substance abusers, and so on. Members of most vulnerable populations come from different cultures and have different beliefs and values. Vulnerable populations are at risk of experiencing poorer outcomes in response to interventions because of the multiple stressors that affect their daily lives.

Vulnerable populations are more likely to develop health problems. Which of the following is true of these populations? A) They are specific populations with unique health care problems. B) They are limited to the very young and older adult age groups. C) They live in communities with similar cultures, beliefs, and values. D) They frequently experience positive outcomes in response to community health interventions.

Answer: D Vulnerable population are defined as clients who are more likely to develop health problems as a result of excess risks, who have limits in access to health care services, or who are dependent on others for care.

Vulnerable populations of clients are those who are more likely to develop health problems as a result of: A) Chronic diseases, homelessness, and poverty B) Poverty and limits in access to health care services C) Lack of transportation, dependence on others for care, and homelessness D) Excess risks, limits in access to health care services, and dependence on others for care

You are caring for a patient admitted with an exacerbation of asthma. After several treatments, the ABG results are pH 7.40, PaCO2 40 mm Hg, HCO3 24 mEq/L, PaO2 92 mm Hg, O2 saturation 99%. You interpret these results as which of the following? Within normal limits Slight metabolic acidosis Slight respiratory acidosis Slight respiratory alkalosis

WNL The normal pH is 7.35 to 7.45. Normal PaCO2 levels are 38 to 48 mm Hg and HCO3 is 22 to 26 mEq/L. Normal PaO2 is >80 mm Hg. Normal oxygen saturation is >95%. Since the patient's results all fall within these normal ranges, the nurse can conclude that the patient's blood gas results are within normal limits.

Chap 48. During discharge planning, the nurse is teaching the client ways to prevent a recurrence of a UTI. Which of the following actions is correct?

Ways to prevent a recurrence of a UTI include: Avoid tight-fitting pants or other clothing that create irritation to the urethra and prevents ventilation of the perineal area. Drink eight, 8-ounce glasses of water per day to flush bacteria out of the urinary system. Wear cotton rather than nylon underclothes. Girls and women should always wipe the perineal area from front to back following urination or defecation in order to prevent introduction of gastrointestinal bacteria into the urethra.

You are caring for a patient with metastatic bone cancer. Which of the following clinical manifestations would alert you to the possibility of hypercalcemia in this patient? Weakness Paresthesia Facial spasms Muscle tremors

Weakness Signs of hypercalcemia are lethargy, headache, weakness, muscle flaccidity, heart block, anorexia, nausea, and vomiting. Paresthesia, facial spasms, and muscle tremors are symptoms of hypocalcemia.

Chap 42. The nurse helps a 50-year-old diabetic client who is to begin giving insulin injections identify previously successful coping strategies that may be useful in the current situation. Which of the following stressors is closely related to the new stressor?

Wearing glasses is another example of beginning a new strategy to assist with what will be a lifelong health need even though it is not necessarily a desired change. Interviewing for a job is a very short-lived situational stressor. Coping strategies effective while a teenager may not be relevant at age 50. Experiencing the stress of a divorce is a social/role stressor quite unlike that of a health problem.

Answer: C The community has three components: structure or locale, people, and social systems. To develop a complete community assessment, the nurse must take a careful look at each of the three components to begin to identify needs for health policy, health programs, and health services.

What are the three elements included in a community assessment? A) Environment, families, and social systems B) People, neighborhoods, and social systems C) Structure or locale, people, and social systems D) Health care systems, geographic boundaries, and people

Chap 34. A client has a streptococcal throat infection. The white blood cell count is elevated. When looking at the differential, the nurse expects which type of white blood cell to be elevated?

When a client has a streptococcal infection, neutrophil count is elevated. Eosinophil count is elevated in allergic reactions. Monocyte count is elevated in chronic inflammatory disorders. Lymphocyte count is elevated in viral infections.

Answer: D No individual client assessment should occur in isolation from the environment and conditions of the client's community. Industrial development, types of pollution, and cultural and religious groups are individual elements in the community.

When completing an individual total assessment of a client, the community-based nurse will include consideration of: A) The type of pollution present in the community B) The amount of industrial development in the past 5 years in the community C) The predominant cultural and religious groups found in the community D) The community structures, the population, and the local social system in which the client lives

Chap 49. Which of the following is a very important part of the movement process of defecation?

When peristaltic waves move the feces into the sigmoid colon and the rectum, the sensory nerves in the rectum are stimulated and the individual becomes aware of the need to defecate. The number of times a person defecates a day would be important information to have about bowel habits, but it is not related to the actual process of defecation.

1

Which activity represents secondary prevention? 1. A home health care nurse visits a patient's home to change a wound dressing. 2. A 50-year-old woman with no history of disease attends the local health fair and has her blood pressure checked. 3. The school nurse provides a program to freshman on healthy eating. 4. The patient attends cardiac rehab sessions weekly.

Answer: D Shortage of staff could mean less time and personnel to conduct and participate in research. Nursing teams that have teamwork skills can aid research. The desire to change is an incentive for research. Pressure from higher levels in the organization is also an incentive to research.

Which of the following could be a barrier to nursing research? A) Presence of teams in nursing B) Pressure from the administration C) Staff wishes to change a policy D) Shortage of professional nursing staff

Answer: A Day care is an example of respite care because it allows the family to maintain normalcy while the client is under their care. A nursing home client receives 24-hour care in the facility. Home care is an intermittent service in which only certain tasks are performed. Nurse extenders may be hired to perform a specific task, such as bathing.

Which of the following is an example of respite care? A) Day care B) Home care C) Nursing home D) Nurse extender

Answer: A National League for Nursing (NLN) is the correct answer. The master of science in nursing (MSN) degree is earned through advanced educational preparation in nursing. Public Health Administration (PHA) is concerned with areas of public health. The National Institutes of Health (NIH) addresses health on a national level.

Which of the following professional organizations was created to address concerns of members in the nursing profession? A) NLN B) MSN C) PHA D) NIH

Answer: C Prescriptive theories address nursing interventions for a phenomenon and predict the consequence of a specific nursing intervention. Descriptive theories describe the phenomena, speculate on the reason the phenomena occur, and predict nursing phenomena. Grand theories are broad and complex and provide a structural framework for broad, abstract ideas about nursing.

Which of the following statements about prescriptive theories is accurate? A) They describe phenomena. B) They have the ability to explain nursing phenomena. C) They reflect practice and address specific phenomena. D) They provide a structural framework for broad abstract ideas.

Answer: B, C, D Evidence-based practice helps nurses to solve dilemmas in the clinical setting because it combines scientific research with clinical expertise and local values. Evidence-based practice does require nurses to review and critique research and practice findings. Nurses are expected to always meet the standards of practice.

Which of the following statements is true about evidence-based practice? (Select all that apply.) Evidence-based practice: A) Is based only on the results of research B) Assists nurses in meeting standards of practice C) Helps nurses solve dilemmas in the clinical setting D) Requires nurses to review and critique research and practice findings

Answer: D Florence Nightingale is the correct choice. Barton founded the Red Cross. Dix organized hospitals, nurses, and supply lines to support the troops of the Union Army. Wald opened the first community health service for the poor.

Who acted to decrease mortality by improving sanitation in the battlefields, which resulted in a decline in illness and infection? A) Dorothea Dix B) Lillian Wald C) Clara Barton D) Florence Nightingale

Chap 42. The client is informed of a cancer diagnosis but claims to be fine. Which of the following is the most indicative physical evidence to the nurse of the client's stress?

With stress, respirations increase, pupils dilate, peripheral blood vessels constrict, and the heart rate increases.

Chap 52. A nurse is evaluating a nursing student's understanding of body water. Which of the following statements indicates a need for further teaching?

Women have a lower percentage of body water than men. Approximately 60% of the average healthy adult's weight is water, the primary body fluid. In good health this volume remains relatively constant and the person's weight varies by less than 0.2 kg (0.5 lb) in 24 hours, regardless of the amount of fluid ingested. Infants have the highest proportion of water, accounting for 70% to 80% of their body weight. Water makes up a greater percentage of a lean person's body weight than an obese person's.

Chap 34. The nurse is having difficulty obtaining a capillary blood sample from a client's finger to measure blood glucose using a blood glucose monitor. Which procedure will increase the blood flow to the area to ensure an adequate specimen?

Wrapping the finger in a warm cloth for 30-60 seconds will increase blood flow to the area. The hand is lowered to increase venous flow. The finger is pierced lateral to the middle of the pad perpendicular to the skin surface.

A client says, "I've noticed how many people are out walking in my neighborhood. Is walking good for you?" What is the best response to help the client through the stages of change toward regular exercise?

Yes, walking is great exercise. Do you think you could go for a 5-minute walk this next week?"

The lungs act as an acid-base buffer by A) increasing the resp rate and depth when CO2 levels in the blood are high, reducing acid load B) increasing resp rate and depth when CO2 levels in the blood are low, reducing base load C) decreasing resp rate and depth when CO2 levels in the blood are high, reducing acid load D) decreasing resp rate and depth when CO2 levels in the blood are low, increasing acid load

a Rationale: As a compensatory mechanism, the respiratory system acts on the CO2 + H2O side of the reaction by altering the rate and depth of breathing to "blow off" (through hyperventilation) or "retain" (through hypoventilation) CO2.

A patient with sleep apnea would like to avoid using a CPAP device if possible. To help him reach this goal the nurse suggests that the patient A) lose excess weight B) take a nap during the day C) eat a high-protein snack at bedtime D) use mild sedatives or alcohol at beditme

a Rationale: Excessive weight worsens sleep apnea, and weight loss reduces sleep apnea. A referral to a weight loss program or bariatric surgery may be indicated.

Implementation of nursing care for the pt with hyponatremia includes A) fluid restriction B) admin of hypotonic IV fluids C) admin of cation-exchange resin D) increased water intake for pts on NG suction

a Rationale: In hyponatremia that is caused by water excess, fluid restriction often is all that is needed to treat the problem.

methylxanthine

a bronchodilator - theophylline/aminophylline - relaxes bronchial smooth muscles. SE: palpitations, agitation, tachycardia, n/v. nursing care: therapeutic range 10-20 mcg/dl

ipratropium (atrovent)

a cholinergic antagonist - relieve bronchospasm. SE: headache, nausea, dry mouth. Nursing care: contraindicated in pt with soybean allergies or peanut allergies

cor pulmonale

a condition of enlargement of the right ventricle as a result of chronic disease within the lungs

dexamethasone

a corticosteroid drug (trade names Decadron or Dexamethasone Intensol or Dexone or Hexadrol or Oradexon) used to treat allergies or inflammation

captopril

a drug (trade name Capoten) that blocks the formation of angiotensin in the kidneys resulting in vasodilation

polycystic kidney disease

a genetic disorder characterized by the growth of numerous fluid-filled cysts in the kidneys - one of the most commonly occuring congential disorders

What is vincristine?

a leukemia/hodgkins/lymphoma drug; given ONLY IV

best definition of total quality management(TQM)

a method of system operation and management used to achieve continuous quality improvement

True (failure to collect all urine invalidates the test)

a nurse is collecting a 24 hour urine specimen. Is it T/F that the nurse must retart the entire test if the pt discards one of the urine specimens durin the course of the test?

it is important to assess a patient's actual cultural belief's because

a patient may not adhere to the usual health beliefs of her culture

A nurse is reviewing the lab results for a client and notes that the differential white blood cell count indicates a shift to the right. The nurse suspects that the client's diagnosis is most likely to be: a) pernicious anemia b) pneumonia c) sepsis d) coronary artery disease

a) pernicious anemia A differential white blood cell count is the leukocyte count broken down (differentiated) according to the cell type. A right shift represents an increased number of mature neutrophils, which is seen with pernicious anemia and after tissue breakdown

Sleep loss is associated with (select all that apply) A) increased BMI B) increased insulin resistance C) impaired cognitive functioning D) increased immune responsiveness E) increased daytime body temperature

a, b, c Rationale: Sleep loss is associated with decreases in immune function and body temperature and with endocrine changes, including a decrease in growth hormone levels. Impaired cognitive function and performance on simple behavioral tasks occur within 24 hours of sleep loss. The effects of sleep loss are cumulative. Chronic loss of sleep places the individual at risk for decreased cognitive function, depression, impaired daytime functioning, social isolation, and overall reduction in quality of life. Individuals who report less than 6 hours of sleep each night have higher body mass index (BMI) and are more likely to be obese. The risks for developing diabetes and glucose intolerance are increased in individuals with a history of insufficient sleep.

A nurse in a medical unit is caring for a client with heart failure. The client suddenly develops extreme dyspnea, tachycardia, and lung crackles and the nurse suspects pulmonary edema. The nurse immediately asks another nurse to contact the physician and prepares to implement which priority interventions? Select all that apply a) administer oxygen b) insert a foley catheter c) administering lasix d) administering morphine sulfate IV e) transferring the client to the coronary care unit f) placing the client in low Fowler's side lying position

a, b, c, d Pulmonary edema is a life threatening event that can result from severe heart failure. In pulmonary edema, the left ventricle fails to eject sufficient blood, and pressure increases in the lungs because of the accumulated blood. Oxygen is always prescribed, and the client is placed in high Fowler's position to ease the work of breathing. Lasix, a rapid acting diuretic will eliminiate accumulated fluid. A foley catheter is inserted to measure output accurately. IV morphine sulfate reduced venous return (preload), decreases anxiety, and also reduces the work of breathing. Transporting the client to the coronary care unit is not a priority intervention. In fact, this may not be necessary at all if the client's response to treatment is successful

Which are appropriate therapies for pt with DM (select all that apply) A) use of statins to treat dyslipidemia B) use of diiuretics to treat nephropathy C) use of ACE inhibitors to treat nephropathy D) use of laser photocoagulation to treat retinopathy

a, c, d Rationale: In patients with diabetes that have microalbuminuria or macroalbuminuria, angiotensin-converting enzyme (ACE) inhibitors (e.g., lisinopril [Prinivil, Zestril]) or angiotensin II receptor antagonists (ARBs) (e.g., losartan [Cozaar]) should be used. Both classes of drugs are used to treat hypertension and have been found to delay the progression of nephropathy in patients with diabetes. The statin drugs are the most widely used lipid-lowering drugs. Laser photocoagulation therapy is indicated to reduce the risk of vision loss in patients with proliferative retinopathy, macular edema, and in some cases of nonproliferative retinopathy.

You are caring for a pt with newly diagnosed type 1 diabetes. What information is essential to include in your pt teaching before discharge from the hospital (select all that apply) A) insulin administration B) elimination of sugar from diet C) need to reduce physical activity D) Hypoglycemia prevention, symptoms, and treatments

a, d Rationale: The nurse ensures that the patient understands the proper use of insulin. The nurse teaches the patient how to recognize and treat signs and symptoms of hypoglycemia and hyperglycemia.

A nurse is assessing a client who is experiencing seizure activity. The nurse understands that it is necessary to determine information about which of the following items as part of a routine assessment of seizures. Select all that apply a) postical status b) duration of seizure c) changes in pupil size or eye deviation d) seizure progression and type of movements e) what the client ate in the 2 hours before

a,b,c,d Typically, seizure assessment includes the time the seizure began, parts of the body affected, type of movements, and progression of the seizure, change in pupil size or eye deviation or nystagmus, client condition during the seizure, and postictal staus

bacteriacidal

able to kill bacteria

Where are vertex sounds heard?

above symphysus pubis

What do you ask every new admit?

advanced directive - yes or no

1-2 weeks

after insertion of the flexible catheter into the peritoneal cavity what is the expected waiting period before beginning a schedule of peritoneal dialysis?

Amylase

aids in digestion of carbs 25 - 151 units /L elevated in pancreatitis (thinks its 3 times) some meds can give flase + or - invalidated results if within 72 hrs after cholecystography with radiopaque dyes

sliding scale

always given with short acting insulin such as reg/ humulog

a hospital refuses to allow a visitor with a seeing eye dog to enter the hospital. this violates what

americans with disabilities act

What test checks for cardiac and pulm abnl in fetus?

amniocentesis perfromed before 20 wk

tamoxifen

an antagonist for estrogen that is used in the treatment of breast cancer

urge incontinence

an inability to suppress signals from the bladder to the brain to prevent urination

prilosec

antacid (trade name Prilosec) that suppresses acid secretion in the stomach

What is phenergan?

antiemetic

dramamine

antihistamine and antiemetic, used to treat motion sickness

codeine, dextromethophan

antitussive, surpresses cough reflex by direct effect on respiratory center. SE: n/v and sedation. nursing care: teach take only as directed

What dose vistaril tx?

anxiety itching-allergic skin rxn preop

Hypertensive Crisis signs and symptoms important interventions

any clinical condition requiring immediate BP reduction usually diastolic pressure higher than 120 HA drowsiness, confusion blurred vision neurologic status changes tachycardia & tachypenia dyspnea cyanosis seizures Be sure to assess BP every 5 mins

when a patient falls, you document in the nurse's notes

any patient-stated cause of fall

What does Indocin tx?

arthritis (osteo, rhematoid, gouty) bursitis tendonitis

the hospitalized client's uncle, who is a physician asks the nurse for the client's chart. the uncle is not the physician on the case. the best response for the nurse is to

ask the client for written consent before allowing the relative to see the chart

before surgery the pt. should stop using

aspirin

after being told that surgery would probably benefit the client, the client refuses to have the surgery. in accepting the client's choice of action, the nurse is acting on which ethical principle

autonomy

Magnesium - common food sources

avocado, canned white tuna, cauliflower, green leafy vegetables, milk, oatmeal, peanut butter, peas, pork, beef, chicken, potatoes, raisins, yogurt

when performing passive range of motion exercise

avoid moving the joint to the point of discomfort

Strategies to reduce sleepiness during nighttime working include A) exercising before work B) sleeping for at least 2 hours before work time C) taking melatonin before working the night shift D) walking for 10 minutes every 4 hours during the night shift

b Rationale: For night shift work, scheduling the sleep period just before going to work increases alertness and vigilance, improves reaction times, and decreases accidents during work.

Analyze the following diagnostic findings for your pt with type 2 diabetes. Which result wil need further assessment? A) BP 126/80 mm Hg B) A1C 9% C) FBG 130 mg/dL (7.2 mmol/L) D) LDL cholesterol 100 mg/dL (2.6 mmol/L)

b Rationale: Lowering hemoglobin A1C (to average of 7%) reduces microvascular and neuropathic complications. Tighter glycemic control (normal A1C < 6%) may further reduce complications but increases hypoglycemia risk.

While caring for a patient with a history of narcolepsy with cataplexy, the nurse can delegate which activity to the CNA? A) teaching about the timing of medications B) walking the patient to and from the bathroom C) developing a plan of care with a family member D) planning an appropriate diet that avoids caffeine-containing foods

b Rationale: Specific activities that may be delegated to nursing assistive personnel (NAP) include routine vital signs on stable patients, feeding or assisting patients at mealtimes, ambulating stable patients, and helping patients with bathing and hygiene. Nursing interventions that require independent nursing knowledge, skill, or judgment, such as assessment, patient teaching, and evaluation of care, cannot be delegated.

Pain is best described as? A) a creation of a person's imagination B) an unpleasant, subjective experience C) a maladaptive response to stimulus D) a neurologic event resulting from activation of nociceptors

b Rationale: The International Association for the Study of Pain (IASP) defines pain as "an unpleasant sensory and emotional experience associated with actual or potential tissue damage or described in terms of such damage."

A patient who has suffered severe burns in a motor vehicle accident will soon be started on parenteral nutrition (PN). Which of the following principles should guide the nurse's administration of the patient's nutrition? a) Administration of PN requires clean technique. b) Central PN requires rapid dilution in a large volume of blood. c) Peripheral PN delivery is preferred over the use of a central line. d) Only water-soluble medications may be added to the PN by the nurse.

b) Central PN requires rapid dilution in a large volume of blood. Central PN is hypertonic and requires rapid dilution in a large volume of blood. Because PN is an excellent medium for microbial growth, aseptic technique is necessary during administration. Administration through a central line is preferred over the use of peripheral PN and no medications may be added to PN by the nurse.

A home care nurse is providing instructions to a client with an arterial ischemic leg ulcer about home care management and self care management. Which statement, if made by the client, indicates a need for further instruction? a) i need to be sure not to go barefoot in my home b) i need to be sure that i elevate my leg above my heart level for at least an hour every day c) if i cut my toenails, i need to be sure that i cut them straight across d) it is all right to apply lanolin to my feet, but I shouldn't place it between my toes

b) I need to be sure that I elevate my leg above my heart level for at least an hour every day Foot care instructions for the client with peripheral arterial disease are the same as those for a client with diabetes mellitus. The client with arterial disease however, should avoid raising the legs above the level of the heart unless instructed to do so as part of an exercise program or if venous stasis is present.

Which of the following assessments should the nurse prioritize in the care of a patient who has recently begun receiving parenteral nutrition (PN)? a) Skin integrity and bowel sounds b) Electrolyte levels and daily weights c) Auscultation of the chest and tests of blood coagulability d) Peripheral vascular assessment and level of consciousness (LOC)

b) electrolyte and daily weights The use of PN necessitates frequent and thorough assessments. Key focuses of these assessments include daily weights and close monitoring of electrolyte levels. Assessments of bowel sounds, integument, peripheral vascular system, LOC, chest sounds, and blood coagulation may be variously performed, but close monitoring of fluid and electrolyte balance supersedes these in importance.

An appropriate nonopioid analgesic for mild pain is (select all that apply) A) oxycodone B) ibuprofen (advil) C) lorazepam (ativan) D) acetaminophen (tylenol) E) codeine with acetaminophen (tylenol 3)

b, d Rationale: Nonopioid analgesics include acetaminophen, aspirin and other salicylates, and nonsteroidal antiinflammatory drugs (NSAIDs).

When would you take carafate?

before eating b/c coats the stomach given for PUD (duodenal ulcers)

What is painless hematuria mean?

bladder ca

If the baby is in an occiput position, where are the heart sounds heard?

btw posterior sounds (sides) and midline below umbilicus

The typical fluid replacement for teh patient with a FVD is A) dextran B) 0.45% saline C) lactated Ringer's D) 5% dextrose in 0.45% saline

c Rationale: Administration of an isotonic solution expands only the extracellular fluid (ECF). There is no net loss or gain from the intracellular fluid (ICF). An isotonic solution is the ideal fluid replacement for a patient with an ECF volume deficit. Examples of isotonic solutions include lactated Ringer's solution and 0.9% NaCl.

It is especially important for the nurse to assess for which clinical manifestation(s) in a pt who has just undergone a total thyroidectomy? A) weight gain B) depressed reflexes C) positive chvostek's sign D) confusion and personality changes

c Rationale: Any condition that causes a decrease in the production of parathyroid hormone (PTH) may result in hypocalcemia. This may occur with removal of a portion of or injury to the parathyroid glands during thyroid or neck surgery. A positive Chvostek sign indicates low serum calcium levels.

What is the priority action for the nurse to take if the patient with type 2 diabetes complains of blurred vision and irritability? A) call the physician B) administer insulin as ordered C) check the pt blood glucose level D) assess for other neurologic symptoms

c Rationale: Blood glucose testing should be performed whenever hypoglycemia is suspected so that immediate action can be taken if necessary.

A patient is recieving a PCA infusion following surgery to repair a hip fracture. She is sleeping soundly but awakens when the nurse speaks to her in a normal tone of voice. Her respirations are 8 beats per minute. The most appropriate nursing action in this situation is to? A) stop the PCA infusion B) obtain an oxygen saturation level C) continue to closely monitor the patient D) administer naloxone (Narcan) and contact the physician

c Rationale: Close monitoring is indicated for this patient with a sedation score of 3 and respiratory rate of 8 breaths/minute. If the respirations fall below 8 breaths/minute and the sedation level is 5 or greater, the nurse should vigorously stimulate the patient and try to keep the patient awake.

A pt is receiving a loop diuretic. The nurse should be alert for which symptoms? A) restlessness and agitation B) paresthesias and irritability C) weak, irregular pulse and poor muscle tone D) increased BP and muscle spasms

c Rationale: Loop diuretics may result in renal loss of potassium (i.e., hypokalemia). Clinical manifestations of hypokalemia include fatigue, muscle weakness, leg cramps, nausea, vomiting, paralytic ileus, soft, flabby muscles, paresthesias, decreased reflexes, weak, irregular pulse, polyuria, hyperglycemia, and electrocardiographic changes

A cancer patient who reports ongoing, constant moderate pain with short periods fo severe pain during dressing changes A) is probably exaggerating his pain B) should be referred for surgical treatment of his pain C) should be receiving both a long-acting and short-acting opoid D) should receive regularly scheduled short-acting opoids plus acetaminophen

c Rationale: Moderate to severe pain usually requires an opioid analgesic. Constant, moderate pain is treated with a long-acting opioid; procedural severe pain is treated with a short-acting opioid.

Providing opioids to a dying patient who is experiencing moderate to severe pain A) may cause addiction B) will probably be ineffective C) is an appropriate nursing action D) will likely hasten the person's death

c Rationale: Opioids are an appropriate intervention for moderate to severe pain experienced by a dying patient, and they may be titrated upward many times over the course of therapy to maintain adequate pain control.

An important nursing responsibility related to pain is to? A) leave the patient alone to rest B) help the patient appear to not be in pain C) believe what the patient says about the pain D) assume responsibility for eliminating the patient's pain

c Rationale: Pain is a subjective experience, and patients need to feel confident the nurse will believe their reports of pain.

A diabetic pt has a serum glucose level of 824 mg/dL (45.7 mmol/L) and is unresponsive. Following assessment of the pt, the nurse suspects diabetic ketoacidosis rather than hyperosmolar hyperglycemic syndrome based on the finding of what? A) polyuria B) severe dehydration C) rapid, deep respirations D) decreased serum potassium

c Rationale: Signs and symptoms of DKA include manifestations of dehydration such as poor skin turgor, dry mucous membranes, tachycardia, and orthostatic hypotension. Early symptoms may include lethargy and weakness. As the patient becomes severely dehydrated, the skin becomes dry and loose, and the eyeballs become soft and sunken. Abdominal pain is another symptom of DKA that may be accompanied by anorexia and vomiting. Kussmaul respirations (i.e., rapid, deep breathing associated with dyspnea) are the body's attempt to reverse metabolic acidosis through the exhalation of excess carbon dioxide. Acetone is identified on the breath as a sweet, fruity odor. Laboratory findings include a blood glucose level greater than 250 mg/dL, arterial blood pH less than 7.30, serum bicarbonate level less than 15 mEq/L, and moderate to large ketone levels in the urine or blood ketones.

A client with a hx of lung disease is at risk for developing respiratory acidosis. The nurse assesses this client for which signs and symptoms characteristic of this disorder? a) bradycardia and hyperactivity b) decreased respiratory rate and depth c) headache, restlessness, and confusion d) bradypnea, dizziness, and paresthesias

c) headache, restlessness, and confusion When a client is experiencing respiratory acidosis, the respiratory rate and depth increase in an attempt to compensate. The client also experiences headache, restlessness, mental status changes such as drowsiness and confusion, visual disturbances, diaphoresis, cyanosis as the hypoxia becomes more acute, hyperkalemia, a rapid irregular pulse, and dysrhythmias

A nurse is caring for an infant with a tracheostomy and prepares to suction the infant. Which of the following would be an incorrect nursing intervention in this clinical situation? a) insert the catheter the length of the tracheostomy tube with the suction off b) apply intermittent suction and withdraw the catheter with a twisting motion c) limit insertion and suctioning time to 15 seconds to prevent hypoxia d) reoxygenate between suction catheter passage and allow sufficient recovery time with each pass

c) limit insertion and suctioning time to 15 seconds to prevent hypoxia When suctioning a tracheostomy in an infant, it is necessary to limit insertion and suction time to 5 seconds to prevent hypoxia.

dry sterile dressing

check order, clean gloves, loosen dressing, observe wound, measure, rmv gloves, prep sterile field, moisten sterile dressing and clean, dry wound, apply ointments, cover

aminoglycosides, sulfonamides, diuretics, NSAIDS, contrast media

chemicals (nephrotoxins) capable of causing intrarenal ARF

What is bryants traction used for?

children < 3yr, < 35 # w/femur fx

eye meds(opthalmic)

clean eyes, tilt head back, put gtts, apply pressur on inner canthus

sterile wound irrigation

clean gloves, position cliient so drainage runs down, drape, discard gloves, open irrigation tray sterily, pour solution, sterile gloves, open packs, draw up solution, irrigate until runs clear, pat dry

3

clients with postatitis should be encouraged to drink _____ liters per day

lasix

commonly used diuretic (trade name Lasix) used to treat hypertension and edema

What is versed given for?

conscious sedation

The nurse is unable to flush a central venous access device and suspects occlusion. The best nursing intervention would be to A) apply warm moist compresses to the insertion site B) attempt to force 10 mL of normal saline into the device C) place the pt on the left side with head down position D) instruct the pt to change positions, raise are, and cough

d Rationale: Catheter occlusion interventions include instructing the patient to change position, raise an arm, and cough; assessing for and alleviating clamping or kinking; flushing with normal saline using a 10-mL syringe (do not force flush); using fluoroscopy to determine cause and site; and using anticoagulant or thrombolytic agents.

A nurse believes that patients with the same type of tissue injury should have the same amount of pain. This statement reflects A) a belief that will contribute to appropriate pain management B) an accurate statement about pain mechanisms and an expected goal of pain therapy C) a premise that this belief will have no effect on the type of care provided to people in pain D) a lack of knowledge about pain mechanisms, which is likely to contribute to poor pain management

d Rationale: Genetic makeup and variability among individuals affects the plasticity of the central nervous system; this phenomenon helps to explain individual differences in responses to pain. Poor knowledge of pain mechanisms often leads to poor pain management.

When comparing the pathophysiology of Type 1 and Type 2 diabetes, which statement would be correct for a pt with type 2 diabetes who was admitted to the hospital with pneumonia? A) the pt must receive insulin therapy to prevent the development of ketoacidosis B) the pt has islet cell antibodies that have destroyed the ability of the pancreas to produce insulin C) the pt has minimal or absent endogenous insulin secretion and requires daily insulin injections D) the pt may have sufficient endogenous insulin to prevent ketosis but is at risk for development of hyperosmolar hyperglycemic syndrome

d Rationale: Hyperosmolar hyperglycemic syndrome (HHS) is a life-threatening syndrome that can occur in the patient with diabetes who is able to produce enough insulin to prevent diabetic ketoacidosis (DKA) but not enough to prevent severe hyperglycemia, osmotic diuresis, and extracellular fluid depletion.

During the postoperative care of a 76-year-old pt, the nurse monitors the pt's I&O carefully, knowing that the pt is at risk for fluid and electrolyte imabalances primarily because A) older adults have an impaired thirst mechanism and need reminding to drink fluids B) water accounts for a greater percentage of body weight in the older adult than in younger adults C) older adults are more likely to lose extracellular fluid during the surgical process D) small losses of fluid are more significant because body fluids account for only about 50% of body weight in older adults

d Rationale: In the older adult, body water content averages 45% to 55% of body weight.

Which of the following words is most likely to be used to describe neuropathic pain? A) dull B) mild C) aching D) burning

d Rationale: Neuropathic pain is caused by damage to peripheral nerves or structures in the central nervous system (CNS). Typically described as numbing, hot or burning, shooting, stabbing, sharp, or electric shock-like in nature, neuropathic pain can be sudden, intense, short lived, or lingering.

Which of the following is true regarding rapid eye mevement (REM) during sleep? A) the EEG patteren is quiescent B) it occurs only once in the night C) it is seperated by distinct physiologic stages D) the most vivid dreaming occurs during this phase

d Rationale: Rapid eye movement (REM) sleep accounts for 20% to 25% of sleep. REM sleep follows non-REM sleep. In a healthy adult, REM sleep occurs 4 to 5 times during a period of 7 to 8 hours of sleep. This stage is considered paradoxic because the brain waves resemble wakefulness. REM sleep is thought to be important for memory consolidation and is the period when the most vivid dreaming occurs.

A pt has the following ABG result: pH 7.52, PaCO2 .30 mm HG, HCO3 -24 mEq/L. The nurse determines that these results indicate A) metabolic acidosis B) metabolic alkalosis C) respiratory acidosis D) respiratory alkalosis

d Rationale: Respiratory alkalosis (carbonic acid deficit) occurs with hyperventilation. The primary cause of respiratory alkalosis is hypoxemia from acute pulmonary disorders. Anxiety, central nervous system (CNS) disorders, and mechanical overventilation also increase ventilation rate and decrease the partial pressure of arterial carbon dioxide (PaCO2) level. This leads to decreased carbonic acid and alkalosis

The nurse is reviewing the laboratory test results for a 71-year-old patient with metastatic lung cancer. The patient was admitted with a diagnosis of malnutrition. Serum albumin level is 4.0 g/dl and prealbumin is 10 mg/dl. The nurse should draw which of the following conclusions? a) The albumin level is normal, and therefore the patient does not have protein malnutrition. b) The albumin level is increased, which is a common finding in patients with cancer who have malnutrition. c) Both the serum albumin and prealbumin levels are reduced, consistent with the admitting diagnosis of malnutrition. d) Although the serum albumin level is normal, the prealbumin level more accurately reflects the patient's nutritional status.

d) Although the serum albumin level is normal, the prealbumin level more accurately reflects the patient's nutritional status. Serum albumin has a half-life of approximately 20 to 22 days; therefore the serum level may lag behind actual protein changes by more than 2 weeks and is therefore not a good indicator of acute changes in nutritional status. Prealbumin has a half-life of 2 days and is a better indicator of recent or current nutritional status.

A nurse reviews the laboratory results for a client with a suspected ectopic pregnancy. The nurse should expect that the results of the beta subunit of human chorionic gonadotropin to be which of the following if the client had an ectopic pregnancy? a) WNL b) not present c) high d) low

d) low an abnormal pregnancy is suspected if b-hCG is present but at lower levels than expected. The absence of b-hCG would indicate no pregnancy, whereas WNL could indicate normal pregnancy. High levels would indicate a molar pregnancy

What position do you put arteries in for better perfusion?

dAngle Arteries

What are v/s for shock?

dec BP inc HR inc RR

What do you admin for SIADH?

declomycin, diuretics

morphine

decreases RR

What reverese iron toxicity?

deferoxamine

What sedative to give for pancreatitis?

demerolol, NOT morphine

lanolin

dermatologic

hemodialysis

dialysis of the blood to remove toxic substances or metabolic wastes from the bloodstream using a an artificial semipermeable membrane

What is caput succedaneum?

diffuse edema that crosses fetal scalp suture lines that will disappear in 1-3 days

what would be consider an invasion of patient privacy

discussing the comatose patient's condition with his father in law posting a blog on your personal website about the patient and their day discussing the outcome of a test with another nurse while in an elevator

nursing interventions for severe hypoglycemia

do not give anything by mouth, position on side, admin glucagon, call 911, stay with pt.

How to prevent dumping syn?

eat reclining position lie down 20-30 min after meals restrict fluids before/during meals eat low fiber/ch2o meals eat sm, freq meal

hyperglycemia actions

encourage to drink sugar free drinks, admin insulin

15 to 20%

end stage renal failure is ___% to ____% of GFR

Creatine kinase (CK)

enzyme found in muscle and brain tissue, resulting from cell trauma 26 - 174 units /L used to detect:myocardial, skeletal muscle or CNS damage

What is the med of choice for anaphlyaxis?

epi

What's a transesophageal fistula?

esophagus doesn't fully develop 3 C's: chocking, coughing, cyanosis

What is russels traction used for?

femur, lower leg

What are sx of grave's dz/hyperthyoidism?

fine, soft hair accel physical and mental fxn heat sensitivity

isoniazid (INH)

first line tx TB - interfers w/cell wal. SE deficiency in B6, peripheral neuritis; liver dysfunction. nursing care: observie for jaundice, frequent hearing tests

Phosphorus - common foods

fish, organ meats, nuts, pork, beef, chicken, whole-grain breads and cereal

corticosteriods, immunosuppressive drugs, plasmaperesis

goodpastures syndrome is managed with...

On

gown, mask, gloves

Bolus feeding

hang bag above stomach, clamp tubing, fill bag with formula, prime tubing, reclamp tube, attach to tube and open clamp, adjust flow, flush tube with water, disconnect

Where are breech sounds heard?

highup on the fundus near the umbillicus

nephrotic syndrome

in this syndrome, glomeruli permeability is altered, allowing albumin to move from blood to urine (changes osmotic pressure of blood). Kidney reabsorbs sodium and water, resulting in generalized body edema. Causes liver to make lipoproteins (cholesterol), leading to hyperlipidemia. As immunoglobulins are EXCRETED, the child's immunity is decreased.

fluticasone and salmetrol (Advair Discus)

inhaled glucocorticoid w/long acting beta 2 adrenergic that is used for maintenance of Asthma. The nurse should monitor for candiadisis

the oblique side lying(lateral) position is helpful because

it takes the pressure of the trochanter and shoulder

malnutrition contributes to the susceptibility to infection because

it upsets homeostatic balance in the body

During CBI, what is a correct position?

leg straight with foley bag tapped to it; no other restrictions

What is the med of choice for VTACH?

lidocaine

Albumin

main plasma protein in blood 3.4 - 5 g/dL Increase- dehydration, diarrhea, metastic carcinoma Decrease - acute infection, ascites & alcoholism

What is a possible first sign of CF?

meconium ileus at birth not eating doesn't poop insolable

cathartic

meds used to treat constipation

What to give for elevated IOP?

miotics (pilocarpine); NOT ATROPINE

when on digoxin

monitor apical pulse for 1 full minute

What pain drug is >< in pancreatitis?

morphine = it causes spasm of the spincter of Oddi use DEMEROL

infiltration r/t IV

most common problem: when fluid or med leaks out of vein into tissue, edema occurs, feels cool, fluid in tissue will reabsorb eithin 24 hrs.

Akathisia

motor restlessness tx with anitPD meds

What would be sx of hypokalemia?

muscle weakness, dysrhymias inc w/ bananas, apricots, celery, beans, carrots, oranges, rasins

saw palmetto

name the herbal agent used for BPH that can falsely lower PSA results and mask signs of prostate cancer resulting in delayed diagnosis

Proscar (finastride)

name the medication used in BPH which lowers DHT (a major cause of prostate growth factor)

propylthiouracil (PTU), methimazole (Tapazole)

name two medications used to treat thyroid storm

morphine

narcotic

urine

no more than 750-1000ml's can be taken out at a time or will cause shock

1.010

normal urine specific gravity is 1.010 - 1.030. A client with diabetes insipidus will have specific gravity of < ______

an example of administrative law that affects nursing is

nurse practice acts

an example of communication through the visual channel is

observing that the client looks away from the nurse when discussing a certain subject

Compensation

pH is in normal range of 7.35 - 7.45

Respiratory Acidosis

pH: decreased PCO2: elevated HCO3: normal Causes: hypoventilation, retaining CO2- Hypercapnia

Metabolic Acidosis

pH: decreased PCO2: normal HCO3: decreased Addition of large amounts of fixed acids to body fluids; vomiting, suctioning

Murphys sign

pain with palpation of gallbladder seen with cholecystitis

6 rights of medication administration

patient, time, dose, route, medication, documentation

carbohydrates, proteins

patients with ARF should eat a diet high in ______ and restricted on _________

an advantage to the patient of a managed health care system is

paying lower health insurance costs and small copayments

What does a coffee brown emesis indicate?

peptic ulcer

an example of illness prevention activities would be

performing vision screenings

anus

perineal prostatectomy nursing considerations include primarily preventing infection because incision is the _____

What type of vision do glaucoma pt lose?

peripheral

What's a beefy red tongue mean?

pernicious anemia

What is a symbol of a fat embolism?

petchiae

the correct actions when donning a pair of sterile gloves

picking up the first glove by grasping it on the fold of the cuff holding the glove with its fingers downward

What are dx of DI?

polyuria, thirst, weakness, dehydration

Placement for infant with spina bifida?

positon prone so that sack doesn't rupture

venous, can

post TURP dark red blood with lower viscosity and few clots is indcative of ______ bleeding and can/cannot be controlled with traction on the cathether (taping catheter to the thight or abdomen)

What v/s to check first with K+?

pulse b/c indicate dysrhythmia

What are koplicks spots?

red spots with blue center, characteristics of PRODOMAL phase of measles

reportable

retropubic prostatectomy nursing considerations include assessing abdominal incision for infection. Urine on dressing is an expected/reportable finding

Narcan or Noloxene

reverses effects of other narcotics

6, 2, 9, rifampin

rifampin (Rifadin), isoniazid (INH), for _____ months... pyrazinamide (Tebrazid) and ethambutol (Myambutol) for the first ____ months. If pt have HIV the regimen is ____ months. Which one turns secretions/urine orange?

depressant

slows down mental/physical processes

when caring for an unconscious client the nurse should

speak to the client in a normal voice

an advance directive is a consent that

spells out the incapacitated patient's wishes regarding test and treatments

the practice of nursing is regulated by

state board of nursing

Consent for medical treatment can be given by a minor with a

std

continuous feeding tube(pump)

stop feeding q 4-8 hrs to check residual, flush q 4-6

antistreptolysin-O (ASO - titer)

streptococcal antibody test. elevated or increasing titers indicate a recent or prolonged infections of Group A betahemolytic streptococcus

How is the MMR Vaccine given?

subQ

onset of hypoglycemia

sudden

if a patient requires a pelvic examination you would position the patient on the table in which position

supine

What is imp for labeling for ABG?

temperature FiO2/if on any L of O2

What test is done to test for myasthenia gravis?

tensilon - will improve sx

Shilling Test

test for pernicious anemia-how well one absorbs vit b12

in which situation may the nurse legally use restraints

the critically ill client becomes violent and is removing IV lines and medical monitoring devices

while many factors can affect access to health care in the US, the most significant barrier is probably

the escalating cost of services

when auscultating heart sounds S1 and S2

the fifth intercostal space at the midclavicular line with the diaphragm

a statement about relationships among concepts or facts, based on existing information

theory

increase acidity

these foods increase/decrease acidity: cranberries, plums, whole grains, meat, cheese

decrease

these foods increase/decrease acidity: peaches, apples, carbonated beverages

rotate injection sites

to slow formation of scar tissue

Protein

total amount of albumin and globulins in plasma 6 - 8 g/dL

ototoxicity

toxic effect r/t damage to 8th cranial nerve resulting in dizziness, tinnitus, gradual hearing loss

nephrotoxicity

toxicity due to blood and protien in the urine

an example of secondary health care services is the clients who

undergoes an appendectomy for a rupture appendix

Natriuretic peptides (ANP, BNP, CNP)

used to id clients with CHF ANP: 22 - 27 pg/ml BNP less than 100 pg/ml CNP not yet determined

antidote foe coumadin

vitamin k

with ear meds if there are 2 differant meds

wait 5-10 mins inbetween meds

when a nurse is sued, the "reasonable person" standard asks

what a reasonable nurse would do in a similar situation

anorexia, pallor, generalized edema

what are common s/s associated with nephrotic syndrome

no isolation (not communicable)

what are the contact precautions for Legionnaires diease? droplet/airborne/contact/no isolation

hyperkalemia, hypernatremia, hypercholesterolemia,

what are the expected elevated labs for oliguric phase of ARF

epicanthal folds, pointed nose, small chin, low set ears

what are the expected malformations with an infant who has polycystic kidney disease

dehydration, poor turgor, hypotension, flattened neck veins

what are the expected s/s of the diuretic phase of ARF

anemia, peripheral neuropathy, gout. uremia

what are the late s/s of chronic glomerulonephritis. What is major s/s of renal failure?

Proscar (finastride), alpha blockers Hytrin (terazosin), Cardura (doxazosin), Flomax (tamsulosin)

what are the nonsurgical tx for BPH

lugols solution (decreases vascularity of the gland)

what can be given to stop the release of thyroid hormone that is stored in the thyroid or given to prevent a thyroid storm

proteinuria, hematuria AND casts

what does a urinalysis reveal in the pt with Chronic Glomerulonephritis?

increase acidity - limit animal protein to 5-7 servings per week

what is the dietary management for pt with calcium phosphate stones

spinach, strawberries, rhubarb, chocolate, tea, peanuts, wheat bran (foods all high in oxylate)

what is the dietary management for pt with oxylate stones

increase alkalinity, limit purines

what is the dietary management for pt with uric acid stones

500ml - 600ml

what is the insensible replacement for clients with acute glomerulonephritis?

noncontrast helical CT

what is the most sensitive means for detecting renal calculi?

0.6g/Kg/day

what is the suggested protein amount for pt on a low protein diet r/t acute glomerulonephritis

increased immunosupressive doses

what is the treatment for acute graft rejection?

immediately stop or slow dialysis, infuse hypertonic saline, albumin or mannitol

what is the treatment for dysequilibrium syndrome

a patient's says I don't know what to do about this problem. the most therapeutic response is

what options are you considering

renal biopsy, abdominal pain, decreased BP, increased pulse rate, decreased UOP

what procedure is being performed with the pt is required to lay on abdomen, remain in bed several hours post procedure and pressure must be applied for 10 minutes to affected kidney. pt should be monitored for s/s of internal bleeding which include:

HTN, tachycardia, peripheral edema, respiratory crackles,

what s/s are expected in the oliguric phase of ARF

frequency, nocturia, reduced size/force of stream

what s/s are typically reported with BPH

decreased serum albumin, elevated creatinine/urea nitrogen

what would you expect a blood sample for a pt with acute glomerulonephritis to reveal?

held, can cause severe hypotension

when a patient is schedule for hemodialysis long term antihypertensives should be given/held. Why?

suppress the immune response (causes inflammation and more damage)

why would corticosteriods and immunosuppressants be given in cases like nehprotic syndrome?

protein, 0.6g/kg/day

with renal insufficiency, despite cause, what dietary item is restricted when blood urea nitrogen levels are elevated? How much?

sodium, 3-4gm per day

with renal insufficiency, despite cause, what dietary item is restricted when patient has HTN? How much?

hyperacute graft rejection (less that 48 hours after transplant)

you have a pt that just underwent a kidney transplant, 1 day post surgery they become febrile, hypternsive with pain at the transplant site. What type of rejection are they experiencing?

acute graft rejection

your patient underwent kidney transplant 1-3 weeks ago, they begin to experience oliguria/anuria, become febrile with increased BP, lethargy, fluid retention, increased serum creatinine, increased blood urea nitrogen, elevated potassium what stage of organ rejection are they in?

The nurse receives report from the previous shift. Which of the following patients should the nurse see FIRST? 1. A patient post coronary artery bypass graft (CABG) having the atrioventricular (AV) wires removed later in the day. 2. A patient with type 1 diabetes scheduled for a cardiac catheterization later today. 3. A patient 1 day postoperative with an epidural catheter in place. 4. A patient diagnosed with cardiomyopathy being evaluated for a heart transplant.

(1) although the patient requires a high level of nursing care, no indication that the patient is unstable (2) patient requires preoperative assessment and teaching, no indication that the patient is unstable (3) correct —epidural used for pain relief, monitor for urinary incontinence, hypotension, respiratory depression, and nausea and vomiting (4) requires monitoring but patient with epidural takes priority

A client with an irregular pulse rate of 81 and a potassium level of 3.0 mEq/L has digoxin (Lanoxin) ordered. Which of the following actions, if taken by the nurse, is BEST? 1. Give the digoxin. 2. Hold the digoxin. 3. Notify the physician. 4. Recheck the pulse.

(1) although the pulse is normal, level of potassium must be considered (2) notify physician about low potassium (3) correct—hypokalemia can precipitate digoxin toxicity; physician should be called to obtain order for potassium supplement (4) notify physician about the potassium level

The nurse observes a client who is taking phenelzine (Nardil) eat another client's lunch. After a few minutes, the client complains of headache, nausea, and rapid heartbeat, and begins to vomit. The nurse anticipates administering which of the following medications? 1. Buspirone (BuSpar). 2. Fluoxetine (Prozac). 3. Prochlorperazine (Compazine). 4. Nifedipine (Procardia).

(1) antianxiety; side effects include light-headedness, confusion, hypotension, palpitations (2) SSRI antidepressant; side effects include palpitation, bradycardia, nausea and vomiting (3) antiemetic; side effect include drowsiness, orthostatic hypotension (4) correct—antihypertensive; client experiencing hypertensive crisis due to ingesting tyramine; side effects include dizziness, headache, nervousness

A client has orders for cefoxitin (Mefoxin) 2 g IV piggyback in 100 ml 5% dextrose in water. The primary IV is 5% dextrose in lactated Ringer's and is infusing by gravity. It is MOST important for the nurse to take which of the following actions? 1. Administer the medication slowly, at 20 to 25 cc/h. 2. Change the primary IV solution. 3. Hang the piggyback infusion bag higher than the primary infusion bag. 4. Obtain an infusion pump prior to administration.

(1) antibiotic should be administered within 1 hour (2) unnecessary for safe infusion (3) correct—when using a gravity drip, piggyback fluid level needs to be higher than primary infusion (4) unnecessary for safe infusion

The nurse cares for a client after right cataract surgery. The nurse should intervene if which of the following is observed? 1. Client is in the supine position. 2. The head of the bed is elevated 30 degrees. 3. The client is lying on the right side. 4. An eye shield is over the right eye.

(1) appropriate position (2) decreases swelling and pain (3) correct—client should not be positioned with operative side in a dependent position or against the bed (4) shield is appropriate

The nurse supervises care given to clients on a medical/surgical unit. The nurse should intervene if which of the following is observed? 1. A nurse and client wear masks during a dressing change for the central catheter used for total parenteral nutrition. 2. A nurse injects insulin through a single-lumen percutaneous central catheter for a client receiving total parenteral nutrition. 3. A nurse applies lip balm to his/her lips immediately after performing a blood draw to obtain a specimen. 4. A nurse wears a disposable particulate respirator when administering rifampin to a client with tuberculosis.

(1) appropriate procedure, prevents airborne contamination (2) insulin is the only medication that can be given, compatible with TPN (3) correct—applying lip balm or handling contact lenses is prohibited in work areas where exposure to bloodborne pathogens may occur (4) use airborne precautions for TB, private room with negative air pressure, minimum of six exchanges per hour

An elderly man diagnosed with chronic schizophrenia is followed in a partial hospitalization program. The client has been on long-term antipsychotic medication and recently developed symptoms of tardive dyskinesia. The nurse's documentation should include which of the following? 1. Assessment of ADL (self-care) ability. 2. Mini-Mental Status Examination (MMSE). 3. Abnormal Involuntary Movement Scale (AIMS). 4. Modified Overt Aggression Scale (MOAS).

(1) assessment of client's abilities to complete his activities of daily living (ADLs) needs to be completed and revised with a client who is aging and chronically mentally ill (2) measures cognitive function (3) correct is most widely accepted examination to test for the presence of tardive dyskinesia (4) assessment tool for determining severity of aggression; usually utilized to determine nature, severity, and prevalence of aggression in an inpatient population

An infant is admitted with vomiting and diarrhea. The infant's anterior fontanelle is depressed and temperature is 103.2°F (39.5°C). Which of the following nursing actions is MOST appropriate? 1. Obtain daily weights and evaluate weight loss. 2. Observe the infant's ability to take in fluids. 3. Place a full bottle of Pedi-Lyte at the bedside. 4. Start an intravenous infusion.

(1) assessment; correct information, but is not what the question asks for (2) correct—assessment; will assist in determining if hydration can be done through oral fluids alone (3) implementation; does not do anything to improve the situation; placing a full bottle at the bedside doesn't guarantee that the infant is taking fluids (4) implementation; would be implemented later

A 4 lb 10 oz baby boy is delivered at 32 weeks' gestation. The infant is admitted to the neonatal intensive care unit and placed in an incubator. The infant has mottling of the skin and acrocyanosis with irregular respirations of 60. The nurse should recognize that these findings indicate which of the following? 1. Hypoglycemia. 2. Cold stress. 3. Birth asphyxia. 4. Hypovolemia.

(1) blood sugar less than 25 mg/dL; would see cyanosis, apnea, tachypnea, irregular respirations, diaphoresis, jitteriness, weak cry, lethargy, convulsions, coma (2) correct—symptoms describe cold stress (3) would see meconium-stained amniotic fluid (4) would see symptoms of shock

An elderly alcoholic client receives a long-acting benzodiazepine (Librium) for 2 days for symptom management and reduction. The client states, "Get those bugs off of me and clean them out of here." The nurse knows the client is exhibiting symptoms of which of the following? 1. A reaction to the sedative medication. 2. A worsening course of the withdrawal syndrome. 3. An exacerbation of the schizophrenia process. 4. The process of aging and the effects of delirium.

(1) client has been medicated with benzodiazepines and did not experience untoward reactions (2) correct—client has most probably progressed to another level of abstinence withdrawal from polypharmacy chemical dependence; characteristic symptoms include tremors, increased heart rate, and fever, as well as psychological problems of confusion, delusions, and hallucinations (3) schizophrenic client usually experiences an episode of auditory hallucinations, not visual or tactile hallucinations (4) combination effect of the normal aging process and dementia could precipitate a similar reaction; however, the normal aging process does not produce delirium but rather dementia

Which of the following statements, if made by the nurse, is accurate about the exercise program required for a patient with rheumatoid arthritis? 1. "If you are having a 'bad' day, postpone your exercises until the next day." 2. "Passive exercises are better for you than active exercises." 3. "When inflammation is severe, decrease the number of repetitions of the exercise." 4. "You can substitute your normal household tasks for your exercises to provide variety."

(1) consistency is important to maintain joint mobility (2) active exercises are better than passive or active-assistive exercises (3) correct—should reduce repetitions when patient experiences more pain (4) should do exercises that have been prescribed for patient

A young adult immobilized for trauma to the spinal cord has periods of diaphoresis, a draining abdominal wound, and diarrhea. On the basis of the nursing assessment, which of the following is the MOST important nursing diagnosis? 1. Risk for constipation related to immobilization. 2. Risk for impaired skin integrity related to immobilization and secretions. 3. Risk for wound infection related to involuntary bowel secretions. 4. Risk for fluid volume excess related to secretions.

(1) constipation is not a problem because the client has diarrhea (2) correct—skin is very susceptible to breakdown because of immobility and bodily secretions; needs numerous nursing interventions to prevent this (3) not most important (4) may be risk of fluid volume deficit due to diarrhea and secretions

The nurse plans a diet for a child diagnosed with cystic fibrosis (CF). Which of the following dietary requirements should be considered by the nurse? 1. High protein, high fat, and high calories. 2. High protein, low fat, and high calories. 3. Low protein, low fat, and low carbohydrate. 4. High protein, high fat, and low carbohydrate.

(1) contains high fat (2) correct—impaired intestinal absorption due to cystic fibrosis necessitates a diet higher in protein and calories; fat is decreased because it may interfere with absorption of other nutrients (3) not adequate for this child (4) contains high fat

A client with clear lung sounds and unlabored breathing receives aminophylline IV. Which of the following is the MOST appropriate nursing action if the client's IV infiltrates? 1. Apply warm soaks to the infiltration site, start a new IV, and continue IV medications. 2. Wait 2 hours, reassess the client, and restart the IV if the client has wheezing or labored breathing. 3. Restart the IV and continue the previous medication schedule. 4. Call the physician and recommend that the IV medications be changed to PO.

(1) continued IV medication may not be necessary based on the current assessment (2) physician should be notified if IV medications are not infusing as scheduled (3) client has improved breathing, so IV medications may not be indicated (4) correct—before a new IV is started on this client, physician should be called and PO medications recommended

The school nurse observes a group of preschool children in the playroom. The nurse recognizes which of the following activities as appropriate behavior for a 5-year-old boy? 1. The boy plays with a large truck with another child. 2. The boy talks on a toy telephone and imitates his father. 3. The boy works on a puzzle with several other children. 4. The boy holds and cuddles a large stuffed animal.

(1) cooperative play occurs in school-aged children (2) correct—imitative behavior seen at this age (3) too advanced for this age (4) too regressed for this age

The mother of a 7-year-old child is dying. The nurse anticipates the child will have which of the following concepts of death? 1. Death is punishment for his/her actions. 2. Death is inevitable and irreversible. 3. Death is temporary and gradual. 4. Death as a concept based on past experience.

(1) correct-7-year-olds see death as a punishment (2) by age of 9, most children begin to develop an adult concept of death and begin to understand that death is irreversible (3) is a preschool child's concept of death (4) is an adolescent's concept of death

The nurse cares for an elderly client who is receiving IV fluids of 0.9% NaCl at 125 mL/h into the left arm. During a routine assessment, the nurse finds that the client has distended neck veins, shortness of breath, and crackles in both lung bases. Which of the following actions should the nurse take FIRST? 1. Decrease the IV rate to 20 mL/h and notify the physician. 2. Decrease the IV rate to 100 mL/h and continue to monitor the client. 3. Discontinue the IV and start oxygen at 6 L/min. 4. Assess for infiltration of the IV solution.

(1) correct—KVO (20 cc/h) will keep access open (2) need to notify physician; rate still too much since patient is in fluid overload (3) IV line may be necessary; diuretics may be ordered (4) description indicates circulatory overload, not infiltration

A child has a closed transverse fracture of the right ulna. Which of the following actions, if performed by the nurse before the application of a cast, is MOST important? 1. Check the radial pulses bilaterally and compare. 2. Evaluate the skin temperature and tissue turgor in the area. 3. Assess sensation of each foot while the child closes her eyes. 4. Apply baby powder to decrease skin irritation under the cast.

(1) correct—assess neurovascular status, check pain, pallor, paralysis, paresthesia, pulselessness (2) assessment; temperature indicates decreased circulation but is subjective and not most important (3) assessment; upper (not lower) extremity fracture (4) implementation; should not be done because it would increase skin irritation

An older woman is hospitalized with a fractured left hip. While awaiting surgery, the client is placed in Buck's traction with a 7-pound weight. Which of the following instructions about moving should be given by the nurse to encourage the patient to participate in her care? 1. "Pull up on the overhead trapeze while you push down on your right foot to lift your body." 2. "With your right arm, grasp the bedside rail on the opposite side and pull yourself over gently." 3. "I'll raise the head of the bed 45 degrees, and then you lean forward and rotate your hips to the left." 4. "Swing your right leg over your left leg and turn from your waist down, keeping your legs straight."

(1) correct—body must move as single, straight unit (2) turning or twisting from the waist down interferes with countertraction (3) prevents proper pull of weights (4) can't turn from side to side; can only move up and down

A mother brings her 10-year-old and 3-year-old daughters to the pediatrician's office because the younger girl complains of dysuria. The physician orders a catheterization to obtain a urine specimen. The nurse should take which of the following actions? 1. Describe the procedure to the child in short, concrete terms while talking calmly. 2. Allow the child to play with the equipment during the procedure. 3. Involve the girl's older sister in explaining the procedure. 4. Show the child a diagram of the urinary system.

(1) correct—children this age need simple explanations (2) might contaminate the equipment; must be a sterile procedure (3) not likely to listen to sister (4) not appropriate for this age

A client is admitted with irritable bowel syndrome. The nurse anticipates that the client's history will reflect which of the following? 1. Pattern of alternating diarrhea and constipation. 2. Chronic diarrhea stools occurring 10 to 12 times per day. 3. Diarrhea and vomiting with severe abdominal distention. 4. Bloody stools with increased cramping after eating.

(1) correct—condition is often called spastic bowel disease; no inflammation is present (2) refers to inflammatory bowel disease such as ulcerative colitis or Crohn's disease (3) refers to inflammatory bowel disease such as ulcerative colitis or Crohn's disease (4) bloody stools do not occur with irritable bowel syndrome

The nurse assists a nursing assistant in providing a bed bath to a comatose patient with incontinence. The nurse should intervene if which of the following actions is noted? 1. The nursing assistant answers the phone while wearing gloves. 2. The nursing assistant log rolls the patient to provide back care. 3. The nursing assistant places an incontinent pad under the patient. 4. The nursing assistant positions the patient on the left side, head elevated.

(1) correct—contaminated gloves should be removed before answering the phone (2) correct way to roll a patient to maintain proper alignment (3) appropriate to use incontinence pad for this patient (4) appropriate position to prevent aspiration and protect the airway

A patient is admitted to the hospital for a hypoglossectomy with lymph node dissection. The patient's preoperative care includes frequent oral hygiene with hydrogen peroxide. The nurse knows the purpose of this treatment includes which of the following? 1. Minimizes the bacterial count in the mouth. 2. Softens the mucous membranes of the tongue before surgery. 3. Stimulates the microcirculation of the mouth. 4. Hydrates the tissues of the gums.

(1) correct—destroys bacteria found in mouth, reduces the chance of infection (2) is not the action of hydrogen peroxide (3) circulation is unaffected by a mouth rinse (4) has slight drying effect on mucous membranes

An 8-year-old has been receiving chemotherapy for 6 months. During her nursing care she asks, "Am I going to die?" Which of the following responses by the nurse is BEST? 1. "Are you afraid of dying?" 2. "Why do you ask that question?" 3. "Only God knows that answer." 4. "We won't leave you alone."

(1) correct—encourages ventilation of thoughts and feelings regarding the concern (2) inappropriate (3) ignores the child's concern with dying (4) ignores the child's concern with dying

A 6-month-old infant has had all of the required immunizations. The nurse knows that this would include which of the following? 1. Two doses of diphtheria, tetanus, and pertussis vaccine. 2. Measles, mumps, and rubella vaccines. 3. A booster dose of the inactivated polio vaccine. 4. Chickenpox and smallpox vaccines.

(1) correct—first dose of the DPT may be given at 2 months of age, the second is given around 4 months (2) MMR is given at 15 months (3) polio is given at 2 and 4 months and again at 12 to 18 months (4) recommended for first responders

A client is admitted for a series of tests to verify the diagnosis of Cushing syndrome. Which of the following assessment findings, if observed by the nurse, support this diagnosis? Select all that apply. 1. Buffalo hump. 2. Intolerance to heat. 3. Hyperglycemia. 4. Hypernatremia. 5. Intolerance to cold. 6. Irritability.

(1) correct—hypersecretion of adrenal hormones; other indications include weight gain, moon face, purple striae, osteoporosis, mood swings, and high susceptibility to infections (2) indication of hyperthyroidism (3) correct—hypersecretion of adrenal hormones; other indications include weight gain, moon face, purple striae, osteoporosis, mood swings, and high susceptibility to infections (4) correct—hypersecretion of adrenal hormones; other indications include weight gain, moon face, purple striae, osteoporosis, mood swings, and high susceptibility to infections (5) indication of hypothyroidism (6) indication of hypoparathyroidism

The nurse teaches nutrition classes at the community center. Which of the following foods should the nurse encourage a low-income client to eat to satisfy essential protein needs? 1. Legumes. 2. Red meat. 3. Seafood. 4. Cheese.

(1) correct—legumes are an economical source rich in protein (2) high in protein, but more expensive to purchase (3) high in protein, but more expensive to purchase (4) high in protein, but more expensive to purchase

The nurse makes patient assignments on the obstetrics unit. Which of the following patients should the nurse assign to an RN who has been reassigned to the obstetrics unit from outpatient surgery? 1. A patient at 16 weeks' gestation admitted with hyperemesis and receiving IV fluids. 2. A patient at 26 weeks' gestation in premature labor and receiving terbutaline (Brethine). 3. A patient at 32 weeks' gestation with a placenta previa and ruptured membranes. 4. A patient at 37 weeks' gestation with pregnancy-induced hypertension and epigastric pain.

(1) correct—monitor IV therapy, administer antiemetics and nutritional supplements (2) monitor patient's response to medication and the status of the fetus (3) prepare for delivery, closely monitor fetal response (4) indicates impending seizures, prepare for delivery

A 25-year-old primigravida diagnosed with type 1 diabetes mellitus reviews the insulin regimen with the nurse. The nurse explains to the client that her insulin needs will change in which of the following ways? 1. Increase during pregnancy and decrease after delivery. 2. Decrease during pregnancy and increase after delivery. 3. Increase during pregnancy and remain increased after delivery. 4. Decrease during pregnancy and fluctuate after delivery.

(1) correct—needs increase during pregnancy due to hormonal interference in glucose metabolism (2) needs increase during pregnancy due to hormonal interference in glucose metabolism (3) insulin needs will decrease after delivery (4) needs increase during pregnancy

The nurse observes the fetal heart monitor for a client in active labor. The fetal heart tracing shows early fetal decelerations. The nurse is aware that this is 1. a slowing early in the contraction, and is usually a normal finding. 2. a slowing early in the contraction, and is usually an abnormal finding. 3. a slowing at the peak of the contraction, and is usually a normal finding. 4. a slowing at the peak of the contraction, and is usually an abnormal finding.

(1) correct—occurs in response to compression of fetal head; uniform shape corresponds to rise in intrauterine pressure as uterus contracts, does not indicate fetal distress (2) does not indicate fetal distress (3) slowing is early in the contraction (4) slowing is early in uterine contraction and is not abnormal

The nurse cares for patients on the psychiatric unit. An extremely angry patient with bipolar illness tells the nurse he just learned his wife has filed for divorce and he needs to use the phone. Which of the following responses by the nurse is MOST appropriate? 1. Allow the patient to use the phone. 2. Confront the patient about his anger and inappropriate plan of action. 3. Do not allow the patient to use the phone because he is an involuntary patient. 4. Set limits on the patient's phone use because he has been unable to control his behavior.

(1) correct—patient is able to use phone unless otherwise indicated by court order or physician's order (2) has not lost civil right to use phone (3) denies patient his civil rights (4) inappropriate

The nurse instructs a client who is receiving imipramine (Tofranil). It is MOST important for the nurse to instruct the client to immediately report which of the following? 1. Sore throat, fever, increased fatigue, vomiting, diarrhea. 2. Dry mouth, nasal stuffiness, weight gain. 3. Rapid heartbeat, frequent headaches, yellowing of eyes or skin. 4. Weakness, staggering gait, tremor, feeling of drunkenness.

(1) correct—possible side effects of Tofranil, a tricyclic antidepressant medication, which can be resolved by altering the dosage or changing the medication (2) describes side effects of antidepressants, which client can learn to manage at home without changing the medication (3) not side effects of Tofranil (4) not side effects of Tofranil

The nurse cares for a client one day after a thoracotomy. Nursing actions listed on the care plan include turn, cough, and deep breathe q 2 h. The nurse understands that the purpose of this nursing action includes which of the following? 1. Promote ventilation and prevent respiratory acidosis. 2. Increase oxygenation and removal of secretions. 3. Increase pH and facilitate balance of bicarbonate. 4. Prevent respiratory alkalosis by increasing oxygenation.

(1) correct—primary purpose of this nursing measure is to improve and/or maintain good gas exchange, especially removal of carbon dioxide in order to prevent respiratory acidosis (2) answer choice #1 is better in that it refers to ventilation rather than oxygenation (3) increasing the pH is not desirable (4) respiratory alkalosis is not prevented by this nursing measure

A patient is treated in the telemetry unit for cardiac disease. The patient receives propranolol hydrochloride (Inderal) 20 mg PO at 9 A.M. When the nurse enters the room to give the medication to the patient, the nurse finds the patient wheezing with a nonproductive cough and shortness of breath. INITIALLY, the nurse should take which of the following actions? 1. Hold the medication and count the respirations. 2. Hold the medication and call the physician. 3. Take an apical pulse and then give the medication. 4. Give the mediation as ordered.

(1) correct—side effects include increased airway resistance; patient is experiencing bronchospasm; should assess and then call the physician (2) should assess the patient's condition first (3) patient is experiencing a side effect; medication should not be given (4) medication should be held; patient is experiencing a side effect

The nurse reviews client assignments on a medical/surgical unit. The nurse determines that the assignment is appropriate if the nursing assistant is caring for which of the following clients? 1. A client with AIDS dementia complex who requires a urine specimen. 2. A client complaining of postoperative pain after repair of a torn rotator cuff. 3. A client with GI bleeding due to a duodenal ulcer who is receiving packed cells. 4. A client with type 1 diabetes receiving prednisone for a herniated disk.

(1) correct—standard, unchanging procedure (2) assign to the RN (3) assign to the RN (4) assign to the RN

A mother brings her 9-month-old infant to the pediatrician's office with complaints of a fever of 102.2°F (39°C) and frequent vomiting. The nurse expects which of the following reflexes to still be present? 1. Babinski's reflex. 2. Moro's reflex. 3. Tonic neck reflex. 4. Grasp reflex.

(1) correct—stroking outer sole of foot upward causes toes to hyperextend and fan and great toe to dorsiflex; disappears after 1 year of age (2) sudden jarring causes extension and abduction of extremities and fanning of fingers with index finger and thumb forming a C shape; disappears after 3 to 4 months (3) when head is turned to side, arm and leg extend on that side, and opposite arm and leg flex; disappears by age 3 to 4 months (4) touching palms of hands or soles of feet causes flexion of hands and toes; palmar grasp disappears after 3 months of age, plantar grasp lessened by 8 months of age

The nurse cares for a client who is receiving a tube feeding around the clock. Which of the following nursing actions is MOST appropriate? 1. Rinse the bag and change the formula every 4 hours. 2. Rinse the bag and change the formula every shift. 3. Change the bag and formula every shift. 4. Rinse the bag and change the formula every 2 hours.

(1) correct—there is an increased growth of organisms after 4 hours (2) inappropriate due to increased organism growth (3) inappropriate due to increased organism growth (4) not a necessary action to maintain asepsis

The nurse cares for a client admitted with a diagnosis of acute hypoparathyroidism. It is MOST important for the nurse to have which of the following items available? 1. Tracheostomy set. 2. Cardiac monitor. 3. IV monitor. 4. Heating pad.

(1) correct—tracheostomy set is the most important for the client's safety due to risk for laryngospasm (2) nice to have, but not the most important (3) nice to have, but not the most important (4) unnecessary

A client has a total laryngectomy with a permanent tracheostomy. The nurse plans nutritional intake for the next 3 days. Which of the following is necessary for the nurse to consider regarding the client's nutrition? 1. To facilitate healing of the surgical area, a nasogastric tube may be utilized and tube feedings may be implemented. 2. The client will be unable to maintain any oral intake as long as the tracheotomy is in place. 3. Nutritional and/or gastric feedings will not be attempted for approximately 3 weeks to decrease the incidence of aspiration. 4. Because the client is dependent on the ventilator, nutritional intake will be delayed.

(1) correct—tube feedings frequently started as the initial nutritional intake; prevents trauma to suture area (2) although client has permanent tracheotomy, will be able to eat normally after area has healed (3) nutritional intake will begin when bowel sounds return and client can tolerate intake (4) client is not dependent on ventilator

The nurse cares for an elderly adult client with multi-infarct dementia. Which of the following actions, if taken by the nurse, is BEST? 1. Place the client in soft hand restraints or chair restraints. 2. Monitor wandering behaviors during a 7-day period. 3. Keep the lounge's television volume on a low level. 4. Encourage a diet high in protein, iron, and vitamins.

(1) do not restrain unless all other options have been exhausted (2) correct—appropriate assessment to determine if client wanders during specific times of the day; assess before implementing (3) need to prevent sensory overload; should assess first (4) offer well-balanced diet

The nurse works with a client who has just indicated a wish to kill herself. The client then asks the nurse not to tell anyone. Which of the following responses by the nurse is BEST? 1. Encourage the client not to do anything without thinking it through very carefully. 2. Explain to the client that anything she tells the nurse is kept strictly confidential. 3. Report this to staff members in order to protect the client. 4. Encourage the client to tell the nurse more about what she is feeling.

(1) does not answer client's immediate concern or give client accurate information about what the nurse will do (2) does not answer client's immediate concern or give client accurate information about what the nurse will do (3) correct—nurse must let the client know that this information will be shared with the staff so that the client's safety can be preserved (4) does not answer client's immediate concern or give client accurate information about what the nurse will do

The nurse cares for a multipara client who delivered a female infant 1 hour ago. The nurse observes that the client's breasts are soft; the uterus is boggy to the right of the midline and 2 cm below the umbilicus; moderate lochia rubra. It is MOST important for the nurse to take which of the following actions? 1. Perform a straight catheterization. 2. Offer the client the bedpan. 3. Put the baby to breast. 4. Massage the uterine fundus.

(1) encourage the client to void before catheterizing (2) correct—boggy uterus deviated to right indicates full bladder, encourage client to void (3) will increase uterine tone, but the problem is a full bladder (4) findings indicate a full bladder

The nurse cares for a client after an electroconvulsive therapy (ECT) treatment. The nurse should report which observation to the client's physician? 1. Headache. 2. Disruption in short- and long-term memory. 3. Transient confusional state. 4. Backache.

(1) expected effect (2) expected effect (3) expected effect (4) correct—client undergoing ECT needs to be instructed about what s/he could experience during and after ECT; expected effects include headache, disrupted memory (short- and long-term), and general confused state; backache is not a usual effect; thorough description of the pain in relation to severity, duration, location, and what makes pain better needs to be assessed and reported to the physician

An 11-year-old boy falls off his bicycle and sustains a minor head injury, which is treated at the outpatient clinic. The nurse instructs the boy's mother about his care at home. The nurse determines that further teaching is necessary if the mother makes which of the following statements? 1. "My son may have dizziness for 24 hours." 2. "My son can drink carbonated beverages if he vomits." 3. "My son may complain of nausea." 4. "My son will probably have a headache."

(1) expected for at least 24 hours (2) correct—vomiting unexpected; should be reported to physician immediately; also unexpected is blurred vision, drainage from ear or nose, weakness, slurred speech, worsening headache (3) expected for at least 24 hours (4) expected for at least 24 hours; should not get more intense

Nursing management prior to an intravenous pyelogram (IVP) would include which of the following? 1. A fat-free meal the evening before the examination and radiopaque tablets at bedtime. 2. Placement of a retention urinary catheter to facilitate dilation of the bladder sphincter. 3. Cleansing enemas the evening before to provide for adequate visualization of the urinary tract. 4. Explaining the importance of following directions regarding voiding during the test.

(1) fat-free meal is associated with a gallbladder series (2) a retention Foley catheter may be in place, but not for the purpose of dilating the bladder sphincter (3) correct—because of the need to visualize the abdominal area, cleansing enemas the evening before an IVP are usually ordered (4) there are few directions the client needs to follow during the test

The nurse cares for an elderly client who has just had a prosthetic hip implant. The nurse should position the client in which of the following positions? 1. With the affected hip internally rotated and flexed. 2. With the affected hip adducted when turned. 3. In the supine position with the knees elevated 90 degrees. 4. Side-lying with the affected hip in a position of abduction.

(1) flexion beyond 60 degrees, adduction and internal rotation should be avoided in the early postoperative period (2) flexion beyond 60 degrees, adduction and internal rotation should be avoided in the early postoperative period (3) flexion beyond 60 degrees, adduction and internal rotation should be avoided in the early postoperative period (4) correct—position of abduction should be maintained

The nurse recognizes that the client diagnosed with an obsessive-compulsive ritual is attempting to achieve which of the following? 1. Control of other people. 2. Increased self-esteem. 3. Avoid severe levels of anxiety. 4. Express and manage anxiety.

(1) inaccurate (2) inaccurate (3) correct—obsessive-compulsive rituals are an attempt to avoid or alleviate increasing levels of anxiety; client is not trying to increase his self-esteem or control others with the ritualistic behaviors; these behaviors do not have a significant impact on others; client does not want to repeat the act but feels compelled to do so (4) ritual is not a method of expressing anxiety but a strategy to avoid it

A patient is returned to the unit after surgery with a cuffed tracheostomy tube in place. The nurse knows that the purpose of the cuff on the tracheostomy tube includes which of the following? 1. Guarantees secure placement of the tracheostomy tube in the airway. 2. Prevents ischemia of the tracheal wall by distributing the pressure applied to it. 3. Decreases the chance of aspiration into the trachea. 4. Protects the trachea from ischemia and edema.

(1) inaccurate, not the purpose of the cuff on a tracheostomy tube (2) complication of using a cuffed tracheostomy tube (3) correct—seals trachea, helps to prevent aspiration (4) trauma from overinflated tube may cause edema

For a client with a neurologic disorder, which of the following nursing assessments is MOST helpful in determining subtle changes in the client's level of consciousness? 1. Client posturing. 2. Glasgow coma scale. 3. Client thinking pattern. 4. Occurrence of hallucinations.

(1) indicates increased intracranial pressure (2) correct—Glasgow coma scale score best evaluates changes in a client's level of consciousness by evaluating eye-opening, motor, and verbal responses (3) more appropriate for the psychiatric client (4) more appropriate for the psychiatric client

Which of the following statements, if made by a client to the nurse, indicates that the client is using the defense mechanism of conversion? 1. "I love my family with all my heart, even though they don't love me." 2. "I was unable to take my final exams because I was unable to write." 3. "I don't believe I have diabetes. I feel perfectly fine." 4. "If my wife was a better housekeeper I wouldn't have such a problem."

(1) indicates reaction formation (2) correct—client has converted his anxiety over school performance into a physical symptom that interferes with his ability to perform (3) indicates denial (4) indicates projection

A client has partial-thickness and full-thickness burns over 75% of his body. The nurse is MOST concerned if which of the following is observed? 1. Epigastric pain. 2. Restlessness. 3. Tachypnea. 4. Lethargy.

(1) insignificant for burn client (2) may be due to pain (3) correct—body responds to early hypovolemic shock by adrenergic stimulation; vasoconstriction compensates for the loss of fluid, resulting in cool, clammy skin, tachycardia, tachypnea, and pale color (4) may be due to pain

The nurse assesses a client with severe bilateral peripheral edema. Which of the following is the BEST way for the nurse to determine the degree of edema in a limb? 1. Measure both limbs with the tape measure and compare. 2. Depress the skin and rank the degree of pitting. 3. Describe the swelling in the affected area. 4. Pinch the skin and note how quickly it returns to normal.

(1) is not the best way to evaluate for peripheral edema (2) correct—severity of edema is characterized by grading it 1+ (2-mm pitting) to 4+ (8-mm pitting) (3) not as objective (4) is used for evaluating hydration

A client diagnosed with bipolar disorder is in a manic phase with combative behavior. Which of the following is the INITIAL priority nursing action? 1. Provide adequate hygiene and nutrition. 2. Decrease environmental stimuli. 3. Slowly involve the client in unit activities. 4. Administer and monitor sedative and mood-stabilizing medications.

(1) is very important to ensure adequate hygiene and nutrition, but behavioral control and client/milieu safety are an initial priority (2) decreasing environmental stimulation is an additional strategy that, when utilized in conjunction with psychopharmacologic intervention, can reduce hyperactivity and aggressive acts; just decreasing environmental stimulation will not diminish client's internal sense of agitation and aggression (3) this action is inappropriate at this time (4) correct—is most important to gain control with a physically aggressive client in manic phase; client has significant sympathetic nervous system stimulation and will require psychopharmacologic intervention with both sedative medications and mood-stabilizing agents

The nurse cares for a patient several days after an above-knee amputation (AKA). Which of the following symptoms are characteristic of an infected residual limb wound? 1. The patient is anxious and restless. 2. There is a small amount of dark drainage on the dressing. 3. The patient complains of persistent pain at the operative site. 4. The skin is cool above the operative site.

(1) may be due to changes in body image or pain (2) expected, not indicative of an infection (3) correct—pain is characteristic of inflammation and infection (4) warm skin above operative site would indicate infection

The nurse cares for a client who presents with confusion, mood lability, impaired communication, and lethargy. The nurse should question which of the following orders? 1. Dexamethasone suppression test. 2. Thyroid studies. 3. Drug toxicology screen. 4. Trendelenburg test.

(1) may be ordered to determine the presence of major depression (2) may be ordered to check for an endocrine cause for the symptoms before the diagnosis of dementia is made (3) may be ordered to see if the client's symptoms are caused by excessive use of medications or alcohol (4) correct—test is used with a client who may have varicose veins, no relationship to the symptoms described in this situation

The nurse caring for a client on suicide precautions makes the following observations: the client is verbalizing other options besides suicide, appears to be responding to antidepressant medication, is sleeping and eating better, and has indicated a willingness to interact more with family members. Based on these data, which of the following nursing actions is MOST appropriate? 1. Recommend that the physician decrease the client's medication dosage. 2. Recommend that the treatment team reevaluate the client's treatment plan. 3. Give the client privileges to walk around the hospital by himself. 4. Ask the family to begin planning for the client's discharge.

(1) may reverse the client's progress (2) correct—data suggest that client is beginning to benefit from treatment; entire treatment team should share data and make a decision about the suicide precautions so that restrictions are changed gradually on the basis of a full-data picture (3) may be the team's decision, but not until a thorough review of the case is completed (4) premature

The physician orders morphine sulfate 8 mg IM q 3 to 4 h for pain PRN. In which of the following situations should the nurse consider withholding the medication until further assessment is completed? 1. The patient complains of acute pain from a partial-thickness burn affecting the lower left leg. 2. The patient's blood pressure is 140/90, pulse is 90, and respiration is 28. 3. The patient's level of consciousness fluctuates from alert to lethargic. 4. The patient exhibits restlessness, anxiety, and cold, clammy skin.

(1) morphine used for moderate to severe pain; medication should be given (2) BP slightly elevated, respirations elevated, may be the result of pain; medication should be given (3) correct—morphine depresses CNS, especially respiratory center in medulla (4) may be the result of pain

The nurse checks for placement of a nasogastric (NG) tube prior to initiating a tube feeding for a client. Which of the following results indicates to the nurse that the tube feeding can begin? 1. A small amount of white mucus is aspirated from the NG tube. 2. The contents aspirated from the NG tube have a pH of 3. 3. No bubbles are seen when the nurse inverts the NG tube in water. 4. The client says he can feel the NG tube in the back of his throat.

(1) mucus may be from lungs (2) correct—stomach contents are acidic (3) not a safe way to check placement (4) not a reliable indication

A neonate weighing 7 lb 4 oz with Apgar scores of 7 and 8 at 1 and 5 minutes, respectively, is admitted to the nursery. Because the infant's mother is diagnosed with a type 1 diabetes, the nurse knows the infant is at GREATEST risk for developing which of the following? 1. Hypovolemia. 2. Hypoglycemia. 3. Hyperglycemia. 4. Cold stress.

(1) no change in blood volume for infant of diabetic mother (2) correct—fetus produces increased insulin to match mother's increased glucose level during pregnancy; infant continues to have high insulin output after birth, resulting in hypoglycemia (3) infant would be at risk of hypoglycemia due to increased insulin production (4) thermal receptors in skin are stimulated due to cold environment; increases metabolic rate; infant needs to maintain normal body temperature while producing minimal amount of heat generated from metabolic processes; not expected with diabetic mother

The nurse cares for patients on the pediatric unit. The mother of a 2-year-old who is one day postoperative tells the nurse, "My child is so restless and overactive." The nurse should take which of the following actions? 1. Direct the LPN/LVN to obtain the child's vital signs. 2. Ask the mother if the child's sutures are still intact. 3. Tell the nursing assistant to take the child for a walk. 4. Check to see when the child last received pain medication.

(1) no indication that there are any problems (2) passing the buck (3) implementation; should first assess (4) correct—young children typically become restless and overactive if in pain; grimacing, clenching teeth, rocking, and aggressive behavior may also be observed

A 2-day-old infant in the newborn nursery does not appear interested in taking formula from the mother or the nurse. An appropriate nursing diagnosis is high risk for 1. impaired swallowing. 2. failure to thrive. 3. fluid volume deficit. 4. altered health maintenance.

(1) no information about swallowing provided with question (2) this is a medical diagnosis, not a nursing diagnosis (3) correct—may become dehydrated (4) not specific for problem described

The nurse prepares a dopamine (Intropin) infusion on a client. Before beginning the infusion the nurse should take which of the following actions? 1. Evaluate the urine output. 2. Obtain the client's weight. 3. Determine the patency of the IV line. 4. Measure pulmonary artery pressures.

(1) not a critical assessment at this time (2) contains correct information, but is not a priority (3) correct—if extravasation occurs, there is sloughing of the surrounding skin and tissue; patent IV line is essential to prevent serious side effects (4) not a critical assessment at this time

A client is diagnosed with obsessive-compulsive disorder manifested by the compulsion of hand-washing. The nurse knows that which of the following BEST describes the client's need for the repetitive acts of hand-washing? 1. Hand-washing represents an attempt to manipulate the environment to make it more comfortable. 2. Hand-washing externalizes the anxiety from a source within the body to an acceptable substitute outside the body. 3. Hand-washing helps the client avoid undesirable thoughts and maintain some control over guilt and anxiety. 4. Hand-washing helps maintain the client in an active state to resist the effects of depression.

(1) not a manipulation on the client's part (2) not an accurate statement regarding the compulsive behavior of this client (3) correct—compulsive behavior is an unconscious attempt to control and/or relieve the tension and anxiety the client is experiencing (4) client is not subject to depression but to high levels of anxiety

A client has a three-way Foley catheter following a transurethral resection. The nurse should rapidly infuse the irrigating solutions if which of the following is observed? 1. The urinary output is increased. 2. Bright-red drainage or clots are present. 3. Dark-brown drainage is present. 4. The client complains of pain.

(1) not a reason to infuse irrigating solution rapidly (2) correct—three-way Foley catheter should be irrigated rapidly when bright-red drainage or clots are present; irrigation rate should be decreased to about 40 gtt/min when the drainage clears (3) not an indication to infuse irrigating solution rapidly (4) not an indication to infuse irrigating solution rapidly

The nurse receives report from the previous shift. Which of the following clients should the nurse see FIRST? 1. A client receiving a blood transfusion who complains of a dry mouth. 2. A client is scheduled to receive heparin and the PTT is 70 seconds. 3. A client is receiving ciprofloxacin (Cipro) and complains of a fine macular rash. 4. A client is receiving IV potassium and complains of burning at the IV site.

(1) not an immediate concern (2) PTT is within normal limits; should give medication (3) correct—indicates hypersensitivity reaction; should stop medication and notify the physician (4) should decrease rate to prevent irritation of the vein, but hypersensitivity reaction requires first attention

The nurse obtains a history from the father of a 6-year-old boy with a history of epilepsy who was admitted with uncontrolled seizures. It is MOST important for the nurse to ask which of the following questions? 1. "What part of the body was affected by the seizure?" 2. "What is the family history of seizure disorders?" 3. "What was your son doing before the seizure?" 4. "How long has it been since his last episode of seizures?"

(1) not most important question (2) should be included in detailed history, but will not prevent an immediate reoccurrence (3) correct—seizure may result from triggering mechanism (loud noise, music, flickering light, prolonged reading, drugs) (4) should be included in detailed history, but will not prevent an immediate reoccurrence

An older client diagnosed with pneumonia is admitted to the medical/surgical unit. The nurse should place the patient in a room with which of the following patients? 1. A 20-year-old in traction for multiple fractures of the left lower leg. 2. A 35-year-old with recurrent fever of unknown origin. 3. A 50-year-old recovering alcoholic with cellulitis of the right foot. 4. An 89-year-old with Alzheimer's disease awaiting nursing home placement.

(1) patients with fractures are considered "clean"; don't place with an infectious patient (2) don't know the cause of the fever (3) correct—generalized nonfollicular infection that involves deeper connective tissue, both patients have infections (4) elderly are high risk for developing pneumonia

A client asks what the difference is between a gastric ulcer and a duodenal ulcer. The nurse's response should be based on which of the following statements? 1. "Gastric ulcers have an increased association with clients who experience increased psychological pressures." 2. "The pain of a duodenal ulcer usually occurs 2 to 4 hours after meals." 3. "Clients with gastric ulcers often gain weight, as food alleviates the pain." 4. "Antacids such as Maalox are seldom prescribed for clients with duodenal ulcers."

(1) refers to duodenal ulcers (2) correct—clients with duodenal ulcers experience pain after meals, e.g., midmorning and midafternoon (3) clients with gastric ulcer may be malnourished because food may cause nausea or vomiting (4) antacids are given to duodenal ulcer clients

The nurse teaches a health class at the local library to a group of senior citizens. Which of the following behaviors should the nurse emphasize to facilitate regular bowel elimination? 1. Avoid strenuous activity. 2. Eat more foods with increased bulk. 3. Decrease fluid intake to decrease urinary losses. 4. Use oral laxatives so that a bowel pattern emerges.

(1) regular exercise program facilitates bowel elimination (2) correct—contained in whole grains, legumes, vegetables, fruits, seeds, nuts, bulk promotes peristalsis (3) normal fluid intake of 1,500 ml/day facilitates bowel elimination (4) laxatives used as last resort because they become habit-forming

The nurse knows that which of the following symptoms is supportive of a diagnosis of Guillain-Barré syndrome? 1. Hemiplegia, hypertension, tachycardia. 2. Respiratory failure, flaccid paralysis, urinary retention. 3. Peripheral edema, hypertension, pulmonary congestion. 4. Diminished reflexes, pain, paresthesia.

(1) relates to a CVA (2) correct—classic symptoms include respiratory failure and flaccidity due to paralysis of the muscles and urinary retention due to loss of sensation (3) relates to pulmonary edema (4) relates to peripheral nerve problems

The nurse observes a student nurse caring for a client. In addition to following standard precautions, the student nurse is wearing a gown and gloves. The nurse determines care is appropriate if the student nurse performs which of the following activities? 1. Gives isoniazid (INH) to a client with tuberculosis. 2. Administers an IM injection to a client with rubella. 3. Delivers a food tray to a client with hepatitis. 4. Changes the dressing for a client with a draining abscess.

(1) requires airborne precautions, particulate respirator (2) requires droplet precautions; nurse should wear a mask (3) requires standard precautions (4) correct—requires contact precautions

The nurse responds to a train derailment. After making an initial assessment, which of the following clients should the nurse see FIRST? 1. A pregnant woman who states that her clothing is wet. 2. A young man with blood pulsating from a cut on the right leg. 3. A preschool child who is screaming and crying uncontrollably. 4. An unconscious woman with the right leg shorter than the left leg.

(1) requires further assessment; could be amniotic fluid or could be urine. (2) correct—indicates arterial bleeding; apply direct pressure; high risk for shock (3) stable patient (4) possible hip fracture; no indication of respiratory difficulty stated

The nurse obtains a client's temperature of 103°F(39.4°C). The nurse knows body compensatory mechanisms include which of the following? 1. Decreased respiratory rate and bradycardia. 2. Normal blood pressure and pulse. 3. Increased respiratory rate and tachycardia. 4. Diaphoresis with cool, clammy skin.

(1) respirations and heart rate will increase with fever (2) blood pressure and pulse usually increase with fever (3) correct—hyperthermia increases the oxygen requirements, which results in faster breathing as well as an increase in the pulse rate (4) diaphoresis may occur, but the skin will be warm

The nurse anticipates which of the following when assessing a client with a diagnosis of a ruptured lumbar disc? 1. Sensation loss in an upper extremity. 2. Clonic jerks in the affected foot. 3. Paresthesia in the affected leg. 4. Chorea in the upper and lower extremities.

(1) results from cervical lesions (2) can occur in a person who has been paralyzed from a spinal cord injury (3) correct—lumbar lesions can cause paresthesia, pain, muscle weakness, and atrophy in the lower extremities (4) is a sign of Huntington chorea, resulting from atrophy of parts of the brain

A client with newly diagnosed type 1 diabetes says to the nurse, "I know that I have to take good care of my feet. When I buy new shoes, is there anything special I should do?" Which of the following responses by the nurse is BEST? 1. "It is best to buy new shoes in the morning." 2. "Have each foot measured every time you buy new shoes." 3. "Buy shoes a half-size larger than your foot size so the fit is roomy." 4. "Buy vinyl shoes because they won't lose their shape easily."

(1) should buy shoes in the afternoon when feet are larger than in the morning (2) correct—feet enlarge with age, break in shoes gradually rather than all at one time, have measurements for shoes taken while standing (feet are larger) (3) buy correct shoe size (4) leather shoes recommended because they "breathe," vinyl could cause foot to perspire and aggravate fungal infections

The nurse cares for clients on a psychiatric unit and is suddenly faced with multiple issues. Which of the following situations require the nurse's IMMEDIATE attention? 1. A client with bipolar disorder walks into the day room in her underwear and begins dancing. 2. A client with depression says to the nurse, "My plan is complete, and I'm ready to go for it." 3. A client recovering from substance abuse complains that another client is harassing him. 4. A client with schizophrenia tells the nurse that it's "God's will" that he destroy the "evil TV."

(1) should remove to quiet area, decrease environmental stimuli (2) correct—could indicate impending suicide; requires immediate follow-up (3) potential suicide is more immediate concern (4) command hallucination; potential suicide takes priority

A client is admitted for regulation of insulin dosage. The client takes 15 units of Humulin N insulin at 8 A.M. every day. At 4 P.M., which of the following nursing observations indicates a complication from the insulin? 1. Acetone odor to the breath, polyuria, and flushed skin. 2. Irritability, tachycardia, and diaphoresis. 3. Headache, nervousness, and polydipsia. 4. Tenseness, tachycardia, and anorexia.

(1) signs of hyperglycemia (2) correct—Humulin N insulin is an intermediate-acting insulin that peaks from 8 to 12 hours after administration; this is when signs and symptoms of hypoglycemia will occur (3) signs of hyperglycemia (4) signs of hyperglycemia

The nurse checks the incision of a patient 48 hours after surgery for a hernia repair. Which of the following findings indicates a possible complication? 1. There is swelling under the sutures. 2. There is crusting around the incision line. 3. The incision line is red. 4. The incision line is approximated.

(1) slight swelling is expected during healing (2) slight crusting of incision line is normal (3) correct—should be pink, not red; indicates possible infection; other signs include increased warmth, tenderness, pain, and purulent or odorous drainage (4) shows healing is taking place

The nurse cares for clients in the medical clinic. A nursing assessment of a client with a hiatal hernia is MOST likely to reveal which of the following? 1. A bulge in the lower right quadrant. 2. Pain at the umbilicus radiating down into the groin. 3. A burning sensation in the midepigastric area each day before lunch. 4. Complaints of awakening at night with heartburn.

(1) suggests an inguinal hernia (2) suggests an inguinal hernia (3) pain usually does not develop during the day with an empty stomach (4) correct—classic symptom of hiatal hernia associated with reflux

A client is admitted to the neurosurgery unit for the removal of a cerebellar tumor. The nurse expects the patient to make which of the following statements about symptoms? 1. "I have been having difficulty with my hearing." 2. "I lose my balance easily." 3. "I can't tell the difference between a sweet and sour taste." 4. "It is not easy for me to remember names and faces."

(1) temporal lobe contains auditory center, loss of hearing would involve CN VIII acoustic (2) correct—cerebellum maintains balance (3) CN IX, glossopharyngeal responsible for differentiation of taste (4) not specific symptom of cerebellum dysfunction

The nurse should include which of the following in a teaching plan for a client receiving tetracycline? 1. Take the medication with milk or antacids to decrease GI problems. 2. The medication should always be taken with meals. 3. Use a maximum-protection sunscreen when outdoors. 4. Crackers and juice will help decrease gastric irritation.

(1) tetracycline should never be taken with milk or antacids because these inhibit the medication's action (2) should take with full glass of water at least 1 hour before or 2 hours after meals (3) correct—because of problems related to photosensitivity, client should wear sunscreen, wide-brimmed hats, and long sleeves when at risk for sun exposure (4) should take with full glass of water at least 1 hour before or 2 hours after meals

The nurse in the outpatient clinic instructs a client diagnosed with right-sided weakness to walk down stairs using a cane. What behavior, if demonstrated by the client, indicates to the nurse that teaching is successful? 1. The client puts the right leg on the step, then the cane, followed by the left leg. 2. The client leads with the cane, followed by the right leg and then the left leg. 3. The client advances the right leg, followed by the left leg and the cane. 4. The client puts the cane on the step and advances the left leg, followed by the right leg.

(1) to go down stairs, advance weak leg and cane first; to go up stairs, advance strong leg, then weak leg and cane (2) correct—to go down stairs, advance cane and weak leg, then strong leg; memory trick: the good goes up, the bad goes down (3) should advance cane and weak leg first (4) weaker leg and cane advance first

The nurse knows that which of these plans is MOST successful in caring for a client with dementia? 1. Teach new skills for adjusting to the aging process. 2. Adjust the environment to meet the client's individual needs. 3. Encourage competitive activities to keep the client physically strong. 4. Provide unstructured activities with frequent changes to increase stimulation.

(1) unable to learn new skills (2) correct—client with dementia does not have cognitive abilities to learn new skills or to adapt; environment must be adapted for client with attention to safety and predictability (3) requires skills the client with dementia does not have (4) requires skills the client with dementia does not have

A client is scheduled for electromyography (EMG). What should the nurse tell the client about the procedure? 1. "Your hair will be carefully washed prior to the procedure." 2. "This is a noninvasive procedure that takes about 30 minutes." 3. "A sedative will be given to you shortly before the procedure." 4. "You will not be allowed to eat 4 to 6 hours before the procedure."

(1) usually performed on the legs (2) correct—electrodes are attached to legs, length of time for impulse transmission is measured (3) may impair test results (4) procedure does not involve general anesthesia or GI system

A teenager diagnosed with anorexia nervosa is admitted to the hospital. In planning to care for the client, the nurse would expect the client to 1. view her appearance as "skinny." 2. be hypoactive and withdrawn. 3. want to talk about and plan her meals. 4. have a close relationship with her mother.

(1) usually view their appearance as fat (2) inaccurate for client with anorexia nervosa (3) correct—display a marked preoccupation with food (4) inaccurate for client with anorexia nervosa

Which observation indicates to the nurse that the client needs further teaching before self-administering insulin? 1. The client draws up the regular insulin first, then the NPH. 2. The client gently rotates the insulin bottle before withdrawing the dose. 3. The client rotates injection sites following the guide on the printed diagram. 4. The client administers the insulin while it is still cold from the refrigerator.

(1) when mixing regular insulin with other types of insulin, the client should draw up the clear (regular) before the cloudy (NPH) (2) bottle of insulin should never be vigorously shaken, but rather gently mixed (3) imperative to rotate injection sites to avoid tissue irritation/infection and ensure proper absorption (4) correct—insulin should be administered at room temperature; temperature extremes should be avoided

A client has been taking propranolol (Inderal) 40 mg BID and furosemide (Lasix) 40 mg daily for several months. Two weeks ago, the physician added verapamil (Calan) 80 mg TID to the client's medication regimen. The client returns to the outpatient clinic for evaluation. It is MOST important for the nurse to assess for which of the following? 1. Tachycardia. 2. Diarrhea. 3. Peripheral edema. 4. Impotence.

(1) will cause bradycardia (2) usually causes constipation (3) correct—Calan is a calcium channel blocker, depresses myocardial contractility, decreases work of ventricles and O2 demand, dilates coronary arteries; when used with other antihypertensives can cause hypotension and heart failure (4) not most important or frequent side effect

A client is scheduled for a myelogram at the outpatient clinic. The physician's office nurse reinforces the physician's explanation of the procedure. Which of the following statements, if made by the nurse, correctly describes a myelogram? 1. "The test involves x-ray examination of the entire spinal column to determine the extent of myelin breakdown." 2. "The test involves injection of a contrast medium into a suspected ruptured vertebral disk, allowing radiographic visualization of the disk." 3. "The test involves a lumbar puncture with injection of contrast medium, allowing x-ray visualization of the vertebral canal." 4. "The test involves x-ray examination of the vertebral column following injection of air into the subarachnoid space."

(1) x-ray examination cannot determine the extent of myelin breakdown (2) no such procedure (3) correct—contrast medium or air is injected into spinal subarachnoid space through a spinal puncture; identifies tumors, cysts, herniated vertebral discs (4) no such procedure

automated peritoneal dialysis

(APD) dialysis machine that controls the fill, dwell, and drain phases, and cycles four to eight exchanges per night with 1 to 2 hours per exchange.

one of the most common charges brought against nurses

...

Normal PR interval time

0.12-0.20 seconds

TSH (Thyroid stimulating hormone or thyrotropin)

0.2 - 5.4 microunits/mL

Therapeutic drug level Lithium (Lithobid)

0.5 - 1.2 mEq/L

Therapeutic drug level Digoxin (Lanoxin)

0.5 - 2 ng/ml

Serum Creatinine

0.6 - 1.3 mg/dL

A nurse is developing a plan of care for a client who is scheduled for surgery. The nurse w/include which of the following activites in the nuring care plan for the client on the day of surgery?

1. Have the client void immediately before surgery. (The nurse w/assist client w/voiding immediately before surgery so that bladder will be empty. Oral hygiene is allowed, but client s/not swallow any water. Client usually has restriction of food and fluids for 8 hrs. before surgery rather than 24 hrs. A slight increase in blood pressure and pulse is common during preoperative period; this is generally result of anxiety.

A nurse monitors a postoperatve client for signs of complications. Which of the following signs w/the nurse determine to be indicative of a potential complication?

1. Increasing restlessness noted in a client is a sign that requires continuous and lose monitoring, because it could be a potential indication of a complication such as hemorhage or shock. Neg Homan's sign is normal. + sign indicative of thrombophlebitis. Faint bowel sounds in all 4 quadrants is normal. BP 120/70, pulse of 90 relativel normal.

A nurse monitors the postoperative client frequently for the presence of secretion in the lungs, knowing that accumulated secretions can lead to:

1. Pneumonia The most common postoperative respiratory problems are atelectasis, pneumonia, and pulmonary emboli. Pneumonia is the inflammation of lung tissue that causes a productive cough, dyspnea, and crackles. Pulmonary edema usually results from L-sided heart failure, and it can be caused by medications, fluid overload, and smoke inhalation. CO2 retention results from the inability to exhale CO2 in clients w/conditions such as COPD. Fluid imbalance can be a deficit or excess related to fluid loss or overload.

The nurse is providing health teaching for a client using herbal compounds such as valerian for sleep. What points need to be included?

1. Should not be used indefinitely 2. May interfere with prescribed medications 3. Over time they can lead to further sleep problems 4. Are not regulated by the Food and Drug Administration (FDA)

Urine Specific Gravity

1.016 - 1.022

Therapeutic drug level Lidocaine (Xylocaine)

1.5 - 5 mcg/ml

Magnesium

1.6 - 2.6 mEq/L

The nurse recognizes that a client is experiencing insomnia when the client reports

1.Difficulty staying asleep 2.Extended time to fall asleep 3.Feeling tired after a night's sleep These symptoms are often reported by clients with insomnia. Clients report nonrestorative sleep. Arising once at night to urinate (nocturia) is not in and of itself insomnia. Falling asleep at inappropriate times is indicative of narcolepsy.

Therapeutic drug level Phenobarbital (Luminal)

10 - 30 mcg/mL

Intradermal injections

10 degree angle, 25-27G needle, 5/8-1/2" needle, barely cover bevel under skin, do not do near a vein, do to the side, used for PPD&allergy testing

Normal intraocular pressure

10 to 21 mm Hg Increased IOP- glaucoma

10. A client with pulmonary edema has been on diuretic therapy. The client has an order for additional furosemide (Lasix) in the amount of 40 mg intravenous push. Knowing that the client will also be started on digoxin (Lanoxin), the nurse should review which laboratory result? 1. Sodium level 2. Digoxin level 3. Creatinine level 4. Potassium level

10. 4 Rationale: The serum potassium level is measured in the client receiving digoxin and furosemide. Heightened digoxin effect leading to digoxin toxicity can occur in the client with hypokalemia. Hypokalemia also predisposes the client to ventricular dysrhythmias. Test-Taking Strategy Use the process of elimination. Eliminate option 2 because the client will just be beginning digoxin therapy. No data indicate the presence of renal insufficiency; therefore, eliminate option 3. Furosemide therapy can cause hyponatremia and hypokalemia, but remember that the risk of hypokalemia has more severe consequences in this situation. Review the nursing considerations related to administering furosemide if you had difficulty with this question.

11. A home health nurse instructs a client about the use of a nitrate patch. The nurse tells the client that which of the following will prevent client tolerance to nitrates? 1. "Do not remove the patches." 2. "Have a 12-hour 'no-nitrate' time." 3. "Have a 24-hour 'no-nitrate' time." 4. "Keep nitrates on 24 hours, then off 24 hours."

11. 2 Rationale: To help prevent tolerance, clients need a 12-hour "no-nitrate" time, sometimes referred to as a pharmacological vacation away from the medication. Options 1, 3, and 4 are incorrect. Test-Taking Strategy: Use the process of elimination, focusing on the subject, preventing tolerance to nitrates. This subject and knowledge regarding administering this medication will direct you to option 2. Review the administration of nitrate patches if you had difficulty with this question.

When to test child for Pb poisoning?

12 months

12. A nurse assesses the sternotomy incision of a client on the third day after cardiac surgery. The incision shows some slight "puffiness" along the edges and is nonreddened, with no apparent drainage. Temperature is 99° F orally. The white blood cell count is 7500 cells/mm3. How should the nurse interpret these findings? 1. Incision is slightly edematous but shows no active signs of infection. 2. Incision shows early signs of infection, although the temperature is nearly normal. 3. Incision shows early signs of infection, supported by an elevated white blood cell count. 4. Incision shows no sign of infection, although the white blood cell count is elevated.

12. 1 Rationale: Sternotomy incision sites are assessed for signs and symptoms of infection, such as redness, swelling, induration, and drainage. Elevated temperature and white blood cell count after 3 to 4 days postoperatively usually indicate infection. Test-Taking Strategy Use the process of elimination. Eliminate options 3 and 4 because the white blood cell count is within normal range. From the remaining options, focus on the data in the question. A nonreddened incision with no apparent drainage indicates no signs of infection. Review the signs of infection if you had difficulty with this question.

12. A client is admitted to a medical unit with nausea and bradycardia. The family hands a nurse a small white envelope labeled "heart pill." The envelope is sent to the pharmacy and it is found to be digoxin (Lanoxin). A family member states, "That doctor doesn't know how to take care of my family." Which of the following statements would convey a therapeutic response by the nurse? 1. "Don't worry about this. I'll take care of everything." 2. "You are concerned your loved one receives the best care." 3. "You're right! I've never seen a doctor put pills in an envelope." 4. "I think you're wrong. That physician has been in practice over 30 years."

12. 2 Rationale: This is a therapeutic, nonjudgmental response. The statement reflects the family's concern but remains nonjudgmental. Option 1 dismisses the family's concerns and disempowers the family. Option 3 creates doubt about the physician's practice without actually knowing the circumstances. Option 4 is argumentative and nontherapeutic. Test-Taking Strategy: Use therapeutic communication techniques. Reflection of the client's or family's concerns is the most therapeutic. Review these techniques if you had difficulty with this question.

Hemoglobin - male adult

14 - 16.5 g/dL

14. A nurse is preparing to ambulate a client on the third day after cardiac surgery. The nurse would plan to do which of the following to enable the client to best tolerate the ambulation? 1. Remove telemetry equipment. 2. Provide the client with a walker. 3. Premedicate the client with an analgesic. 4. Encourage the client to cough and deep breathe.

14. 3 Rationale: The nurse should encourage regular use of pain medication for the first 48 to 72 hours after cardiac surgery because analgesia will promote rest, decrease myocardial oxygen consumption resulting from pain, and allow better participation in activities such as coughing, deep breathing, and ambulation. Options 2 and 4 will not help in tolerating ambulation. Removal of telemetry equipment is contraindicated unless prescribed. Test Taking Strategy Use the process of elimination. Focus on the subject, how best to tolerate the ambulation. Coughing and deep breathing will not actively help endurance, so eliminate option 4. Removal of telemetry equipment is contraindicated unless ordered. From the remaining options, focusing on the subject will direct you to option 3. Review comfort measures for the client following cardiac surgery if you had difficulty with this question.

18. A nurse notices frequent artifact on the electrocardiographic monitor for a client whose leads are connected by cable to a console at the bedside. The nurse examines the client to determine the cause. Which of the following items is unlikely to be responsible for the artifact? 1. Frequent movement of the client 2. Tightly secured cable connections 3. Leads applied over hairy areas 4. Leads applied to the limbs

18. 2 Rationale: Motion artifact, or "noise," can be caused by frequent client movement, electrode placement on limbs, and insufficient adhesion to the skin, such as placing electrodes over hairy areas of the skin. Electrode placement over bony prominences also should be avoided. Signal interference also can occur with electrode removal and cable disconnection. Test-Taking Strategy Use the process of elimination, focusing on the subject, artifact and note the strategic word unlikely. Recalling the causes of artifact will direct you to option 2. Review these causes if you had difficulty with this question.

31.The nurse prepares a dose of a medication ordered by the subcutaneous route and calculates the dose to be 4.5 mL. What is the first nursing action that the nurse should take?</question> <question_image/> <choice_1>Verify the written order.</choice_1> <choice_2>Call the physician.</choice_2> <choice_3>Call the pharmacist.</choice_3> <choice_4>Ask another nurse to check the dosage calculation.</choice_4>

1</correct> <rationale>If there is confusion related to a medication order, refer to and verify the original written order. Be careful to read abbreviations and dosage correctly. Asking another nurse or the pharmacist, or calling the physician are correct interventions, but not the first intervention, because the first step in the medication process is the writing of the order. Once that is verified, the nurse could choose any of the other options, which are correct.</rationale>

17. A nurse is working for an HMO and caring for one of the members. The member asks if he can see any doctor of his choice in the community. The nurse explains: <choice_1>"You may see any doctor of your choice but there will be additional costs if the doctor is outside the HMO."</choice_1> <choice_2>"No, you must see a doctor within the HMO."</choice_2> <choice_3>"No, you must see a doctor within the HMO or you will cancel your current health insurance policy."</choice_3> <choice_4>"Yes, you may see anyone you want."</choice_4>

1</correct> <rationale>Client/member freedom of choice cannot be taken away. However, HMOs require members to stay with doctors within the network unless they wish to pay all or a larger part of the cost of going outside the system. Options 2 and 3 would remove the client/member's freedom of choice and option 4 would be incorrect.</rationale>

25. Which of the following snacks should the nurse offer the hospitalized toddler?</question> <question_image/> <choice_1>Crackers</choice_1> <choice_2>Peanuts</choice_2> <choice_3>Grapes</choice_3> <choice_4>Cereal bar</choice_4>

1</correct> <rationale>Crackers are a soft consistency when chewed and swallowed. Toddlers can easily choke on small foods such as peanuts, popcorn, and grapes, and on firm consistency foods such as cereal bars.</rationale>

6.>The nurse is participating in a seminar about legal and ethical practice of nursing for continuing education credit. Which statement by a nurse best describes the relationship between law and ethics for the practice of nursing? <choice_1>"The ethics of a discipline attempt to formulate and justify responses to moral dilemmas and may or may not be regulated by law."</choice_1> <choice_2>"Laws dictate the ethics of nursing as they reflect societal choices about the ordering of relationships in society."</choice_2> <choice_3>"Ethics represent the moral customs of an individual nurse; therefore, they cannot be regulated by the law."</choice_3> <choice_4>"Ethical practice decreases the threat of a lawsuit, which is the primary source of legal influence on nursing practice."</choice_4>

1</correct> <rationale>Law is not the sole source of the ethical practice of nursing; numerous legal sources influence nursing practice. An individual should understand the ethics of a profession before becoming a member of that profession because those ethics may differ from personal ones.</rationale>

3. A new graduate nurse orientee plans to show an adolescent client a video about self-injection technique. A staff nurse remarks, "I gave the client written literature yesterday, so the video probably isn't necessary." The nurse orientee proceeds with showing the video and discussing the skill with the adolescent after engaging in decision making related to which of the following? 1. Autonomy 2. Informed consent 3. Paternalism 4. Noncompliance

1</correct> <rationale>The nurse is exercising autonomy, the right to make one's own decision. Nurses who follow this principle recognize that each client is unique. In this situation, perhaps because of the developmental level, the nurse assessed that a video would be a better teaching-learning method than written literature. Paternalism restricts the freedom of the individual because another determines choices. Noncompliance occurs when an individual is fully aware of the consequences yet chooses the action anyway. Informed consent is providing agreement to undergo treatment following a description of a procedure with the risks, benefits, and alternatives explained.

40.A major portion of a construction project has collapsed. The emergency department (ED) has been notified that numerous victims are being transported to the ED. The first action of the ED nurses should be which of the following?</question> <question_image/> <choice_1>Assess the department for resources—staff, beds, equipment.</choice_1> <choice_2>Implement the personnel recall system.</choice_2> <choice_3>Discharge stable clients.</choice_3> <choice_4>Set up a temporary morgue.</choice_4>

1</correct> <rationale>The nurses must first assess current emergency department resources. No decisions can be made without a comprehensive assessment, such as outlined in option 1. The other options are not as encompassing, and a comprehensive assessment is needed with a possible impending disaster.</rationale>

21. The nurse is starting the first round of data collection at the beginning of the shift. The first room the nurse enters belongs to a client with a diagnosis of unstable diabetes. The nurse finds the client confused, cold and clammy, and having difficulty breathing. The first priority of care would be to:</question> <question_image/> <choice_1>Deliver oxygen.</choice_1> <choice_2>Test the client's blood sugar.</choice_2> <choice_3>Call the doctor.</choice_3> <choice_4>Run down the hall trying to find help.</choice_4>

1</correct> <rationale>Remembering the ABCs, the first priority is always breathing. Once oxygen has been applied, testing blood sugar, summoning help, and having someone call the physician would all be appropriate care measures.</rationale>

5.An individual has a seizure while walking down the street. During the seizure, a nurse from a physician's office is noticed driving past without stopping to assist. The individual sues the nurse for negligence but fails to win a judgment for which of the following reasons <choice_1>The nurse had no duty to the individual.</choice_1> <choice_2>The nurse did what most nurses would do in the same circumstance.</choice_2> <choice_3>The nurse did not cause the client's injuries.</choice_3> <choice_4>The nurse was off-duty at that time.</choice_4>

1</correct> <rationale>The nurse must have a relationship with the client that involves providing care. The relationship is usually a component of employment. Options 2 and 4 are false. Option 3 is a true statement, but is not the one that applies to this case.</rationale>

49.The nurse is leaving the room of a client who has methicillin-resistant <i>Staphylococcus aureus</i> (MRSA) microorganisms in a wound and the urine. Place the following personal protective equipment in order of removal.</question> <question_image/> <choice_1>Eye protection</choice_1> <choice_2>Gloves</choice_2> <choice_3>Mask</choice_3> <choice_4>Gown</choice_4>

2, 3, 4, 1</correct> <rationale>Gloves are removed first because they would be most contaminated. The mask would be removed next, followed by the gown. Eye protection is removed last, followed by washing the hands.</rationale>

24. When planning for discharge from the birthing center on the following day, the nurse learns that the father will drive the new mother and infant home. When teaching the new parents about infant restraint systems, the nurse should include that the restraint system be (select all that apply):</question> <question_image/> <choice_1>Forward facing</choice_1> <choice_2>Rear facing</choice_2> <choice_3>In the back seat</choice_3> <choice_4>In the front seat</choice_4> <choice_5>Of a bland or neutral color</choice_5>

2, 3</correct> <rationale>A child restraint system should always be in the back seat and rear facing. After a child is 1 year of age and 20 pounds, the seat may be in the rear and front facing. Although bright colors are stimulating to an infant, the color of the system does not matter.</rationale>

46.Which of the following actions by the nurse comply with core principles of surgical asepsis? Select all that apply.</question> <question_image/> <choice_1>Wash hands before and after client care.</choice_1> <choice_2>Keep the sterile field in view at all times.</choice_2> <choice_3>Wear personal protective equipment.</choice_3> <choice_4>Add contents to the sterile field holding the package 6 inches above the field.</choice_4> <choice_5>Consider the outer 1.5 inches of the sterile field as contaminated.</choice_5>

2, 4</correct> <rationale>Options 2 and 4 are core principles of surgical asepsis. Options 1 and 3 are core principles of medical asepsis. Option 5 is an incorrect principle of surgical asepsis. The outer 1 inch of a sterile field is considered contaminated.</rationale>

2. A client develops atrial fibrillation with a ventricular rate of 140 beats/min and signs of decreased cardiac output. Which of the following medications should the nurse first anticipate administering? 1. Atropine sulfate 2. Warfarin (Coumadin) 3. Lidocaine (Xylocaine) 4. Metoprolol (Lopressor)

2. 4 Rationale: b-Blockers such as metoprolol slow conduction of impulses through the AV node and decrease the heart rate. In rapid atrial fibrillation, the goal first is to slow the ventricular rate and improve the cardiac output and then attempt to restore normal sinus rhythm. Test-Taking Strategy: Use the process of elimination. Eliminate option 1 because atropine sulfate will further increase the heart rate and will further decrease the cardiac output. Eliminate option 3 because lidocaine is only useful in suppressing ventricular dysrhythmias. Although warfarin (Coumadin) is administered to clients with atrial fibrillation to prevent clots from forming in the atria it will have no effect in decreasing the ventricular rate or restoring normal sinus rhythm. Review these medications if you had difficulty with this question.

A nurse is checking a client's surgical incision and notes an increase in the amount of drainage, a separation of the incision line, and the appearance of underlying tissue. Which of the following is the intial action?

2. Apply sterile dressing soaked w/normal saline to the wound. Wound dehiscence is the separation of the wound edges at the suture line. S/sx include increased drainage and appearance of underlying tissues. It usually occurs as a complication 6-8 days. The client s/be instructed to remain quiet and to avoid coughing or straining, and he or she s/be positioned to prevent further stress on the wound. Sterile dressings soaked w/sterile normal saline s/be used to cover the wound. The physician needs to be notified.

A nurse obtains vital signs on a postoperative client who just returned to nursing unit. The client's BP is 100/60 mm Hg, the pulse 90 beats/min., and respiration rate is 20 breaths/min.. On basis of these findings,which of the following nursing actions s/be performed?

2. Continue to monitor vital signs. A slightly lower-than-normal BP and increased pulse rate are common after surgery. Warm blankets are applied to maintain the client's body temp. The level of consciousness can be determined by checking the client's response to light touch and verbal stimuli rather than by shaking the client. There is no reason to contact the RN immediately.

A nurse checks the client's surgical incision for signs of infection. Which of the following w/be indicative of a potential infection?

2. The presence of purulent drainage. S/Sx of a wound infection include warm, red, and tender skin around the incision. The client may have fever and chills. Purulent material may exit from drains or from separated wound edges. Infection may be caused by poor aseptic technique or a wound that was contaminated before surgical exploration; it appears 3-6 days after surgery. Serous drainage is not indicative of a wound infection. A temp. of 98.8 F is not an abnormal finding in a postoperative client. The fact that a client feels cold is not indicative of an infection, although chills and fever are signs of infection.

20. A client has frequent bursts of ventricular tachycardia on the cardiac monitor. Why should the nurse be most concerned about with this dysrhythmia? 1. It can develop into ventricular fibrillation at any time. 2. It is almost impossible to convert to a normal rhythm. 3. It is uncomfortable for the client, giving a sense of impending doom. 4. It produces a high cardiac output that quickly leads to cerebral and myocardial ischemia.

20. 1 Rationale: Ventricular tachycardia is a life-threatening dysrhythmia that results from an irritable ectopic focus that takes over as the pacemaker for the heart. The low cardiac output that results can lead quickly to cerebral and myocardial ischemia. Clients frequently experience a feeling of impending doom. Ventricular tachycardia is treated with antidysrhythmic medications, cardioversion (client awake), or defibrillation (loss of consciousness). Ventricular tachycardia can deteriorate into ventricular fibrillation at any time. Test-Taking Strategy Use the process of elimination and note the strategic words most concerned. Option 2 is incorrect and is eliminated first. From the remaining options, focusing on the strategic words will direct you to option 1 because this option identifies the life-threatening condition. Review the concerns associated with ventricular tachycardia if you had difficulty with this question.

41.The nurse should explain to the mother of a 12-month-old infant that a forward-facing infant seat is safest once the infant weighs at least how many pounds? Fill in the number.

20</correct> <rationale>The infant must weigh at least 20 pounds in order to be safe in a forward-facing infant seat and must be 1 year or older.</rationale>

21. A client is at risk for pulmonary embolism and is on anticoagulant therapy with warfarin sodium (Coumadin). The client's prothrombin time is 20 seconds, with a control of 11 seconds. How would the nurse interpret these results? 1. Client needs to have test repeated. 2. Client results are within the therapeutic range. 3. Client results are higher than the therapeutic range. 4. Client results are lower than the needed therapeutic level.

21. 2 Rationale: The therapeutic range for prothrombin time is 1.5 to 2 times the control for clients at high risk for thrombus. Based on the client's control value, the therapeutic range for this individual would be 16.5 to 22 seconds. Therefore the result is within the therapeutic range. Test-Taking Strategy: Use the process of elimination. Look at the control value. Remembering that the purpose of anticoagulant therapy is to prolong clotting times will assist in eliminating options 3 and 4. Eliminate option 1, because there is no basis for repeating the test. Because the prothrombin value identified in the question is not even double the control, select option 2 from the remaining options. Review the therapeutic prothrombin level for a client at risk for pulmonary embolism if you had difficulty with this question.

21. A nurse is caring for a client with unstable ventricular tachycardia. The nurse instructs the client to do which of the following, if prescribed, during an episode of ventricular tachycardia? 1. Lie down flat in bed. 2. Remove any metal jewelry. 3. Breathe deeply, regularly, and easily. 4. Inhale deeply and cough forcefully every 1 to 3 seconds.

21. 4 Rationale: Cough cardiopulmonary resuscitation (CPR) sometimes is used in the client with unstable ventricular tachycardia. The nurse tells the client to use cough CPR, if prescribed, by inhaling deeply and coughing forcefully every 1 to 3 seconds. Cough CPR may terminate the dysrhythmia or sustain the cerebral and coronary circulation for a short time until other measures can be implemented. Options 1, 2, and 3 will not assist in terminating the dysrhythmia. Test-Taking Strategy To answer this question, you must be familiar with the treatment for unstable ventricular tachycardia. Remember that cough CPR sometimes is used in the client with unstable ventricular tachycardia. Review the concept of cough CPR if you are not familiar with it.

22. A client is having frequent premature ventricular contractions. A nurse would place priority on assessment of which of the following? 1. Sensation of palpitations 2. Causative factors, such as caffeine 3. Precipitating factors, such as infection 4. Blood pressure and oxygen saturation

22. 4 Rationale: Premature ventricular contractions can cause hemodynamic compromise. The shortened ventricular filling time with the ectopic beat leads to decreased stroke volume and, if frequent enough, to decreased cardiac output. The client may be asymptomatic or may feel palpitations. Premature ventricular contractions can be caused by cardiac disorders, states of hypoxemia or by any number of physiological stressors, such as infection, illness, surgery, or trauma, and by intake of caffeine, nicotine, or alcohol. Test-Taking Strategy Note the strategic words priority on assessment. Use the ABCs—airway, breathing, and circulation—to direct you to option 4. Review the effects of premature ventricular contractions if you had difficulty with this question.

23. A client has developed atrial fibrillation, with a ventricular rate of 150 beats/min. The nurse should assess the client for which associated signs or symptoms? 1. Flat neck veins 2. Nausea and vomiting 3. Hypotension and dizziness 4. Hypertension and headache

23. 3 Rationale: The client with uncontrolled atrial fibrillation with a ventricular rate more than 100 beats/min is at risk for low cardiac output because of loss of atrial kick. The nurse assesses the client for palpitations, chest pain or discomfort, hypotension, pulse deficit, fatigue, weakness, dizziness, syncope, shortness of breath, and distended neck veins. Test-Taking Strategy Use the process of elimination. Flat neck veins are normal or indicate hypovolemia, so eliminate option 1. Nausea and vomiting (option 2) are associated with vagus nerve activity and do not correlate with a tachycardic state. From the remaining options, think of the consequences of falling cardiac output to direct you to option 3. Review the effects of atrial fibrillation if you had difficulty with this question.

23. A client is on enalapril (Vasotec) for the treatment of hypertension. The nurse teaches the client that he should seek emergent care if he experiences which adverse effect? 1. Nausea 2. Insomnia 3. Dry cough 4. Swelling of the tongue

23. 4 Rationale: Enalapril (Vasotec) is an angiotensin-converting enzyme inhibitor. Angioedema is an adverse effect. Swelling of the tongue and lips can result in airway occlusion. Nausea, insomnia, and a cough can occur as side (not adverse) effects of the medication. Test-Taking Strategy: Note the strategic word adverse. Use the ABCs—airway, breathing, and circulation—to direct you to option 4. Review the adverse effects of this medication if you had difficulty with this question.

24. Which of the following would be an expected outcome of nesiritide (Natrecor) administration? 1. Client will have an increase in urine output. 2. Client will have an absence of dysrhythmias. 3. Client will have an increase in blood pressure. 4. Client will have an increase in pulmonary capillary wedge pressure.

24. 1 Rationale: Nesiritide is a recombinant version of human B-type natriuretic peptide, which vasodilates arteries and veins. It is used for the treatment of decompensated heart failure, increases renal glomerular filtration, and increases urine output. Options 2, 3, and 4 are incorrect. Test-Taking Strategy: Use the process of elimination. Nesiritide does not have antidysrhythmic properties. Dysrhythmias may be a side effect of the medication, so option 2 should be eliminated. Eliminate option 3 because the medication is a vasodilator and causes a decrease in blood pressure. Eliminate option 4 because the medication decreases pulmonary capillary wedge pressure (PCWP). Review the effects of this medication if you had difficulty with this question.

Therapeutic drug level Amikacin (Amikin)

25 - 30 mcg/mL

25. A client with rapid rate atrial fibrillation asks a nurse why the physician is going to perform carotid sinus massage. Which of the following would be reflective of a correct explanation provided by the nurse? 1. The vagus nerve slows the heart rate. 2. The diaphragmatic nerve slows the heart rate. 3. The diaphragmatic nerve overdrives the rhythm. 4. The vagus nerve increases the heart rate, overdriving the rhythm.

25. 1 Rationale: Carotid sinus massage is one maneuver used for vagal stimulation to decrease a rapid heart rate and possibly terminate a tachydysrhythmia. The others include inducing the gag reflex and asking the client to strain or bear down. Medication therapy often is needed as an adjunct to keep the rate down or maintain the normal rhythm. Options 2, 3, and 4 are incorrect descriptions of this procedure. Test-Taking Strategy Knowledge of anatomy and physiology alone may be sufficient to answer this question. Eliminate options 3 and 4 because a rapid rate dysrhythmia would need to be slowed. Recalling the functions of the vagus nerve and the diaphragmatic nerve will direct you to option 1. The vagus nerve affects heart rate. The diaphragmatic nerve affects respiration. If you are unfamiliar with the functions of these nerves, review this content.

25. A client is admitted to a hospital with acute myocardial infarction and is started on tissue plasminogen activator (tPA, Activase) by infusion. Of the following parameters, which one would a nurse determine requires the least frequent assessment to detect complications of therapy with tissue plasminogen activator? 1. Neurological signs 2. Presence of bowel sounds 3. Blood pressure and pulse 4. Complaints of abdominal and back pain

25. 2 Rationale: Thrombolytic agents dissolve existing clots, and bleeding can occur anywhere in the body. The nurse monitors for any obvious signs of bleeding and also for occult signs of bleeding, which would include hemoglobin and hematocrit values, blood pressure and pulse, neurological signs, assessment of abdominal and back pain, and the presence of blood in the urine or stool. Test-Taking Strategy: Note the strategic words least frequent assessment. Remember that bleeding is the primary complication of thrombolytic therapy. Therefore, look for the option that is not related to bleeding. A change in neurological signs could indicate cerebral bleeding, abdominal and back pain could indicate abdominal bleeding, and change in blood pressure and pulse could be general indicators of hemorrhage. The presence of bowel sounds is unrelated to this medication. Review nursing considerations for the client receiving tissue plasminogen activator if you had difficulty with this question.

27. A nurse is preparing to defibrillate a client in ventricular fibrillation. After placing the paddles on the client's chest and before discharging them, which of the following should be done? 1. Ensure that the client has been intubated. 2. Set the defibrillator to the "synchronize" mode. 3. Administer lidocaine hydrochloride (Xylocaine). 4. Confirm that the rhythm is actually ventricular fibrillation.

27. 4 Rationale: Until the defibrillator is attached and charged, the client is resuscitated by using cardiopulmonary resuscitation. Once the defibrillator has been attached, the electrocardiogram is checked to verify that the rhythm is ventricular fibrillation or pulseless ventricular tachycardia. Leads also are checked for any loose connections. A nitroglycerin patch, if present, is removed. The client does not have to be intubated to be defibrillated. Lidocaine may be given subsequently but is not required before defibrillation. The machine is not set to the synchronous mode because there is no underlying rhythm with which to synchronize. Test-Taking Strategy Use the process of elimination, focusing on the subject, ventricular fibrillation. Note that option 4 directly addresses this subject and also addresses assessment of the client. Review the procedure for defibrillation if you had difficulty with this question.

47.Which of the following precautions would the nurse implement when admitting a client to the nursing unit with herpes zoster?</question> <question_image/> <choice_1>Airborne precautions</choice_1> <choice_2>Contact precautions</choice_2> <choice_3>Droplet precautions</choice_3> <choice_4>Neutropenic precautions</choice_4>

2</correct> <rationale>Herpes zoster is caused by the herpes virus varicella zoster. It can be transmitted by direct contact with the client. It is not transmitted via droplets or air currents. Neutropenic precautions are not indicated because the client is not at risk for contracting an infection from the nurse or other individuals.</rationale> <cognitive_level>Application</cognitive_level>

33.Which of the following medication orders should the nurse question?</question> <question_image/> <choice_1>Morphine sulfate (Morphine) 4mg IV every 3 to 4 hours as needed for pain</choice_1> <choice_2>Ceftriaxone (Rocephin) IVPB every 8 hours</choice_2> <choice_3>Furosemide (Lasix) 40 mg po daily</choice_3> <choice_4>Metoprolol (Lopressor) 50 mg po twice a day</choice_4>

2</correct> <rationale>Option 2 does not have a medication dosage listed. All other options have required information for dispensing medications.</rationale>

19.The nurse is working in a very busy long-term care facility and recognizes the importance of good time management skills to the delivery of safe nursing care. The nurse recognizes the goal of time management is to:</question> <question_image/> <choice_1>Find more time to accomplish care delivery.</choice_1> <choice_2>Use time wisely by setting a reasonable amount of time to spend on tasks.</choice_2> <choice_3>Manage time perfectly from the first day of the new job.</choice_3> <choice_4>Do the best job possible without worrying about priorities.

2</correct> <rationale>The goal of time management is not finding more time, but using time more wisely by setting a reasonable amount of time to spend on individual tasks. It takes practice and doesn't occur on the first day of the new job. One aspect of time management is setting priorities.</rationale>

7. A female client being treated in an outpatient setting for blood clots in the leg is taking anticoagulant medication. The client reports to her neighbor, a nurse, that she has a headache. The nurse offers the individual aspirin for the headache, which she takes. The client suffers a bleeding episode secondary to interaction between the aspirin and the anticoagulant. The legal nurse consultant interprets that which of the following elements of malpractice is missing from this case? <choice_1>Breech of duty</choice_1> <choice_2>Duty owed</choice_2> <choice_3>Injury</choice_3> <choice_4>Causation between nurse's action and injury</choice_4>

2</correct> <rationale>In this situation, there was no nurse-client relationship. Although the neighbor offering the aspirin was a nurse, this action did not occur as a component of the nurse's employment. All of the other requirements were present.

22. The nurse determines a new mother is in greatest need of more education about infant care and safety when the mother states:</question> <question_image/> <choice_1>"I am pretty sure that I am going to breastfeed my baby."</choice_1> <choice_2>"After feeding, the baby should be put on her tummy to prevent choking."</choice_2> <choice_3>"Solid foods are not necessary during the baby's first 4 to 6 months."</choice_3> <choice_4>"My baby will sleep frequently and should be awakened every 3 to 4 hours for feeding."</choice_4>

2</correct> <rationale>Infants should always be put to sleep on the back. Options 1, 3, and 4 are correct statements related to infant care and therefore pose no risk to the infant and no concern to the nurse.

9. The physician orders a medication in a dose that is considered toxic. The nurse gives the medication to the client, who later suffers a cardiac arrest and dies. Which of the following consequences can the nurse expect? <choice_1>The doctor, not the nurse, can be charged with negligence because the doctor ordered the dose.</choice_1> <choice_2>The nurse and the doctor can dually be charged with negligence.</choice_2> <choice_3>Because the nurse actually gave the medication, only the nurse can be charged with negligence.</choice_3> <choice_4>Negligence will not be charged, as this event could happen to any reasonable person.</choice_4>

2</correct> <rationale>Nurses, along with physicians, can be charged with negligence for failing to recognize the incorrectly prescribed dosage of a commonly known drug. The other responses are incorrect interpretations of possible consequences

26. What is the best method for the nurse to use to encourage the use of bicycle helmets by school-aged children?</question> <question_image/> <choice_1>Advocate for legislation on helmet laws.</choice_1> <choice_2>Teach parents to role-model helmet use while riding bicycles.</choice_2> ' <choice_3>Verbally reprimand children who report not wearing helmets while riding.</choice_3> <choice_4>Recommend the parents purchase stylish helmets to increase compliance.</choice_4>

2</correct> <rationale>Parent role models of behavior are the best method to develop good habits in children. The other options, although possibly valid (except option 3), are not the <i>best</i> answer.</rationale>

12. Which of the following tasks would not be appropriate for the LPN/LVN to accept when delegated by the registered nurse (RN)? <choice_1>Instructing the LPN/LVN to reinforce teaching of the RN's assigned clients prior to discharge</choice_1> <choice_2>Administering IV push morphine sulfate to the client with post-operative pain</choice_2> <choice_3>Removing a dressing from a postoperative client's abdomen</choice_3> <choice_4>Collecting and documenting vital signs and reporting changes to the RN</choice_4>

2</correct> <rationale>The decision to delegate should be consistent with the nursing process (appropriate assessment, planning, implementation, and evaluation). The person responsible for client assessment, diagnosis, care planning, and evaluation is the registered nurse. LPN/LVN functions include reinforcing teaching and removal of dressings. However, LPN/LVNs are not allowed to administer IV push medications and should refuse this assignment as it falls outside of their scope of practice.

1 T

3 tsp

37. A client on the hospital unit has fallen. Place the following nursing interventions in order of priority.</question> <question_image/> <choice_1>Identify all witnesses.</choice_1> <choice_2>Call the physician.</choice_2> <choice_3>Monitor and provide urgent care.</choice_3> <choice_4>Notify the house supervisor.</choice_4> <choice_5>Fill out the incident report.</choice_5>

3, 4, 2, 1, 5</correct> <rationale>The primary action of the nurse is emergency assessment and first aid. If the nurse contacts the nursing supervisor, there will be nursing help to contact the physician and speak with witnesses. After caring for the client and assessing the situation, the nurse is prepared to fill out the incident report.</rationale>

16. An LPN/LVN is about to make first rounds after receiving an intershift report at 3 p.m. In what order should the LPN/LVN see the following clients? Fill in the order below <choice_1>A 54-year-old client 4 hours post-cardiac catheterization who has mild discomfort at the access site</choice_1> <choice_2>A client newly diagnosed with diabetes mellitus who needs reinforcement of sick day management guidelines</choice_2> <choice_3>A client who arrived 30 minutes ago from the postanesthesia care unit</choice_3> <choice_4>A client who is ready for discharge but will not have transportation to home available until 5 p.m.</choice_4> <choice_5>A client with pneumonia who has received two doses of IV antibiotics and has an oxygen saturation of 93%</choice_5>

3, 5, 1, 4, 2</correct> <rationale>Priority setting can be implemented using a variety of models. The client who is postoperative should be seen first because the client is newly arrived on the unit and is at most risk of becoming unstable or experiencing a change in clinical condition. The client with pneumonia should be seen next because the infection involves the airway, although oxygen saturation levels are higher than the critical value of 90% or less. The client who is 4 hours post-cardiac catheterization should be seen next to evaluate the site and conduct general examination of the affected extremity. The client who will be discharged should be seen next to determine that there are no last minute needs or issues. The client who needs teaching should be seen last because this is not a physiological need.</rationale> <cognitive_ll>Analysis</cognitive_level> <client_need>Safe Effective Care Environment: Coordinated Care</client_need> <integrated_process>Nursing Process: Planning</integrated_process> <content_area>Leadership/Management</content_area> <strategy>Determine which client is at most risk of becoming unstable to pick client 3, followed by monitoring the client whose airway is potentially at risk (client 5). The client with the cardiac catheter could become unstable but has been on the unit for 4 hours, so this client can be seen third. The fourth client scheduled for discharge should be checked fourth, because of time—it will not take long to address any remaining issues or concerns. The client needing teaching will need the most time and can be planned for last.</strategy> <special_handling>0</special_handling> </record>

3. A client with myocardial infarction has been transferred from a coronary care unit to a general medical unit with cardiac monitoring via telemetry. A nurse plans to allow for which of the following client activities? 1. Strict bed rest for 24 hours after transfer 2. Bathroom privileges and self-care activities 3. Ad lib activities because the client is monitored 4. Unsupervised hallway ambulation with distances under 200 feet

3. 2 Rationale: On transfer from the coronary care unit, the client is allowed self-care activities and bathroom privileges. Supervised ambulation in the hall for brief distances is encouraged, with distances gradually increased (50, 100, 200 feet). Test-Taking Strategy Use the process of elimination. Eliminate options 3 and 4 first because they are excessive, given that the client has just been transferred from the coronary care unit. Option 1 is not appropriate because the client would be doing less activity than in the coronary care unit before transfer. Review activity prescriptions for the client with a myocardial infarction if you had difficulty with this question.

3. In reviewing the medication records of the following group of clients, the nurse determines that which client would be at greatest risk for developing hyperkalemia? 1. Client receiving furosemide (Lasix) 2. Client receiving bumetanide (Bumex) 3. Client receiving spironolactone (Aldactone) 4. Client receiving hydrochlorothiazide (HCTZ)

3. 3 Rationale: Spironolactone is a potassium-sparing diuretic and competes with aldosterone at receptor sites in the distal tubule, resulting in excretion of sodium, chloride, and water and retention of potassium and phosphate. Use of the medications noted in options 1, 2, and 4 could result in hypokalemia. Test-Taking Strategy: Use the process of elimination. Eliminate options 1 and 2 because they are both loop diuretics, which lead to the side effect of hypokalemia. Next eliminate option 4 because it is a thiazide diuretic, which acts on the distal tubule and inhibits sodium, chloride, and potassium reabsorption. Review the effects of these medications if you had difficulty with this question.

A nurse is collecting data from a client who is scheduled for surgery in 1 week in the ambulatory care surgical center. The nurse notes that the client has a history of arthritis and has been taking acetylsalicylic acid (aspirin). The nurse reports the information to the physician and anticipates that the physician will prescribe which of the following?

3. Discontinue the aspirin 48 hrs. before the scheduled surgery. Anticoagulants alter normal clotting factors and increase the risk of hemorrhage. Aspirin has properties that can alter the clotting mechanism and s/thus be discontinued at least 48 hrs. before surgery.

30. A nurse is evaluating a client's response to cardioversion. Which of the following observations would be of highest priority to the nurse? 1. Blood pressure 2. Status of airway 3. Oxygen flow rate 4. Level of consciousness

30. 2 Rationale: Nursing responsibilities after cardioversion include maintenance first of a patent airway, and then oxygen administration, assessment of vital signs and level of consciousness, and dysrhythmia detection. Test Taking Strategy Use the process of elimination, noting the strategic words highest priority. Use the ABCs—airway, breathing, and circulation—to direct you to option 2. Review care of the client following cardioversion if you had difficulty with this question.

31. A nurse is performing cardiopulmonary resuscitation on a client who has had a cardiac arrest. An automatic external defibrillator is available to treat the client. Which of the following activities will allow the nurse to assess the client's cardiac rhythm? 1. Hold the defibrillator paddles firmly against the chest. 2. Apply adhesive patch electrodes to the chest and move away from the client. 3. Apply standard electrocardiographic monitoring leads to the client and observe the rhythm. 4. Connect standard electrocardiographic electrodes to a transtelephonic monitoring device.

31. 2 Rationale: The nurse or rescuer puts two large adhesive patch electrodes on the client's chest in the usual defibrillator positions. The nurse stops cardiopulmonary resuscitation and orders anyone near the client to move away and not touch the client. The defibrillator then analyzes the rhythm, which may take up to 30 seconds. The machine then indicates if defibrillation is necessary. Test-Taking Strategy Use the process of elimination. If you are not familiar with this piece of equipment, look first at the word automatic in the name. This implies that a person is not as involved in the process as with a conventional defibrillator and will help eliminate option 1. Because standard electrocardiogram monitoring leads do not play an active role once resuscitation is underway (options 3 and 4), you can eliminate these comparative or alike options. Review the procedure related to the use of an automatic external defibrillator if you had difficulty with this question.

32. A nurse employed in a cardiac unit determines that which of the following clients is the least likely to have implantation of an automatic internal cardioverter-defibrillator (AICD)? 1. A client with syncopal episodes related to ventricular tachycardia 2. A client with ventricular dysrhythmias despite medication therapy 3. A client with an episode of cardiac arrest related to myocardial infarction 4. A client with three episodes of cardiac arrest unrelated to myocardial infarction

32. 3 Rationale: An automatic internal cardioverter-defibrillator (AICD) detects and delivers an electrical shock to terminate life-threatening episodes of ventricular tachycardia and ventricular fibrillation. These devices are implanted in clients who are considered high risk, including those who have survived sudden cardiac death unrelated to myocardial infarction, those who are refractive to medication therapy, and those who have syncopal episodes related to ventricular tachycardia. Test-Taking Strategy Use the process of elimination and note the strategic words least likely. Ventricular dysrhythmias that induce syncope or occur while the client is on medication are likely to be true indications for the AICD, so eliminate options 1 and 2 first. From the remaining options, the main difference is whether or not the cardiac arrest was related to myocardial infarction. Of these two, the one most likely to be responsive to AICD would be the client without myocardial infarction because those dysrhythmias are spontaneous. Review the indications for the use of an AICD, if you had difficulty with this question.

33. A nurse is caring for a client who has just had implantation of an automatic internal cardioverter-defibrillator. The nurse immediately would assess which of the following items based on priority? 1. Anxiety level of the client and family 2. Presence of a Medic-Alert card for the client to carry 3. Knowledge of restrictions of postdischarge physical activity 4. Activation status of the device, heart rate cutoff, and number of shocks it is programmed to deliver

33. 4 Rationale: The nurse who is caring for the client after insertion of an automatic internal cardioverter-defibrillator needs to assess device settings, similar to after insertion of a permanent pacemaker. Specifically, the nurse needs to know whether the device is activated, the heart rate cutoff above which it will fire, and the number of shocks it is programmed to deliver. Options 1, 2, and 3 are also nursing interventions but are not the priority. Test-Taking Strategy Use Maslow's Hierarchy of Needs theory. Option 4 is the option that identifies the physiological need. Review care to the client following insertion of an automatic internal cardioverter-defibrillator if you had difficulty with this question.

34. A nurse is caring for a client immediately after insertion of a permanent demand pacemaker via the right subclavian vein. Which of the following activities will assist with preventing dislodgement of the pacing catheter? 1. Limiting movement and abduction of the left arm 2. Limiting movement and abduction of the right arm 3. Assisting the client to get out of bed and ambulate with a walker 4. Having the physical therapist do active range-of-motion exercises to the right arm

34. 2 Rationale: In the first several hours after insertion of a permanent or a temporary pacemaker, the most common complication is pacing electrode dislodgment. The nurse helps prevent this complication by limiting the client's activities of the arm on the side of the insertion site. Test-Taking Strategy Use the process of elimination. Note that the pacemaker was inserted on the right side. Therefore, to prevent pacing electrode dislodgment, motion must be limited on that side. Options 3 and 4 involve movement of the right arm and are eliminated first. Limiting the movement of the left arm (option 1) is of no benefit to the client. Thus, option 2 is the correct option. Review care of the client following insertion of a pacemaker if you had difficulty with this question.

35. A client diagnosed with thrombophlebitis 1 day ago suddenly complains of chest pain and shortness of breath and is visibly anxious. The nurse should immediately assess the client for signs and symptoms of which of the following? 1. Pneumonia 2. Pulmonary edema 3. Pulmonary embolism 4. Myocardial infarction

35. 3 Rationale: Pulmonary embolism is a life-threatening complication of deep vein thrombosis and thrombophlebitis. Chest pain is the most common symptom, which is sudden in onset, and may be aggravated by breathing. Other signs and symptoms include dyspnea, cough, diaphoresis, and apprehension. Test-Taking Strategy Focus on the client's diagnosis to answer the question. Recalling the complications related to thrombophlebitis will direct you to option 3. Review these complications and the associated signs and symptoms if you had difficulty with this question.

36. A client seeks treatment in a physician's office for unsightly varicose veins, and sclerotherapy is recommended. Before leaving the examining room, the client says to the nurse, "Can you tell me again how this sclerotherapy is done?" Which of the following statements would reflect accurate teaching by the nurse? 1. "The varicosity is surgically removed." 2. "The vein is tied off at the upper end to prevent stasis from occurring." 3. "The vein is tied off at the lower end to prevent stasis from occurring." 4. "An agent is injected into the vein to damage the vein wall and close the vein off."

36. 4 Rationale: Sclerotherapy is the injection of a sclerosing agent into a varicosity. The agent damages the vessel and causes aseptic thrombosis, which results in vein closure. With no blood flow through the vessel, there is no distention. The surgical procedure for varicose veins is vein ligation and stripping. This procedure involves tying off the varicose vein and large tributaries and then removing the vein with hook and wires via multiple small incisions in the leg. Test-Taking Strategy Use the process of elimination and note the name of the procedure, sclerotherapy. A vessel that is sclerosed is blocked. This will direct you to option 4. Review this procedure if you had difficulty with this question.

37. A client is having a follow-up physician office visit after vein ligation and stripping. The client describes a sensation of "pins and needles" in the affected leg. Which of the following would be an appropriate action by the nurse based on evaluation of the client's comment? 1. Instruct the client to apply warm packs. 2. Report the complaint to the physician. 3. Reassure the client that this is only temporary. 4. Advise the client to take acetaminophen (Tylenol) until it is gone.

37. 2 Rationale: Hypersensitivity or a sensation of "pins and needles" in the surgical limb may indicate temporary or permanent nerve injury following surgery. The saphenous vein and saphenous nerve run close together in the distal third of the leg. Because complications from this surgery are relatively rare, this symptom should be reported. Test-Taking Strategy Use the process of elimination. Pins and needles sensations usually indicate nerve irritation or damage. If you know this, you can eliminate options 1 and 4. Reassuring the client about something being "only temporary" is often not an appropriate action, unless this is known to be absolutely true. Review the complications associated with vein ligation and stripping if you had difficulty with this question.

38. Postoperatively, a nurse is caring for a client who had a percutaneous insertion of an inferior vena cava filter and was on heparin therapy before surgery. The nurse would inspect the surgical site most closely for evidence of which of the following? 1. Bleeding and infection 2. Thrombosis and infection 3. Bleeding and wound dehiscence 4. Wound dehiscence and evisceration

38. 1 Rationale: After inferior vena cava filter insertion, the nurse inspects the surgical site for bleeding and signs and symptoms of infection. Otherwise, care is the same as for any other postoperative client. Test-Taking Strategy Use the process of elimination. Because inferior vena cava filters are inserted percutaneously through a deep vein, options 3 and 4 are eliminated because no abdominal incision is made. From the remaining options, noting that the client has been on anticoagulant therapy before surgery because of the high risk of pulmonary embolism will direct you to option 1. Review care of the client following insertion of an inferior vena cava filter if you had difficulty with this question.

15. A staff nurse decides to attend a continuing education class on the use of advanced technology in health care delivery. The nurse manager should interpret this participation in staff development and continuing education as which of the following? <choice_1>A waste of time because CEU credits are not a mandate for promotion or relicensure</choice_1> <choice_2>Not important because advanced technology is too expensive and only drives up the cost of health care without increasing direct client care</choice_2> <choice_3>Essential to nursing care because advanced technology impacts several facets of health care delivery</choice_3> <choice_4>Only important for nurse managers because they allocate the unit's resources for the use of advanced technology in client care

3</correct> <rationale>Staff education is essential to maintaining clinical competence and client safety; therefore, options 1 and 2 are incorrect. Information technology is important to all nurses not just to nurse managers to organize and manage nursing and health care delivery. Option 4 is incorrect as well.</rationale> <cognitive_level>Application</cognitive_level> <client_need>Safe Effective Care Environment: Coordinated Care</client_need> <integrated_process>Nursing Process: Evaluation</integrated_process> <content_area>Leadership/Management</content_area> <strategy>The core of the question is understanding the impact of advanced technology on client care and its underlying costs. Use general nursing knowledge and the process of elimination to make a selection.</strategy>

34.The nurse is aware that a confused elderly client is at risk for falls. Which of the following interventions would the nurse avoid using with this client?</question> <question_image/> <choice_1>Orient the client to the call light system.</choice_1> <choice_2>Keep the hospital bed in low position.</choice_2> <choice_3>Keep the full bed rails up at all times.</choice_3> <choice_4>Assist with appropriate toileting every 2 hours.</choice_4>

3</correct> <rationale>Full bed rails are a type of physical restraint. A confused client may attempt to climb over the rails, increasing the risk for fall and injury. The other options are positive interventions for reducing risk for falls.</rationale>

11. An LPN/LVN who has been in practice for 6 months is due for the first performance evaluation. In preparing for the evaluation, the nurse should look to which standard against which to evaluate personal performance during the first 6 months of employment? <choice_1>ANA standards of care</choice_1> <choice_2>The state nurse practice act</choice_2> <choice_3>The written job description</choice_3> <choice_4>The organization's standard of clinical care</choice_4>

3</correct> <rationale>The best way that the nurse can effectively self-evaluate performance of his or her job is to compare individual performance against the written job description. Job descriptions help identify activities that each staff member may perform. The ANA standards of care help set the parameters for minimal standards and should be used as guidelines. Individual state boards of nursing identify the legal boundaries of nursing practice to safeguard the public. The state nurse practice act assists nurse leaders in knowing what tasks are within the scope of their state's nurse practice act and the scope of practice for their staff members. The job descriptions are designed to support the organization's work and aid in standards of performance.</rationale>

28.The home health nurse is visiting an elderly client with diabetes mellitus. The nurse becomes concerned and implements safety education when which of the following occurs?</question> <question_image/> <choice_1>The neighbors bring a warm lunch to the elderly client.</choice_1> <choice_2>The children install new air-conditioning units in the kitchen and bedroom.</choice_2> <choice_3>The grandchildren fold laundry and place the baskets by the door to the bedroom.</choice_3> <choice_4>The client stores the diabetic testing supplies on the kitchen table.</choice_4>

3</correct> <rationale>Laundry baskets that are set on the floor will pose a risk for falling for the elderly client. All hallways, floors, stairways, and furniture should be free of clutter. Neighbors bringing lunch and family controlling climate for the elderly client are good safety interventions. Keeping diabetic supplies on a kitchen table with easy access will facilitate diabetic testing.</rationale>

10. A nurse and teacher are discussing legal issues related to the practice of their professions. The teacher asks what is the primary purpose of the Nurse Practice Act in that state. The nurse replies that the Nurse Practice Act is intended to do which of the following? <choice_1>Accredit schools of nursing</choice_1> <choice_2>Enforce ethical standards of behavior</choice_2> <choice_3>Protect the public</choice_3> <choice_4>Define the scope of nursing practice</choice_4>

3</correct> <rationale>A Nurse Practice Act serves to protect the public by setting minimum qualifications for nursing in relation to skills and competencies. One way it fulfills the responsibility to protect the public is by defining the scope of nursing practice in that state. The state's board of nursing approves schools to operate but does not accredit them. It does not enforce ethical standards.

703. A client is having frequent premature ventricular contractions. A nurse would place priority on assessment of which of the following? 1. Sensation of palpitations 2. Causative factors, such as caffeine 3. Precipitating factors, such as infection 4. Blood pressure and oxygen saturation

4 Rationale: Premature ventricular contractions can cause hemodynamic compromise. The shortened ventricular filling time with the ectopic beat leads to decreased stroke volume and, if frequent enough, to decreased cardiac output. The client may be asymptomatic or may feel palpitations. Premature ventricular contractions can be caused by cardiac disorders, states of hypoxemia, or by any number of physiological stressors, such as infection, illness, surgery, or trauma, and by intake of caffeine, nicotine, or alcohol. Test-Taking Strategy: Note the strategic words priority on assessment. Use the ABCs—airway, breathing, and circulation—to direct you to option 4. Review the effects of premature ventricular contractions if you had difficulty with this question.

20. The nurse working in a sub-acute nursing home is assigned four clients. Place these clients in order of first to last priority of care.</question> <question_image/> <choice_1>Client A is complaining of feeling sad and lonely.</choice_1> <choice_2>Client B has just recently returned from the doctor's office after undergoing a stress test.</choice_2> <choice_3>Client C needs to be repositioned and has several pressure ulcers.</choice_3> <choice_4>Client D is complaining of feeling short of breath.</choice_4>

4, 2, 3, 1</correct> <rationale>Shortness of breath is always first priority. The client who had a stress test requires careful monitoring for potential complications. Client C needs to be repositioned to prevent further skin breakdown. While Client A's emotional concerns are important, they are of lower priority than the other three clients.</rationale>

A nurse is caring for a postoperative client who has a drain inserted into the surgical wound. Which of the following nursing actions w/be inappropriate for the care of the drain?

4. Secure the drain by curling or folding it and taping it firmly to the body. Aseptic technique must be used when emptying the drainage container or changing the dressing to avoid contamination of the wound. Usualy drainage from the wound is pale, red, and watery, whereas active bleeding will be bright red in color. The drain s/be checked for patency to provide an exit for the fluid or blood to promote healing. The nurse needs to ensure that drainage flows freely and that there are no kinks in the drains. Curling or folding the drain prevents the flow of the drainage.

40. A client is scheduled for a cardiac catheterization using a radiopaque dye. Which of the following assessments is most critical before the procedure? 1. Intake and output 2. Height and weight 3. Allergy to iodine or shellfish 4. Baseline peripheral pulse rates

40. 3 Rationale: A cardiac catheterization requires an informed consent because it involves injection of a radiopaque dye into the blood vessel. The risk of allergic reaction and possible anaphylaxis is a concern and the presence of allergies must be assessed before the procedure. Although options 1, 2, and 4 are accurate, they are not the most critical preprocedure assessments. Test-Taking Strategy Use the process of elimination and note the strategic words most critical. Recalling the concern related to allergy to the dye and the risk of anaphylaxis makes option 3 correct. Review preprocedure interventions for a cardiac catheterization if you had difficulty with this question.

41. A nurse is assessing the neurovascular status of a client who returned to the surgical nursing unit 4 hours ago after undergoing aortoiliac bypass graft. The affected leg is warm, and the nurse notes redness and edema. The pedal pulse is palpable and unchanged from admission. How would the nurse correctly interpret the client's neurovascular status? 1. The neurovascular status is normal because of increased blood flow through the leg. 2. The neurovascular status is moderately impaired, and the surgeon should be called. 3. The neurovascular status is slightly deteriorating and should be monitored for another hour. 4. The neurovascular status is adequate from an arterial approach, but venous complications are arising.

41. 1 Rationale: An expected outcome of surgery is warmth, redness, and edema in the surgical extremity because of increased blood flow. Therefore, options 2, 3, and 4 are incorrect interpretations. Test-Taking Strategy Use the process of elimination. Option 2 can be eliminated because the pedal pulse is unchanged from admission. Venous complications from immobilization resulting from surgery would not be apparent within 4 hours, so eliminate option 4. From the remaining options, think about the effects of sudden reperfusion in an ischemic limb. There would be redness from new blood flow and edema from the sudden change in pressure in the blood vessels. Review the expected assessment findings following this surgical procedure if you had difficulty with this question.

42. A nurse is evaluating the condition of a client after pericardiocentesis performed to treat cardiac tamponade. Which of the following observations would indicate that the procedure was unsuccessful? 1. Rising blood pressure 2. Clearly audible heart sounds 3. Client expressions of relief 4. Rising central venous pressure

42. 4 Rationale: Following pericardiocentesis, a rise in blood pressure and a fall in central venous pressure are expected. The client usually expresses immediate relief. Heart sounds are no longer muffled or distant. Test-Taking Strategy Use the process of elimination and note the strategic word unsuccessful. Successful therapy is measured by the disappearance of the original signs and symptoms of cardiac tamponade. Therefore, look for the option that identifies a sign consistent with continued tamponade. Review signs of cardiac tamponade and the expected effects of pericardiocentesis if you had difficulty with this question.

43. A nurse is assessing a client with an abdominal aortic aneurysm. Which of the following assessment findings by the nurse is probably unrelated to the aneurysm? 1. Pulsatile abdominal mass 2. Hyperactive bowel sounds in the area 3. Systolic bruit over the area of the mass 4. Subjective sensation of "heart beating" in the abdomen

43. 2 Rationale: Not all clients with abdominal aortic aneurysm exhibit symptoms. Those who do may describe a feeling of the "heart beating" in the abdomen when supine or being able to feel the mass throbbing. A pulsatile mass may be palpated in the middle and upper abdomen. A systolic bruit may be auscultated over the mass. Hyperactive bowel sounds are not related specifically to an abdominal aortic aneurysm. Test-Taking Strategy Use the process of elimination and note the strategic word unrelated. Note that options 1, 3, and 4 are comparative or alike in that they identify a circulatory component. Review the signs of abdominal aortic aneurysm if you had difficulty with this question.

sub-q injections

45-90 degree angle, 25G needle, 1/2-1" length, below dermis/above muscle

45. Cardiac magnetic resonance imaging (MRI) is prescribed for a client. The nurse identifies that which of the following is a contraindication for performance of this diagnostic study? 1. Client has a pacemaker. 2. Client is allergic to iodine. 3. Client has diabetes mellitus. 4. Client has a biological porcine valve.

45. 1 Rationale: The magnetic fields used for magnetic resonance imaging (MRI) can deactivate the pacemaker. Options 2, 3, and 4 are not contraindications for an MRI. Test-Taking Strategy Focus on the name of the test and note the strategic word magnetic. Remember that the magnetic fields of the MRI can deactivate the pacemaker. Review the contraindications for an MRI if you had difficulty with this question.

35.The nurse has applied elbow splints on a confused client to prevent the client from removing the intravenous (IV) line. Which of the following interventions is required?</question> <question_image/> <choice_1>Document the appearance of the client's IV site every hour.</choice_1> <choice_2>Remove the restraints every 8 hours.</choice_2> <choice_3>Ask for a renewal of the physician's restraint order every 72 hours.</choice_3> <choice_4>Assess and document client condition at least every hour.</choice_4>

4</correct> <rationale>The client should be checked at least hourly, and the nurse is required to document status. The IV site should be checked every hour, but documentation may be done only once per shift unless a problem occurs. Physical restraints impede a client's freedom, and thus their use needs to be ordered every 24 hours. Because restraints may also impede circulation, they should be removed according to agency policy, which is generally every 1 to 2 hours rather than every 8 hours.</rationale>

23. The result of a toddler's lead screening is 12mg/dL. The nurse should say which of the following to the mother at this time?</question> <question_image/> <choice_1>"His lab values are just fine."</choice_1> <choice_2>"Have you noticed any blood in his stools?"</choice_2> <choice_3>"When were his last immunizations?"</choice_3> <choice_4>"Tell me about where you live."</choice_4>

4</correct> <rationale>The lead value of 12mg/dL is high. Lead levels below 10mg/dL are acceptable. Levels of 10-19mg/dL require an environmental history. Levels above 20mg/dL require a full medical evaluation. Asking a question regarding the child's address is the first step in evaluating the environment. Older homes may have lead paint and lead in the plumbing. Option 1 is inaccurate because the level is high (not normal), and options 2 and 3 are unrelated to lead poisoning.</rationale>

4.A client asks why a diagnostic test has been ordered and the nurse replies, "I'm unsure but will find out for you." When the nurse later returns and provides an explanation, the nurse is acting under which principle? 1. Nonmaleficence 2. Veracity 3. Beneficence 4. Fidelity

4</correct> <rationale>Fidelity means to be faithful to agreements and promises. This nurse is acting on the client's behalf to obtain needed information and report it back to the client. Nonmaleficence is duty to do no harm. Veracity refers to telling the truth—for example, not lying to a client about a serious prognosis. Beneficence means doing good, such as by implementing actions (keeping a salt shaker out of sight) that benefit a client (heart condition requiring sodium-restricted diet).

18. The nurse is working in a nursing home that uses the team nursing approach to client care. The nurse is orienting a new CNA who asks if she will be assigned her own clients to care for. The nurse would respond: <choice_1>"Yes, every day you come in you'll have a client list that will be your sole responsibility once you've completed orientation."</choice_1> <choice_2>"Yes."</choice_2> <choice_3>"No, you'll just complete the tasks you are told to complete on whatever client you are asked to care for."</choice_3> <choice_4>"No. In team nursing you will be assigned to a team that will share responsibilities for a specific group of clients."</choice_4>

4</correct> <rationale>In team nursing, RNs, LPNs/LVNs, and CNAs work together to care for a group of clients, with each care provider working to his or her scope of practice. Working as a team means no client is any nurse's sole reponsibility. Option 2 doesn't provide any helpful information, and option 3 doesn't explain that each team member is expected to work within their scope of practice.

42.The nurse would do which of the following when washing the hands as part of medical asepsis before caring for an assigned client in an outpatient clinic?</question> <question_image/> <choice_1>Wash the hands with the hands held higher than the elbows.</choice_1> <choice_2>Adjust the temperature of the water to the hottest possible.</choice_2> <choice_3>Scrub the hands and nails with a scrub brush for 5 minutes.</choice_3> <choice_4>Use a clean paper towel to turn the water off.</choice_4>

4</correct> <rationale>Option 4 is correct because the faucet is considered contaminated. The hands are considered to be more contaminated than the elbows. Therefore, water should flow from least contaminated to most contaminated, eliminating option 1. Option 2 can result in burns to the nurse. Warm water removes less of the protective oils in the skin. Option 3 describes a surgical scrub.</rationale>

43.The nurse's forearm is splattered with blood while inserting an intravenous catheter. What action should the nurse take?</question> <question_image/> <choice_1>Wipe the blood away with an alcohol swab.</choice_1> <choice_2>Wipe the blood away with a tissue.</choice_2> <choice_3>Flush the forearm with hot water, letting the water flow from the elbow toward the fingers.</choice_3> <choice_4>Wash the forearm with soap and water.</choice_4>

4</correct> <rationale>Washing the skin with the combination of soap and water will remove the blood through mechanical friction. While alcohol can kill bacteria, it cannot kill viruses and fungi (option 1). Tissues would not adequately remove the blood (option 2). Hot water can burn the nurse, and water alone is inadequate in removing the blood (option 3).</rationale>

5. A client who is receiving digoxin (Lanoxin) daily has a serum potassium level of 3.0 mEq/L and is complaining of anorexia. A physician orders a digoxin level to rule out digoxin toxicity. A nurse checks the results, knowing that which of the following is the therapeutic serum level (range) for digoxin? 1. 0.5 to 2 ng/mL 2. 1.2 to 2.8 ng/mL 3. 3 to 5 ng/mL 4. 3.5 to 5.5 ng/mL

5. 1 Rationale: Therapeutic levels for digoxin range from 0.5 to 2 ng/mL. Therefore, options 2, 3, and 4 are incorrect. Test-Taking Strategy: Knowledge of the therapeutic serum digoxin level will direct you to option 1. If you had difficulty with this question, learn the therapeutic level for digoxin.

Therapeutic Drug level Propranolol (Inderal)

50 - 100 ng/mL

Iron - female adult

50 - 170 mcg/dL

Normal WBC

5000-10,000 cells/mm3 When there is a drop, neutropenic precautions should be implemented

6. A nurse notes bilateral 12 edema in the lower extremities of a client with myocardial infarction who was admitted 2 days ago. The nurse would plan to do which of the following next? 1. Order daily weights starting on the following morning. 2. Review the intake and output records for the last 2 days. 3. Request a sodium restriction of 1 g/day from the physician. 4. Change the time of diuretic administration from morning to evening.

6. 2 Rationale: Edema, the accumulation of excess fluid in the interstitial spaces, can be measured by intake greater than output and by a sudden increase in weight. Diuretics should be given in the morning whenever possible to avoid nocturia. Strict sodium restrictions are reserved for clients with severe symptoms. Test-Taking Strategy Use the process of elimination, noting the strategic word next. Use the steps of the nursing process to prioritize. Option 2 is the only option that addresses assessment of data. Review care of the client with a myocardial infarction if you had difficulty with this question.

6. A client is being treated with procainamide (Procanbid) for a cardiac dysrhythmia. Following intravenous administration of the medication, the client complains of dizziness. What intervention should the nurse take first? 1. Administer ordered nitroglycerin tablets. 2. Measure the heart rate on the rhythm strip. 3. Obtain a 12-lead electrocardiogram immediately. 4. Auscultate the client's apical pulse and obtain a blood pressure.

6. 4 Rationale: Signs of toxicity from procainamide include confusion, dizziness, drowsiness, decreased urination, nausea, vomiting, and tachydysrhythmias. If the client complains of dizziness, the nurse should assess the vital signs first. Although options 2 and 3 may be interventions, these would be done after the vital signs are taken. Nitroglycerin is a vasodilator and will lower the blood pressure. Test-Taking Strategy: Use the steps of the nursing process to eliminate options 1 and 3. From the remaining options, remember always to assess the client first, not the monitoring devices. Therefore, option 4 is correct. Review the signs of toxicity and the nursing interventions if you had difficulty with this question.

The nurse is caring for a patient admitted to the hospital for asthma who weighs 186 lb (84.5 kg). During dietary counseling, the patient asks the nurse how much protein he should ingest each day. How many grams of protein does the nurse recommend should be included in the diet based on the patient's current weight? 24 41 68 93

68 The daily intake of protein should be between 0.8 and 1 g/kg of body weight. Thus this patient should take in between 68 and 84 g of protein per day in the diet.

692. A client is admitted to an emergency department with chest pain that is being ruled out for myocardial infarction. Vital signs are as follows: at 11 AM, pulse (P), 92 beats/min, respiratory rate (RR), 24 breaths/min, blood pressure (BP), 140/88 mm Hg; 11:15 AM, P, 96 beats/min, RR, 26 breaths/min, BP, 128/82 mm Hg; 11:30 AM, P, 104 beats/min, RR, 28 breaths/min, BP, 104/68 mm Hg; 11:45 AM, P, 118 beats/min, RR, 32 breaths/min, BP, 88/58 mm Hg. The nurse should alert the physician because these changes are most consistent with which of the following complications? 1. Cardiogenic shock 2. Cardiac tamponade 3. Pulmonary embolism 4. Dissecting thoracic aortic aneurysm

692. 1 Rationale: Cardiogenic shock occurs with severe damage (more than 40%) to the left ventricle. Classic signs include hypotension, a rapid pulse that becomes weaker, decreased urine output, and cool, clammy skin. Respiratory rate increases as the body develops metabolic acidosis from shock. Cardiac tamponade is accompanied by distant, muffled heart sounds and prominent neck vessels. Pulmonary embolism presents suddenly with severe dyspnea accompanying the chest pain. Dissecting aortic aneurysms usually are accompanied by back pain. Test-Taking Strategy: Use the process of elimination. Recalling that the early serious complications of myocardial infarction include dysrhythmias, cardiogenic shock, and sudden death will direct you to option 1. No information in the question is associated with options 2, 3, or 4. Review the complications of myocardial infarction if you had difficulty with this question.

693. A client admitted to the hospital with chest pain and history of type 2 diabetes mellitus is scheduled for cardiac catheterization. Which of the following medications would need to be withheld for 48 hours before and after the procedure? 1. Regular insulin 2. Glipizide (Glucotrol) 3. Repaglinide (Prandin) 4. Metformin (Glucophage)

693. 4 Rationale: Metformin (Glucophage) needs to be withheld 48 hours before and after cardiac catheterization because of the injection of contrast medium during the procedure. If the contrast medium affects kidney function, with metformin in the system, the client would be at increased risk for lactic acidosis. The medications in options 1, 2, and 3 do not need to be withheld 48 hours before or after cardiac catheterization. Test-Taking Strategy: Use the process of elimination. Eliminate options 2 and 3 first. Although these medications may be withheld on the morning of the procedure because of the client's NPO status, there is no indication for withholding the medication the day prior to and the day postprocedure. Regular insulin may be administered if elevated blood glucose levels from infused intravenous solutions occur on the day of the procedure. Review preprocedure and postprocedure interventions if you had difficulty with this question.

694. A client is in sinus bradycardia with a heart rate of 45 beats/min, complains of dizziness, and has a blood pressure of 82/60 mm Hg. Which of the following should the nurse anticipate will be prescribed? 1. Defibrillate the client. 2. Administer digoxin (Lanoxin). 3. Continue to monitor the client. 4. Prepare for transcutaneous pacing.

694. 4 Rationale: Hypotension and dizziness are signs of decreased cardiac output. Transcutaneous pacing provides a temporary measure to increase the heart rate and thus perfusion in the symptomatic client. Digoxin will further decrease the client's heart rate. Defibrillation is used for treatment of pulseless ventricular tachycardia and ventricular fibrillation. Continuing to monitor the client delays necessary intervention. Test-Taking Strategy: Use the process of elimination. Eliminate option 3 because the client is symptomatic and requires intervention. Option 2 is eliminated because digoxin will further decrease the client's heart rate. Defibrillation is used for treatment of pulseless ventricular tachycardia and ventricular fibrillation; therefore eliminate option 1. Review the indications for transcutaneous pacing if you had difficulty with this question.

696. A client with myocardial infarction suddenly becomes tachycardic, shows signs of air hunger, and begins coughing frothy, pink-tinged sputum. Which of the following would the nurse anticipate when auscultating the client's breath sounds? 1. Stridor 2. Crackles 3. Scattered rhonchi 4. Diminished breath sounds

696. 2 Rationale: Pulmonary edema is characterized by extreme breathlessness, dyspnea, air hunger, and the production of frothy, pink-tinged sputum. Auscultation of the lungs reveals crackles. Rhonchi and diminished breath sounds are not associated with pulmonary edema. Stridor is a crowing sound associated with laryngospasm or edema of the upper airway. Test-Taking Strategy: Use the process of elimination. Recalling that fluid produces sounds that are called crackles will assist you in eliminating options 1, 3, and 4. If you had difficulty with this question, review the manifestations found in pulmonary edema.

697. A client with myocardial infarction is developing cardiogenic shock. Because of the risk of myocardial ischemia, for which of the following should the nurse carefully assess the client? 1. Bradycardia 2. Ventricular dysrhythmias 3. Rising diastolic blood pressure 4. Falling central venous pressure

697. 2 Rationale: Classic signs of cardiogenic shock as they relate to this question include low blood pressure and tachycardia. The central venous pressure would rise as the backward effects of the severe left ventricular failure became apparent. Dysrhythmias commonly occur as a result of decreased oxygenation and severe damage to greater than 40% of the myocardium. Test-Taking Strategy: Use the process of elimination and focus on the strategic words myocardial ischemia. Recall that ischemia makes the myocardium irritable, producing dysrhythmias. Also, knowledge of the classic signs of shock helps eliminate the incorrect options. Review the clinical manifestations associated with cardiogenic shock if you had difficulty with this question.

698. A client who had cardiac surgery 24 hours ago has a urine output averaging 20 mL/hr for 2 hours. The client received a single bolus of 500 mL of intravenous fluid. Urine output for the subsequent hour was 25 mL. Daily laboratory results indicate that the blood urea nitrogen level is 45 mg/dL and the serum creatinine level is 2.2 mg/dL. Based on these findings, the nurse would anticipate that the client is at risk for which of the following? 1. Hypovolemia 2. Acute renal failure 3. Glomerulonephritis 4. Urinary tract infection

698. 2 Rationale: The client who undergoes cardiac surgery is at risk for renal injury from poor perfusion, hemolysis, low cardiac output, or vasopressor medication therapy. Renal insult is signaled by decreased urine output and increased blood urea nitrogen and creatinine levels. The client may need medications to increase renal perfusion and possibly could need peritoneal dialysis or hemodialysis. No data in the question indicate the presence of hypovolemia, urinary tract infection, or glomerulonephritis. Test-Taking Strategy: Use the process of elimination. Eliminate options 3 and 4 first because no data indicate infection or inflammation. Noting that the urine output is inadequate will assist you in eliminating option 1. Review the complications associated with cardiac surgery if you had difficulty with this question.

7. A nurse is conducting a health history of a client with a primary diagnosis of heart failure. Which of the following disorders reported by the client is unlikely to play a role in exacerbating the heart failure? 1. Atrial fibrillation 2. Nutritional anemia 3. Peptic ulcer disease 4. Recent upper respiratory infection

7. 3 Rationale: Heart failure is precipitated or exacerbated by physical or emotional stress, dysrhythmias, infections, anemia, thyroid disorders, pregnancy, Paget's disease, nutritional deficiencies (thiamine, alcoholism), pulmonary disease, and hypervolemia. Test-Taking Strategy Use the process of elimination and note the strategic word unlikely. Remembering that heart failure is exacerbated by factors that increase the workload of the heart will assist you in eliminating options 1, 2, and 4. Review the precipitating factors associated with heart failure if you had difficulty with this question.

ABG - pH

7.35 - 7.45

Glucose (fasting)

70 - 110 mg/dL

700. A client is wearing a continuous cardiac monitor, which begins to sound its alarm. A nurse sees no electrocardiographic complexes on the screen. Which of the following should be the priority action of the nurse? 1. Call a code blue. 2. Call the physician. 3. Check the client status and lead placement. 4. Press the recorder button on the electrocardiogram console.

700. 3 Rationale: Sudden loss of electrocardiographic complexes indicates ventricular asystole or possibly electrode displacement. Accurate assessment of the client and equipment is necessary to determine the cause and identify the appropriate intervention. Options 1, 2, and 4 are unnecessary. Test-Taking Strategy: Use the steps of the nursing process. Always assess the client directly before taking any action. Option 3 is the only option that addresses assessment. Review care of the client on a cardiac monitor if you had difficulty with this question.

702. A client has frequent bursts of ventricular tachycardia on the cardiac monitor. What should the nurse be most concerned about with this dysrhythmia? 1. It can develop into ventricular fibrillation at any time. 2. It is almost impossible to convert to a normal rhythm. 3. It is uncomfortable for the client, giving a sense of impending doom. 4. It produces a high cardiac output that quickly leads to cerebral and myocardial ischemia.

702. 1 Rationale: Ventricular tachycardia is a life-threatening dysrhythmia that results from an irritable ectopic focus that takes over as the pacemaker for the heart. The low cardiac output that results can lead quickly to cerebral and myocardial ischemia. Clients frequently experience a feeling of impending doom. Ventricular tachycardia is treated with antidysrhythmic medications, cardioversion (client awake), or defibrillation (loss of consciousness). Ventricular tachycardia can deteriorate into ventricular fibrillation at any time. Test-Taking Strategy: Use the process of elimination and note the strategic words most concerned. Option 2 is incorrect and is eliminated first. From the remaining options, focusing on the strategic words will direct you to option 1 because this option identifies the life-threatening condition. Review the concerns associated with ventricular tachycardia if you had difficulty with this question.

704. A client has developed atrial fibrillation, with a ventricular rate of 150 beats/min. The nurse should assess the client for which associated signs or symptoms? 1. Flat neck veins 2. Nausea and vomiting 3. Hypotension and dizziness 4. Hypertension and headache

704. 3 Rationale: The client with uncontrolled atrial fibrillation with a ventricular rate more than 100 beats/min is at risk for low cardiac output because of loss of atrial kick. The nurse assesses the client for palpitations, chest pain or discomfort, hypotension, pulse deficit, fatigue, weakness, dizziness, syncope, shortness of breath, and distended neck veins. Test-Taking Strategy: Use the process of elimination. Flat neck veins are normal or indicate hypovolemia, so eliminate option 1. Nausea and vomiting (option 2) are associated with vagus nerve activity and do not correlate with a tachycardic state. From the remaining options, think of the consequences of falling cardiac output to direct you to option 3. Review the effects of atrial fibrillation if you had difficulty with this question.

712. A nurse is assessing the neurovascular status of a client who returned to the surgical nursing unit 4 hours ago after undergoing aortoiliac bypass graft. The affected leg is warm, and the nurse notes redness and edema. The pedal pulse is palpable and unchanged from admission. How would the nurse correctly interpret the client's neurovascular status? 1. The neurovascular status is normal because of increased blood flow through the leg. 2. The neurovascular status is moderately impaired, and the surgeon should be called. 3. The neurovascular status is slightly deteriorating and should be monitored for another hour. 4. The neurovascular status is adequate from an arterial approach, but venous complications are arising.

712. 1 Rationale: An expected outcome of aortoiliac bypass graft surgery is warmth, redness, and edema in the surgical extremity because of increased blood flow. Therefore options 2, 3, and 4 are incorrect interpretations. Test-Taking Strategy: Use the process of elimination. Option 2 can be eliminated because the pedal pulse is unchanged from admission. Venous complications from immobilization resulting from surgery would not be apparent within 4 hours, so eliminate option 4. From the remaining options, think about the effects of sudden reperfusion in an ischemic limb. There would be redness from new blood flow and edema from the sudden change in pressure in the blood vessels. Review the expected assessment findings following this surgical procedure if you had difficulty with this question.

714. A nurse is caring for a client who had a resection of an abdominal aortic aneurysm yesterday. The client has an intravenous infusion with a rate of 150 mL/hr, unchanged for the last 10 hours. The client's urine output for the last 3 hours was 90, 50, and 28 mL (28 mL most recent). The client's blood urea nitrogen level is 35 mg/dL and serum creatinine level is 1.8 mg/dL, measured this morning. Which of the following actions should the nurse take next? 1. Call the physician. 2. Check the urine specific gravity. 3. Check to see if the client had a sample for serum albumin level drawn. 4. Put the intravenous line on a pump so that the infusion rate is sure to stay stable.

714. 1 Rationale: Following abdominal aortic aneurysm resection or repair, the nurse monitors the client for signs of renal failure. Renal failure can occur because often much blood is lost during the surgery and, depending on the aneurysm location, the renal arteries may be hypoperfused for a short period during surgery. The nurse monitors hourly intake and output and notes the results of daily blood urea nitrogen and creatinine levels. Urine output lower than 30 to 50 mL/hr is reported to the physician. Test-Taking Strategy: Focus on the information in the question and the abnormal assessment data. This question indicates elevations in blood urea nitrogen and creatinine levels and a significant drop in hourly urine output. These assessment findings should direct you to option 1. Review the complications associated with this surgical procedure if you had difficulty with this question.

715. A client with angina complains that the anginal pain is prolonged and severe and occurs at the same time each day, most often at rest in the absence of precipitating factors. How would the nurse best describe this type of anginal pain? 1. Stable angina 2. Variant angina 3. Unstable angina 4. Nonanginal pain

715. 2 Rationale: Variant angina, or Prinzmetal's angina, is prolonged and severe and occurs at the same time each day, most often at rest. Stable angina is induced by exercise and relieved by rest or nitroglycerin tablets. Unstable angina occurs at lower and lower levels of activity or at rest, is less predictable, and is often a precursor of myocardial infarction. Test-Taking Strategy: Use the process of elimination, focusing on the data in the question. Noting the strategic words at rest will direct you to option 2. If you had difficulty with this question, review the characteristics of the various types of angina.

716. A nurse in a medical unit is caring for a client with heart failure. The client suddenly develops extreme dyspnea, tachycardia, and lung crackles and the nurse suspects pulmonary edema. The nurse immediately asks another nurse to contact the physician and prepares to implement which priority interventions? Select all that apply. 1. Administering oxygen 2. Inserting a Foley catheter 3. Administering furosemide (Lasix) 4. Administering morphine sulfate intravenously 5. Transporting the client to the coronary care unit 6. Placing the client in a low Fowler's side-lying position

716. 1, 2, 3, 4 Rationale: Pulmonary edema is a life-threatening event that can result from severe heart failure. In pulmonary edema, the left ventricle fails to eject sufficient blood, and pressure increases in the lungs because of the accumulated blood. Oxygen is always prescribed, and the client is placed in a high Fowler's position to ease the work of breathing. Furosemide, a rapid-acting diuretic, will eliminate accumulated fluid. A Foley catheter is inserted to measure output accurately. Intravenously administered morphine sulfate reduces venous return (preload), decreases anxiety, and also reduces the work of breathing. Transporting the client to the coronary care unit is not a priority intervention. In fact, this may not be necessary at all if the client's response to treatment is successful. Test-Taking Strategy: Note the strategic words priority interventions and focus on the client's diagnosis. Recall the pathophysiology associated with pulmonary edema and use the ABCs—airway, breathing, and circulation—to help determine priority interventions. Review priority interventions for the client with pulmonary edema if you had difficulty with this question.

717. A client with atrial fibrillation is receiving a continuous heparin infusion at 1000 units/hr. The nurse would determine that the client is receiving the therapeutic effect based on which of the following results? 1. Prothrombin time of 12.5 seconds 2. Activated partial thromboplastin time of 60 seconds 3. Activated partial thromboplastin time of 28 seconds 4. Activated partial thromboplastin time longer than 120 seconds

717. 2 Rationale: Common laboratory ranges for activated partial thromboplastin time are 20 to 36 seconds. Because the activated partial thromboplastin time should be 1.5 to 2.5 times the normal value, the client's activated partial thromboplastin time would be considered therapeutic if it was 60 seconds. Test-Taking Strategy: Use the process of elimination. Option 1 is eliminated because the prothrombin time assesses response to warfarin (Coumadin) therapy. Eliminate option 3 because at 28 seconds the client is receiving no therapeutic effect from the continuous heparin infusion. Eliminate option 4 because this value is beyond the therapeutic range and the client is at risk for bleeding. Review laboratory tests to monitor the effectiveness of heparin therapy if you had difficulty with this question.

718. A nurse provides discharge instructions to a postoperative client who is taking warfarin sodium (Coumadin). Which statement, if made by the client, reflects the need for further teaching? 1. "I will take my pills every day at the same time." 2. "I will avoid alcohol consumption." 3. "I have already called my family to pick up a Medic-Alert bracelet." 4. "I will take Ecotrin (enteric-coated aspirin) for my headaches because it is coated."

718. 4 Rationale: Ecotrin is an aspirin-containing product and should be avoided. Alcohol consumption should be avoided by a client taking warfarin sodium. Taking prescribed medication at the same time each day increases client compliance. The Medic-Alert bracelet provides health care personnel emergency information. Test-Taking Strategy: Use the process of elimination and note the strategic words need for further teaching. These words indicate a negative event query and ask you to select an option that is an incorrect statement. Recalling that warfarin (Coumadin) is an anticoagulant and that Ecotrin is an aspirin-containing product will direct you to option 4. Review client teaching points related to warfarin if you had difficulty with this question.

721. A nurse is monitoring a client who is taking propranolol (Inderal). Which assessment data would indicate a potential serious complication associated with propranolol? 1. The development of complaints of insomnia 2. The development of audible expiratory wheezes 3. A baseline blood pressure of 150/80 mm Hg followed by a blood pressure of 138/72 mm Hg after two doses of the medication 4. A baseline resting heart rate of 88 beats/min followed by a resting heart rate of 72 beats/min after two doses of the medication

721. 2 Rationale: Audible expiratory wheezes may indicate a serious adverse reaction, bronchospasm. β-Blockers may induce this reaction, particularly in clients with chronic obstructive pulmonary disease or asthma. Normal decreases in blood pressure and heart rate are expected. Insomnia is a frequent mild side effect and should be monitored. Test-Taking Strategy: Use the process of elimination, eliminating options 3 and 4 because these are expected effects from the medication. Note the strategic words potential serious complication. These strategic words will direct you to option 2. Review the adverse effects of this medication if you had difficulty with this question.

723. A client is diagnosed with an acute myocardial infarction and is receiving tissue plasminogen activator, alteplase (Activase, tPA). Which of the following is a priority nursing intervention? 1. Monitor for renal failure. 2. Monitor psychosocial status. 3. Monitor for signs of bleeding. 4. Have heparin sodium available.

723. 3 Rationale: Tissue plasminogen activator is a thrombolytic. Hemorrhage is a complication of any type of thrombolytic medication. The client is monitored for bleeding. Monitoring for renal failure and monitoring the client's psychosocial status are important but are not the most critical interventions. Heparin is given after thrombolytic therapy, but the question is not asking about follow-up medications. Test-Taking Strategy: Use the process of elimination and note the strategic word priority. Remember, bleeding is a priority. Review care of the client on tissue plasminogen activator if you had difficulty with this question.

724. A nurse is planning to administer hydrochlorothiazide (HydroDIURIL) to a client. The nurse understands that which of the following are concerns related to the administration of this medication? 1. Hypouricemia, hyperkalemia 2. Increased risk of osteoporosis 3. Hypokalemia, hyperglycemia, sulfa allergy 4. Hyperkalemia, hypoglycemia, penicillin allergy

724. 3 Rationale: Thiazide diuretics such as hydrochlorothiazide are sulfa-based medications, and a client with a sulfa allergy is at risk for an allergic reaction. Also, clients are at risk for hypokalemia, hyperglycemia, hypercalcemia, hyperlipidemia, and hyperuricemia. Test-Taking Strategy: Use the process of elimination. Recalling that thiazide diuretics carry a sulfa ring will direct you to option 3. Review the nursing considerations related to administering this medication if you had difficulty with this question.

725. A home health care nurse is visiting a client with elevated triglyceride levels and a serum cholesterol level of 398 mg/dL. The client is taking cholestyramine (Questran). Which of the following statements, if made by the client, indicates the need for further education? 1. "Constipation and bloating might be a problem." 2. "I'll continue to watch my diet and reduce my fats." 3. "Walking a mile each day will help the whole process." 4. "I'll continue my nicotinic acid from the health food store."

725. 4 Rationale: Nicotinic acid, even an over-the-counter form, should be avoided because it may lead to liver abnormalities. All lipid-lowering medications also can cause liver abnormalities, so a combination of nicotinic acid and cholestyramine resin is to be avoided. Constipation and bloating are the two most common side effects. Walking and the reduction of fats in the diet are therapeutic measures to reduce cholesterol and triglyceride levels. Test-Taking Strategy: Use the process of elimination and note the strategic words need for further education. These words indicate a negative event query and ask you to select an option that is an incorrect statement. Remembering that over-the-counter medications should be avoided when a client is taking a prescription medication will direct you to option 4. Review client teaching points related to this medication if you had difficulty with this question.

726. A client is on nicotinic acid (niacin) for hyperlipidemia and the nurse provides instructions to the client about the medication. Which statement by the client would indicate an understanding of the instructions? 1. "It is not necessary to avoid the use of alcohol." 2. "The medication should be taken with meals to decrease flushing." 3. "Clay-colored stools are a common side effect and should not be of concern." 4. "Ibuprofen (Motrin) taken 30 minutes before the nicotinic acid should decrease the flushing."

726. 4 Rationale: Flushing is a side effect of this medication. Aspirin or a nonsteroidal anti-inflammatory drug can be taken 30 minutes prior to taking the medication to decrease flushing. Alcohol consumption needs to be avoided because it will enhance this side effect. The medication should be taken with meals but this will decrease gastrointestinal upset; taking the medication with meals has no effect on the flushing. Clay-colored stools are a sign of hepatic dysfunction and should be immediately reported to the physician. Test-Taking Strategy: Use the process of elimination. Option 1 can be eliminated because alcohol must be abstained from. Option 2 can be eliminated because taking the medication with meals helps decrease the gastrointestinal symptoms. The clay-colored stools in option 3 is a sign of hepatic dysfunction and should be immediately reported to the physician. Review the client teaching points related to this medication if you had difficulty with this question.

727. A 66-year-old client complaining of not feeling well is seen in a clinic. The client is taking several medications for the control of heart disease and hypertension. These medications include atenolol (Tenormin), digoxin (Lanoxin), and chlorothiazide (Diuril). A tentative diagnosis of digoxin toxicity is made. Which of the following assessment data would support this diagnosis? 1. Dyspnea, edema, and palpitations 2. Chest pain, hypotension, and paresthesia 3. Double vision, loss of appetite, and nausea 4. Constipation, dry mouth, and sleep disorder

727. 3 Rationale: Double vision, loss of appetite, and nausea are early signs of digoxin toxicity. Additional signs of digoxin toxicity include bradycardia, difficulty reading, other visual alterations such as green and yellow vision or seeing spots or halos, confusion, vomiting, diarrhea, decreased libido, and impotence. Test-Taking Strategy: Use the process of elimination. Recalling that gastrointestinal and visual disturbances occur with digoxin toxicity will direct you to option 3. If you had difficulty with this question, review the signs of digoxin toxicity.

730. A client is receiving thrombolytic therapy with a continuous infusion of streptokinase (Streptase). The client suddenly becomes extremely anxious and complains of itching. A nurse hears stridor and on examination of the client notes generalized urticaria and hypotension. Which of the following should be the priority action of the nurse? 1. Administer oxygen and protamine sulfate. 2. Stop the infusion and call the physician. 3. Cut the infusion rate in half and sit the client up in bed. 4. Administer diphenhydramine

730. 2 Rationale: The client is experiencing an anaphylactic reaction to streptokinase, which is allergenic. The infusion should be stopped, the physician notified, and the client treated with epinephrine, antihistamines, and corticosteroids. Test-Taking Strategy: Recall that an allergic reaction and possible anaphylaxis are risks associated with streptokinase therapy. Also, focusing on the signs and symptoms in the question will assist in answering the question. When a severe allergic reaction occurs, the offending substance should be stopped, and lifesaving treatment should begin. Review the adverse effects of this medication if you had difficulty with this question.

731. A client is admitted with pulmonary embolism and is to be treated with streptokinase (Streptase). A nurse would report which of the following assessments to the physician before initiating this therapy? 1. Adventitious breath sounds 2. Temperature of 99.4° F orally 3. Blood pressure of 198/110 mm Hg 4. Respiratory rate of 28 breaths/min

731. 3 Rationale: Thrombolytic therapy is contraindicated in a number of preexisting conditions in which there is a risk of uncontrolled bleeding, similar to the case in anticoagulant therapy. Thrombolytic therapy also is contraindicated in severe uncontrolled hypertension because of the risk of cerebral hemorrhage. Therefore the nurse would report the results of the blood pressure to the physician before initiating therapy. Test-Taking Strategy: Use the process of elimination and focus on the client's diagnosis. Options 1, 2, and 4 may be present in the client with pulmonary embolism but are not necessarily signs that warrant reporting before this therapy is initiated. Review the contraindications associated with the administration of this medication if you had difficulty with this question.

732. The nurse is monitoring a client who is taking digoxin (Lanoxin) for adverse effects. Which findings are characteristic of digoxin toxicity. Select all that apply. 1. Tremors 2. Diarrhea 3. Irritability 4. Blurred vision 5. Nausea and vomiting

732. 2, 4, 5 Rationale: Digoxin (Lanoxin) is a cardiac glycoside. The risk of toxicity can occur with the use of this medication. Toxicity can lead to life-threatening events and the nurse needs to monitor the client closely for signs of toxicity. Early signs of toxicity include gastrointestinal manifestations such as anorexia, nausea, vomiting, and diarrhea. Subsequent manifestations include headache, visual disturbances such as diplopia, blurred vision, yellow-green halos, photophobia, drowsiness, fatigue, and weakness. Cardiac rhythm abnormalities can also occur. The nurse also monitors the digoxin level. Therapeutic levels for digoxin range from 0.5 to 2 ng/mL. Test-Taking Strategy: Specific knowledge regarding the characteristics of digoxin toxicity is needed to answer this question. Recall that the early signs are gastrointestinal manifestations. Next, recall that visual disturbances occur. If you had difficulty with this question review the manifestations associated with digoxin toxicity.

Apgar scoring interventions

8- 10 no interventions required, support newborn's spontaneous efforts 4 - 7 gently stimulate; rub newborn back and admin O2 0 - 3 newborn requires resusciatation

8. A home health care nurse is visiting an older client at home. Furosemide (Lasix) is prescribed for the client and the nurse teaches the client about the medication. Which of the following statements, if made by the client, indicates the need for further teaching? 1. "I will sit up slowly before standing each morning." 2. "I will take my medication every morning with breakfast." 3. "I need to drink lots of coffee and tea to keep myself healthy." 4. "I will call my doctor if my ankles swell or my rings get tight."

8. 3 Rationale: Tea and coffee are stimulants and mild diuretics. These are a poor choice for hydration. Taking the medication at the same time each day improves compliance. Because furosemide is a diuretic, the morning is the best time to take the medication so as not to interrupt sleep. Notification of the health care provider is appropriate if edema is noticed in the hands, feet, or face or if the client is short of breath. Sitting up slowly prevents postural hypotension. Test-Taking Strategy: Use the process of elimination, noting the strategic words need for further teaching. These words indicate a negative event query and ask you to select an option that is an incorrect statement. Recalling that tea and coffee are stimulants and that diuretics potentially can worsen dehydration will direct you to option 3. In addition, coffee and tea are not healthy items to consume. Review client teaching points related to this medication if you had difficulty with this question.

Calcium

8.6 - 10 mg/L

Amniotic fluid

800 - 1200 ml at end of pregnancy surrounds, cushions,protects, maintains fetal body temp and measures kidney function litmus blue=amniotic green =urine

9. A client who has developed severe pulmonary edema would most likely exhibit which of the following? 1. Mild anxiety 2. Slight anxiety 3. Extreme anxiety 4. Moderate anxiety

9. 3 Rationale: Pulmonary edema causes the client to be extremely agitated and anxious. The client may complain of a sense of drowning, suffocation, or smothering. Test-Taking Strategy Use the process of elimination. Noting the strategic word severe will direct you to option 3. Review the clinical manifestations associated with severe pulmonary edema if you had difficulty with this question.

9. A nurse is caring for a client receiving a heparin intravenous (IV) infusion. The nurse anticipates that which laboratory study will be prescribed to monitor the therapeutic effect of heparin? 1. Hematocrit 2. Hemoglobin 3. Prothrombin time 4. Activated partial thromboplastin time

9. 4 Rationale: The prothrombin time will assess for the therapeutic effect of warfarin sodium (Coumadin), and the activated partial thromboplastin time (aPTT) will assess the therapeutic effect of heparin. Hematocrit and hemoglobin values assess red blood cell concentrations. Baseline assessment, including an aPTT value, should be completed, as well as ongoing daily aPTT values while the client is taking heparin. Heparin doses are determined based on the result of the aPTT. Test-Taking Strategy: Use the process of elimination. Eliminate options 1 and 2 because they are comparative or alike and are unrelated to heparin therapy. From the remaining options, recall the relationship between the prothrombin time and warfarin and the aPTT and heparin. Review care of a client on heparin infusion if you had difficulty with this question.

61.The nurse is caring for a client with hepatitis A. Which of the following client statements indicates that teaching conducted by the nurse about disease transmission was effective? Select all that apply.</question> <question_image/> <choice_1>"We must avoid kissing."</choice_1> <choice_2>"We can use the same bath towels."</choice_2> <choice_3>"We must avoid eating with the same utensils."</choice_3> <choice_4>"We must wear masks."</choice_4> <choice_5>"No special precautions are needed."</choice_5>

<correct>1, 3</correct> <rationale>Hepatitis A is an infectious disease transmitted by the fecal-oral route. Standard precautions are mandatory. Contact precautions are instituted if the client is incontinent of stool. Family members should avoid close contact with the client. They should not kiss the client or use the same eating utensils and bath towels. Masks are not necessary because the disease is not transmitted by the respiratory tract.</rationale>

72.Which of the following statements by the client could indicate a potential problem for the couple planning to use coitus interruptus?</question> <question_image/> <choice_1>"I really don't want to get pregnant right now, so we need a very effective method."</choice_1> <choice_2>"I think I can always pull out before I ejaculate."</choice_2> <choice_3>"We don't have any other sex partners."</choice_3> <choice_4>"We want a contraceptive method that is inexpensive and completely natural."</choice_4>

<correct>1</correct> <rationale>Because some semen is released before ejaculation, coitus interruptus has an 18% failure rate and would not be considered a very effective method for a couple wanting to avoid pregnancy.</rationale>

66.What information does the nurse need to gather before scheduling a client's endometrial biopsy?</question> <question_image/> <choice_1>Usual length of menstrual cycle</choice_1> <choice_2>Blood type and Rh factor</choice_2> <choice_3>Presence of any metal implants</choice_3> <choice_4>Last type of birth control used</choice_4>

<correct>1</correct> <rationale>The nurse needs to know the first day of the last normal menstrual period and the length of the menstrual cycle. Endometrial biopsy is performed on day 21 to 27 of the menstrual cycle to assess the endometrial response to progesterone and the degree of development of the luteal phase endometrium.</rationale> <cognitive_level>Application</cognitive_level>

98.The pregnant client tells the nurse that she is lactose intolerant. When considering the recommendation of a calcium supplement, which of the following data should be collected?</question> <question_image/> <choice_1>History of kidney stones</choice_1> <choice_2>Presence of leg cramps</choice_2> <choice_3>Color of mucous membranes and conjunctiva</choice_3> <choice_4>Resting heart rate</choice_4>

<correct>1</correct> <rationale>Increased calcium intake can lead to formation of kidney stones. A calcium supplement is not expected to affect leg cramps, color of mucous membranes and conjunctiva, or resting heart rate.</rationale>

62.Which of the following statements indicates to the nurse that a couple is coping with the stress of infertility treatment?</question> <question_image/> <choice_1>"We are really trying to maintain a little romance in our relationship." </choice_1> <choice_2>"My wife was so upset that she threw the syringe at me yesterday."</choice_2> <choice_3>"My husband couldn't even have an erection when he was supposed to."</choice_3> <choice_4>"We have two or three glasses of wine each night to help us relax."</choice_4>

<correct>1</correct> <rationale>Maintaining a healthy relationship is important during infertility treatments, which can be very stressful. Options 2, 3, and 4 may indicate ineffective coping strategies and warrant further investigation.</rationale>

50.A client with suspected severe acute respiratory syndrome (SARS) arrives at the emergency department. Which of the following physician orders should the nurse implement first?</question> <question_image/> <choice_1>Airborne and contact isolation</choice_1> <choice_2>IV D<sub>5</sub>NS at 100 mL/hr</choice_2> <choice_3>Nasopharyngeal culture for reverse-transcription polymerase chain reaction</choice_3> <choice_4>Sputum for enzyme immunoassay testing</choice_4> <choice_5/>

<correct>1</correct> <rationale>SARS is a highly contagious viral respiratory illness that is spread by close person-to-person contact. SARS is transmitted by airborne respiratory droplets and by touching surfaces and objects contaminated with infectious droplets. Instituting infection control measures would be the first priority of the nurse. This action would protect both health care workers and other clients in the emergency department. Then all other interventions can be safely implemented.</rationale>

29. The nurse supervisor observes the new LPN/LVN administering medications on the unit. The nursing supervisor concludes there is a risk for medication error when the nurse does which of the following?</question> <question_image/> <choice_1>Answers a physician's page while passing medications</choice_1> <choice_2>Uses military time for documentation</choice_2> <choice_3>Asks for help with a dosage calculation</choice_3> <choice_4>Does not give a medication that the client questions</choice_4>

<correct>1</correct> <rationale>The nurse should never interrupt the process for administering medications. Errors are typically made when the nurse is interrupted. Military time is frequently used by institutions for documentation. The nurse should always ask for assistance with dosage calculations when in doubt. The nurse should never give a medication that a client questions. Always double check the order, dosage, and medication, and give the client an explanation.</rationale>

60.The nurse assigned to the respiratory care unit is working with four clients who have pneumonia. The nurse should assign the only remaining private room on the nursing unit to the client infected with which of the following organisms?</question> <question_image/> <choice_1>Penicillin-resistant <i>Streptococcus pneumoniae</i> pneumonia</choice_1> <choice_2><i>Pseudomonas aeruginosa</i> pneumonia</choice_2> <choice_3><i>Pneumocystis carinii</i> pneumonia</choice_3> <choice_4><i>Legionella pneumophila</i> pneumonia</choice_4>

<correct>1</correct> <rationale>While each option contains "pneumonia," the causative agent is different for each. Option 1 includes a pathogenic microorganism that is difficult to treat and requires droplet precautions.</rationale>

95.A pregnant client, who is a vegetarian, is concerned about her folic acid intake and asks the nurse to recommend some foods that she should include in her diet. Which of the following should the nurse recommend?</question> <question_image/> <choice_1>Peanuts</choice_1> <choice_2>Hamburger</choice_2> <choice_3>Bananas</choice_3> <choice_4>Apple juice</choice_4>

<correct>1</correct> <rationale>Both peanuts and hamburger are good sources of folic acid, but since the client is a vegetarian, peanuts is a better recommendation. The other options do not contain significant amounts of folic acid.</rationale>

58.The nurse is assisting the client who has methicillin-resistant <i>Staphylococcus aureus</i> in collecting a clean-catch urine specimen. Which of the following protective equipment is unnecessary?</question> <question_image/> <choice_1>N95 particulate respirator</choice_1> <choice_2>Gown</choice_2> <choice_3>Eye protection</choice_3> <choice_4>Sterile gloves</choice_4>

<correct>1</correct> <rationale>Methicillin-resistant <i>Staphylococcus aureus</i> requires transmission-based contact precautions. Eye protection would be worn to protect the mucous membranes of the eyes when splatters of body fluids or excretions are possible. A gown would be worn when the nurse is in direct contact with the client. Contact precautions require gloves. N95 respirators are needed when caring for the client with tuberculosis, so it is inappropriate for this scenario.</rationale>

90.During a prenatal visit in the second trimester, which of the following, if reported by the client, would be a cause for concern?</question> <question_image/> <choice_1>Thirst and urinary frequency</choice_1> <choice_2>+1 deep tendon reflexes</choice_2> <choice_3>Constipation</choice_3> <choice_4>Backache in the lower sacral area</choice_4>

<correct>1</correct> <rationale>Urinary frequency usually disappears in the second trimester. Thirst and urinary frequency may be a sign of developing gestational diabetes and warrants further investigation. Deep tendon reflexes are assessed during a physical examination and are not reported to a health care provider by the client.</rationale>

80.In teaching the client about factors that can decrease the effectiveness of oral contraceptives, which of the following should be included by the nurse?</question> <question_image/> <choice_1>Antibiotic use</choice_1> <choice_2>Weight gain</choice_2> <choice_3>Amenorrhea</choice_3> <choice_4>Iron-deficiency anemia</choice_4>

<correct>1</correct> <rationale>Antibiotic use can decrease the effectiveness of oral contraceptives. Oral contraceptives can help prevent iron-deficient anemia by decreasing menstrual blood flow. Weight gain and anemia are not related to the effectiveness of birth control pills.</rationale>

54.The nurse would formulate which of the following as the most appropriate goal for the client with droplet precautions?</question> <question_image/> <choice_1>The client will identify three ways to reduce the spread of infection.</choice_1> <choice_2>The client will limit the risk of exposure to the causative agent.</choice_2> <choice_3>The client will be taught how to take antimicrobial medication.</choice_3> <choice_4>The client will understand how to protect other family members.</choice_4>

<correct>1</correct> <rationale>Option 1 is the only goal that is client-focused, specific, and measurable. Options 2 and 4 are client-focused but vague. Option 3 focuses on the nursing action of teaching.</rationale>

The client, who delivered her first child 2 days ago, is being discharged from the hospital. She is interested in a contraceptive method that is not associated with intercourse and will not interfere with lactation. The nurse concludes that which of the following probably would be the best method for this client?</question> <question_image/> <choice_1>Progestin-only oral contraceptives (mini-pills)</choice_1> <choice_2>Female condoms</choice_2> <choice_3>Diaphragm</choice_3> <choice_4>Triphasic pills</choice_4>

<correct>1</correct> <rationale>Oral contraceptives with a combination of estrogen and progestin are not recommended in the first 6 weeks of lactation. In addition, the long-term effects on the infant are not known. The use of female condoms and a diaphragm are associated with sexual intercourse. Progestin-only pills are safe for lactating women.</rationale>

92.When considering maternal serum alpha-fetoprotein testing for a client, the nurse would conclude that there is a contraindication for the test if the client:</question> <question_image/> <choice_1>Is at 25 weeks' gestation.</choice_1> <choice_2>Would not consider termination of the pregnancy.</choice_2> <choice_3>Does not have a family history of neural tube defects.</choice_3> <choice_4>Had an ultrasound at 8 weeks' gestation.</choice_4>

<correct>1</correct> <rationale>This test, which measures the level of maternal serum alpha-fetoprotein, is most sensitive between 16 to 18 weeks' gestation. However, it can be performed up to 22 weeks' gestation.</rationale>

48.A client with tuberculosis asks the nurse if visitors will need to wear masks. What response by the nurse is most accurate?</question> <question_image/> <choice_1>"Everyone who enters your room must wear a mask to protect themselves from tuberculosis."</choice_1> <choice_2>"Masks would not be necessary for visitors who have had tuberculosis before."</choice_2> <choice_3>"It is less important for your family to wear masks, since they live in close contact with you."</choice_3> <choice_4>"Only visitors who are at risk for tuberculosis need to wear a mask."</choice_4>

<correct>1</correct> <rationale>Tuberculosis is highly contagious and spread by inhalation of airborne droplets. Airborne precautions would be initiated, requiring everyone to wear a special particulate respirator fit-tested mask. Individuals who have had tuberculosis in the past can be re-exposed and develop the active form of the disease again.</rationale>

27. A school nurse is planning a health class on accidents and injuries for a high school class. Which topic is most important to include?</question> <question_image/> <choice_1>Occupational-related injuries at work</choice_1> <choice_2>Motor vehicle-related injuries</choice_2> <choice_3>Fall-related injuries</choice_3> <choice_4>Injury due to residential fires</choice_4>

<correct>2</correct> <rationale>Driving a car and having the independence to ride with friends is an important milestone for high school-aged adolescents. Some adolescents experiment with alcohol and drugs, putting them at increased risk for motor vehicle accidents. Option 1 is a risk for working adults, and options 3 and 4 are risk factors for the elderly.</rationale>

45.The nurse would do which of the following to protect the client from infection at the portal of entry?</question> <question_image/> <choice_1>Place a sputum specimen in a biohazard bag for transport to the lab.</choice_1> <choice_2>Empty a Jackson-Pratt drain, using sterile technique.</choice_2> <choice_3>Dispose of soiled gloves in a waste container.</choice_3> <choice_4>Wash hands after providing client care.</choice_4>

<correct>2</correct> <rationale>Option 2 is an action aimed at interrupting the portal of entry link in the chain of infection. By using sterile technique, the nurse reduces the risk of introducing pathogens into the client's wound via the drain. Option 1 is an action that breaks the chain of infection at the reservoir link. Options 3 and 4 control the mode of transmission.</rationale

89.With regard to normal changes in the cardiovascular system during pregnancy, the nurse should reinforce teaching to the pregnant client about which of the following?</question> <question_image/> <choice_1>Her pulse rate will decrease.</choice_1> <choice_2>She may experience dizziness if she lays on her back.</choice_2> <choice_3>She will have a decrease in red blood cells.</choice_3> <choice_4>She may experience a feeling of fullness in her chest.</choice_4>

<correct>2</correct> <rationale>Pressure on the vena cava from the gravid uterus may cause a decrease in blood flow to the right atrium and result in a decrease in blood pressure. Dizziness is a symptom of hypotension. The pulse rate could stay the same or increase as the workload of the heart increases during the course of pregnancy. There is an increase in the number of red blood cells to meet physiological demand. Option 4 is not a cardiovascular change during pregnancy, although abdominal fullness occurs as the pregnancy progresses.</rationale>

69.The clinic nurse is interviewing a client couple for an initial infertility workup. Which of the following topics should the nurse plan to address?</question> <question_image/> <choice_1>Whether the couple has medical insurance</choice_1> <choice_2>How infertility is affecting their lives</choice_2> <choice_3>Whether the man has seafood allergies</choice_3> <choice_4>Whether the woman works outside the home</choice_4>

<correct>2</correct> <rationale>The psychological, cultural, and social ramifications of infertility can be extensive. You need to ascertain if the couple needs assistance in coping with their infertility and treatment.</rationale>

52.The nurse is changing an abdominal dressing on a client who has an infection spread by droplets. Which of the following pieces of personal protective equipment would the nurse use?</question> <question_image/> <choice_1>Clean gloves</choice_1> <choice_2>Mask</choice_2> <choice_3>Gown</choice_3> <choice_4>Eye protection</choice_4>

<correct>2</correct> <rationale>A mask is necessary for anyone within 3 feet of a client with an infection spread by particle droplets. There is not enough information in the question to support the use of any other equipment.</rationale>

100.The pregnant client, who is 34 weeks' gestation, calls the prenatal clinic complaining of cramping pain in her abdomen. After the diagnosis of Braxton-Hicks contractions is made, the nurse should give the client which of the following recommendations?</question> <question_image/> <choice_1>"Go to bed and wait for your real labor to begin."</choice_1> <choice_2>"Empty your bladder frequently and change positions if these contractions are bothering you."</choice_2> <choice_3>"Avoid using your Lamaze breathing with these contractions because it might precipitate preterm labor."</choice_3> <choice_4>"Just ignore these contractions; we will let you know if there is a problem."</choice_4>

<correct>2</correct> <rationale>Braxton-Hicks contractions are probably caused by stretching of the myometrium. They are usually relieved by position changes, frequent emptying of the bladder, resting in a lateral recumbent position, and walking or light exercise.</rationale>

71.The client has an obstruction between the uterus and the fallopian tubes. In obtaining a health history, the nurse collects information about which of the following that may have caused this problem?</question> <question_image/> <choice_1>Rubella infection prior to adolescence</choice_1> <choice_2>Pelvic inflammatory disease caused by gonorrhea</choice_2> <choice_3>Smoking two packs of cigarettes per day</choice_3> <choice_4>Ingestion of 2 ounces of alcohol daily</choice_4>

<correct>2</correct> <rationale>Infectious processes of the reproductive tract such as PID may result in tubal scarring and therefore tubal blockage. Rubella infection in childhood usually results in the development of active immunity to the disease. Smoking and alcohol present health risks to the woman but not related to tubal patency.</rationale> <cognitive_level>Analysis</cognitive_level>

75.The client is interested in using female condoms and wants to know if there are any disadvantages. The nurse's best response would be:</question> <question_image/> <choice_1>"The female condom provides good protection against pregnancy but does not protect against sexually transmitted infections."</choice_1> <choice_2>"The female condom may be difficult to insert and may be uncomfortable to both partners."</choice_2> <choice_3>"The female condom is very effective; let me write you a prescription for some."</choice_3> <choice_4>"The female condom is made of latex and should not be used when allergy is present."</choice_4>

<correct>2</correct> <rationale>Made of polyurethane, the female condom does not require a prescription but can be difficult to insert and can cause discomfort. It is effective against both sexually transmitted infections and pregnancy.</rationale>

51.In addition to standard precautions, which other type(s) of isolation precautions should the nurse use when caring for the client with severe acute respiratory syndrome (SARS)?</question> <question_image/> <choice_1>Droplet precautions</choice_1> <choice_2>Airborne precautions and contact precautions</choice_2> <choice_3>Contact precautions and droplet precautions</choice_3> <choice_4>Airborne precautions</choice_4>

<correct>2</correct> <rationale>SARS is a highly contagious viral respiratory illness that is spread by close person-to-person contact. SARS is transmitted by airborne respiratory droplets and by touching surfaces and objects contaminated with infectious droplets. Personal protective equipment would include protective gowns, gloves, N95 respirators, and eye protection. Airborne precautions would also include placing the client in a private room with negative air pressure flow. The correct answer is option 2. Airborne and contact precautions would provide the necessary protection outlined above. Options 1 and 3 are incorrect. Droplet precautions would not protect the nurse who touches contaminated items. Droplet precautions do not provide a negative air pressure room. Option 4 is incorrect. Contact precautions alone would not provide adequate protection from airborne particles.</rationale>

The client is experiencing an inability to become pregnant after she has had one full-term pregnancy. The nurse would know this client is suffering from which health problem?</question> <question_image/> <choice_1>Primary infertility</choice_1> <choice_2>Secondary infertility</choice_2> <choice_3>Unexplained infertility</choice_3> <choice_4>Combined factor infertility</choice_4>

<correct>2</correct> <rationale>Secondary infertility is the term for couples that have had one pregnancy but are unable to conceive again. Primary infertility describes the inability to conceive even once. Options 3 and 4 are not terms that are used when discussing fertility.</rationale>

97.The pregnant client has been started on an iron supplement. Which of the following information should the nurse include as a priority when reinforcing prenatal teaching about the iron supplement?</question> <question_image/> <choice_1>It should be taken 30 minutes after eating a full meal.</choice_1> <choice_2>It is better absorbed if taken with a liquid containing Vitamin C.</choice_2> <choice_3>It will eliminate the need for prenatal vitamins.</choice_3> <choice_4>It should be taken at the same time as the prenatal vitamin.</choice_4>

<correct>2</correct> <rationale>Iron is absorbed best on an empty stomach (not after a full meal) and in the presence of Vitamin C. It may or may not be taken at the same time as other vitamin supplementation. It does not replace the need for other vitamins.</rationale>

83.Which of the following statements would indicate to the nurse that teaching was effective for the client who is to receive a Norplant subdermal implant?</question> <question_image/> <choice_1>"By the end of 5 years, the capsules will be absorbed by my body."</choice_1> <choice_2>"If I get the Norplant implant, for 5 years I will have about the same risk of pregnancy as if I had surgical sterilization."</choice_2> <choice_3>"I will need to wait until I am 18 to receive a Norplant implant."</choice_3> <choice_4>"I will need to come to the clinic to have the implant reinserted every 3 months."</choice_4>

<correct>2</correct> <rationale>Norplant is a subdermal contraceptive implant that has about the same failure rate as surgical sterilization, is effective for 5 years, and must be surgically removed.</rationale>

81.In addition to prevention of pregnancy, oral contraceptives would provide benefits for a client with which of the following problems?</question> <question_image/> <choice_1>Pelvic inflammatory disease</choice_1> <choice_2>Severe facial acne</choice_2> <choice_3>Chloasma</choice_3> <choice_4>Gallbladder disease</choice_4

<correct>2</correct> <rationale>Oral contraceptives can reduce acne, result in signs and symptoms of early pregnancy including chloasma, and accelerate the progress of gallbladder disease. Birth control pills do not provide protection against STIs that can result in PID.</rationale>

85.In reinforcing instructions for the client who is to receive Depo-Provera, the nurse should tell the client which of the following?</question> <question_image/> <choice_1>Like oral contraceptives, Depo-Provera increases the risk of venous thrombosis.</choice_1> <choice_2>The most common side effects of Depo-Provera are amenorrhea and irregular uterine bleeding.</choice_2> <choice_3>Depo-Provera has a higher failure rate than oral contraceptives.</choice_3> <choice_4>Depo-Provera will interfere with lactation.</choice_4>

<correct>2</correct> <rationale>The most common side effects of Depo-Provera are amenorrhea and irregular bleeding. With a failure rate similar to oral contraceptives, Depo-Provera does not interfere with lactation. Typically, the estrogen component of hormonal contraceptives is associated with thromboembolic disease; Depo-Provera contains only progestin.</rationale>

13. The charge nurse on the night shift reports that the narcotic count is incorrect. The nurse has already spoken to the staff nurse believed responsible for the incorrect count and has reason to believe that substance abuse by the nurse is the cause. If substance abuse by the staff nurse proves to be the cause of the incorrect count, what is the most appropriate next step? <choice_1>Recount the narcotics with the staff nurse and take disciplinary action.</choice_1> <choice_2>Ask the staff nurse to leave the unit immediately and report the incident to the American Nurses Association.</choice_2> <choice_3>Complete an incident report, report findings to the pharmacy, and notify the nursing administration for the facility.</choice_3> <choice_4>Submit the findings to the Council on Nursing Practice

<correct>3</correct> <rationale>An incident report must be completed because of the inaccurate narcotic count. Narcotics are controlled substances and fall under federal law and regulation. Both the pharmacy and nursing administration must be notified. If the staff nurse is found to be using a controlled substance, this finding must be reported to the state board of nursing. Individual state boards of nursing identify the legal boundaries of nursing practice, including disciplinary action, through nurse practice acts (which differ among the states). The American Nurses Association, through the Code of Ethics for Nurses, provides guidance to nurses and protection for clients and their families but does not have the authority to discipline nurses.</rationale> <cognitive_level>Application</cognitive_level> <client_need>Safe Effective Care Environment: Coordinated Care</client_need> <integrated_process>Nursing Process: Planning</integrated_process> <content_area>Leadership/Management</content_area> <strategy>The core issue of the question is an understanding of the nature and purpose of professional nursing organizations and institutions. Each state board is responsible for the regulation of nursing and articulates the principles for delegation and disciplinary action. Options 1 and 2 represent actions that a nurse manager would have to take to protect the public good. The functions of professional nursing organizations do not include statutory laws but rather ethical codes of conduct for both nursing students and professional nurses. Option 4 refers to professional organizations like ANA who may have a Council on Nursing Practice which strive to develop standards of practice for professional nurses.</strategy>

91.The nurse is collecting data on a client who is 12 weeks' gestation. The examiner would expect to find the fundus at which of the following locations?</question> <question_image/> <choice_1>3 cm below the sternum</choice_1> <choice_2>The level of the umbilicus</choice_2> <choice_3>The level of the symphysis pubis</choice_3> <choice_4>3 cm below the umbilicus</choice_4>

<correct>3</correct> <rationale>By the twelfth week of gestation, the uterus should have increased in size to be palpable at the symphysis pubis. Factors affecting this finding include abnormal fetal growth or the presence of a multiple gestation.</rationale>

79.The client has come to the clinic to discuss use of a cervical cap for contraception. Which of the following would the nurse view as a contraindication to use of the cervical cap?</question> <question_image/> <choice_1>History of blood clots</choice_1> <choice_2>Age greater than 35 years</choice_2> <choice_3>Abnormal Pap smear 6 months ago</choice_3> <choice_4>Elevated liver enzymes</choice_4>

<correct>3</correct> <rationale>Long-term exposure to secretions, spermicides, and bacteria trapped inside the cap can result in abnormal Pap smear results. This client has a history of an abnormal Pap smear; cervical cap use could negatively impact this finding, and another method should be explored for this client. The other options have no relationship to use of the cervical cap.</rationale>

99.During the first prenatal exam, the nurse discovers that the client has not had a second vaccination for measles, mumps, and rubella. The nurse would expect the provider to do which of the following?</question> <question_image/> <choice_1>Administer the vaccine during this visit.</choice_1> <choice_2>Wait until the third trimester to administer the vaccine.</choice_2> <choice_3>Administer the vaccine following delivery.</choice_3> <choice_4>Omit the vaccine because these are childhood diseases not acquired by adults.</choice_4>

<correct>3</correct> <rationale>The measles, mumps, and rubella vaccine contains live, attenuated virus and could cause disease and harm to the fetus during pregnancy. It should be given after delivery, and the woman should avoid conceiving for 3 months.</rationale>

63.The client has been diagnosed with <i>Trichomoniasis vaginitis</i>. The nurse explains while reinforcing client teaching that this infection can affect fertility by:</question> <question_image/> <choice_1>Utilizing the glycogen in vaginal secretions, leaving no nutrition for the spermatozoa.</choice_1> <choice_2>Creating a blockage of the fallopian tubes that prohibits spermatozoa from reaching an ovum.</choice_2> <choice_3>Decreasing the pH of the vaginal secretions, thereby destroying most spermatozoa.</choice_3> <choice_4>Increasing the temperature inside the vagina, which decreases the motility of spermatozoa.</choice_4>

<correct>3</correct> <rationale>Vaginal fluid pH is slightly alkaline, as is semen. Spermatozoa cannot survive in an acidic environment. Trichomoniasis vaginitis increases the acidity of the vaginal and cervical secretions, thus reducing the number of viable sperm.</rationale>

68.The client couple is planning intracytoplasmic sperm injection, followed by intrauterine embryo transfer. Which of the following statements indicates that the nurse's teaching was effective?</question> <question_image/> <choice_1>"His sperm swim too fast for me to become pregnant."</choice_1> <choice_2>"My eggs have thick walls and don't let his sperm in."</choice_2> <choice_3>"Any extra embryos can be frozen for implantation later."</choice_3> <choice_4>"We will have to wait several weeks to see if any eggs get fertilized."</choice_4>

<correct>3</correct> <rationale>In vitro fertilization usually creates multiple embryos, of which up to four are implanted. Cryopreservation of excess embryos is common, and they can be implanted at a later date.</rationale>

65.Which of the following client statements indicates the need for additional teaching?</question> <question_image/> <choice_1>"I should come back for a postcoital test 1 to 2 days before I expect to ovulate."</choice_1> <choice_2>"I should schedule my hysterosalpingogram for the week after ovulation."</choice_2> <choice_3>"We should abstain for 14 days prior to coming back for the sperm penetration test."</choice_3> <choice_4>"I should schedule my endometrial biopsy for the last week of my menstrual cycle."</choice_4>

<correct>3</correct> <rationale>Sperm penetration test, which tests for the ability of sperm to penetrate an egg, should be performed after 2 to 7 days of abstinence.</rationale>

74. The client who is married and has three children has come to the family planning clinic asking about a birth control method that is sanctioned by the Roman Catholic Church. She wants the most effective method possible. The nurse's best recommendation is which of the following?</question> <question_image/> <choice_1>Billings or cervical assessment method</choice_1> <choice_2>Ovulation testing kit</choice_2> <choice_3>Symptothermal method</choice_3> <choice_4>Basal body temperature method</choice_4>

<correct>3</correct> <rationale>The symptothermal method combines cervical mucus and BBT measurements and results in a lower failure rate than single assessments of the fertile period. This method is completely natural and acceptable to the beliefs of this religious group. Ovulation testing kits do not give enough warning of ovulation to prevent pregnancy.</rationale>

94.The nurse determines the client understands her prenatal nutritional education when she states:</question> <question_image/> <choice_1>"I understand that if I don't eat foods with folic acid, my baby will have birth defects."</choice_1> <choice_2>"I understand that eating citrus fruits, especially oranges, will help me meet my need for folic acid."</choice_2> <choice_3>"I understand that if my level of folic acid is low, it could cause my baby to have a neural tube defect."</choice_3> <choice_4>"I understand that I should limit my intake of folic acid because it can build up in the liver and cause birth defects."</choice_4>

<correct>3</correct> <rationale>Maternal folic acid deficiency has been linked to infant neural tube defects. Folic acid may be obtained from prenatal vitamin supplements as well as foods. The other responses contain incorrect statements and do not indicate understanding of prenatal nutrition.</rationale> <cognitive_level>Analysis</cognitive_level>

84.The nurse is preparing to administer an injection of Depo-Provera. Which of the following would result in safe and effective administration of this drug?</question> <question_image/> <choice_1>Check to see that it has been at least 8 weeks since the client's last injection.</choice_1> <choice_2>Determine that the client's hemoglobin level is within normal range.</choice_2> <choice_3>Using a 23-gauge needle, inject the medication into the deltoid muscle.</choice_3> <choice_4>Check the client's medical record for a history of pelvic inflammatory disease.</choice_4>

<correct>3</correct> <rationale>The medication is administered intramuscularly every 80 to 90 days. Anemia, while important to the client's health, is not related to Depo-Provera use. The drug does not provide protection against sexually transmitted infections; counseling regarding the consistent use of condoms would be an effective intervention to prevent the reoccurrence of pelvic inflammatory disease.</rationale>

76.Which of the following clients would be the best candidate for insertion of an intrauterine device?</question> <question_image/> <choice_1>A client who is married, has one child, and wants to get pregnant in about 6 months</choice_1> <choice_2>A client who is unmarried, has no children, and has numerous sexual partners</choice_2> <choice_3>A client who is married, has two children, and does not want more children for at least 3 years</choice_3> <choice_4>A client who is unmarried, has one child, and has a history of pelvic inflammatory disease</choice_4>

<correct>3</correct> <rationale>Intrauterine devices are usually recommended for women who have been pregnant and are in a monogamous relationship so that they are at a low risk for sexually transmitted disease.</rationale>

73.Which of the following, if stated by the client, would indicate that teaching about cervical mucus changes as an indicator of ovulation has been understood?</question> <question_image/> <choice_1>"If my cervical mucus is yellowish and thick, I am probably fertile."</choice_1> <choice_2>"The thin, clear mucus will block sperm from getting to my cervix."</choice_2> <choice_3>"If my cervical mucus is thick and white, I will need to avoid intercourse or use a back-up method of contraception."</choice_3> <choice_4>"If my cervical mucus is thin and stretchable, I am probably fertile."</choice_4>

<correct>4</correct> <rationale>Cervical mucus that is thin and clear indicates a rising level of estrogen and impending ovulation. Stretchability of the cervical mucus, or spinnbarkeit, is indicative of the fertile period and promotes motility of the sperm. Options 1 and 3 represent cervical mucus during the infertile period when sexual intercourse is unlikely to result in pregnancy.</rationale>

57.A client asks, "How did I get scarlet fever?" Which of the following would be the best response by the nurse?</question> <question_image/> <choice_1>"Scarlet fever is transmitted through sexual intercourse."</choice_1> <choice_2>"You can get scarlet fever if you share contaminated needles or get a blood transfusion."</choice_2> <choice_3>"Most people get it by eating contaminated food."</choice_3> <choice_4>"You inhaled infected droplets in the air."</choice_4>

<correct>4</correct> <rationale>Scarlet fever is transmitted by particle droplets larger than 5 microns. Scarlet fever is not transmitted through sexual intercourse or the blood or by consuming contaminated food.</rationale>

56.The nurse would expect to institute transmission-based precautions for a client with which of the following?</question> <question_image/> <choice_1>Pneumonia caused by <i>Pseudomonas aeruginosa</i></choice_1> <choice_2><i>Pneumocystis carinii</i> pneumonia</choice_2> <choice_3>A sacral wound contaminated by <i>Escherichia coli</i></choice_3> <choice_4>A draining leg wound with methicillin-resistant <i>Staphylococcus aureus</i></choice_4>

<correct>4</correct> <rationale>Transmission-based precautions are required for all antibiotic-resistant microorganisms regardless of their mode of transmission. The other options indicate the need for medical and surgical asepsis in the care of the client but not the use of transmission-based precautions.</rationale>

59.The nurse is preparing to irrigate a wound infected with vancomycin-resistant <i>enterococci</i>. Which of the following should the nurse wear?</question> <question_image/> < choice_1>Gloves, gown, and particulate respirator</choice_1> <choice_2>Gloves and surgical mask</choice_2> <choice_3>Gloves, eye protection, and particulate respirator</choice_3> <choice_4>Gloves, gown, eye protection, and surgical mask</choice_4>

<correct>4</correct> <rationale>An infection with vancomycin-resistant <i>enterococci</i> requires transmission-based contact precautions. Since the nurse will be irrigating the wound and splatters of body fluids or exudates are possible, eye protection and surgical mask should be worn to protect the mucous membranes of the eyes, nose, and mouth. A gown would be worn when the nurse is in direct contact with the client. Contact precautions require gloves.</rationale>

87.The client has come to the clinic for her first prenatal visit. During the pelvic examination, the examiner indicates that the vaginal mucosa has a bluish color. The nurse documents this as a positive:</question> <question_image/> <choice_1>Hegar's sign.</choice_1> <choice_2>Goodell's sign.</choice_2> <choice_3>McDonald's sign.</choice_3> <choice_4>Chadwick's sign.</choice_4>

<correct>4</correct> <rationale>Beginning around the fourth week of pregnancy, vasocongestion in the pelvic area results in a bluish color to the vulva, vagina, and cervix, known as Chadwick's sign. Hegar's sign is a softening of the lower uterine segment, Goodell's sign is a softening of the cervix, and McDonald's sign is an ease in flexing the body of the uterus against the cervix; none of these other signs involve color changes.</rationale>

In teaching a client about the risk of toxic shock syndrome associated with diaphragm use, the nurse should tell the client to do which of the following to decrease her risk?</question> <question_image/> <choice_1>Leave the diaphragm in place for 36 to 48 hours after intercourse.</choice_1> <choice_2>Avoid using soap when cleaning the device.</choice_2> <choice_3>Wear latex or rubber gloves when handling the device.</choice_3> <choice_4>Seek treatment of any vaginal infection before reusing the device.</choice_4>

<correct>4</correct> <rationale>When using the device, the woman should wash her hands with soap and water, remove the device within 24 hours of intercourse, clean the device with soap and water, and seek treatment for vaginal infections before reusing the device.</rationale>

67.The nurse is teaching a class in the community on common myths regarding fertility and infertility. Which of the following statements made by class participants indicates teaching has been successful?</question> <question_image/> <choice_1>"If my husband works out every day, he won't be able to make a baby."</choice_1> <choice_2>"If we have intercourse standing up, we won't be able to conceive."</choice_2> <choice_3>"If we have intercourse on the even days after ovulation, we will conceive a girl."</choice_3> <choice_4>"If my husband sits in the hot tub every night, his sperm count will decrease."</choice_4>

<correct>4</correct> <rationale>Hot tubs, saunas, and tight underwear can raise the temperature of the testes too high for efficient spermatogenesis and lead to decreased sperm numbers and motility.</rationale>

55.The nurse would implement which of the following as a requirement of care specific to the client who has tuberculosis?</question> <question_image/> <choice_1>Disposal of needles and syringes in a rigid, puncture-proof container</choice_1> <choice_2>Handwashing after removing contaminated gloves</choice_2> <choice_3>Wearing a gown if splashing is possible</choice_3> <choice_4>A private room with negative air flow</choice_4>

<correct>4</correct> <rationale>The client with tuberculosis can spread the infection by breathing and requires a private room and airborne precautions. Options 1, 2, and 3 are aspects of standard precautions that would be implemented with any client, regardless of medical diagnosis.</rationale>

38. Which of the following items would the nurse avoid documenting when a reportable incident has occurred?</question> <question_image/> <choice_1>Names of witnesses in the incident report</choice_1> <choice_2>Nursing interventions in the medical record</choice_2> <choice_3>Time the physician was called in the incident report</choice_3> <choice_4>That an incident report was submitted in the medical record</choice_4>

<correct>4</correct> <rationale>The medical record belongs to the client and should contain all of the facts related to the client and the incident. The incident report belongs to the hospital and should contain all of the facts and supportive data related to the client and the incident. The medical record should not refer to the incident report.</rationale>

93.At the first prenatal visit, the client reveals that her last menstrual period began March 18. The nurse calculates her estimated date of delivery to be which of the following?</question> <question_image/> <choice_1>June 25</choice_1> <choice_2>November 18</choice_2> <choice_3>January 11</choice_3> <choice_4>December 25</choice_4>

<correct>4</correct> <rationale>According to Nägele's rule, the estimated date of birth can be calculated by subtracting 3 months from the beginning date of the last menstrual period and then adding 7 days to that date.</rationale>

53.A client with vancomycin-intermediate-resistant <i>Staphylococcus aureus</i> (VISA) is admitted to the nursing unit. What type of precautions would be appropriate for this client?</question> <question_image/> <choice_1>Standard precautions</choice_1> <choice_2>Neutropenic precautions</choice_2> <choice_3>Droplet precautions</choice_3> <choice_4>Contact precautions</choice_4>

<correct>4</correct> <rationale>Clients with antibiotic-resistant microorganisms must be isolated with transmission-based precautions. The organism is transmitted via close person-to-person direct contact and by touching contaminated surfaces and objects. Standard precautions are used with all clients, regardless of medical diagnosis. Reverse isolation is instituted for immunocompromised clients. This organism is not transmitted via droplet nuclei.</rationale>

Troponin T

> 0.1- 0.2 ng/ml indicates MI

Bundles of muscle fibers that compose skeletal muscle are identified as: A. fasciculi. B. fasciculations. C. ligaments. D. tendons.

A

The nurse is caring for a patient whose calcium intake must increase because of high risk factors for osteoporosis. The nurse would recommend which of the following menus? A) Cream of broccoli soup with whole wheat crackers and tapioca for dessert B) Hamburger on soft roll with a side salad and an apple for dessert C) Low-fat turkey chili with sour cream and fresh pears for dessert D) Chicken salad on toast with tomato and lettuce and honey bun for dessert

A

What is the removal of devitalized tissue from a wound called? A) Debridement B) Pressure reduction C) Negative pressure wound therapy D) Sanitization

A

What term refers to a linear skin lesion that runs along a nerve route? A. Zosteriform B. Annular C. Dermatome D. Shingles

A

When assessing for the presence of a herniated nucleus pulposus, the examiner would: A. raise each of the patient's legs straight while keeping the knee extended. B. ask the patient to bend over and touch the floor while keeping the legs straight. C. instruct the patient to do a knee bend. D. abduct and adduct the patient's legs while keeping the knee extended.

A

Which type of pressure ulcer is noted to have intact skin and may include changes in one or more of the following: skin temperature (warmth or coolness), tissue consistency (firm or soft), and/or pain? A) Stage I B) Stage II C) Stage III D) Stage IV

A

During skin inspection the nurse observes three lesions on the client's right knee. The lesions are round, have a raised border, and are grouped in a "smiley face" pattern. Which descriptors does the nurse use to document these observations? A. Annular, circinate, clustered B. Linear, confluent, vesicular C. Circumscribed, universal, pustular D. Serpiginous, coalesced, wheal-like

A Rationale: Annular lesions have a raised border with flat, clear centers. Circinate lesions are round. The grouping is not linear and they are not touching, which makes them clustered rather than confluent. The information given is not enough to determine whether they are circumscribed.

Which intervention does the nurse use to promote "take" of a graft placed on the client's right heel? A. Elevate the client's right foot by placing pillows under the leg from the knee to the ankles. B. Position the client on the abdomen with the right foot hyperextended for at least 4 hours daily. C. Ensure that the grafted area is pressed tightly to the bed to promote adherence to the wound bed. D. Assess the circulation distal to the graft every hour and compare the findings with those from the left foot.

A Rationale: No pressure should be placed on the graft, and care must be taken to ensure it does not move over the wound so the blood vessels can connect the graft with the wound bed. Elevating the area allows better circulation and no pressure. Pressing the graft tightly against the bed would disturb the graft location and compromise circulation. Although placing the client on the abdomen would eliminate pressure on the heel, having the foot hyperextended would move the graft. Assessing circulation is a good thing to do but does not promote graft take.

Which technique for obtaining a specimen for bacterial culture is most correct for the nurse to use with a client who has crusted skin lesions on the upper back? A. Remove several crusts, and swab the underlying exudate. B. Dampen the culture swab with sterile water, and then roll the swab over the central crusts. C. Apply a gauze bandage to the area, remove it after 1 hour, and send the entire gauze to the laboratory. D. Clean the area with an antibacterial solution, remove a crust from the center of the cluster of lesions, and send it to the laboratory.

A Rationale: The infecting agent is present in the material underneath the crusts, not on the outer surface of the crusts. Cleaning the area first with an antibacterial solution could compromise or delay culture results.

Answer: C Review of the literature is the first step in the orderly research process to determine what is already known about the problem. Recruiting clients occurs later in the process, after identifying the problem, researching the literature, and designing the study. Experimenting with new nursing procedures that have not been tested or approved is a risk to clients. Surveys are designed to obtain information from large study populations and would not be a first step in the research process.

A clinical nurse develops a better way to secure an intravenous access device in a client and wants to see if it would benefit other clients. The first step in initiating a study should be to: A) Recruit clients to participate in the study. B) Use the new technique and gather client feedback. C) Review current literature related to the clinical problem. D) Survey clients regarding their preferences and feelings regarding the procedure.

Answer: C Pallor would appear as yellowish brown in brown-skinned people. Pallor would manifest as bluish skin in light-skinned people. Pallor would appear as ashen gray skin in black-skinned people. Shiny skin indicates edema.

A common abnormality encountered during inspection of the skin is pallor. Pallor is easily seen in the face, mucosa of the mouth, and nail beds. How would pallor appear in a brown-skinned client? A) As shiny skin B) As bluish skin C) As yellowish skin D) As ashen gray skin

Answer: A, B, C, D In this case, all four options are correct. The community health nurse is providing information for the community and helping its members learn to access the help that is available, but not dictating the steps that need to be taken.

A community health nurse is caring for members of a Bosnian community. The nurse determines that the children are undervaccinated and that the community is unaware of this resource. As the nurse assesses the community, the nurse determines that there is a health clinic within 5 miles. The nurse meets with the community leaders and explains the need for immunizations, the location of the clinic, and the process for accessing the health care resources. Which of the following is the nurse doing? (Select all that apply.) A) Improving children's health care B) Teaching the community about illness C) Educating about community resources D) Promoting autonomy in decision making

Answer: C A case manager's competency is defined as the ability to establish an appropriate plan of care that is based on assessment of clients and families and coordinates the provision of needed resources and services across a continuum of care. A collaborator's competency is described as engaging in a combined effort with all those involved in care delivery. A change agent's competency is to implement new and more effective approaches to problems. A client advocate presents the client's point of view so that appropriate resources can be obtained.

A competent community-based nurse must be skilled in fulfilling a variety of roles. The ability to establish an appropriate plan of care that is based on assessment of clients and families and coordinates the provision of needed resources and services across a continuum of care defines the competency of: A) Collaborator B) Change agent C) Case manager D) Client advocate

The nurse notes that an advance directive is in the client's medical record. Which of the following statements represents the best description of guidelines a nurse would follow in this case?

A living will is invoked only when the client has a terminal condition or is in a persistent vegetative state.

Chap 34. Which procedure provides information regarding the physiology of an organ?

A nuclear scan demonstrates ability of tissue to absorb the chemical. All of the other answers provide anatomic information.

Answer: A, C Clients being discharged home need education regarding how to take their medication and when to call their health care provider. There is not enough information here to determine if options 2 and 4 are appropriate, although hand hygiene after toileting is always important.

A nurse is planning a client's discharge from a subacute care unit to home. Education should be provided on which of the following topics? (Select all that apply.) A) Medication administration B) Stress reduction techniques with blood pressure assessment C) Circumstances in which the client should call the health care provider D) Hand-washing hygiene when assisting with transfer to the bathroom

Answer: B A counselor helps clients identify and clarify health problems and choose appropriate courses of action to solve those problems. An educator helps the community gain greater skills, including through the presentation of educational programs. A collaborator is an individual who engages in a combined effort with other individuals to develop a mutually acceptable plan that will achieve common goals.

A nurse is practicing in an occupational health setting. There are a large number of employees who smoke, and the nurse designs an employee assistance program for smoking cessation. This is an example of which nursing role? A) Educator B) Counselor C) Collaborator D) Case manager

Answer: B, C The case manager coordinates the efforts of all disciplines to achieve the most efficient and appropriate plan of care for the client, with a focus on discharge planning. Therefore, coordination of transfer to a step-down rehabilitation unit and follow-up after discharge to evaluate that needs have been met are the correct answers.

A nurse is working in an acute care hospital that uses a case management model. About which of the following activities should the nurse communicate with the case manager? (Select all that apply.) A) Management of a client transfer to the radiology department B) Coordination of a client transfer to the step-down rehabilitation unit C) Follow-up after a client's discharge to evaluate whether needs have been met D) Permission for a family to bring in special food for a client

Answer: C Evaluation research is aimed at finding out how well a program, practice, policy, or procedure is working. A survey studies a large group to identify general information, opinions, attitudes, or perceptions. A grounded theory is a theory developed through the collection and analysis of qualitative data. Experimental research collects information about human subjects who are divided into a control group and a comparison group.

A nurse manager wants to determine how well a new policy is working in the clinical area. It would be appropriate to use: A) Survey methods B) Grounded theory C) Evaluation research D) Experimental research

Answer: B A qualitative study involves inductive reasoning to develop generalizations or theories from specific observations or interviews. Historical research establishes facts and relationships concerning past events. Correlational research explores the interrelationships among variables of interest without any intervention by the researcher. An experimental study used tightly controlled subject groups, variables, and procedures to eliminate bias and ensure that findings can be generalized to similar groups of subjects.

A nurse researcher interviews senior oncology nurses, asking them to describe how they deal with the loss of a client. The analysis of the interviews yields common themes describing the nurses' grief. This is an example of which type of study? A) Historical study B) Qualitative study C) Correlational study D) Experimental study

Answer: A Because the clients at the clinic are allowed their choice of the traditional versus the new exercise program, the sampling in this study is not random sampling and can bias study results.

A nurse researcher is designing an exercise study that involves 100 clients who attend a wellness clinic. As the clients come to the clinic, each has a choice as to whether they want to be in the new exercise program or remain in the traditional program. The nurse plans to measure the clients' self-report of exercise before and 6 months after the program begins. What factor might influence the results of this study in an unfavorable way? A) Bias B) Anonymity C) Sample size D) Sampling method

Answer: D Taking blood pressure measurements is illness prevention. Health promotion includes activities like exercise classes. Secondary care is often known as traditional care. It would include rehabilitation after a stroke in an individual with a history of elevated blood pressure.

A nurse volunteers to take blood pressure measurements after church services. This is an example of which level of health care service? A) Secondary care B) Restorative care C) Health promotion D) Illness prevention

Answer: C The nurse who has held the same position for 2 to 3 years and understands the specific area and client population is termed a competent nurse. The expert is a nurse with diverse experience who can focus on a specific problem and offer multidimensional solutions. The proficient nurse has more than 2 to 3 years' experience and applies knowledge and experience to a situation. The advanced beginner nurse has at least some level of experience.

A nurse who has filled a position on the same unit for 2 years understands the unit's organization and the care of the clients on that nursing unit. Benner defines this nurse as able to anticipate nursing care and to formulate long-range goals; this nurse is given the title: A) Expert nurse B) Proficient nurse C) Competent nurse D) Advanced beginner

Answer: B This process may be carried out with other members of the health care team, and client and family members may be included. All nurses use critical thinking. An advanced care nurse has advanced educational preparation. An evidence-based practitioner draws on research findings as well as clinical expertise and client values. A multidisciplinary practice includes health care members from various fields of activity, such as physical therapy and dietary therapy, along with nursing.

A nurse who uses critical thinking in the decision-making process to provide effective quality care to individuals is known as: A) An advanced care nurse B) A clinical decision maker C) A member of a multidisciplinary practice D) An evidence-based practitioner

4

A nurse works in a special care unit for children with severe immunology problem and is caring for a three-year-old boy from Greece.The boy's father is there with him while his mother and sister are back in Greece. The nurse is having difficulty communicating with the father. What action does the nurse take? 1. Care for the boy as she would any other patient. 2. Ask the manager to talk with the father and keep him out of the unit 3. Have another unit care for the boy because maybe that nurse will do better with the father. 4. Search for help with interpretation and understanding of cultural differences by contacting someone from the Greek community.

4

A patient at the community clinic asks the nurse about health promotion activities that she can do because she is concerned about getting DM since her grandfather and father both have the disease. This patient reflects that the patient is in what stage of the health belief model? 1. Perceived threat of illness 2. Likelihood of taking preventative health action 3. Analysis of perceived health benefits of preventative action 4. Perceived susceptibility to the disease

4

A patient with a 20-year medical history of DM had a lower led amputation. Which statement made by the patient indicates that he is experiencing a problem with body image. 1."I just don't have the energy to get out of bed in the morning," 2. "I have been attending church regularly with my wife since I have got out of the hospital." 3. "My wife has taken over paying bills since I have been in the hospital." 4. "I don't go out much because everyone stares at me."

Answer: D Change must be perceived as advantageous, compatible with existing values, and easily adaptable to be successful and accepted. Up-front cost, managerial framework, building plans, contractors, compliance with building codes, and regulations for governmental agencies are all incorporated in proposals but do not provide convincing reasoning that leads to change.

A proposal written by a community-based nurse for a new, higher quality older adult care center will have increased probability of acceptance if the proposal includes: A) All building plans and a list of contractors to complete the job B) Compliance with the codes and building requirements of local government agencies C) The up-front cost and managerial framework of the new older adult center D) Description of how advantageous, realistic, compatible, and adaptable the change will be when implemented

Answer: D Informed consent means that the research subjects are given full and complete information about the purpose of the study, procedures, data collection, potential harm and benefits, and alternate methods of treatment. Confidentiality rules guarantee that any information the subject provides will not be reported to people outside the research team. Bias is any personal opinion or judgment that may be interjected into the results.

A researcher gives a subject full and complete information about the purpose of a study. This is an example of: A) Bias B) Anonymity C) Confidentiality D) Informed consent

Chap 42. An individual complains of difficulty concentrating and a headache. The nurse observes that the client has a fearful facial expression and is easily distracted. What level of anxiety is the individual manifesting?

A severely anxious person would have concentration difficulties, distractibility, headache or dizziness, and fearful facial expression. In the mild level, individuals are alert. In the moderate level, there is narrowed focus of attention and selective inattention is present. Mild gastric symptoms may be present in the moderate level. In panic, perception is distorted and the person is unable to function. The individual experiences feelings of impending doom.

Answer: D Describing, explaining, predicting, and/or prescribing interrelationships among concepts are stated purposes of research.

A theory is a set of concepts, definitions, relationships, and assumptions or propositions to explain a phenomenon. The purposes of the components of a theory are to: A) Describe concepts or connect two concepts that are factual B) Formulate a perceptual experience to describe or label a phenomenon C) Express the global view about the individual, situations, or factors of interest to a specific discipline D) Describe, explain, predict, and/or prescribe interrelationships among the concepts that define the phenomenon

Answer: B A theory is a set of concepts, definitions, relationships, and assumptions that explains a phenomenon. Theories do not formulate legislation, measure nursing functions, or reflect any domain of nursing practice.

A theory is a set of concepts, definitions, relationships, and assumptions that: A) Formulates legislation B) Explains a phenomenon C) Measures nursing functions D) Reflects the domain of nursing practice

Chap 49. A client has a transverse colostomy. What kind of fecal drainage is to be expected?

A transverse colostomy produces a malodorous, mushy, uncontrolled drainage. A descending colostomy produces increasingly solid fecal drainage that can be controlled. Malodorous, liquid fecal drainage is produced by an ascending colostomy, while an ileostomy produces constant liquid fecal drainage.

The nurse must follow standards of care to avoid potential litigation such as negligence suits. Which of the following describe a potential nursing malpractice situation? (Select all that apply.)

A) Failure to question a health care provider about the appropriateness of a client order B) Failure to make a nursing diagnosis C) Failure to properly use medical equipment ordered for client monitoring D) Failure to follow the "six rights" of medication administration E) Failure to provide discharge instructions

A nurse is administering care to a client immediately after nephrectomy and renal transplantation. The nurse administers IV fluids as prescribed, recognizing that the hourly rate is usually calculated on the basis of: a) the number of milliliters in the previous hours' urine output b) one half the previous hours urine output c) Strict hourly rate of 100 mL d) Strict hourly rate of 150 mL

A) number of mLs in the previous hours' urine output IV fluids are managed very carefully after nephrectomy and renal transplants. Fluids are usually given according to a formula that takes into account the previous hour's urine output. The desired urine output is generally high.

Which of the following safe sun practices would the RN include in the teaching care plan for a pt who has photosensitivity? (select all) A. wear protective clothing B. apply sunscreen liberally and often C. tanning booths decrease the likelihood of sunburn D. avoid exposure to the sun, esp during midday E. wear any sunscreen as long as it is purchased in a drugstore

A, B, D. Patients should recognize that sun safety guidelines include sun avoidance, especially during the midday hours; protective clothing; and broad-spectrum sunscreen (e.g., SPF 15, SPF 30 if a history of skin cancer or sun sensitivity). Sunscreens should be applied 20 to 30 minutes before going outdoors and be reapplied every 2 hours and after swimming. Patients should avoid tanning booths and sun lamps.

Dermatologic manifestation(s) of Cushing syndrome can include (select all that apply) A. acne B. telangiectasia C. increased sweating D. generalized hyperpigmentation E. brown pigmentation in the legs

A, B. Dermatologic manifestations of glucocorticoid excess (as in Cushing syndrome) include atrophy; striae; epidermal thinning; telangiectasia; acne, decreased subcutaneous fat over extremities; thin, loose dermis; impaired wound healing; increased vascular fragility; mild hirsutism; and excessive collection of fat over clavicles, back of neck, abdomen, and face.

You are caring for a patient admitted with diabetes mellitus, malnutrition, and massive GI bleed. In analyzing the morning lab results, the nurse understands that a potassium level of 5.5 mEq/L could be caused by which of the following factors in this patient (select all that apply)? A) The potassium level may be increased if the patient has renal nephropathy. B) The patient may be excreting extra sodium and retaining potassium because of malnutrition. C) The potassium level may be increased as a result of dehydration that accompanies high blood glucose levels. D) There may be excess potassium being released into the blood as a result of massive transfusion of stored hemolyzed blood.

A, C, D Hyperkalemia may result from hyperglycemia, renal insufficiency, and/or cell death. Diabetes mellitus, along with the stress of hospitalization and illness, can lead to hyperglycemia. Renal insufficiency is a complication of diabetes. Malnutrition does not cause sodium excretion accompanied by potassium retention; thus it is not a contributing factor to this patient's potassium level. Stored hemolyzed blood can cause hyperkalemia when large amounts are transfused rapidly.

2. Why is adequate oral hygiene an essential part of nursing care? Select all that apply. a. It promotes the patient's sense of wellbeing. b. It prevents deterioration of the oral cavity. c. It contributes to decreased incidence of aspiration pneumonia. d. It eliminates the need for flossing. e. It decreases oropharyngeal secretions

A,B,C.

2. The nurse is teaching parents of a toddler about the use of car seats. The nurse demonstrates an understanding of this measure by stating that car seats are mandatory in which of the following conditions? a. In all 50 states b. In 36 of the 50 states c. If a seat belt is not available d. On interstate highways

A.

3.A nurse is assessing the following children. Which child would the nurse identify as having the greatest risk for choking and suffocating? a. A toddler playing with his 9-year-old brother's construction set b. A 4-year-old eating yogurt for lunch c. An infant covered with a small blanket and asleep in the crib d. A 3-year-old drinking a glass of juice

A.

4.Which recommendation by the nurse to an adolescent patient with acne would be most appropriate? a. Wash the skin twice a day. b. Use cosmetics liberally to cover blackheads. c. Use emollients on the area. d. Squeeze blackheads as they appear.

A.

The RN assessed the skin lesions as circumscribed, superficial, elevated, solid, and greater than 0.5 cm in diameter. They would be called: A. plaques B. papules C. pustules D. wheals

A. A plaque is a circumscribed, elevated, superficial, solid lesion; it is > 0.5 cm in diameter.

The staff mix available for the medical-surgical unit includes RNs, LPNs/LVNs, and nursing assistants. Which of these clients does the nurse plan to assign to an experienced LPN/LVN? A. Adult client who has had suturing of a facial tear that occurred when the client fell off a bike onto a dirt road B.Adult client who needs to be admitted for a grafting of a second-degree burn on the right leg C.Middle-aged adult client who needs discharge teaching before going home after receiving steroids for Stevens-Johnson syndrome D. Older adult client with stage I pressure ulcers who needs to be turned every 2 hours

A. An LPN/LVN would be familiar with wound monitoring for potentially contaminated wounds and would recognize manifestations of infection.

6. Which of the following cardiovascular effects of aging should the nurse anticipate when providing care for older adults (select all that apply)? A. Arterial stiffening B. Increased blood pressure C. Increased maximal heart rate D. Decreased maximal heart rate E. Increased recovery time from activity

A. Arterial stiffening B. Increased blood pressure D. Decreased maximal heart rate E. Increased recovery time from activity

The nurse is teaching the client about decreasing the risk for melanomas and other skin cancers. Which primary prevention technique is most important for the nurse to include? A. Avoiding or reducing skin exposure to sunlight B. Avoiding tanning beds C. Being aware of skin markings and performing skin self-examination D. Wearing SPF 40 sunscreen

A. Avoiding or reducing one's exposure to the sun is the most important prevention technique. This includes avoiding direct sunlight, using sunscreen, and wearing protective clothing (including hats).

5. The nurse is providing care for a patient who has decreased cardiac output related to heart failure. The nurse recognizes that cardiac output is A. Calculated by multiplying the patient's stroke volume by the heart rate. B. The average amount of blood ejected during one complete cardiac cycle. C. Determined by measuring the electrical activity of the heart and the patient's heart rate. D. The patient's average resting heart rate multiplied by the patient's mean arterial blood pressure.

A. Calculated by multiplying the patient's stroke volume by the heart rate. Cardiac output is determined by multiplying the patient's stroke volume by heart rate, thus identifying how much blood is pumped by the heart over a 1-minute period. Electrical activity of the heart and blood pressure are not direct components of cardiac output.

In a patient admitted with cellulitis of the left foot, which of the following clinical manifestations would you expect to find on assessment of the left foot? A. Redness and swelling B. Pallor and poor turgor C. Cyanosis and coolness D. Edema and brown skin discoloration

A. Cellulitis is a diffuse, acute inflammation of the skin. It is characterized by redness, swelling, and heat in the affected area. These changes accompany the processes of inflammation and infection.

The client with a foot ulcer says, "I feel helpless." What is the nurse's best response? A. Encourages participation in care of the wound B. Encourages visitors C. Says, "I know how you feel" D. Assures the client that it will be all right

A. Encouraging participation in wound care gives the client a sense of autonomy.

Individuals with dark skin are more likely to develop: A. keloids B. wrinkles C. skin rashes D. skin cancer

A. Individuals with dark skin are predisposed to certain skin and hair conditions such as keloids, which are overgrowths of collagenous tissue at the site of a skin injury.

While providing teaching to the client undergoing excisional biopsy, which statement will the nurse include? A. "Administration of local anesthetic agents may cause burning." B. "The biopsy results will be available within 2 hours of the procedure." C. "The dressing must remain in place for the first 48 hours." D. "Redness and swelling at the puncture site are expected."

A. Local anesthetic agents may cause a burning sensation for the client.

The nursing instructor reviews instructions with the nursing student on caring for the older adult client with a pressure ulcer. What action by the nursing student indicates a need for further instruction about proper skin care for this client? A. Massages bony prominences B. Avoids reddened areas C. Repositions the client every 1 to 2 hours D. Uses a moisturizing lotion

A. Massaging bony prominences should be avoided in older adult clients.

The nurse would assess a patient admitted with cellulitis for which of the following localized signs? A. pain B. fever C. chills D. malaise

A. Pain, redness, heat, and swelling are all localized signs of cellulitis. Fever, chills, and malaise are generalized, systemic manifestations of inflammation and infection.

Important pt teaching after a chemical peel includes: A. avoidance of sun exposure B. application of firm bandages C. limitations of vigorous exercise D. use of ice to relieve discomfort

A. Patient teaching after a chemical peel should include instructions to use sunscreen and to avoid sun exposure for 6 months to prevent hyperpigmentation.

A mother and her two children have been diagnosed with pediculosis corporis. an appropriate measure in treating this condition is: A. applying pyrethrins to the body B. topical application of griseofulvin C. moist compresses applied frequently D. administration of systemic antibiotics

A. Pediculosis corporis (i.e., body lice) is treated with γ-benzene hexachloride or pyrethrins.

During change-of-shift report, the outgoing nurse reports a new finding of petechiae in a new patient admitted with a yet-to-be diagnosed hematologic disorder. On assessment of this patient, the incoming nurse may expect to find: A. Tiny, purple spots on skin. B. Large ecchymotic areas on skin. C. Hyperkeratotic papules and plaques. D. Small, raised red areas on the soles of the feet.

A. Petechiae present as tiny, purple spots on the skin. Large ecchymotic areas are purpura; hyperkeratotic papules and plaques represent actinic keratosis; and small raised red areas on the soles of the feet signify Osler's nodes

Which of the following interventions would be most helpful in managing a patient newly admitted with cellulitis of the right foot? A. Applying warm, moist heat B. Limiting ambulation to three times daily C. Keeping the foot at or below heart level D. Wrapping the foot snugly in warm blankets

A. The application of warm, moist heat speeds the resolution of inflammation and infection when accompanied by appropriate antibiotic therapy. It does this by increasing local circulation to the affected area to bring macrophages to the area and carry off cellular debris.

The nurse is teaching the client about postoperative care following oral cancer surgery. Because of damage to the epidermis, what topic does the nurse plan to discuss with the client? A. Body image counseling B.Respiratory protection C. Self-suctioning D. Tobacco cessation education

A. The epidermis is the outer layer of the skin. Damage to the epidermis can cause body image disturbance with clients.

in teaching a pt with malignant melanoma about this disorder, the RN recognizes that the prognosis of the pt is most dependent on: A. the thickness of the lesion B. the degree of color change in the lesion C. how much the lesion has spread superficially D. the amount of ulceration present in the lesion

A. The most important prognostic factor is tumor thickness at the time of diagnosis. Two methods are used to determine thickness. The Breslow measurement indicates the depth of the tumor in millimeters, and the Clark level indicates the depth of invasion of the tumor. The higher the number, the deeper the melanoma.

Which of the following laboratory results is the best indicator that a patient with cellulitis is recovering from this infection? A. WBC of 8200/μl B. WBC of 2900/μl C. WBC of 16,300/μl D. WBC of 12,700/μl

A. The normal white blood cell count is generally 4000 to 11,000/μl. For this reason, the patient's level would be returning to normal if it was 8200/μl, indicating recovery from cellulitis.

In teaching the client about skin cancer prevention, which instruction will the nurse include? A. "Avoid sun exposure between 11 AM and 3 PM." B. "Examine skin quarterly for possible cancerous or precancerous lesions." C. "Wear transparent clothing to protect the skin from the sun." D. "It is safe to use a tanning bed."

A. The sun's rays are strongest between 11 AM and 3 PM and can cause more damage during this time.

Which statement by the client with psoriasis indicates to the nurse that additional teaching about his condition is required? A. "A tanning bed will supply the ultraviolet light I need." B. "Medicine can prevent the growth of new skin cells." C. "I can never be cured." D. "Stress can cause my flare-ups."

A. Ultraviolet (UV) radiation is commonly used in the treatment of psoriasis, but the use of commercial tanning beds is specifically not recommended for these clients. This statement indicates that the client requires further teaching.

To assess the skin for temperature and moisture, the most appropriate technique for the RN is to use is: A. palpation B. inspection C. percussion D. auscultation

A. Using the back of your hand on the patient's skin best assesses its temperature.

When assessing the activity-exercise pattern in relation to the skin, the RN questions the pt regarding: A. protection against sun exposure B. the use of moisturizing shampoo C. self-care habits related to daily hygiene D. the presence of dark circles under the eyes

A. When assessing the activity-exercise pattern, the nurse asks the following questions: Do your leisure or work activities involve the use of any chemicals that are irritating to your skin? Do you do anything to protect yourself from the sun?

The client has an odorous purulent wound. How does the nurse best support this client? A. Changes the dressing frequently B. Encourages a diet high in protein C. Suggests whirlpool therapy D. Places room deodorizers in the room

A> Frequent dressing changes help the client feel clean.

The nurse anticipates that the client with a deep necrotizing wound caused by a brown recluse spider bite may require which type of healing therapy? A. Hyperbaric oxygen B. Nutrition therapy C. Topical growth factors D. Vacuum-assisted wound closure

A> Hyperbaric oxygen therapy is usually reserved for life- or limb-threatening wounds such as burns, necrotizing soft tissue infections, brown recluse spider bites, osteomyelitis, and diabetic ulcers.

The nurse is teaching the client with loss of sensation and movement in the lower extremities secondary to spinal cord injury about protecting skin integrity. Which daily prevention strategy will the nurse include in the client's teaching plan? A. Lift hips off the chair at least every 30 minutes. B. Eat a low-fat diet. C. Massage reddened areas. D. Complete a pressure map.

A> Lifting hips off the chair at least every 30 minutes relieves pressure and can prevent pressure ulcers.

-pril

ACE inhibitors

Chap 34. During an assessment, the nurse learns that the client has a history of liver disease. Which of the following diagnostic tests might be indicated for this client?

ALT is an enzyme that contributes to protein and carbohydrate metabolism. An increase in the enzyme indicates damage to liver. The liver contributes to the metabolism of protein which results in the production of ammonia. If the liver is damaged, the ammonia level will increase. Myoglobin, cholesterol, and BNP are relevant for heart disease.

Acute Lymphocytic Leukemia

Abnormal leukocytes are found in blood forming tissue Prognosis is favorable

Acute Renal Failure ARF Pathophysiology Precipitating Factors Assessment Nursing Intervention

Abrupt deterioration of the renal system, a reversible syndrome; occurs when metabolites accumulate in the body and urinary output changes; 3 major types (prerenal, intrarenal, postrenal); 3 phases (oliguric, diuretic, recovery) Monitor intake and output, give only enough fluids in the oliguric phase to replace losses, usually 400-500mL/hour; document and report any change in fluid volume status; monitor lab values of both serum and urine to assess electrolyte status, especially hyperkalemia; assess level of consciousness for subtle changes, weigh daily: in oliguric phase- client may gain up to 1 lb/day; prevent cross infection; Kayexalate may be prescribed if K is too high; provide low protein, moderate fat, high carbohydrate diet; monitor cardiac rate and rhythm; monitor drug levels and interactions

The nurse is caring for a client with end-stage lung disease. The client wants to go home on oxygen therapy and be comfortable. The family wants the client to undergo a new surgical procedure. The nurse explains the risk and benefits of the surgery to the family and discusses the client's wishes with the family. The nurse is acting as the client's:

Advocate

Chap 49. The nurse is promoting regular defecation for a client in the nurse's care. Which of the following actions by the nurse is NOT correct?

Although the squatting position best facilitates defecation, on a toilet seat, the best position for most people seems to be leaning forward. A client should be encouraged to defecate when the urge is recognized. Regular exercise helps clients develop a regular defecation pattern. For clients who have difficulty sitting down and getting up from the toilet, an elevated toilet seat can be attached to a regular toilet.

Chap 49. The nurse is most likely to report which of the following findings to the primary care provider for a client who has an established colostomy?

An established stoma should be dark pink like the color of the buccal mucosa and is slightly raised above the abdomen. The skin under the appliance may remain pink/red for a while after the adhesive is pulled off. Feces from an ascending ostomy are very liquid, less so from a transverse ostomy, and more solid from a descending or sigmoid stoma.

Chap 43. What is the best route for administering pain medication to a dying client?

Analgesics need to be administered by intravenous infusion because of decreased blood circulation. Due to poor absorption of the GI tract and possible nausea, the oral route is not the best choice. Because of decreased blood circulation, the subcutaneous and intramuscular routes are not the best routes.

2. A nurse forgets to administer a client's diuretic and the client experiences an episode of pulmonary edema. This medication error would be considered negligence if it constituted which of the following? 1. The purposeful failure to perform a health care procedure 2. The unintentional failure to perform a health care procedure 3. The act of substituting a different medication for the one ordered</choice_3> 4. Failure to follow a direct order by a physician

Answer 2: Negligence is the unintentional failure of an individual to perform or not perform an act that a reasonable person would or would not do in the same or similar circumstances. Options 3 and 4 do not fit the description of the event, and option 1 is the opposite of option 2.

1. A client is referred to a surgeon by the general practitioner. After meeting the surgeon, the client decides to find a different surgeon to continue treatment. The nurse supports the client's action, utilizing which ethical principle 1. Beneficence 2.Veracity 3. Autonomy 4. Privacy

Answer 3: Autonomy is the right of individuals to take action for themselves. Beneficence is duty to help others by doing what is best for them, whereas negligence is a legal term. Veracity is truthfulness. Privacy is the nondisclosure of information by the health care team.

14.As a member of the long-term care facility quality improvement team, the nurse has been asked to evaluate the quality of nursing care on the unit. The nurse has decided to ask the nursing staff for assistance in this endeavor. Which of the following would be appropriate to ask the nursing staff to do? <choice_1>Track the number of supplies used by clients on the unit.</choice_1> <choice_2>Document the time spent on direct client care.</choice_2> <choice_3>Administer a client and family satisfaction survey.</choice_3> <choice_4>Assess clients and report acuity daily.</choice_4>

Answer: 3 <rationale>Client satisfaction surveys are an important tool to monitor and evaluate patient and family needs. This information helps health care organizations meet those needs. Options 1, 2, and 4 are extremely helpful but do not improve client satisfaction and outcomes. Tracking supplies, documenting nursing time, and reporting on client acuity provides information that can be used in preparing a budget or unit staffing requirements.</rationale> <cognitive_level>Application</cognitive_level> <client_need>Safe Effective Care Environment: Coordinated Care</client_need> <integrated_process>Nursing Process: Planning</integrated_process> <content_area>Leadership/Management</content_area> <strategy>The core issue of the question is quality management. The purpose of quality management is to improve performance and meet client needs. The best way to assess client satisfaction is to ask the client directly. Established standards of practice, policies, and procedures safeguard clients and protect them from potential injury and harm. Quality management programs seek to ensure quality care and improve client satisfaction.</strategy>

A client with coronary artery disease complains of substernal chest pain. After assessing the client's heart rate and blood pressure, a nurse administers nitroglycerin, 0.4 mg, sublingually. After 5 minutes, the client states, "My chest still hurts." Select the appropriate actions that the nurse should take. Select all that apply. 1. Call a Code Blue. 2. Contact the physician. 3. Contact the client's family. 4. Assess the client's pain level. 5. Check the client's blood pressure. 6. Administer a second nitroglycerin, 0.4 mg, sublingually.

Answer: 4, 5, 6 Rationale: The usual guidelines for administering nitroglycerin tablets for chest pain include administering one tablet every 5 minutes PRN for chest pain, for a total dose of three tablets. If the client does not obtain relief after taking a third dose of nitroglycerin, the physician is notified. Because the client is still complaining of chest pain, the nurse would administer a second nitroglycerin tablet. The nurse would assess the client's pain level and check the client's blood pressure before administering each nitroglycerin dose. There are no data in the question that indicate the need to call a Code Blue. Additionally, it is not necessary to contact the client's family unless the client has requested this. Test-Taking Strategy: Focus on the data in the question. Use the steps of the nursing process to determine that assessing the client's pain level and checking the client's blood pressure are appropriate actions. Next, recalling the usual guidelines for administering nitroglycerin tablets will assist in determining that an appropriate action is to administer a second nitroglycerin tablet, 0.4 mg, sublingually. Review care of the client with chest pain and the guidelines for the administration of nitroglycerin if you had difficulty with this question.

-lam

Anti-anxiety drugs

-statin

Anti-hyperlipidemics (Cholestrol lower)

-vir

Anti-virals

Enoxaparin Lovenox

Antiplatelet Low molecular weight heparin Prevention of thrombolytic formation Monitor for signs of bleeding; Give SQ; monitor CBC; use soft toothbrush, avoid cuts

Answer: D As a science, nursing draws on scientifically tested knowledge applied in the practice setting.

As an art, nursing relies on knowledge gained from practice and reflection on past experiences. As a science, nursing relies on: A) Experimental research B) Nonexperimental research C) Physician-generated research D) Scientifically tested knowledge

Causes of Respiratory Acidosis

Asthma, Atelectasis, brain trauma, bronchiectasis, bronchitis, CNS depressants, emphysema, hypoventilation, pulmonary edema, pneumonia, pilmonary emboli

EKG P Wave

Atrial Systole Represents depolarization of the atrial muscle Should be rounded and without peaking or notching

Potassium - common food sources

Avocado, bannanas, cantaloupe, carrots, fish, mushrooms, oranges, potatoes, pork, beef, veal, raisins, spinach, strawberries, tomatoes

The health care provider's order is 1000 mL 0.9% NaCl IV over 6 hours. Which rate do you program into the infusion pump? A) 125 mL/hr B) 167 mL/hr C) 200 mL/hr D) 1000 mL/hr

B

The patient at greatest risk for developing multiple adverse effects of immobility is a: A) 1-year-old child with a hernia repair. B) 80-year-old woman who has suffered a hemorrhagic cerebrovascular accident (CVA). C) 51-year-old woman following a thyroidectomy. D) 38-year-old woman undergoing a hysterectomy.

B

When testing for muscle strength, the examiner should: A. observe muscles for the degree of contraction when the individual lifts a heavy object. B. apply an opposing force when the individual puts a joint in flexion or extension. C. measure the degree of force that it takes to overcome joint flexion or extension. D. estimate the degree of flexion and extension in each joint.

B

A female business professional has extremely dry skin on her legs. In addition to using lotions after bathing, she asks the nurse about other measures to help reduce the dryness. What is the nurse's best response? A. "Wear long-legged pajamas to sleep in rather than nightgowns." B. "Avoid wearing pantyhose or nylon stockings for more than 2 hours at a time." C. "Leave the fat-containing soap on your skin when bathing rather than rinsing it off." D. "Bathe in water that is as warm as you can stand to stimulate the release of body oils from your sebaceous glands."

B Rationale: Clothing that fits tightly and rubs can dry the skin. Prolonged contact with nylon stockings or pantyhose causes or exacerbates dry skin on the legs. Avoiding these clothing items can reduce this dryness. Wearing pajamas to sleep in, leaving soap on the skin, and bathing in very warm water can contribute to dry skin.

A nurse works on a cardiac unit. The nurse is taking care of a client who recently underwent coronary bypass surgery. Which of the following represent legal sources of standards of care nurses use to deliver safe health care? (Select all that apply.)

B) Regulations identified in the Joint Commission manual C) Policies and procedures of the employing hospital D) Nurse Practice Act of the state in which the nurse is working E) American Nurses Association standards of nursing practice

A nurse is sued for failure to monitor a client appropriately. Which statements are correct about professional negligence lawsuits? (Select all that apply.)

B) The person filing the lawsuit has the burden of proof. D) The plaintiff must prove that a breach in the prevailing standard of care caused an injury.

An older adult has limited mobility as a result of a surgical repair of a fracture hip. During assessment you note that the patient cannot tolerate lying flat. Which of the following assessment data support a possible pulmonary problem related to impaired mobility? (Select all that apply.) A) B/P = 128/84 B) Respirations 26 per minute on room air C) HR 114 D) Crackles heard on auscultation E) Pain reported as 3 on scale of 0 to 10 after medication

B, C, D

While performing patient teaching regarding hypercalcemia, which of the following statements are appropriate (select all that apply)? A) Have patient restrict fluid intake to less than 2000 ml/day. B) Renal calculi may occur as a complication of hypercalcemia. C) Weight-bearing exercises can help keep calcium in the bones. D) The patient should increase daily fluid intake to 3000 to 4000 ml. E) Treatment of heartburn can best be managed with Tums on a prn basis.

B, C, D A daily fluid intake of 3000 to 4000 ml is necessary to enhance calcium excretion and prevent the formation of renal calculi, a potential complication of hypercalcemia. Tums are a calcium-based antacid that should not be used in patients with hypercalcemia. Weight-bearing exercise does enhance bone mineralization.

Age-related changes in the skin include (select all that apply) A. oily scalp B. a loss of collagen C. thicker, brittle nails D. thinner, fragile nails E. improved blood supply

B, C. Decreased subcutaneous fat, muscle laxity, degeneration of elastic fibers, and collagen stiffening lead to increased wrinkling, sagging breasts and abdomen, redundant flesh around eyes, and slowness of skin to flatten when pinched together (i.e., tenting). Decreased oil leads to dry, coarse hair and a scaly scalp. Diminished blood supply leads to the following changes: decrease in rosy appearance of the skin and mucous membranes, skin that is cool to the touch, and diminished awareness of pain, touch, temperature, and peripheral vibration. Decreased peripheral blood supply leads to thick, brittle nails with diminished growth. The dermis loses volume and has fewer blood vessels.

in teaching a pt who is using topical corticosteroids to treat acute dermatitis, the RN should tell the pt that: (select all that apply) A. the cream form is the most efficient system of delivery B. short term use of topical corticoisteroids usually does not cause systemic side effects C. creams and ointments should be applied with a glove in small amounts to prevent further infection D. abruptly discontinuing the use of topical corticosteroids may cause a reappearance of the dermatitis E. systemic side effects may be experienced from topical corticosteroids if the person is malnourished

B, D. Systemic corticosteroids often have undesirable systemic effects. Topical corticosteroids for short-term therapy have fewer systemic effects. Rebound dermatitis is common when therapy is stopped abruptly; this effect can be reduced by tapering the use of topical corticosteroids.

1.When planning for completion of a patient's personal hygiene, it is most important to consider which of the following? a. When the patient had his or her most recent bath b. The patient's usual hygiene practices and preferences c. Where the bathing fits in the nurse's schedule d. The time that is convenient for the patient care assistant

B.

10. A student nurse is learning to assess blood pressure. What does the blood pressure measure? a. Flow of blood through the circulation b. Force of blood against arterial walls c. Force of blood against venous walls d. Flow of blood through the heart

B.

12.During morning care, the patient asks the nurse to shave him with a disposable razor. Before shaving him, what should the nurse do? a. Have him sign a permission form. b. Check to see if the patient is taking anticoagulants. c. Tell him that only a family member may shave a patient. d. Position him flat in bed.

B.

15. A patient has intravenous fluids infusing in the right arm. When taking a blood pressure on this patient, what would the nurse do in this situation? a. Take the blood pressure in the right arm b. Take the blood pressure in the left arm c. Use the smallest possible cuff d. Report inability to take the blood pressure

B.

15.The nurse teaches a patient at home to use clean technique when changing a wound dressing. This is which of the following? a. The nurse's preference b. Safe for the home setting c. Unethical behavior d. Grossly negligent

B.

15.When making an occupied bed, which of the following is most important for the nurse to do? a. Keep the bed in the low position b. Use a bath blanket or top sheet for warmth and privacy c. Constantly keep the side rails raised on both sides d. Move back and forth from one side to the other when adjusting the linens

B.

2.Your patient has developed a low-grade fever and states that she has felt very tired lately. You interpret these findings as indicating which stage of infection? a. Incubation period b. Prodromal stage c. Full stage of illness d. Convalescent period

B.

3. During a bath, the nurse observes that a patient has dry skin. Which action would be best? a. Bathe the patient more frequently. b. Use an emollient on the dry skin. c. Massage the skin with alcohol. d. Discourage fluid intake.

B.

6.Which action would be the priority when administering oral care to a dependent patient? a. Assisting the patient to the dorsal recumbent position b. Wearing disposable gloves c. Using a firm toothbrush to cleanse teeth and gums d. Irrigating forcefully with hydrogen peroxide

B.

8. Before assessing respirations, the nurse reviews normal rates for adults. Which rate would the nurse identify as normal? a. 1 to 6 breaths/min b. 12 to 20 breaths/min c. 60 to 80 breaths/min d. 100 to 120 breaths/min

B.

Which of the following assessment findings of a 70-year-old male patient's skin should the nurse prioritize? A. The patient's complaint of dry skin that is frequently itchy. B. The presence of an irregularly shaped mole that the patient states is new. C. The presence of veins on the back of the patient's leg that are blue and tortuous. D. The presence of a rash on the patient's hand and forearm to which the patient applies a corticosteroid ointment.

B. Although all of the noted assessment findings are significant, the presence of an irregular mole that is new is suggestive of a neoplasm and warrants immediate follow-up.

The older adult female client asks the nurse, "Why is my hair thinning?" After assuring the client that this is a normal sign of aging, what is the nurse's best follow-up response? A. "How does this make you feel?" B. "How is this affecting you?" C. "Wear a hat outside to stay warm." D. "You could wear a wig."

B. Asking the client how she is affected assesses the need for direct additional counseling.

Diagnostic testing is recommended for skin lesions when: A. a health history cannot be obtained B. a more definitive diagnosis is needed C. percussion reveals an abnormal finding D. treatment with prescribed medication has failed

B. Biopsy is one of the most common diagnostic tests used in the evaluation of a skin lesion. A biopsy is indicated in all conditions in which a malignancy is suspected or a specific diagnosis is questionable.

Which of the following medications are the most likely to have an effect on the patient's integumentary system? A. Diuretic B. Corticosteroid C. Benzodiazepine D. Calcium channel blocker

B. Corticosteroids can have unwanted integumentary side effects. Integumentary effects are less likely to occur with benzodiazepines, calcium channel blockers, and diuretics.

The nurse working in the same-day surgery unit has just received report and plans to assess which client first? A. Adult with a basal cell carcinoma excised who needs discharge teaching about wound care B. Young adult who has had rhinoplasty and is swallowing frequently C. Middle-aged adult who reports 7/10 pain after removal of a cyst D. Older adult ready to be transferred to the long-term care facility after débridement of a pressure ulcer

B. Frequent swallowing after rhinoplasty may indicate bleeding, which requires immediate action by the nurse.

the RN identifies that a pt with a diagnosis of which of the following disorders is most at risk for spreading the disease? A. tinea pedis B. impetigo on the face C. candidiasis D. psoriasis on the palms and soles

B. Impetigo is caused by a bacterial infection (group A β-hemolytic streptococci or staphylococci) and is highly contagious. Good skin hygiene and infection control practices are necessary to prevent the spread of this infection. Tinea pedis and candidiasis are fungal infections. Psoriasis is an autoimmune chronic dermatitis and is not contagious.

3. The nurse is admitting a patient who is scheduled to undergo a cardiac catheterization. Which of the following allergies is most important for the nurse to assess before this procedure? A. Iron B. Iodine C. Aspirin D. Penicillin

B. Iodine The physician usually will use an iodine-based contrast to perform this procedure. Therefore it is imperative to know whether or not the patient is allergic to iodine or shellfish.

On inspection of the pt's skin, the RN notes hypertrophied scarring at the site of a prior injry to the skin. this assessment abnormality is called: A. vitiligo B. keloid C. telangiectasia D.

B. Keloid is an overgrowth of collagenous tissue at the site of a skin injury.

Which activity for the long-term care client does the nurse plan to assign to the LPN/LVN? A. Develop a care plan for a client who has blisters caused by herpes zoster. B. Administer an antihistamine to a client who is describing pruritus. C. Teach a client how to self-assess for changes in skin lesions. D. Perform a baseline skin assessment for a newly admitted client.

B. LPNs/LVNs are familiar with safe administration of medications, including monitoring for medication effectiveness and adverse effects.

The nurse would recognize which of the following patients as likely to have the poorest prognosis? A. A 60-year-old diagnosed with nodular ulcerative basal cell carcinoma B. A 59-year-old man who is being treated for stage IV malignant melanoma C. A 70-year-old woman who has been diagnosed with late squamous cell carcinoma D. A 51-year-old woman whose biopsy has revealed superficial squamous cell carcinoma

B. Late detection of malignant melanoma is associated with a poor outcome. Basal cell carcinomas often have very effective treatment success rates. Although late SCC has worse outcomes than superficial SCC, these are both exceeded in mortality by late-stage malignant melanoma.

Which skin condition will the emergency department nurse assess first? A. Localized redness to the surgical site B. Pitting edema C. Poor skin turgor D. Red bony prominences

B. Pitting edema indicates an electrolyte, cardiac, or renal insufficiency.

The nurse understands that deep tissue wounds, such as chronic pressure ulcers, take longer to heal because they heal by which intention? A. First B. Second C. Third D. Mixed

B. Second intention healing is characterized by a cavity-like defect. This requires gradual filling in of the dead space with connective tissue in deeper tissue injuries or wounds with tissue loss.

The nurse in the outpatient clinic is caring for four clients who require cultures of skin lesions. Which action will the nurse take first? A. Add potassium hydroxide to the specimen to check for a possible fungal infection, and inspect it under the microscope. B. Soak the crust of a possible bacterial lesion with normal saline. C. Instruct the client who has had a punch biopsy about wound care. D. Place the viral culture tubes for a client with possible herpes zoster

B. Soaking the crust of a possible bacterial lesion with normal saline does not require immediate action.

Which nursing documentation is correct in describing multiple lesions with well-defined borders that are located in one area? A. Clustered round lesions to the chest B. Five clustered circumscribed lesions on the chest C. Five diffuse circinate lesions on the chest D. Several lesions in one area that have well-defined borders

B. This entry is specific, with correct terminology.

9. A 59-year-old man has presented to the emergency department with chest pain. Which of the following components of his subsequent blood work is most clearly indicative of a myocardial infarction (MI)? A. CK-MB B. Troponin C. Myoglobin D. C-reactive protein

B. Troponin Troponin is the biomarker of choice in the diagnosis of MI, with sensitivity and specificity that exceed those of CK-MB and myoglobin. CRP levels are not used to diagnose acute MI.

7. Auscultation of a patient's heart reveals the presence of a murmur. This assessment finding is a result of A. Increased viscosity of the patient's blood. B. Turbulent blood flow across a heart valve. C. Friction between the heart and the myocardium. D. A deficit in heart conductivity that impairs normal contractility.

B. Turbulent blood flow across a heart valve. Turbulent blood flow across the affected valve results in a murmur. A murmur is not a direct result of variances in blood viscosity, conductivity, or friction between the heart and myocardium.

Inspection of an obese, female patient reveals the presence of a foul odor that emanates from the patient's abdominal skin folds. The nurse would suspect that the odor is most likely caused by A. Ecchymosis. B. Colonization by yeast or bacteria. C. Age-related integumentary changes. D. Atrophy of the skin under the abdominal folds.

B. Unusual foul odors, especially those found in intertriginous areas, are often the result of colonization by yeast or bacteria. Ecchymosis is the presence of bruising whereas an unusual odor would not normally be attributed to age-related changes or skin atrophy.

During assessment of a pt, the RN notes an area of red, sharply defined plaques covered with silvery scales that are mildly itchy on the pt's knees and elbows, this is known as: A. lentigo B. psoriasis C. actinic keratosis D. seborrheic keratosis

B.Clinical manifestations of psoriasis include sharply demarcated, silvery scaling plaques on reddish skin, commonly on the scalp, elbows, knees; palms, soles, and fingernails; itching, burning, and pain; localized or general, intermittent or continuous pattern; and symptoms that vary in intensity from mild to severe.

Which method will the nurse use to assess skin lesions for cancer? A. American Cancer Society Skin Assessment B. Asymmetry, border, color, diameter, evolving C. Dermatologist skin review D. Size, location, and inflammation

B> The ABCDE method is the accepted technique for assessing skin lesions.

When examining a client with emphysema, what physical finding is the nurse likely to see?

Barrel chest, dry or productive cough, decreased breath sounds, dyspnea, crackles in lung fields

3

Based on the theoretical model of change, what is the most appropriate response to a patient who states: "Me, exercise? I haven't done that since junior high gym class and I hated it then." 1. "That's fine. Exercise is bad for you anyways." 2. "Ok. I want you to walk three miles four times a week and I will see you in one month." 3. "I understand. Can you think of one reason why being more active could be more helpful for you?" 4. "I'd like you to ride your bike three times a week and eat at least four fruits and vegetables every day."

Chap 38. The nursing diagnosis Risk for Impaired Skin Integrity related to sensory-perception disturbance would best fit a client who:

Because of the paraplegia (paralysis of lower body), the client is unable to feel discomfort. The client will be taught to lift self using chair arms every 10 minutes if possible. "Cut foot by stepping on broken glass." is an actual problem versus a potential problem. In "Wears glasses because of poor vision.", the client wears glasses that help correct the poor vision. "Is legally blind and smokes in bed." is more of a Risk for Injury diagnosis.

-olol

Beta Blockers

What should the O2 flow rate be for the client with COPD?

Between 1 and 2 L per nasal cannula; too much O2 may eliminate the COPD client's stimulus to breath. A COPD client has a hypoxic drive to breathe.

After surgery the patient with a closed abdominal wound reports a sudden "pop" after coughing. When the nurse examines the surgical wound site, the sutures are open, and pieces of small bowel are noted at the bottom of the now-opened wound. Which corrective intervention should the nurse do first? A) Allow the area to be exposed to air until all drainage has stopped B) Place several cold packs over the area, protecting the skin around the wound C) Cover the area with sterile, saline-soaked towels and immediately notify the surgical team; this is likely to indicate a wound evisceration D) Cover the area with sterile gauze, place a tight binder over it, and ask the patient to remain in bed for 30 minutes because this is a minor opening in the surgical wound and should reseal quickly

C

On assessing your patient's sacral pressure ulcer, you note that the tissue over the sacrum is dark, hard, and adherent to the wound edge. What is the correct stage for this patient's pressure ulcer? A) Stage II B) Stage IV C) Unstageable D) Suspected deep tissue damage

C

Risk factors that may lead to skin disease and breakdown include: A. loss of protective cushioning of the dermal skin layer. B. decreased vascular fragility. C. a lifetime of environmental trauma. D. increased thickness of the skin.

C

Which description best fits that of serous drainage from a wound? A) Fresh bleeding B) Thick and yellow C) Clear, watery plasma D) Beige to brown and foul smelling

C

Which of the following is an indication for a binder to be placed around a surgical patient with a new abdominal wound? A) Collection of wound drainage B) Reduction of abdominal swelling C) Reduction of stress on the abdominal incision D) Stimulation of peristalsis (return of bowel function) from direct pressure

C

Which skin care measures are used to manage a patient who is experiencing fecal and urinary incontinence? A) Keeping the buttocks exposed to air at all times B) Using a large absorbent diaper, changing when saturated C) Using an incontinence cleaner, followed by application of a moisture-barrier ointment D) Frequent cleaning, applying an ointment, and covering the areas with a thick absorbent towel

C

You assess four patients. Which patient is at greatest risk for the development of hypocalcemia? A) 56-year-old with acute kidney renal failure B) 40-year-old with appendicitis C) 28-year-old who has acute pancreatitis D) 65-year-old with hypertension and asthma

C

Your patient had 200 mL of ice chips and 900 mL intravenous (IV) fluid during your shift. Which total intake should you record? A) 700 mL B) 900 mL C) 1000 mL D) 1100 mL

C

Which precaution is most important for the nurse to teach a client prescribed adalimumab (Humira)? A. Drinking a full glass of water when taking each drug dose B. Reducing the drug dosage when psoriasis symptoms decrease C. Reporting symptoms of infection to the prescriber immediately D. Avoiding sunlight and tanning beds for the duration of drug therapy

C Rationale: Humira suppresses inflammatory and immune responses to some degree. This makes the client more susceptible to infection and may suppress some of the usual manifestations of infection. Together, these actions can allow a minor infection to become more severe very quickly. Any potential infection, no matter how minor, should receive immediate medical attention.

A number of strategies have the potential for creating work environments that enable nurses to demonstrate more caring behaviors. Some of these include:

C) Providing flexibility, autonomy, and improved staffing

10. A male patient has weakened knees due to arthritis. The home healthcare nurse determines that he understands the need for safety modifications at home by which of the following? a. Using the towel bar for support to stand up from the commode b. Leaning on the pedestal table in his bedroom when he dresses c. Sitting primarily in chairs with armrests d. Using a small stepladder to reach an item on an upper shelf

C.

10.Which of the following lists the recommended sequence for removing soiled personal protective equipment when the nurse prepares to leave the patient's room? a. Gown, goggles, mask, gloves, and exit the room b. Gloves, wash hands, remove gown, mask, and goggles c. Gloves, goggles, gown, mask, and wash hands d. Goggles, mask, gloves, gown, and wash hands

C.

11.For a nurse under normal conditions with unsoiled hands, effective hand hygiene between patients requires which of the following? a. At least a 15-second scrub with plain soap and water b. At least a 23-minute scrub with an antimicrobial soap c. Use of an alcohol-based antiseptic hand rub d. That a mask be worn when scrubbing

C.

12.The nurse is planning a health teaching session for new parents about poisoning emergencies. Which of the following would the nurse emphasize that the parents do first? a. Use salt water to induce vomiting b. Rush the child to the emergency department c. Call the poison control center d. Routinely administer syrup of ipecac for any poison ingestion

C.

13.A nurse is documenting a blood pressure of 120/80 mm Hg. The nurse interprets the 120 to represent which of the following? a. Pulse rate b. Diastolic pressure c. Systolic pressure d. Pulse deficit

C.

14.The nurse is about to bathe a female patient who has an intravenous access in place in her forearm. The patient's gown, which does not have snaps on the sleeves, needs to be removed prior to bathing. What should the nurse do? a. Temporarily disconnect the IV tubing at a point close to the patient and thread it through the gown sleeve. b. Cut the gown with scissors to allow arm movement. c. Thread the bag and tubing through the gown sleeve, keeping the line intact. d. Temporarily disconnect the tubing from the IV container, threading it through the gown.

C.

2. A nurse is assessing vital signs on several hospitalized children. The nurse would plan to use the oral route to assess temperature for which patient? a. 6-month-old infant b. Patient receiving oxygen therapy by mask c. 15-year-old healthy adolescent d. Unconscious patient

C.

3. Efforts by healthcare facilities to reduce the incidence of HAIs include an awareness of which of the following? a. The CDC requires all states to report HAI rates from each hospital. b. The gastrointestinal tract is a common site for HAIs. c. Joint Commission considers death or serious injury from HAIs a sentinel event d. The causal agent for most HAIs is viral.

C.

4. A patient develops a urinary tract infection after an indwelling urinary catheter has been inserted. This would most accurately be termed as which type of infection? a. A viral infection b. A chronic infection c. An iatrogenic infection d. An opportunistic infection

C.

7. Mr. James has an eye infection with a moderate amount of discharge. Which action would be most appropriate for the nurse to use when cleaning his eyes? a. Using hydrogen peroxide b. Wiping from the outer canthus to the inner canthus c. Positioning him on the same side as the eye to be cleansed d. Using only one cotton ball per eye

C.

The nurse is caring for a patient admitted with emphysema, angina, and hypertension. Before administering the prescribed daily dose of atenolol 100 mg PO, the nurse assesses the patient carefully. Which of the following adverse effects is this patient at risk for given the patient's health history? Hypocapnia Tachycardia Bronchospasm Nausea and vomiting

C. Bronchospasm Atenolol is a cardioselective β1-adrenergic blocker that reduces blood pressure and could affect the β2-receptors in the lungs with larger doses or with drug accumulation. Although the risk of bronchospasm is less with cardioselective β-blockers than nonselective β-blockers, atenolol should be used cautiously in patients with COPD.

During morning rounds, the nurse discovers that the older adult client has been incontinent during the night. To protect the skin, what will the nurse do first? A. Apply a barrier cream. B. Assess the area for skin breakdown. C. Clean the client. D. Place the client in a side-lying position.

C. Cleaning and drying the client is the first priority for skin protection.

Which statement by the client with psoriasis indicates that teaching about the condition has been effective? A. "I know that I need to avoid warm climates." B. "I need to cover up the affected areas to prevent spread to my family." C. "I should practice good handwashing technique." D. "Psoriasis can be cured with steroids."

C. Infections such as strep throat can exacerbate psoriatic flare-ups. Handwashing can help prevent infection.

8. While assessing the cardiovascular status of a patient, the nurse performs auscultation. Which of the following practices should the nurse implement into the assessment during auscultation? A. Position the patient supine. B. Ask the patient to hold his or her breath. C. Palpate the radial pulse while auscultating the apical pulse. D. Use the bell of the stethoscope when auscultating S1 and S2.

C. Palpate the radial pulse while auscultating the apical pulse. In order to detect a pulse deficit, simultaneously palpate the radial pulse when auscultating the apical area. The diaphragm is more appropriate than the bell when auscultating S1 and S2. A sitting or side-lying position is most appropriate for cardiac auscultation. It is not necessary to ask the patient to hold his or her breath during cardiac auscultation.

Which of the following laboratory tests would be most important to check in a patient presenting with purpura? A. Urinalysis B. Serum electrolytes C. Coagulation studies D. White blood cell count

C. Purpura are areas of ecchymoses that may signify a bleeding disorder. Therefore it is most important for the nurse to assess the patient's coagulation studies.

A common site for the lesions associated with atopic dermatitis is the: A. buttocks B. temporal area C. antecubital space D. plantar surfaces of the feet

C. The most common location for atopic dermatitis in adults is the antecubital or popliteal space.

During the physical examination of a pt's skin, the RN would: A. use a flashlight if the room is poorly lit B. note cool, moist skin as a normal finding C. pinch up a fold of skin to assess for turgor D. perform a lesion-specific examination first and then a general inspection

C. Turgor refers to the elasticity of the skin. The nurse should assess turgor by gently pinching an area of skin under the clavicle or on the back of the hand. Skin with good turgor should move easily when lifted and should immediately return to its original position when released.

The discharged obese client will require frequent dressing changes for a skin condition on the left foot. How will the nurse assess whether the client is able to perform this task at home? A. Asks the client if he is squeamish B. Demonstrates how to change the dressing C. Determines whether the client can reach the affected area D. Provides all the necessary dressing materials

C. Whether the obese client can access the dressing site is the most important thing to assess. If the dressing site cannot be accessed by the client, it will be difficult for the client to perform frequent dressing changes at home.

A client who has had an excisional biopsy of a skin lesion in the same-day surgery unit is ready for discharge. Which nursing activity will the nurse assign to an LPN/LVN working with this client? A. Teach the client about signs of incisional infection. B. Instruct the client about how to do dressing changes. C. Apply an antibiotic ointment and place a sterile dressing on the incision. D. Complete the written discharge instructions for the long-term care facility.

C. Wound care is included in practical nursing education.

What is the best way for the nurse to prevent the client's stage I pressure ulcer from advancing to stage II? A. Massage the reddened areas. B. Pad the ulcer. C. Promote mobility and/or frequent repositioning. D. Suggest an egg crate mattress.

C> Frequent repositioning and/or promoting mobility is the best way to prevent further deterioration of this client's pressure ulcer.

The young client has been diagnosed with ringworm, but the mother would like the child to return to school. To avoid spreading the infection, what will the nurse suggest to the mother? A. "Wash your hands frequently." B. "Your child may return to school but must be isolated from the rest of the class." C. "Keep the site covered with a bandage." D. "Keep your child out of school until the infection has cleared."

C> Keeping the site covered prevents spread of the infection.

The nurse is instructing the client on skin and sun protection. Which statement by the client indicates the need for further teaching? A. "My skin is better protected from the sun because I am dark skinned." B. "Sunscreen should be applied liberally." C. "I use a tanning bed to avoid the sun's harmful rays." D. "My sunglasses are UVA and UVB protected."

C> Tanning beds are just as damaging to the skin as the sun's rays; this statement indicates that the client needs further teaching.

Which nursing interventions can the nurse working in a long-term care facility delegate to a nursing assistant? A. Use the Braden scale to determine pressure ulcer risk for a newly admitted client. B. Complete daily sterile dressing changes for a client with a venous leg ulcer. C. Reposition every 2 hours a client who has had a stroke and is incontinent. D. Admit a newly transferred client who had pedicle flap surgery 1 week ago.

C> The nursing assistant has the education and scope of practice to reposition a client.

The nurse prepares to administer vancomycin (Lyphocin, Vancocin) to a client diagnosed with methicillin-resistant Staphylococcus aureus (MRSA) infection. How will the nurse administer this medication? A. Administer by bolus. B. Give IV push. C. Infuse over 60 minutes. D. Mix vancomycin with primary intravenous (IV) bag.

C> Vancomycin (Lyphocin, Vancocin) is irritating to the veins and can trigger thrombophlebitis; it should be given over at least 60 minutes.

Chap 34. The nurse doing the health teaching to a client for testing feces for occult blood informs the client that some items can produce false-positive results. Which of the following statements is consistent with what the nurse should emphasize?

Certain foods can produce inaccurate test results. False-positive results can occur if the client has recently ingested red meat or raw radishes and melons. Taking more than 250 mg of vitamin C from food or supplements may produce a false-negative result even if the client is bleeding. The presence of color is not what is being assessed with occult blood, so eating colored vegetables does not influence the test results unless they are raw turnips, radishes, or horseradish. Tea does not have any impact on occult blood results.

Kyphosis, a change in the musculoskeletal system, leads to:

Changes in the configuration of the spine that affect the lungs and thorax

Atrial Fibrillation

Chaotic activity in the AV node , No true P waves visible, Irregular ventricular rhythm TX: Anticoagulant therapy due to risk for stroke, antidysrhythmic drugs, cardioversion to tx atrial dysrhythmias

Parkinson Disease

Chronic, progressive, debilitating neurologic disease of the basal ganglia and substantia nigra, affecting motor ability and characterized by tremor at rest, increased muscle tone (rigidity), slowness in the initiation and execution of movement (bradykinesia), and postural instability (difficulties with gait and balance) Nursing Assessment: rigidity of extremities, masklike facial expressions, drooling, stooped posture, tremors at rest, emotional lability. Drugs: anticholinergics, dopamine replacements, monoamine oxidase type b inhibitor

acute glomerulonephritis

Classic sign of this is Cola colored (red-brown) urine. Other signs are: sudden onset of flank pain, irritability, malaise, fever, gross or microscopic hematuria, dysuria, edema, hypertension & oliguria.

Chap 34. A 78-year-old male client needs to complete a 24-hour urine specimen. In planning his care, which of the following measures is most important?

Communication of the test to all staff is likely to result in completing the collection. At the beginning of the test, instructing him to empty his bladder and saving this voiding to start the collection is incorrect because you want to discard the first voiding. A clean receptacle, not a sterile one, is used to collect the urine; the purpose of the test will determine if refrigeration is needed.

Chap 43. If a client loses a spouse after 50 years and begins giving away possessions, this is a clue to the nurse that the client is experiencing:

Complicated grieving may be characterized by extended time of denial, depression, severe physiologic symptoms, or suicidal thoughts. Giving away one's possessions can be a warning sign of the potential for suicide.

Glaucoma

Condition characterized by increased intraocular pressure Gradual painless vision loss.

Cataract

Condition characterized by opacity of the lens Early signs include: blurred vision, decreased color perception, diplopia, reduced visual acuity, clouded pupil.

Chap 38. A client has been oriented to person, time, and place but becomes disoriented within 24 hours of admission to the hospital. What is the most probable reason for this?

Confusion can be caused by physiologic disturbances such as chronic medical problems (e.g., dementia, chronic obstructive pulmonary disease, hypertension, stroke) that place them at risk. Chronic depression does not cause an acute change in orientation. Lack of sensory stimulation generally does not cause disorientation. Failure to orient the client would not cause disorientation to person and place.

What symptoms of pneumonia might the nurse expect to see in an older adult?

Confusion, lethargy, anorexia, rapid respiratory rate

Chap 49. A student nurse questions the nurse about the difference between constipation and fecal impaction. Which of the following statements is most accurate?

Constipation is fewer than three bowel movements per week with the passage of hard, dry stool or no stool. It is defined in relation to the person's regular elimination patterns. Fecal impaction is the collection of hardened feces in the folds of the rectum, not the anus.

To assist an adult client to sleep better, the nurse recommends which of the following?

Consuming a small glass of warm milk at bedtime A small glass of milk relaxes the body and promotes sleep. Alcohol, large meals, and exercising all within 1 to 2 hours of bedtime have insomnia-producing effects and may, in fact, stimulate wakefulness. Large meals could also produce indigestion.

"Presence" involves a person-to-person encounter that:

Conveys closeness and a sense of caring

Which pt would be at the greatest risk for the potential development of hypermagnesmia? A) 83 year old man with lung cancer and HTN B) 65 year old woman with HTN taking B-adrenergic blockers C) 42 year old woman with systemic lupus erythematosus and renal faliure D) 50 year old man with BPH and a UTI

Correct answer: c Rationale: Causes of hypermagnesemia include renal failure (especially if patient is given magnesium products), excessive administration of magnesium for treatment of eclampsia, and adrenal insufficiency

Chap 38. In what way does culture affect sensory functioning?

Culture determines the amount of sensory stimulation that a person considers usual or normal, and also affects the amount of stimulation an individual desires and believes to be meaningful. Physiological changes related to sensation are usually due to developmental changes. Culture does not determine the levels of stress; however, it does affect how one deals with the stress. The occurrence of illnesses related to sensory functioning does not depend upon the culture of the individual.

A nurse is teaching a community group about ways to minimize the risk of developing osteoporosis. Which of the following statements made by a woman in the audience reflects a need for further education? A) "I usually go swimming with my family at the YMCA 3 times a week." B) "I need to ask my doctor if I should have a bone mineral density check this year." C) "If I don't drink milk at dinner, I'll eat broccoli or cabbage to get the calcium that I need in my diet." D) "I'll check the label of my multivitamin. If it has calcium, I can save money by not taking another pill. "

D

A patient with a cardiac history is taking the diuretic furosemide (Lasix) and is seen in the emergency department for muscle weakness. Which laboratory value do you assess first? A) Serum albumin B) Serum sodium C) Hematocrit D) Serum potassium

D

A patient with left-sided weakness asks his nurse, "Why are you walking on my left side? I can hold on to you better with my right hand." What would be your best therapeutic response? A) "Walking on your left side lets me use my right hand to hold on to your arm. In case you start to fall, I can still hold you." B) "Would you like me to walk on your right side so you feel more secure?" C) "Either side is appropriate, but I prefer the left side. If you like, I can have another nurse walk with you who will hold you on the right side." D) "By walking on your left side I can support you and help keep you from injury if you should start to fall. By holding your waist I would protect your shoulder if you should start to fall or faint.

D

A student nurse has been assigned to teach fourth graders about hygiene. While preparing, the student nurse adds information about the sweat glands. Which of the following should be included while discussing this topic? A. There are two types of sweat glands: the eccrine and the sebaceous. B. The evaporation of sweat, a dilute saline solution, increases body temperature. C. Eccrine glands produce sweat and are mainly located in the axillae, anogenital area, and navel. D. Newborn infants do not sweat and use compensatory mechanisms to control body temperature.

D

An older adult who was in a car accident and fractured his femur has been immobilized for 5 days. Which nursing diagnosis is related to patient safety when the nurse assists this patient out of bed for the first time? A) Chronic pain B) Impaired skin integrity C) Risk for ineffective cerebral tissue perfusion D) Risk for activity intolerance

D

The knee joint is the articulation of three bones, the: A. femur, fibula, and patella. B. femur, radius, and olecranon process. C. fibula, tibia, and patella. D. femur, tibia, and patella

D

The nurse puts elastic stockings on a patient following major abdominal surgery. The nurse teaches the patient that the stockings are used after a surgical procedure to: A) Prevent varicose veins. B) Prevent muscular atrophy. C) Ensure joint mobility and prevent contractures. D) Promote venous return to the heart.

D

When repositioning an immobile patient, the nurse notices redness over a bony prominence. What is indicated when a reddened area blanches on fingertip touch? A) A local skin infection requiring antibiotics B) Sensitive skin that requires special bed linen C) A stage III pressure ulcer needing the appropriate dressing D) Blanching hyperemia, indicating the attempt by the body to overcome the ischemic episode.

D

Which of the following describes a hydrocolloid dressing? A) A seaweed derivative that is highly absorptive B) Premoistened gauze placed over a granulating wound C) A debriding enzyme that is used to remove necrotic tissue D) A dressing that forms a gel that interacts with the wound surface

D

While receiving a blood transfusion, your patient develops chills, tachycardia, and flushing. What is your priority action? A) Notify a health care provider B) Insert an indwelling catheter C) Alert the blood bank D) Stop the transfusion

D

The client reports that a previously sensitive area of skin no longer responds to temperature changes or painful stimuli. This finding indicates a functional abnormality for which skin layer? A. Stratum corneum B. Adipose layer C. Epidermis D. Dermis

D Rationale: The dermis or dermal layer of skin contains sensory nerves that transmit the sensations of touch, pressure, temperature, pain, and itch. The stratum corneum, epidermis, and adipose tissue do not transmit sensation. Because this client's skin area was once sensitive and now is not suggests an abnormality in the function of the dermis.

The family of a patient newly diagnosed with hepatitis A asks the nurse what they can do to prevent becoming ill themselves. Which of the following responses by the nurse is most appropriate? a) "The hepatitis vaccine will provide immunity from this exposure and future exposures." b) "I am afraid there is nothing you can do since the patient was infectious before admission." c) "You will need to be tested first to make sure you don't have the virus before we can treat you." d) "An injection of immunoglobulin will need to be given to prevent or minimize the effects from this exposure."

D) An injection of immunoglobulin will need to be given to prevent or minimize the effects from this exposure Immunoglobulin provides temporary (1-2 months) passive immunity and is effective for preventing hepatitis A if given within 2 weeks after exposure. It may not prevent infection in all persons, but it will at least modify the illness to a subclinical infection. The hepatitis vaccine is only used for preexposure prophylaxis.

5.The nurse has opened the sterile supplies and donned two sterile gloves to complete a sterile dressing change, a procedure that requires surgical asepsis. The nurse must do which of the following? a. Keep splashes on the sterile field to a minimum b. Cover the nose and mouth with gloved hands if a sneeze is imminent c. Use forceps soaked in a disinfectant d. Consider the outer 1-inch of the sterile field as contaminated

D.

5.When describing safety issues and related mortality to a local senior citizens group, which of the following would the nurse identify as the leading cause of accidental death for people 79 years of age and older? a. Fires b. Exposure to temperature extremes c. Drug overdose d. Falls

D.

7.When discussing emergency preparedness with a group of first responders, which of the following would be important to include about preparation for a terrorist attack? a. Post traumatic stress disorders can be expected inmost survivors of a terrorist attack b. The FDA has collaborated with drug companies to create stockpiles of emergency drugs c. Even small doses of radiation result in bone marrow depression and cancer d. Blast lung injury is a serious consequence following detonation of an explosive device

D.

8.An older resident who is disoriented likes to wander the halls of his long-term care facility. Which of the following would be most appropriate for the nurse to use as an alternative to restraints? a. Sitting him in a geriatric chair near the nurses' station b. Using the sheets to secure him snugly in his bed c. Keeping the bed in the high position d. Identifying his door with his picture and a balloon

D.

8.Which of the following interventions would the nurse include in the plan of care when providing foot care to an older patient? a. Using scissors to correct an ingrown toenail b. Trimming toenails as short as possible c. Using an alcohol rub if the feet are dry d. Bathing the feet at least daily

D.

9.Which organization(s) initially developed the guidelines for minimum protection standards for infection prevention and control? a. OSHA b. Individual healthcare facilities c. The state governing body d. The CDC

D.

1. Which of the following instructions given to a patient who is about to undergo Holter monitoring is most appropriate? A. "You may remove the monitor only to shower or bathe." B. "You should connect the monitor whenever you feel symptoms." C. "You should refrain from exercising while wearing this monitor." D. "You will need to keep a diary of all your activities and symptoms."

D. "You will need to keep a diary of all your activities and symptoms." A Holter monitor is worn for at least 24 hours while a patient continues with usual activity and keeps a diary of activities and symptoms. The patient should not take a bath or shower while wearing this monitor.

During the postoperative client assessment, which skin condition discovered by the nurse requires an urgent response? A. Clubbing of the nail beds B. Cool extremities C. Café au lait spots D. Warm red area on the calf

D. A warm red area on the calf is indicative of a thrombus and requires urgent attention.

The nurse is evaluating the effectiveness of interventions for pressure ulcer management. Which diagnostic test result with an increased level indicates client progress and effective health care team collaboration? A. Calcium B. Hematocrit C. Numbers of immature white blood cells (WBCs) D. Serum albumin

D. Albumin measures protein, which is necessary for healing. Increased serum albumin indicates successful collaboration with the dietitian.

The nurse identifies the priority problem of skin breakdown related to poor hygiene in a long-term care client who has areas of skin breakdown in the skin folds and the perineal area. Which intervention will be best for the RN to delegate to the nursing assistant? A. Check the client's skin weekly for areas of redness or breakdown. B. Teach the client and family about the importance of good hygiene in skin folds. C. Evaluate the client's ability to provide skin hygiene independently. D. Bathe the client, and apply a protective barrier to skin folds and perineum.

D. Assisting clients with personal hygiene is included in nursing assistant education.

The home health nurse is doing an intake assessment on a client who had a recent shave biopsy of a basal cell carcinoma located on the client's cheek. Which statement by the client may indicate the greatest need for client teaching? A. "Every morning, I check the biopsy site for signs of infection." B. "I have been cleaning my face with soap and water." C. "My appetite is improving lately." D. "I have been working in my garden for several hours every day."

D. Basal cell carcinomas of the skin are associated with sun exposure. The nurse should further assess the client for knowledge about the association between sun exposure and skin cancers and for use of sunscreens.

4. The blood pressure of a 71-year-old patient admitted with pneumonia is 160/70 mm Hg. Which of the following is an age-related change that contributes to this finding? A. Stenosis of the heart valves B. Decreased adrenergic sensitivity C. Increased parasympathetic activity D. Loss of elasticity in arterial vessels

D. Loss of elasticity in arterial vessels An age-related change that increases the risk of systolic hypertension is a loss of elasticity in the arterial walls. Because of the increasing resistance to flow, pressure is increased within the blood vessel and hypertension results.

Which of the following practices should the nurse teach a patient to follow when the patient is applying topical medication? A. Avoid applying medications directly on to dressings B. Use a tongue blade whenever the patient's skin integrity allows C. Avoid covering skin regions that have topical medication in place D. Apply a layer of medication that is just thick enough to ensure coverage

D. Patients should be directed to avoid applying topical medications too thickly. Medications may be applied directly on to dressings, and regions with medications may be covered. A tongue blade is not normally necessary for application.

A pt with psoriasis tells the Rn that she has quit her job as a receptionist because she feels her appearance is disgusting to customers. the nursing diagnosis that best describes this pt response is: A. ineffective coping r/t lack of social support B. impaired skin integrity r/t presence of lesions C. anxiety r/t lack of knowledge of the disease process D. social isolation r/t decreased activities secondary to fear of rejection

D. Psoriasis can be severe and disabling, and people withdraw from social contacts because of visible lesions.

A 14-year-old female and her mother have presented to their nurse practitioner seeking treatment for the daughter's acne. The nurse would recognize that acne is characterized by the presence of multiple: A. Ulcers. B. Wheals. C. Vesicles. D. Pustules.

D. Pustules are elevated, superficial lesions filled with purulent fluid, such as those commonly associated with acne. Wheals, ulcers, and vesicles are not common manifestations of acne.

The nurse admits a client to the clinic who is reporting severe itching to the arms and legs caused by exposure to poison ivy. The nurse anticipates that the health care provider will prescribe which medication? A. Anthralin (Anthaforte, Drithocreme, Lasan) B. Benzyl benzoate (Ascabiol) C. Calcipotriene (Dovonex) D. Diphenhydramine (Benadryl)

D> Treatment is aimed at removal of the triggering substance and relief of symptoms. Because the skin reaction is caused by histamine release, antihistamines such as diphenhydramine (Benadryl) are helpful.

What can positive Horman's sign indicate?

DVT

intrarenal ARF

Damage to renal parenchyma Causative Factors: Prolonged prerenal state, nephrotoxins, intratubular obstruction, infections (glomerulonephritis), renal injusry, vascular lesions, acute pyelonephritis

Myasthenia Gravis

Disorder affecting the neuromuscular transmission of impulses in the voluntary muscles of the body. It is considered an autoimmune disease characterized by the presence of acetylcholine receptor antibodies, which interfere with neuronal transmission Nursing assessment: diplopia, ptosis, sleepy appearance, weakness of laryngeal and pharyngeal muscles, dysphagia, muscle weakness improved by rest, worsened by activity.

Myocardial Infarction Pathophysiology Precipitating Factors Assessment Nursing Intervention

Disruption in or deficiency of coronary artery blood supply, resulting in necrosis of myocardial tissue Causes: thrombus or clotting, shock or hemorrhage Sudden onset of pain in the lower sternal region, rapid irregular, and feeble pulse, decreased LOC, cardiac dysrhythmias, occurring in about 90% of MI clients, cardiogenic shock or fluid retention, creatine kinase (CK) intracellular enzymes that are released into circulation after an MI: rise 3 to 12 hours after the onset of MI, peak at 24 hours, return to normal within 2 to 3 days; CK-MB band is specific to myocardial cells and can help quantify myocardial damage; Cardiac specific troponin is a myocardial muscle protein released into circulation after MI injury with greater sensitivity than CK-MB; narrowed pulse pressure (e.g 90/80 mmHg); bowel sounds are absent or high pitched. Administer IV morphine sulfate for pain and increase O2 perfusion, acts as a peripheral vasodilator and decreases venous return, nitrates, beta blockers, calcium channel blockers, aspirin, antiplatelet aggregates, obtain vitals, ECG rhythm, administer O2 at 2-6L per nasal cannula, obtain cardiac enzymes, assess breath sounds for rales (indicating pulmonary edema), maintain patent IV line for administration of emergency meds, monitor fluid balance, keep in semi-Fowlers position

Chap 38. A client who had a stroke is unable to distinguish written words and symbols. Which of the following is the most appropriate nursing diagnosis for this client?

Disturbed Sensory Perception is the condition where an individual has a change in the amount of incoming stimuli accompanied by an impaired response to these stimuli. The inability to recognize written words and symbols is a disturbance in the interpretation of the stimuli. Impaired Memory means that a person has inability to remember or recall information or behaviors. Acute Disorientation refers to the person not being aware of time, space, or place. Ineffective cognition is a very vague term and is not a nursing diagnosis.

-mide

Diuretics

Causes of Metabolic Alkalosis

Diuretics, excessive vomiting, GI suctioning, hyperaldosteronism, ingestion of and/or infusion of excessive sodium bicarbonate, massive transfusion of whole blood

Answer: A When dealing with clients who are at risk for or may have suffered abuse, it is important to provide protection. Educating the mother on the developmental issues of her infant is important but provides no protection for the victim. Providing protection and eliminating the fear of retribution is a priority upon discovery of abuse. By disregarding the mother's situation, the nurse has failed to intervene for the family in crisis in the community.

During a well-baby visit, the community-based nurse observed patterned bruises and skin abrasions on the face, arms, and throat of the infant's 21-year-old mother. In questioning the mother, the nurse discovers that she is a recent victim of spousal abuse. An important principle in dealing with this client is: A) Ensuring the protection of the mother B) Informing the authorities of the attack C) Educating the mother on well-baby developmental issues D) Continuing with the well-baby examination and disregarding the mother's situation

Answer: D Kyphosis (hunchback) is an exaggeration of the posterior curvature of the thoracic spine and is common in older adults. Lordosis (swayback) is increased lumbar curvature. Scoliosis is lateral spinal curvature. Hypotonic muscle has little tone and feels flabby, usually because of atrophy of muscle mass.

During general inspection of the musculoskeletal system of an older client, the nurse notes kyphosis. Kyphosis is: A) Lateral spinal curvature B) Loss of or decrease in muscle tone C) Increased lumbar curvature D) Exaggeration of the posterior curvature of the thoracic spine

Chap 43. A nursing student has a client who just lost her brother to suicide. The client accepts the situation intellectually but denies it emotionally. Which of Engel's stages of grieving is the client experiencing?

During the shock and disbelief stage, the client refuses to accept loss, has stunned feelings, and accepts the situation intellectually but denies it emotionally. During the developing awareness stage, reality of loss begins to penetrate consciousness, and anger may be directed at the agency, nurses, or others. During the restitution stage, the client conducts rituals of mourning (e.g., funeral). During the stage of resolving the loss, the client attempts to deal with the painful void, is still unable to accept a new love object to replace the lost person or object, may accept a more dependent relationship with a support person, and thinks over and talks about memories of the lost person or object.

Pulse Oximetry

Easy measurement of O2 sat Should be >90%, ideally above 95% No nail polish Can attach to ear, toe, finer Must have good peripheral perfusion to be accurate

Chap 34. A client presents to the ER with shortness of breath and chest pain. The nurse knows that the following procedures may be used to determine a diagnosis:

Electrocardiography provides a graphic recording of the heart's electrical activity by using electrodes placed on the skin that transmit electrical impulses to a graphic recorder. Stress electrocardiography uses electrocardiograms to assess the client's response to an increased cardiac workload during exercise. Angiography is an invasive procedure using radiopaque dye that is injected into the vessels to be examined; this will help to diagnose a cardiac problem. In a lumbar puncture the cerebrospinal fluid is withdrawn to obtain a specimen to determine a diagnosis or to detect malignancy; this is not a test performed to diagnosis a person with shortness of breath or chest pain. A thoracentesis is performed to remove excess fluid or air to ease breathing; this would be done after diagnosis.

The nurse is developing a plan of care for a client experiencing narcolepsy. Which intervention is appropriate to include on the plan?

Encourage client to take one or two 20-minute naps during the day Brief daytime naps of no longer than 20 minutes help reduce subjective feelings of sleepiness. Carbohydrates can increase sleepiness. Limiting fluids will not help the client with narcolepsy, nor will energy preservation.

Chap 49. Which of the following guidelines should be followed when administering an enema to children?

Enemas should be 200-300 mL in children 18 months to 5 years. Insert the tube 5 to 7.5 cm in children and 2.5 to 2.75 cm in infants. Enema temperature should be 37.7°C, which is slightly higher than normal body temperature. The enema solution should be isotonic, as hypertonic solutions can lead to hypovolemia. Enemas should be 200-300 mL in children 18 months to 5 years.

When assessing the older adult, the nurse should review the client's achievement of developmental tasks. For the older adult, these may include all of the following except:

Engaging in more introspective, self-focused activities

The nurse is gathering a sleep history from a client who is being evaluated for obstructive sleep apnea. What common symptom will the client most likely report?

Excessive daytime sleepiness The client will awake with a headache. The other options may exist, but headache is the most common complaint

General health promotion and illness prevention measures the nurse may recommend to older adults include

Exercising regularly

FHR (fetal heart rate) at or near term

120 - 160 beats/mins

Triglycerides

< 200 mg/dL

The nurse in the outpatient clinic assists with the application of a cast to the left arm of a pre-school-aged child. After the cast is applied, the nurse should take which of the following actions? 1. Petal the edges of the cast to prevent irritation. 2. Elevate the client's left arm on two pillows. 3. Apply cool, humidified air to dry the cast. 4. Ask the client to move the fingers to maintain mobility.

(1) done when cast is completely dry, prevents crumbling of plaster into cast (2) correct—minimizes swelling, elevated for first 24 to 48 hours, protects from pressure and flattening of cast (3) would delay drying of cast (4) maintaining mobility of fingers not most important after application of cast

64.The nurse is concerned that which of the following viral infections, if experienced by an adult male, may cause infertility?</question> <question_image/> <choice_1>Varicella zoster</choice_1> <choice_2>Rubella</choice_2> <choice_3>Influenza</choice_3> <choice_4>Mumps</choice_4>

<correct>4</correct> <rationale>Mumps in adult males can cause permanent blockage of the vas deferens, contributing to or resulting in infertility. The other responses are incorrect.</rationale>

The components of a nail examination include: A. contour, consistency, and color. B. shape, surface, and circulation. C. clubbing, pitting, and grooving. D. texture, toughness, and translucency.

A

In which position does the nurse place the client immediately after a rhytidectomy to promote venous return and prevent swelling? A. Fowler's B. Lithotomy C. Lateral Sims' D. Trendelenburg

A Rationale: Only Fowler's position would make the face less dependent, thus promoting venous return and decreasing swelling.

The nurse assesses a client's neurosensory cerebellar functioning. Which of the following assessment techniques is correct? 1. Test the client's deep tendon reflexes to observe for weakness. 2. Check the client's pupils with a penlight and observe for constriction. 3. Have the client stand with eyes closed and observe for swaying. 4. Ask the client to show her teeth and stick out her tongue.

(1) general central nervous system response, not sensory involvement (2) evaluates for increased intraocular pressure (3) correct—coordination is governed by the cerebellum; this test evaluates neurosensory status (4) evaluates the facial and hypoglossal nerves

Previous administrations of chemotherapy agents to a cancer patient have resulted in diarrhea. Which of the following dietary modifications should the nurse recommend? A bland, low-fiber diet A high-protein, high-calorie diet A diet high in fresh fruits and vegetables A diet emphasizing whole and organic foods

A bland, low fiber diet Patients experiencing diarrhea secondary to chemotherapy and/or radiation therapy often benefit from a diet low in seasonings and roughage. Fresh fruits and vegetables are high in fiber and should be minimized during treatment. Whole and organic foods do not prevent diarrhea.

Answer: D Restorative care assists an individual in regaining the maximum possible level of functioning. Home care includes professional and paraprofessional services that are rendered in the home setting. Extended care is intermediate medical or nursing care for individuals with an acute or chronic illness or disability. Assisted care is a setting in which the client is able to function at a higher level of autonomy within a homelike environment but in which care can be given when needed.

A client discharged after suffering a stroke is transferred from a tertiary care facility to another facility for additional care to help the client recover and continue to regain function. This type of care facility is known as: A) Home care B) Assisted care C) Extended care D) Restorative care

Answer: A In the staff model of an MCO, the physicians are salaried employees. In the group model, the MCO contracts with a single group practice. An independent practice association is a group of physicians who are under contact to the organization but are not members of it and whose practices include fee-for-service and capitated clients. The MCO contracts with multiple group practices and/or integrated organizations in the network model.

A client is receiving health care from a health care provider who is a salaried employee. Which model is being followed by the managed care organization (MCO) to which the client belongs? (Select all that apply.) A) Staff model B) Group model C) Network model D) Independent practice association

Cushing Syndrome

A metabolic disorder characterized by abnormally increased secretion of cortisol, cause by increased amounts of adrenocorticotropic hormone secreted by pituitary gland. Cause is usually chronic administration of corticosteroids. Moon face, truncal obesity, buffalo hump, abdominal striae, muscle atrophy, thinning of the skin, amenorrhea. Lab data: hyperglycemia, hypernatremia, hypokalemia, decreased eosinophils and lymphocytes, increased plasma cortisol

Answer: B Quality client care is always the primary focus of nursing practice. Cost control would be a benefit but is not the primary focus. Research is not about technology. Many "old" procedures can be improved through research. Although research is a professional function of nursing, it is not done to serve the profession.

A priority goal for nursing research is: A) Controlling cost for hospitals B) Improving client care C) Keeping up with technological advances D) Maintaining the professional climate in nursing

The nurse is aware that Rh immune globulin (RhoGAM) is administered to prevent complications in which of the following situations? 1. The baby is Rh-negative, the mother is Rh-negative, and the father is Rh-positive. 2. The mother is Rh-negative, the baby is Rh-positive, and there is a negative direct Coombs. 3. The mother is Rh-positive and previously sensitized, and the baby is Rh-negative. 4. The mother is Rh-positive, the baby is Rh-negative, and there is a history of one incomplete pregnancy.

(1) if both mother and baby are Rh-negative, there is no problem (2) correct—RhoGAM is given to an Rh-negative mother who delivers an Rh-positive baby when the baby has a negative Coombs test (3) medication is not given if the mother has been sensitized by a previous pregnancy (4) there is no incompatibility here, but the mother needs to be evaluated regarding sensitization in the incomplete pregnancy

The nurse cares for a child who is in Buck's traction. During the neurovascular assessment, the nurse notes that the foot of the uninjured leg feels warmer to touch than that of the broken leg. The nurse should take which of the following actions? 1. Record the observation. 2. Encourage the child to move the foot. 3. Cover the colder foot with a sock. 4. Notify the physician.

(1) ignores possibility that Ace bandage is too tight (2) does not relieve the circulation problem (3) does not relieve the circulation problem (4) correct—assessment indicates that Ace bandage is too tight and needs readjusting

The MOST appropriate nursing action before administering captopril (Capoten) is to check the client's 1. apical pulse for 60 seconds. 2. blood pressure. 3. urine output. 4. temperature.

(1) important, but not a priority (2) correct—Capoten is an antihypertensive that necessitates assessment of BP before administration (3) important, but not priority (4) unnecessary to assess prior to the administration of the medication

Differentiate between ARF and CRF?

ARF: often reversible, abrupt deterioration of kidney fxn CRF: irreversible, slow deterioration of kidney fxn characterized by increasing BUN and creatinine. Eventually dialysis is required.

Chap 52. How does the normal aging process affect fluid balance?

Although in the elderly the ADH or antidiuretic hormone levels remain normal or may be elevated, the nephrons are less able to conserve water in response to the antidiuretic hormone. This is part of the normal aging process. Multiple medications and the presence of many chronic illnesses are not part of the normal aging process. Thirst response is usually blunted in the elderly and could lead to decreased fluid intake.

Which of the following regulates hospitals to ensure safety in the provision of services, establishes criteria that must be met for a hospital to receive funding from the government, and provides for penalties if guidelines are not followed?

Americans With Disabilities Act (ADA)

Answer: D Additional training in anesthesia medicine would be required to be a certified registered nurse anesthetist.

An APN is pursuing a job change. Which of the following positions would the APN be unable to fill without meeting additional criteria? A) Case manager B) Nurse manager C) Nurse educator D) Certified registered nurse anesthetist

Answer: A Care provider is a staff position, a nurse who provides direct care. The nurse specialist has clinical expertise in a specific area. The nurse practitioner has advanced training in assessment and pharmacology and is able to provide health care in specific settings. The case manager has additional experience and is able to coordinate activities of other members of the health care team.

An APN is the most independently functioning of all professional nurses. All of the following are examples of a clinically focused APN except: A) Care provider B) Case manager C) Nurse specialist D) Nurse practitioner

-cillin

Antibiotics

-oxacin

Antibiotics

-mycin

Antibiotics /Anti-infectives

Heparin

Anticoagulant Administered parenterally (SQ or IV) as an antagonist to thrombin and to prevent the conversion of fibrinogen to fibrin Assess PTT, Hgb, Hct, platelets; Assess stools for occult blood; Avoid IM injection; Notify anyone performing diagnostic testing of medication Antagonist: protamine sulfate

Nurses agree to be advocates for their clients. The practice of advocacy calls for the nurse to do which of the following?

Assess the client's point of view and prepare to articulate this point of view.

The nurse is caring for a patient admitted with a history of hypertension. The patient's medication history includes hydrochlorothiazide (Hydrodiuril) daily for the past 10 years. Which of the following parameters would indicate the optimal intended effect of this drug therapy? Weight loss of 2 lb Blood pressure 128/86 Absence of ankle edema Output of 600 ml per shift

B) Blood pressure of 128/86 Hydrochlorothiazide may be used alone as monotherapy to manage hypertension or in combination with other medications if not effective alone. After the first few weeks of therapy, the diuretic effect diminishes, but the antihypertensive effect remains. Since the patient has been taking this medication for 10 years, the most direct measurement of its intended effect would be the blood pressure.

1. A school nurse is preparing a talk on healthcare topics for adolescents. Which of the following would be important for the nurse to include? a. Less than 10% of students report regular tobacco use b. Reaching a BAL of 0.08 within 2 hours constitutes binge drinking. c. Teenagers are more likely to drink and drive d. Peer pressure has an insignificant affect on adolescent substance us

B.

13. A patient develops food poisoning from contaminated potato salad. What is the means of transmission for the infecting organism? a. Direct contact b. Vector c. Vehicle d. Airborne

C.

Chap 30. In a client with long-term emphysema, the nurse might expect to see which condition when inspecting the nails?

Clubbing is a condition where the nailbed is at least 180 degrees, often caused by lack of oxygen, such as in emphysema. Koilonychia is an abnormality where the nail curves upward from the nail bed, and is often seen in clients with iron-deficiency anemia. Paronychia is the technical term for "ingrown nail." A slow blanch test (greater than 2-3 seconds) may indicate circulatory problems.

List four components of teaching for the client with TB

Cough into tissues, and dispose of immediately into special bags; Long term need for daily medication; good handwashing techniques; report symptoms of deterioration

4.Which of the following would be most important for the nurse to keep in mind regarding the use of side rails for a confused patient? a. They prevent confused patients from wandering b. A history of a previous fall from a bed with raised side rails is insignificant c. Alternative measures are ineffective to prevent wandering d. A person of small stature is at increased risk for injury from entrapment

D

The newly admitted client has all of the following laboratory test values. Which value suggests to the nurse that the client may be at an increased risk for pressure ulcer formation? A. International normalized ratio (INR) of 1.5 B. White blood cell (WBC) count of 5200/mm3 C. Serum sodium concentration of 134 mEq/L D. Serum prealbumin concentration of 15.2 mg/dL

D Rationale: Adequate nutrition, especially protein intake, helps promote healthy skin and prevent tissue breakdown. A serum prealbumin concentration less than 19.5 mg/dL indicates inadequate nutrition and a severe protein deficiency. With so little protein, the skin cannot repair itself and is at great risk for injury even with minor trauma.

11.A nurse knows that the blood pressure is often higher in older adults based on the understanding that which of the following occurs with aging? a. Loss of muscle mass b. Changes in exercise level c. Decreased peripheral resistance d. Decreased elasticity in arterial walls

D.

13.An unresponsive patient is wearing gas-permeable con-tact lenses. How would the nurse remove these lenses? a. Gently irrigate the eye with an irrigating solution from the inner canthus outward. b. Grasp the lens with a gentle pinching motion. c. Don sterile gloves before attempting the removal procedure. d. Ensure that the lens is centered on the cornea before gently manipulating the lids to release it.

D.

13.Joint Commission guidelines regarding the use of restraints recommend which of the following? a. Vest restraints be used because they are the least restrictive type b. Restraints should be used for 48 hours in non-psychiatric patients c. Restraints should be applied to prevent wandering behavior d. Alternative measures must be attempted first

D.

14. A nurse is caring for an obese 62-year-old patient with arthritis who has developed an open reddened area over his sacrum. Which of the following is a priority nursing diagnosis? a. Imbalanced Nutrition: More Than Body Requirements related to immobility b. Impaired Physical Mobility related to pain and dis-comfort c. Chronic Pain related to immobility d. Risk for Infection related to altered skin integrity

D.

14.The nurse orients an older patient to the safety features in her hospital room. Which of the following is a vital component of this admission routine? a. Explain how to use the telephone b. Introduce the patient to her roommate c. Review the hospital policy on visiting hours d. Explain how to operate the call bell

D.

5.The nurse observes a marked inflammation of the gums, and recession and bleeding of the gums and documents this observation using which term? a. Glossitis b. Caries c. Cheilosis d. Periodontitis

D.

Intrarenal Stage of ARF

Damage to renal parenchyma Causative Factors: Prolonged prerenal state, nephrotoxins, intratubular obstruction, infections (glomerulonephritis), renal injusry, vascular lesions, acute pyelonephritis

What dz has ascending muscle weakness?

GBS

Chap 49. Clients should be taught that repeatedly ignoring the sensation of needing to defecate could result in which of the following?

Habitually ignoring the urge to defecate can lead to constipation through loss of the natural urge and the accumulation of feces. Diarrhea will not result—if anything, there is increased opportunity for water reabsorption because the stool remains in the colon, leading to firmer stool. The ability to ignore the urge indicates a strong voluntary sphincter, not a weak one, that could result in incontinence. Hemorrhoids would occur only if severe drying out of the stool occurs and, thus, repeated need to strain to pass stool.

Listening includes not only taking in what a client says but also:

Interpreting and understanding what the client means

Chap 52. Which of the following IV solutions are isotonic?

Isotonic solutions are often used to restore vascular volume. 0.9% NaCl and 5% dextrose in water are isotonic. 0.45% NaCl is hypotonic. 5% dextrose in normal saline and 5% dextrose in 0.45% NaCl are hypertonic

What discharge instructions should be given to a client who has had urinary calculi?

Maintain high fluid intake of 3 to 4L/day. Pursue follow up care (stones tend to recur). Follow prescribed diet based on calculi content. Avoid supine position.

To help reduce confusion of the older adult at night, the nurse may

Make telephone calls to friends or family members to let the older adult hear reassuring voices.

Hypocalcemia & Electrocardiographic Changes

Prolonged ST and QT intervals

Hypornatremia + Hypervolemic - assessment findings

Pulse - rapid & bounding BP - Normal to elevated CVP - normal to elevated

Which of the following terms is defined as a mental self-image of strengths and weaknesses in all aspects of one's personality?

Self-concept

Position for enema?

Sims (left side-lying)

Chap 52. The response of the body to stress as it relates to fluid volume is to:

Stress can increase cellular metabolism, blood glucose concentration, and catecholamne levels. In addition, stress can increase production of ADH, which in turn decreases urine production. The overall response of the body to stress is to increase the blood volume.

Visual acuity declines with age. Presbyopia is a progressive decline in:

The ability of the eyes to accommodate for close, detailed work

Which two factors contribute to the projected increase in the number of older adults?

The aging of the baby boom generation and the growth of the population segment over age 85

2

The nurse is developing a health promotion program on healthy eating and exercise for high school students using the health belief model as a framework. Which statement made by a nursing student is related to the individual's perception of susceptibility to an illness? 1. "I don't have time to exercise because I have to work after school every night." 2. "I'm worried about becoming overweight and getting DM because my father has DM." 3. "The statistics of how many teenagers are overweight is scary." 4. "I've decided to start a walking club at school for interested students."

Answer: D The code of ethics is the philosophical ideals of right and wrong that define the principles the nurse will use to provide care to clients. A code of ethics does not ensure identical care to all clients (which would not be acceptable). The nursing code of ethics does not protect clients from harm or improve self-health care.

The nurse practices nursing in conformity with the code of ethics for professional registered nurses. This code: A) Improves self-health care B) Protects the client from harm C) Ensures identical care to all clients D) Defines the principles by which nurses' provide care to their clients

Answer: D The nurse begins with inspection and then follows with auscultation. It is important to perform auscultation before palpation and percussion, because palpation and percussion may alter the frequency and character of bowel sounds.

The techniques of physical assessment are inspection, palpation, percussion, and auscultation. The order in which these techniques are used is slightly different during abdominal examination than during examination of other body areas. The nurse should perform which two of the following first? A) Palpation and inspection B) Inspection and percussion C) Palpation and auscultation D) Inspection and auscultation

A nurse hears a colleague tell a student nurse that it is best not to touch the clients unless performing a procedure or an assessment. Why is this not the best practice?

Touch forms a connection between nurse and client.

Chap 52. Which of the following events occurs when a person hyperventilates?

When a person hyperventilates, more carbon dioxide than normal is exhaled, carbonic acid levels fall, and the pH rises to greater than 7.45.

topical agents

act on local tissue

Guillain-Barre syndrome Signs and symptoms Major concerns

acute infectious neuronitis of cranial & peripheral nerves Usually preceeded by resp infection or gastroenteritis Major concern: difficulty breathing;monitor resp status closely

septicemia

bloodstream infection: if becomes very bas pt may have a red line along arm

What SE could be irreversible with someone receiving tamoxifen?

blurred vision

What's characteristic of cataract?

blurring, opacity

digitalis

cardiotonic

A client in a psychiatric facility describes seeing snakes on the walls of the room. Which of the following is an accurate nursing diagnosis? 1. Sensory-perceptual alterations: visual. 2. Altered thought processes. 3. Ineffective individual coping. 4. Impaired social interaction.

(1) correct—reflects a pattern of impaired perception, which is supported by the data that client is having a hallucination, defined as a sensory perception for which no external stimuli exist (2) not relevant to the data (3) not relevant to the data (4) not relevant to the data

continuous renal replacement therapy

-uremic toxins & fluids are removed while acid-base status & electrolytes are adjusted slowly & continuously -dialyze patients in a more physiologic way over 24 hours just like the kidneys -contraindicated in a patient with life-threatening manifestations of uremia -highly permeable, hollow fiber hemofilter -uses double lumen catheter placed in femoral, jugular, or subclavian vein -removes plasma water & non-protein solutes -can be used in conjunction w/ HD -continued for 30-40 days -hemofilter change q 24-48 hours -ultra filtrate should be clear yellow -specimens may be obtained for evaluation CRRT vs HD: -continuous rather than intermittent -does not require constant monitoring by specialized HD nurse -does not require complicated HD equipment

1. A client with no history of cardiovascular disease comes to the ambulatory clinic with flu-like symptoms. The client suddenly complains of chest pain. Which of the following questions would best help a nurse discriminate pain caused by a noncardiac problem? 1. "Can you describe the pain to me?" 2. "Have you ever had this pain before?" 3. "Does the pain get worse when you breathe in?" 4. "Can you rate the pain on a scale of 1 to 10, with 10 being the worst?"

1. 3 Rationale: Chest pain is assessed by using the standard pain assessment parameters (e.g., characteristics, location, intensity, duration, precipitating and alleviating factors, and associated symptoms). Options 1, 2, and 4 may or may not help discriminate the origin of pain. Pain of pleuropulmonary origin usually worsens on inspiration. Test-Taking Strategy Use the process of elimination, focusing on the subject, pain resulting from a noncardiac problem. The three incorrect options, although appropriate to use in practice, are general assessment questions only. Option 3 will discriminate between a cardiac and noncardiac cause of pain. Review pain assessment measures for the client with a cardiovascular problem if you had difficulty with this question.

A nurse is monitoring an adult client for postoperative complications. Which of the following w/be the most indicative of a potential postoperative complication that requires further observation?

1. A uninary output of 20mL/Hour. Urine output is maintained at a minimum of at least 30 mL/hr. for an adult. An ouptut of less than 30mL/hr. for each ot 2 consecutive hrs. s/be reported to the physician. A temp. more than 100*F or less than 97*F and a falling systolic blood pressure <90mm Hg are to be reported. The client's preoperative or baseline blood pressure is used to make informed postoperative comparisons. Moderate or light serous drainage from the surgical site is considered normal.

The nurse teaches the mother of a newborn that in order to prevent sudden infant death syndrome (SIDS), the best position in which to place the baby after nursing is

1. Supine 2. Side lying Research demonstrates that the occurrence of SIDS is reduced with these two positions. Placing the infant prone has been implicated as a cause of SIDS. Fowler's position is a semi-sitting position and has not been discussed in the prevention of SIDS.

fleets enema

120 ml most common, prep for colon exams

13. A nurse is caring for a client receiving dopamine. Which of the following potential nursing diagnoses is appropriate for this client? 1. Fluid volume, excess 2. Cardiac output, increased 3. Tissue perfusion, ineffective 4. Sensory perception, disturbed

13. 3 Rationale: The client receiving dopamine therapy should be assessed for ineffective tissue perfusion related to peripheral vasoconstriction. Options 1, 2, and 4 are not related directly to this medication therapy. Test-Taking Strategy: Use the process of elimination. Recalling that dopamine causes peripheral vasoconstriction will direct you to option 3. Review the action of this medication if you had difficulty with this question.

Normal Ammonia Value

14-45mcg/dL

Cholesterol

140 - 199 mg /dL

How many times can a pt take nitro?

3 times, if no relief, go to hospital >< if bp < 90/60

A nurse is caring for a client who is scheduled for surgery. The client is concerned about the surgical procedure. To alleviate the client's fears and misconceptions about surgery, the nurse should:

3. Ask the cliet to discuss information known about the planned surgery.

8.The client has decided to discontinue further treatment for cancer. Although the nurse would like the client to continue treatment, the nurse recognizes the client is competent and supports the client's decision using which of the following ethical principles?<choice_1>Justice</choice_1> <choice_2>Fidelity</choice_2> <choice_3>Autonomy</choice_3> <choice_4>Confidentiality</choice_4>

3</correct> <rationale>Autonomy refers to the right to make one's own decisions. Justice refers to fairness; fidelity refers to trust and loyalty; confidentiality refers to the right to privacy of personal health information.

Following transurethral resection of the prostate gland, hematuria should subside by what postop day?

4th day

1 teacup

6 oz.

How much does 1 grain =?

60 mg

The musculoskeletal system functions include: A. protection and storage. B. movement and elimination. C. storage and control. D. propulsion and preservation.

A

Which of the following assessments do you perform routinely when an older adult patient is receiving intravenous 0.9% NaCl? A) Auscultate dependent portions of lungs B) Check color of urine C) Assess muscle strength D) Check skin turgor over sternum or shin

A

Chap 34. The nurse practitioner requests a laboratory blood test to determine how well a client has controlled the client's diabetes over the past three months. Which blood test will provide this information?

A glycosylated hemoglobin test would best determine diabetes control over the past three months. Fasting blood glucose and capillary blood specimen will provide information about the current blood glucose level. Glucose tolerance test is used to determine if a client has diabetes.

1,3

A male patient has been laid off from his construction job and has many unpaid bills. He is going through a divorce from his marriage of 15 years and has been seeing a pastor to help him through this difficult time. He does not have a primary care provider because he has never really been sick. and his parents never took him as a child. What external variables influence the patient's health practices. (select all that apply) 1. Difficulty paying his bills. 2. Seeing his pastor as a means of support. 3. Family practice of not routinely seeing a healthcare provider. 4. Stress from the divorce and loss of a job.

What precautions are required for clients with TB when placed on respiratory isolation?

A mask for anyone entering room; private room; client must wear mask when leaving room

Chap 30. What evidence would most likely indicate to the nurse that a client had a negative Romberg test? The client:

A negative Romberg test would be indicated when a client was able to maintain an upright posture and foot stance with minimal swaying. A positive Romberg would show a client who couldn't maintain foot stance, moved the feet apart to maintain stance, and exhibited increased swaying.

Answer: D A knowledge-focused trigger is a question regarding new information available on a topic. A problem-focused trigger is one faced while caring for a client or noting a trend. The PICO (population, intervention, comparison, outcome) format is a way to phrase a question to help clarify the question and the parts. A hypothesis is a prediction about the relationship between study variables.

A new nurse on an orthopedic unit is assigned to care for a client undergoing skeletal traction. The nurse asks a colleague, "What is the best practice for cleaning pin sites in skeletal traction?" This question is an example of which of the following? A) Hypothesis B) PICO question C) Problem-focused trigger D) Knowledge-focused trigger

Chap 49. The nurse assesses a client's abdomen several days after abdominal surgery. It is firm, distended, and painful to palpate. The client reports feeling "bloated." The nurse consults with the surgeon, who orders an enema. The nurse prepares to give what kind of enema?

A return flow enema provides relief of postoperative flatus by stimulating bowel motility. Soapsuds enemas, retention enemas, and oil retention enemas manage constipation and do not provide flatus relief.

Chap 38. Which client is at greatest risk for experiencing sensory overload?

A sudden, unexpected admission for surgery may involve many experiences (e.g., lab work, x-rays, signing of forms) while the client is in pain or some form of discomfort. The time for orientation will thus be lessened. After surgery, the client may be in pain and possibly in a critical care setting. "A 40-year-old client in isolation with no family" and "A 28-year-old quadriplegic client in a private room" reflect a greater risk for sensory deprivation, and "A 16-year-old listening to loud music" is a normal activity for a teenager.

Answer: D This client has a fever, potentially secondary to the pneumonia previously diagnosed. His blood pressure is within normal limits. His oxygen saturation is at 92%, so this will need to be addressed second. His respiratory rate is high, which can be a result of the fever.

An 82-year-old widower brought via ambulance is admitted to the emergency department with complaints of shortness of breath, anorexia, and malaise. He recently visited his health care provider and was put on an antibiotic for pneumonia. The client indicates that he also takes a diuretic and a beta blocker, which helps his "high blood." Which vital sign value would take priority in initiating care? A) Respiration rate = 20 breaths per minute B) Oxygen saturation by pulse oximetry = 92% C) Blood pressure = 138/84 D) Temperature = 39° C (102° F), tympanic

Chap 49. A client is being prepped for intestinal surgery. The nurse has an order to administer an enema. Before getting a clarification order as to the type of enema to administer, the nurse is aware of the following types

An enema is a solution introduced into the rectum and large intestine. The action of an enema is to distend the intestine and sometimes to irritate the intestinal mucosa, thereby increasing peristalsis and the excretion of feces and flatus. Enemas are classified as cleansing, carminative, retention, and return flow.

Answer: B Surgical clients are the client population of interest (P) in the PICO (population, intervention, comparison, outcome) question. Chlorhexidine use is the comparison of interest, and povidone-iodine use is the intervention of interest. The operating room nurse is not an element of the PICO question.

An operating room nurse is talking with colleagues during a meeting and asks, "I wonder if we would see fewer wound infections if we used chlorhexidine instead of povidone-iodine to clean the skin of our surgical clients? In this example of a PICO question, the P is: A) Povidone-iodine use B) Surgical clients C) Chlorhexidine use D) Operating room nurses

Chap 43. A nurse is evaluating a nursing student who is caring for a dying client's physiological needs. Which of the following actions demonstrates a need for further teaching?

An unconscious client experiencing airway clearance problems would be placed in a lateral position. A conscious client with an airway clearance problem would be placed in a Fowler's position. If the client is diaphoretic, the nursing student would give the client frequent baths, change the linen, and regularly change the client's position. The nursing student would provide skin care to the client in response to incontinence of urine or feces.

88.With regard to normal changes in the reproductive system during pregnancy, the nurse should reinforce client teaching about which of the following?</question> <question_image/> <choice_1>Vaginal secretions will increase and thicken.</choice_1> <choice_2>Uterus will grow by adding many new cells.</choice_2> <choice_3>Breasts will become red and hard.</choice_3> <choice_4>Cervix will begin to dilate during the second trimester.</choice_4>

Answer: 1 <rationale>During pregnancy, increased estrogen production results in an increased amount and thickening of vaginal secretions. The uterus grows by cell hypertrophy, not by adding more cells. Red and hard breasts or a cervix dilating during the second trimester are not normal findings.</rationale>

Chap 48. Which of the following types of medication affect the client's voiding?

Antihypertensive medication may cause urine retention. Antibiotics, blood thinners, and stool softeners do not affect a client's voiding.

trimethoprim-sulfamethoxazole (Bactrim)

Antimicrobial drug combination: causes synergistic sequential blockade of folic acid synthesis. Active against many gram−negative bacteria including Aeromonas, Enterobacter, H influenzae, Klebsiella, Moraxella, Salmonella, Serratia, Shigella. Possible back−up agent for methicillin−resistant staphylococci.

Chap 52. The results of an arterial blood gas are as follows: pH: 7.5, PaCO2: 40, PaO2: 88, HCO3: 28. Evaluate the acid-base imbalance.

Arterial blood gases are performed to evaluate the client's acid-base balance and oxygenation. pH is the measure of relative acidity or alkalinity. PaCO2 is the partial pressure of carbon dioxide in arterial plasma. PaO2 is the pressure exerted by oxygen dissolved in the plasma, HCO3 is the measure of the metabolic component of acid-base balance. Base excess is a calculated value of bicarbonate levels.

Chap 43. While the nurse is discussing a client's likely death with family members, one of the client's children says, "We plan on taking turns being here for now, but we all want to be here at the time of death. Is there any way we can tell when that time is close?" The nurse's best response is:

As death nears, muscles relax with decreased activity. Muscle rigidity is not a usual pattern. The gag reflex is lost, and mucus accumulates in the back of the throat. Vision is blurred. A lucid moment is not a pattern in death. It is difficult to pinpoint the exact time when death will occur, but the imminence of clinical death can be detected.

Chap 34. A nurse is reading the lab report with the results of the blood specimens that were drawn this a.m. Which of the following is accurate?

As the nurse is reading diagnostic tests, he/she must be aware of the normal values of diagnostic tests and the implications of the test results.

An 18-year-old woman is in the emergency department with fever and cough. The physician asks the nurse to measure vital signs, auscultate lung sounds, listen to heart sounds, determine the level of comfort, and collect blood and sputum samples for analysis. The nurse is performing what aspect of practice?

Assessment

Addison Disease Primary Adrenocortical Deficiency

Autoimmune process commonly found in conjunction with other endocrine diseases of an autoimmune nature; a primary disorder Sudden withdrawal from corticosteroids may precipitate symptoms of Addison disease Addison disease is characterized by lack of cortisol, aldosterone, and androgens; Hyposecretion of adrenal cortex hormones form adrenal gland. Fatigue, weakness, weight loss, anorexia, nausea, vomiting, postural hypotension, hyoglycemia, hyponatremia, hyperkalemia

The nurse incorporates what priority nursing intervention into a plan of care to promote sleep for a hospitalized client?

Avoid awakening client for nonessential tasks Hospitals and extended care facilities usually do not adapt care to an individual's sleep-wake cycle preferences. The nurse should attempt to avoid awakening sleeping clients for nonessential tasks to try and preserve their sleep cycles.

The nurse is developing a teaching a plan for a client diagnosed with (MRSA) infection. The nurse plans to include which instruction in the client's teaching plan? A. Take daily tub baths using a mild soap. B. The infected area should be covered with a clean, dry bandage. C. Wash the infected areas first, then wash the uninfected areas. D. Use bath sponges or puffs when bathing.

B> The infected area should be covered with a clean, dry bandage to prevent the spread of infection.

The nurse notices yellowing at the corners of the sclera in the African-American client admitted for hepatitis. What does the nurse do next? A. Palpates the liver B. Checks the oral mucosa C. Examines the client's hair D. Monitors pulse oximetry

B> To assess dark-skinned clients for jaundice, check for a yellow tinge to the oral mucous membranes, especially the hard palate, and examine the sclera nearest to the iris rather than the corners of the eye.

What's the most imp v/s for Addison's disease?

BP- watch for hypotension

What is a pink puffer?

Barrel chest, which is indicative of emphysema and is caused by use of accessory muscles to breathe. The person works harder to breathe. The person works harder to breathe, but the amount of O2 taken in is adequate to oxygenate the tissues

Chap 52. The client's arterial blood gas results are: pH 7.32; PaCO2 58; HCO3 32. The nurse knows that the client is experiencing which acid-base imbalance?

Because of CO2 retention the PaCO2 is elevated. C02 is involved in production of acid which will result in a decreased pH. HCO3 will vary. Metabolic acidosis involves a loss of bicarbonate but no retention of CO2. Metabolic alkalosis involves a loss of acid or retention of HCO3, but no retention of CO2. Respiratory alkalosis involves a loss of CO2 resulting in an increased pH.

Which of the following medications are the safest to administer to adults needing assistance in falling asleep?

Benzodiazepines The safest group of drugs is the benzodiazepenes. They facilitate the action of the neurons in the central nervous system (CNS) that suppress responsiveness to stimulation, therefore decreasing levels of arousal.

Answer: B Hypertension is often asymptomatic until pressure is very high. Headache (usually occipital), facial flushing, nosebleed, and fatigue are common symptoms of hypertension. Restlessness and dusky or cyanotic skin that is cool to the touch, dizziness, mental confusion, and mottled extremities are all signs and symptoms of hypotension. Unexplained pain and hyperactivity are very vague complaints.

Besides high blood pressure values, what other signs and symptoms may the nurse observe if hypertension is present? A) Unexplained pain and hyperactivity B) Headache, flushing of the face, and nosebleed C) Dizziness, mental confusion, and mottled extremities D) Restlessness and dusky or cyanotic skin that is cool to the touch

Chap 49. Which of the following is most likely to validate that a client is experiencing intestinal bleeding?

Bleeding into the upper GI tract is black and tarry. Large quantities of fat mixed with pale yellow liquid stool can be a sign of malabsorption in an infant. Brown, formed stools are normal stools. Narrow, pencil-shaped stools are characteristic of an obstructive condition of the rectum.

Which of the following indicates that additional assistance is needed to transfer the patient from the bed to the stretcher? A) The patient is 5 feet 6 inches and weighs 120 lbs. B) The patient speaks and understands English. C) The patient received an injection of morphine 30 minutes ago for pain. D) You feel comfortable handling a patient of his size and with his level of cooperation.

C

12.Which hospitalized patient is most at risk for developing a healthcare-associated infection? a. Mr. Y, a 60-year-old patient who smokes two packs of cigarettes daily b. Mrs. J, a 40-year-old patient who has a white blood cell count of 6,000/mm c. Mr. L, a 65-year-old patient who has an indwelling urinary catheter in place d. Mrs. M, a 60-year-old patient who is a vegetarian and slightly underweight

C.

The older adult client who is bedridden has a documented history of protein deficiency. What will the nurse plan to monitor for? A. Anemia B. Decreased wound healing C. Pressure ulcer development D. Weight gain

C> This client is at risk for pressure ulcer if he or she remains bedridden.

-coxib

COX2 enzyme blocker

What's a definitive dx for AAA?

CT scan

The nurse should understand law primarily because the nurse:

Can be an advocate for clients

According to Watson's transpersonal caring theory, the nurse should understand which of the following?

Caring can increase healing and promote well-being.

An APN is pursuing a job change. Which of the following positions would the APN be unable to fill without meeting additional criteria?

Certified registered nurse anesthetist

Angina Pathophysiology Precipitating Factors Assessment Nursing Intervention

Chest discomfort or pain that occurs when myocardial O2 demands exceed supply Common Causes: atherosclerotic heart disease, hypertension, coronary artery spasm, hypertrophic cardiomyopathy, smoking, obesity, stress, diabetes Pain, dyspnea, tachycardia, palpitations, nausea, vomiting, fatigue, diaphoresis, pallor, weakness, syncope, dysrhythmias, During an attack: provide immediate rest, take vitals, record ECG, administer no more than 3 ntirogylcerin tablets, 5 minutes apart, Seek emergency attention if no relief has occurred after taking nitrogylcerin, Percutaneous transluminal coronary angioplasty (PTCA)-- a balloon catheter is repeatedly inflated to split or fracture plaque, and the arterial wall is stretched, enlarging the diameter of the vessel. A rotoblade is used to pulverize the plaque; Arthrectomy-- a catheter with a collection chamber is used to remove plaque; Coronary artery bypass (CABG); coronary laser therapy; coronary artery stent

Bilirubin, indirect (unconjugated)

Circulates primarily in the bloodstrem 0.1 - 1 mg/dL

Chap 48. The nurse is teaching a young female client how to reduce the number of urinary tract infections. Which of the following client behaviors needs to be altered?

Clients should avoid bubble baths, since this can be irritating to the urethra and encourage inflammation and bacterial infection. If the client drinks 8, eight oz glasses of fluid per day, wipes her perineal area from front to back following urination, and wears cotton underwear she is following the proper guidelines to prevent a recurrence of a UTI.

Chap 43. The nurse is providing care to an unconscious client who is dying. Which of the following is not a clinical manifestation of impending clinical death?

Clinical manifestations of impending clinical death include: Slower and weaker pulse. Difficulty swallowing and gradual loss of the gag reflex. Mottling and cyanosis of the extremities. Rapid, shallow, irregular, or abnormally slow respirations.

Chap 38. When a client is unconscious, the health care staff is responsible for providing:

Coma stimulation consists of providing sensory stimulation to promote brain recovery by waking the reticular activating system (RAS).

TORCHs syndrome in the neonate

Combinations of diseases: Taxoplasmosis Rubella (German measles) Cytomegalovirus Herpes Syphyllis

Chap 42. The nurse has recently changed jobs to work with young adults and recognizes that sources of stress common to that population include which of the following?

Common stressors among young adults include marriage, starting a new job, and leaving the parental home. Stressors from aging parents are more common among middle-aged adults; decreased physical abilities is a stressor in elders; and changing body structure serves as a stressor in both children and older adults.

Tuberculosis Pathophysiology Precipitating Factors Assessment Nursing Intervention

Communicable lung disease cause by infection from Mycobacterium tuberculosis bacteria; transmission is airborne, after initial exposure, the bacteria encapsulates into Ghon lesions, bacteria remain dormant until a later time, where clinical symptoms occurs Fever with night sweats, anorexia, malaise, fatigue, cough, hemopytisis, dyspnea, pleuritic chest pain with inspiration, cavitation or calcification as evidenced on chest radiograph, positive sputum culture A positive TB skin test is exhibited by an induration 100mm or greater in diameter 48 hours after the skin test. Anyone who has received a BCG vaccine will have a positive skin test. Provide teaching, collect sputum culture, place client in respiratory isolation, administer anti TB meds

Chap 34. The client is supposed to have a fecal occult blood test done on a stool sample. The nurse is going to use the Hemoccult test. Which of the following indicates that the nurse is using the correct procedure?

Correct procedure includes collecting a sample from two different areas of the stool specimen; assessing for a blue color change; asking if the client has taken vitamin C in the past few days. The reagent is placed on the stool smear after it is applied to the card. A pink color would be considered negative and does not require verification.

Chap 30. A nurse is evaluating a nursing student's understanding of cranial nerves. Which of the following statements demonstrates a need for further teaching? The assessment method for:

Cranial Nerve Functions and Assessment Methods Assessment of Cranial nerve II would be to ask the client to read a Snellen-type chart. Cranial nerve I would be to ask the client to close the eyes and identify different mild aromas, such as coffee, vanilla, peanut butter, orange/lemon, or chocolate. Cranial nerve VI would be to assess the client's directions of gaze. Cranial nerve VII would be to ask the client to smile, raise the eyebrows, frown, puff out the cheeks, and close the eyes tightly.

When obtaining a wound culture to determine the presence of a wound infection, from where should the specimen be taken? A) Necrotic tissue B) Wound drainage C) Drainage on the dressing D) Wound after it has first been cleaned with normal saline

D

The client has had a melanoma lesion removed. For secondary prevention, what is important for the nurse to teach the client? A. Ensure that all lesions are reviewed by a dermatologist or a surgeon. B. Avoid sun exposure. C. Perform a total skin self-examination monthly. D. Perform a total skin self-examination monthly with a partner.

D> Performing a monthly total skin self-examination with another person is the best secondary preventive measure.

Answer: D The nurse may delegate vital signs measurement to unlicensed assistive personnel when the client is in stable condition, the results are predictable, and the technique is standard. The preoperative client is the only client listed who meets these guidelines.

Delegation of some tasks may become one of the decisions the nurse will make while on duty. For which of the following clients would it be most appropriate for unlicensed assistive personnel to measure the client's vital signs? A) A client who recently started taking an antiarrhythmic medication B) A client with a history of transfusion reactions who is receiving a blood transfusion C) A client who has frequently been admitted to the unit with asthma attacks D) A client who is being admitted for elective surgery who has a history of stable hypertension

Chap 38. A nurse planning a seminar on delirium and dementia plans to explain the characteristics differentiating the two. Which of the following describes an alertness that fluctuates—that is, the client may be alert and oriented during the day but become confused and disoriented at night?

Delirium alertness fluctuates. The client may be alert and oriented during the day but become confused and disoriented at night. The dementia client's level of alertness is generally normal.

The three common conditions affecting cognition in older adults are:

Delirium, depression, and dementia

Tensilon test

Diagnosis myasthenic gravis, differeniate myasthenic crisis and cholomergic crisis + client shows muscle improvement after tensilon injection Antidote (needed for cholinergic crisis) atropine sulfate

Chap 48. Which nursing assessment in the home care environment for clients with urinary elimination problems is inappropriate?

Dietary guides related to fiber and fluid balance are given to clients with this problem. The remaining actions are noted in the assessment guide and are appropriate measures to use with clients.

Chap 52. Which of the following information about osmosis is NOT correct?

Diffusion is the continual intermingling of molecules in liquids, gases, or solids brought about by the random movement of the molecules. Osmosis is the movement of water across cell membranes, from the less concentrated solution to the more concentrated solution. Osmosis occurs when the concentration of solutes on one side of a selectively permeable membrane, such as the capillary membrane, is higher than on the other side. Osmosis is an important mechanism for maintaining homeostasis and fluid balance. Osmolality is determined by the total solute concentration within a fluid compartment and is measured as parts of solute per kilogram of water.

-zide

Diuretics

Signs of Hyperkalemia

Dizziness, weakness, cardiac irregularities, muscle cramps, diarrhea, nausea.

What instructions should be given to a client following radiation therapy?

Do not wash off lines; wear soft cotton garments; avoid use of powders and creams on radiation site

Chap 52. The average adult should drink about __________ mL of fluid per day if they exercise moderately and it is a moderate temperature.

During periods of moderate activity at moderate temperature, the average adult drinks about 1,500 mL per day but needs 2,500 mL per day, an additional 1,000 mL.

Answer: C The domain contains the subject, central concepts, values and beliefs, phenomena of interest, and the central problems of the discipline. A paradigm is a model that explains the linkage of science, philosophy and theory that is accepted and applied by the discipline.

Each science has a domain, which is the perspective of the discipline. This domain: A) Represents the recipients of the benefits of the science or discipline B) Is a model that explains the linkage of science, philosophy, and theory that is accepted and applied by the discipline C) Describes the subject, central concepts, values and beliefs, phenomena of interest, and central problems of the discipline D) Is a dynamic state of being in which the developmental and behavioral potential of the individual is realized to the fullest

Hypernatremia - assessment findings

Early - muscle twitching Late: diminished -absent deep tendon reflexes pulmonary edema (with hypervolemia) Extreme Thirst Decreased urinary output, increased specific gravity Dry flushed skin, mucous membranes HR and BP respond to vascular volume status

What nursing measure promotes sleep in school-aged children?

Encourage quiet activities prior to bedtime. Encouraging quiet activities before bedtime helps prepare children for sleep. Evening exercise and watching television can make falling asleep more difficult. Children may sleep better with a night light in the room

Benign Prostatic Hyperplasia (BPH) Pathophysiology Precipitating Factors Assessment Nursing Intervention

Enlargement or hypertrophy of the prostate. BPH tends to occur in men over 40 years of age, intervention is required when symptoms of obstruction occur, the most common treatment is transurethral resection of the prostate gland in which the prostate is removed by endoscopy allowing for a shorter hospital stay. Increased frequency of voiding, with a decrease in amount of each voiding, nocturia, hesitancy, terminal dribbling, decrease in size and force of stream, acute urinary retention, bladder distention Preoperative teaching, maintain patent urinary drainage system to decrease spasms, provide pain relief as prescribed, minimize catheter manipulation by taping catheter to abdomen or leg, maintain gentle traction on urinary catheter, check the urinary drainage system for clots, irrigate bladder as prescribed, observe the color and content of urinary output, monitor vital signs frequently for indication of circulatory collapse, monitor hemoglobin and hematocrit for pattern of decreasing values that indicates, instruct client to increase fluid intake to 3000mL/day.

Chap 52. Which of the following statements about the functions of the body's fluids is most accurate?

Extracellular fluid carries nutrients to and waste products from the cells. Plasma carries oxygen from the lungs to the capillaries of the vascular system. Intracellular fluid contains solutes and provides a medium for the metabolic processes to take place; it is not a transport system. The extracellular fluid is the transport system that carries nutrients to and waste products from the cells.

All of the following are examples of active strategies of health promotion except:

Fluoridation of drinking water

Chap 34. Which technique is NOT correct when collecting a urine specimen for culture and sensitivity by clean catch?

For a male client, using a circular motion to clean the urinary meatus is the correct technique. For a female client, the perineal area should be cleaned from front to back. Always explain to the client that a urine specimen is required, give the reason, and explain the method to be used to collect it. A nurse should always perform hand hygiene and observe other appropriate infection control procedures. The nurse must ensure that the specimen label is attached to the specimen cup, not the lid, and that the laboratory requisition provides the correct information.

What does amphojel tx?

GERD kidney stones watch out for consiptaion

Ethical dilemmas often involve a conflict of opinion. Once the nurse has determined that the dilemma is ethical, which of the following would be a critical first step in negotiating the difference of opinion?

Gather all relevant information regarding the clinical, social, and spiritual aspects of the dilemma.

When the nurse describes a client as "that nasty old man in 354," the nurse is exhibiting which of the following?

Gender bias and ageism

When assessing a patient admitted with nausea and vomiting, which of the following findings supports the nursing diagnosis of deficient fluid volume? Polyuria Decreased pulse Difficulty breathing General restlessness

General restlessness Restlessness is an early cerebral sign that dehydration has progressed to the point where an intracellular fluid shift is occurring. If the dehydration is left untreated, cerebral signs could progress to confusion and later coma.

Chap 48. Which of the following assessments should be ongoing for clients with urinary retention catheters?

Gravity drainage should be checked and maintained, so the receptacle should be below the client's bladder. Observation of the flow, color, odor, and abnormal constituents needs to be done every 2 to 3 hours. Drainage system should be well-sealed or closed and should have no leaks. Tubing should be fastened to the client's bedclothes, not the bed sheet.

What is an early sign of dopamine excess?

HA

What is the first intervention for autonomic dysreflexxia?

HOB elevated

Hyperkalemia - assessment findings

HR - slow, weak, irregular BP - decreased Resp - weakness of sketal muscles leading to resp failure Early Neuro - muscle twitches, cramps, tingling & burning followed by numbness (hands, feet and around mouth) Late Neuro - profound weakness, ascending flaccid paralysis arms and legs GI - hyperactive bowel sounds, diarrhea

Hypercalcemia - assessment findings

HR -bounding peripheral pulses increased (early) bradycardia lead to cardiac arrest (late) BP - Increased Resp - ineffective resp movement Neuro - profound muscle weakness, diminished to absent deep tendon reflexes, disoriented, lethargy, coma Renal - renal calculi, flanlk pain, urinary output varies GI - hypoactive bowel sounds, anorexia, nausea, abd. distention, constipation

Chap 52. Following surgery, the client requires a blood transfusion. The main reason the nurse wants to complete the unit transfusion within a four-hour period is that blood:

Hanging for longer than four hours creates an increased risk of sepsis, which is why the nurse wants to complete the unit transfusion in less time. The remaining items are not likely to happen.

Chap 34. The nurse would call the primary care provider immediately for which of the following lab results?

Hct = 22% for female client is very low and can lead to death. A Hgb = 16 g/dL for male client, WBC = 9 × 103/mL3, and Platelets = 300 × 103/mL3 are within normal range.

Hypoglycemia Symptoms

Headache, nausea, sweating, tremors, lethargy, hunger, confusion, slurred speech, tingling mouth, anxiety Usually occurs rapidly and is potentially life threatening, treat immediately with CHO i.e graham cracker and peanut butter twice Check blood glucose, may seize if <40.

Answer: D Healthy People 2010 was established to create ongoing health care goals, including increasing life expectancy and quality of life, and eliminate health disparities through improved delivery of health care services. Gathering information, assessing needs, and developing and implementing public health policies are steps in achieving the goals set forth by Healthy People 2010.

Healthy People 2010's overall goals are to: A) Assess the health care needs of individuals, families, or communities B) Develop and implement public health policies and improve access to care C) Gather information on incident rates of certain diseases and social problems D) Increase life expectancy and quality of life and eliminate health disparities

Chap 38. Nurses have a role in promoting healthy sensory function. Which of the following guidelines is applicable to provide appropriate sensory input to children?

Healthy sensory function can be promoted by providing appropriate sensory input to children by stimulating as many senses as possible and using a variety of stimuli. Consistency is not necessary in the provision of stimuli. Regular visual screening is necessary but does not ensure appropriateness of sensory input. Blocking stimuli is necessary to prevent sensory overload, and overload is not present in this situation.

Chap 43. Which of the following may be considered normal or "healthy" types of grief?

Healthy/normal types of grief include abbreviated grief (normal grief that is briefly experienced), anticipatory grief (experienced before the loss/death but appropriate), and disenfranchised grief (the emotions are felt privately, but not expressed in public). Unhealthy/abnormal types of grief include complicated grief in several different forms: unresolved grief, which is extended in length and severity; and inhibited grief, in which symptoms are suppressed and other effects, including somatic, are experienced instead.

Apgar scoring

Heart rate, respiratory rate, muscle tone, reflex irritability, and skin color A = appearance P = Pulses G = Grimace A - Activity R = Reflexes score 0 =absent 1 = decreased 2 = strongly positive done at 1 & 5 mins

Chap 30. Which is the best place to assess for hydration in the older adult?

Hydration status in elders is best assessed over the sternum or clavicle due to normal loss of peripheral skin turgor.

Hyponatremia - assessment findings

Hyperactive bowel sounds Nausea, abdominal cramping & diarrhea Increased urinary output, decreased specific gravity Dry mucous membranes H/A, confusion, seizures muscle weakness, diminished deep tendon reflexes shallow ineffective respiratory movement (LATE) cardio vary with changes in cascular volume

The nurse is caring for a patient receiving an initial dose of chemotherapy to treat a rapidly growing metastatic colon cancer. The nurse is aware that this patient is at risk for tumor lysis syndrome (TLS) and will monitor the patient closely for which of the following abnormalities associated with this oncologic emergency? Hypokalemia Hypocalcemia Hypouricemia Hypophosphatemia

Hypocalcemia TLS is a metabolic complication characterized by rapid release of intracellular components in response to chemotherapy. This can rapidly lead to acute renal failure. The hallmark signs of TLS are hyperuricemia, hyperphosphatemia, hyperkalemia, and hypocalcemia.

Hypothyroidism Hashimoto Dz Myxedema

Hypofunction of the thyroid gland, with insufficiency of thyroid hormone Fatigue, thin, dry hair, constipation, bradycardia, hypotension, goiter, periorbital edema, cold intolerance, weight gain, dull emotions and mental processes

Answer: B In Healthy People 2010, the assurance role of public health is defined as making essential community-wide health services available and accessible. In Healthy People 2010, public development and implementation refer to the role of health professionals in providing leadership in development of policies that support the population's health. Population-based public health programs focus on disease prevention, health promotion, and health protection. A healthy environment for each individual, family, and community is the overall goal of Healthy People 2010.

In Healthy People 2010, assurance refers to the role of public health in: A) Providing disease prevention, health protection, and health promotion B) Making essential community-wide health services available and accessible C) Providing leadership in developing policies that support the population's health D) Achieving a healthy environment for each individual, family, and community

Answer: C Wheezes are adventitious breath sounds that are high-pitched, continuous musical sounds, such as a squeak heard continuously during inspiration or expiration. Pleural friction rub has a dry, grating quality and is heard best during inspiration. Crackles can be fine, high-pitched, short, interrupted crackling sounds; moist, low sounds in the middle of inspiration; or coarse, loud, bubbly sounds. Rhonchi are loud, low-pitched, rumbling, coarse sounds heard during inspiration.

In assessing the client's lungs the nurse hears adventitious breath sounds that are high-pitched, continuous musical sounds, such as a squeak heard continuously during inspiration or expiration, usually louder on expiration. These adventitious breath sounds are known as: A) Crackles B) Rhonchi C) Wheezes D) Pleural friction rub

Chap 42. Two people have been in a motor vehicle crash and have similar injuries. According to the transaction-based model, their degree of stress from the crash would be:

In the transaction model, stress is a very personal experience and varies widely among individuals. "Extremely similar since they had the same stimulus" represents the stimulus model, and "The identical physiologic alarm reaction" represents the response model of stress. In "Different depending on their external resources and support levels," external resources and support are factors in determining stress levels, but omit the key aspects of internal/personal influences.

Chap 42. Which physiological response describes the ability of a person to perform a far more strenuous physical activity than normal when faced with a stressor?

Increased blood clotting is the physiological response to epinephrine. The sympathetic system, not the parasympathetic, is stimulated in response to stressors. Bronchial dilation allows for increased oxygen intake. Adrenal hormonal effects allow the person faced with a stressor to do more strenuous activities than normal.

The nurse teaches a client taking phenytoin (Dilantin), an anticonvulsant, that this group of medications causes which symptom of a sleep problem?

Increased daytime sleepiness The anticonvulsants can cause increased daytime sleepiness because they decrease REM sleep time. They do not cause nocturia, increased awakenings, or increased difficulty falling asleep.

The older adult is at risk for falls for various reasons. To help prevent falls the nurse may:

Instruct the older adult to use a night light in the bathroom

Describe preoperative nursing care for a client undergoing a laryngectomy

Involve family and client in manipulation of tracheostomy equipment before surgery; plan acceptable communication methods; refer to speed pathologist; discuss rehabilitation program

Chap 52. An elderly client was hydrated with lactated Ringer's solution in the emergency department for the last hour. During the most recent evaluation of the client the nurse noted a rapid bounding pulse and shortness of breath. Reporting this episode to the primary care provider, the nurse suspects that the client now shows signs of:

Isotonic solutions have the same osmolality as body fluids. Isotonic solutions, such as normal saline and Ringer's lactate initially remain in the vascular compartment, expanding vascular volume. Isotonic imbalances occur when water and electrolytes are lost or gained in equal proportions, and serum osmolality remains constant.

The distribution of nurses to areas of greatest need when there is a nursing shortage represents the application of which principle or theory?

Justice

Chap 34. The client has a urinary health problem. Which of the following procedures is performed using indirect visualization?

KUB is an x-ray of the kidneys, ureters, and bladder. An IVP and retrograde pyelography use injections of contrast media. A cystoscopy uses a lighted instrument (cystoscope) inserted through the urethra resulting in direct visualization.

What's a strawberry tongue mean?

Kawasaki syndrome-leading cause of acquired HD in the USA

Chap 43. The client has been diagnosed with an illness and given a prognosis of less than a month to live, yet the client continues to make plans for speaking engagements for 6 months in the future. This is an example of which of Elizabeth Kübler-Ross's stages of grieving?

Kübler-Ross defined 5 stages of grieving, including: denial, bargaining, depression, acceptance, and anger. This client demonstrates the stage of denial and demonstrates the behavioral response of refusing to believe that loss is happening.

Stroke Right Hemisphere

Language: may be alert and oriented Memory: disoriented, cannot recognize faces Vision: visual/spatial deficits, neglect of left visual fields, loss of depth perception Behavior: impulsive, unaware of neurologic deficits, euphoric, constantly smiles, denies illness, poor judgment, overestimates abilities, impaired sense of humor Hearing: loss of ability to hear tonal variations

Older adults are cautioned about the use of nonprescription sleeping medications because these medications can:

Lead to further sleep disruption even when they initially seemed to be effective. Over-the-counter medications for sleep often cause more problems than benefits. The other answers are incorrect.

Answer: A The goal of Leininger's theory is to provide the client with culturally specific nursing care, in which the nurse integrates the client's cultural traditions, values, and beliefs into the plan of care.

Leininger's theory of cultural care diversity and universality specifically addresses: A) Caring for clients from unique cultures B) Understanding the humanistic aspects of life C) Identifying variables affecting a client's response to a stressor D) Caring for clients who cannot adapt to internal and external environmental demands

Chap 49. During discharge planning, the nurse is teaching the client how to manage diarrhea. Which of the following actions is NOT correct?

Limit foods containing insoluble fiber, such as high-fiber whole wheat breads, whole grain breads and cereals, and raw fruits and vegetables. Drink at least eight glasses of water per day to prevent dehydration. Eat foods with sodium and potassium. Limit fatty foods. Increase foods containing soluble fiber, such as rice, oatmeal, and skinless fruits and potatoes.

Chap 43. While talking to adult children of a dying client, the nurse finds them tearful, with ambivalent feelings toward the client. The client often expresses beliefs of a wasted life. The children say that the client was a parent who often showed love but followed it with criticism, anger, damaging accusations, and emotional abuse. The nurse suggests an intervention that may be helpful to the client and the client's children. The intervention most likely to be helpful is:

Making a videotape of loving memories helps both the client and family to focus on the positive. Relaxation tapes help with stress reduction, but do not help resolve problems experienced by the client and children. Staffing needs do not permit a nurse to be with one client continually, and families require privacy as well. Assurance that the past no longer matters is an assurance lacking concrete properties.

Answer: C The first level of Maslow's hierarchy of needs includes the need for air, food, and water—basic elements of survival. Love and belonging are on the second level, esteem and self-esteem are on the fourth level, and self-actualization is the final level.

Maslow's hierarchy of needs is useful to nurses, who must continually prioritize a client's nursing care needs. The most basic or first-level needs include: A) Self-actualization B) Love and belonging C) Air, water, and food D) Esteem and self-esteem

Chap 38. While performing a history, which of the following sensory perceptions does the nurse assess?

Mental status is assessed while performing a history. Kinesthetic perception, deep tendon reflexes, and cranial nerves are assessed during the physical exam, not the history.

You are admitting a patient with complaints of abdominal pain, nausea, and vomiting. A bowel obstruction is suspected. You assess this patient for which of the following anticipated primary acid-base imbalances if the obstruction is high in the intestine? Metabolic acidosis Metabolic alkalosis Respiratory acidosis Respiratory alkalosis

Metabolic alkalosis Because gastric secretions are rich in hydrochloric acid, the patient who is vomiting will lose a significant amount of gastric acid and be at an increased risk for metabolic alkalosis.

Answer: D Middle-range theories are limited in scope, less abstract than grand theories, address specific phenomena or concepts, and reflect practice. Grand theories are described as broad and complex. Prescriptive theories address nursing interventions and predict the consequence of a specific nursing intervention. Descriptive theories describe phenomena, speculate as to why the phenomena occur, and describe the consequences of phenomena.

Mishel's theory of uncertainty in illness focuses on the experience of clients with cancer who live with continual uncertainty. The theory provides a basis for nurses to assist clients in appraising and adapting to the uncertainty and illness response and can be described as: A) A grand theory B) A descriptive theory C) A prescriptive theory D) A middle-range theory

Which of the following nursing interventions is most appropriate when caring for a patient with dehydration? Auscultate lung sounds q2hr. Monitor daily weight and intake and output. Monitor diastolic blood pressure for increases. Encourage the patient to reduce sodium intake.

Monitor daily weight and intake and output Measuring weight is the most reliable means of detecting changes in fluid balance. Weight loss would indicate the dehydration is worsening, whereas weight gain would indicate restoration of fluid volume. Recall that a 1-kg weight gain indicates a gain of approximately 1000 ml of body water.

Which of the following professional organizations was created to address concerns of members in the nursing profession?

NLN

What drug groups to give WITH food?

NSAIDS Corticosteroids Bipolar drugs Cephlasporins Sulfonamides

Asthma Pathophysiology Precipitating Factors Assessment Nursing Intervention

Narrowing or closure of the airway due to a variety of stimulants Mucosal edema, V/Q abnormalities, increased work of breathing, beta blockers, respiratory infection, allergic reaction, emotional stress, exercise, environmental or occupational exposure, reflux esophagitis Dyspnea, wheezing, chest tightness, assess precipitating factors, medication history Administer bronchodilators, administer fluids and humidification, education, ABGs, ventilatory patterns

Potassium - precaution with IV administration

Never given - IV push, IM or subq routes Dilution - no more than 1 mEq/10 ml Invert - to be distribution evenly Properly labeled Max infusiuon - 5 - 10 mEq/hr, NEVER exceed 20 mEq/hr > 10 mEq/hr on cardiac monitoring & infusion device Assess: frequently - Phlebitis or infilration- stop immediately Renal function B4 and I & O during

It may seem redundant that health care providers pledge to "do no harm" to clients. The purpose of this oath is to reassure the public that the health care team will work to heal clients with as little pain and harm as possible. Which principle underlies this pledge?

Nonmaleficence

EKG U Wave

Not always present Most prominent in presence of hypokalemia

In most ethical dilemmas, the solution to the dilemma requires negotiation among members of the health care team. Why is the nurse's point of view valuable?

Nurses develop a relationship with the client that is unique among all professional health care providers.

Chap 42. A nurse feels vulnerable after a child dies following a lengthy resuscitation effort. A positive coping strategy for the nurse is to:

Nurses must learn positive coping strategies to deal with stress and prevent burnout. The nurse needs to tune into feelings rather than suppress and numb them with sedatives. A child's death is always extraordinary. It is important for the nurse to deal with the grief. The nurse should not make the assumption that a mistake caused the child's death, rather than recognizing that people do the best they can in desperate circumstances and that even children cannot always be resuscitated. However, reviewing resuscitations can be useful if done to improve overall care.

Chap 38. It is important for the nurse to be sensitive to stimulation that is culturally acceptable to a client. The lack of culturally assistive or supportive acts is called:

Nurses should encourage clients who want to have culturally related symbols present and to follow practices with which they are comfortable, provided that these practices do not endanger health.

Chap 30. Which of the following actions is correct for the nurse assessing a client who has just had a cast applied to the lower leg?

Nursing Assessments Addressing Selected Client Situations: 1. Client complains of abdominal pain: Inspect, auscultate, and palpate the abdomen; assess vital signs. 2. Client is admitted with a head injury: Assess level of consciousness using Glasgow Coma Scale (see Table 30-10 in the textbook); assess pupils for reaction to light and accommodation; assess vital signs. 3. The nurse prepares to administer a cardiotonic drug to a client: Assess apical pulse and compare with baseline data. 4. The client has just had a cast applied to the lower leg: Assess peripheral perfusion of toes, capillary blanch test, pedal pulse if able, and vital signs. 5. The client's fluid intake is minimal: Assess tissue turgor, fluid intake and output, and vital signs.

How to remember CN?

Oh, Oh, Oh, To, Touch, And, Feel, A, Girls, Vagina, And, Hymen

Chap 49. A nurse is evaluating a nursing student's understanding of the actions of enema solutions. Which of the following statements demonstrates a need for further teaching?

Oil solutions lubricate the feces and the colonic mucosa. Isotonic solutions distend the colon, stimulate peristalsis, and soften feces. Hypertonic solutions draw water into the colon. Hypotonic solutions distend the colon, stimulate peristalsis, and soften feces. Soapsuds solutions irritate the mucosa and distend the colon.

Chap 48. Oliguria is:

Oliguria is low urine output, usually less than 500 mL a day or 30 mL an hour for an adult. Polyuria (or diuresis) refers to the production of abnormally large amounts of urine by the kidneys, often several liters more than the client's usual daily output. Polydipsia is excessive fluid intake. Anuria refers to a lack of urine production.

CK-MB MI;Cardiac Enzyme

Onset: 2-4 hour Peak: 12-20 hour Return to Normal: 48-72 hour

Cardiac Troponins MI; Cardiac Enzymes

Onset: as early as 1 hour post injury Peak: 48 hours Return to Normal: 5-14 days

Chap 34. During a bone marrow biopsy, the nurse needs to assess the client for:

Pallor, diaphoresis, and faintness due to bleeding or pain are to be observed during a bone marrow biopsy. During a thoracentesis, the nurse needs to observe for dyspnea, pallor, and coughing as signs of distress. Pallor, dyspnea, drop in blood pressure, abnormal pulse rate, skin color, and restlessness due to shock would be observations for a client during the abdominal paracentesis.

Chap 43. After a nurse questions a client about relationship abuse, the client claims to be ready to leave the abusive relationship, although past attempts were not successful due to fear, lack of support, lack of confidence, and financial considerations. The client asks the nurse for help. Which of the following examples of perceived loss may the client be experiencing?

Perceived loss is experienced by one person but cannot be directly verified by others. Loss of partner, residence, and lifestyle can be seen and acknowledged by others, even if they are not favorable. Dreams are something of which only the client is aware. She may have dreamed of a happier relationship that she finally acknowledged was not forthcoming, or the dream may be of a different origin. Only the client knows.

After researching the scientific literature and confirming that an issue is an ethical dilemma, which of the following should a nurse do next?

Perform a values clarification.

Chap 30. Which of the following assessments would the nurse expect to find during a well-baby visit for a 12-month-old infant?

Physical assessment of the infant requires a strong knowledge of child growth and development. It is expected that a 12-month-old infant will have a partially open anterior fontanel. It should be fully closed by 18 months of age. Infants can visually follow objects by the age of 4 weeks. The 12-month-old infant is expected to crawl and to show signs of interest/ability to stand and walk. The stepping reflex disappears by 6 months. Lanugo is a fine, downy hair that is present on premature infants. The femoral artery is a large artery and a strong bilaterally equal pulse is present. Weak femoral pulses may be a sign of a cardiac anomaly.

Chap 49. Which of the following is the correct order for the physical examination of the abdomen?

Physical examination of the abdomen in relation to fecal elimination problems includes inspection, auscultation, percussion, and palpation with specific reference to the intestinal tract. Auscultation precedes palpation because palpation can alter peristalsis. Examination of the rectum and anus includes inspection and palpation.

Chap 49. The mother of a 6-year-old reported that the child goes to the bathroom frequently. After measuring the urine for 24 hours, she reported that it totaled 2000 mL. This is an example of:

Polyuria is the production of an abnormally large amount of urine. A 6-year-old usually has a normal urinary output of 700-100 mL daily. Anuria refers to a lack of urine production; nocturia is voiding two to three times during the night; oliguria is the production of low urine output, usually less than 500 mL a day for adults.

Chap 34. A nurse cares for a client following a liver biopsy. Which nursing care plan reflects proper care?

Positioning the client in a right side-lying position with a pillow under the biopsy site reflects proper care. Positioning in a dorsal recumbent position, with one pillow under the head, does not permit the necessary pressure applied to the biopsy site. Bed rest is only required for several hours. There is no reason to do neurological checks.

A common age-related change in auditory acuity is called:

Presbycusis

A parish nurse for a Catholic church provides a free blood pressure screening the first Sunday of every month. This is what level of prevention?

Primary

The guidelines of the Health Insurance Portability and Accountability Act (HIPAA) support which of the following?

Privacy for all health care issues

Chap 42. When a person is approached with a stressful situation, and responds by trying to improve the situation by making changes or taking action, it is described as:

Problem-focused coping focuses on efforts to improve a situation by making changes or taking some action.

Chap 42. A client who was raised in an abusive family resented the mother for not being protective. Since the death of the father, the client has been taking care of the mother and devoting time to counseling abused women. What defense mechanism is this client using?

Reaction formation causes persons to act exactly the opposite from the way they feel. Sublimation is the displacement of energy associated with primitive drives into more acceptable outlets. Undoing is acting in a way to relieve guilt or unacceptable thoughts by reparation. Identification is the attempt to manage anxiety by imitating the behaviors of someone feared or respected.

EKG QT Interval

Represents the time required to completely depolarize and repolarize the ventricles

After kidney surgery, what are the primary assessments the nurse should make?

Respiratory status (breathing is guarded because of pain); circulatory status (the kidney is very vascular and excessive bleeding can occur); pain assessment; urinary assessment (most important, assessment of urinary output)

Chap 43. A nurse's client just passed away. The nurse understands that rigor mortis is the stiffening of the body that occurs about _____ hours after death.

Rigor mortis is the stiffening of the body that occurs about 2 to 4 hours after death. It results from a lack of adenosine triphosphate (ATP), which causes the muscles to contract, which in turn immobilizes the joints. Rigor mortis starts in the involuntary muscles (heart, bladder, etc.), then progresses to the head, neck, and trunk, and finally reaches the extremities. All other times are incorrect.

The caring aspect of nursing may be negatively affected in clinical practice today primarily because of:

Rise in technology that takes nurses' attention away from clients

Femur fracture

Risk for fat emboli

Chap 52. The nurse has been invited to discuss "the importance of promoting a good fluid and electrolyte balance in children" for a group of parents at the local school parents meeting. Of the following actions, which is not representative of this topic?

Salt causes the body to retain fluids due to an increase in the concentration of sodium and the release of ADH. Caffeine acts as a diuretic in individuals and may lead to loss of excess fluids in the body. The remaining identified measures are all appropriate.

Taste buds atrophy and lose sensitivity, and appetite may decrease. The older adult is less able to discern:

Salty, sour, and bitter tastes

Atrial Flutter

Saw toothed waveform, fluttering in chest, ventricular rhythm states regular TX: Cardioversion to tx atrial dysrhythmias, antidysrhythmic drugs, radiofrequency catheter ablation

A nurse working in a special care unit for children with severe immunologic problems cares for a 3-year-old boy from Greece. The nurse is having difficulty communicating with the father. What is the appropriate action?

Search for help in interpreting and understanding the culture differences by contacting someone from the local Greek community.

Chap 38. A 16-year-old client has just had emergency surgery for a fractured femur after being involved in a motor vehicle crash. The client was unconscious upon admission and the parents signed for client to have the surgery. Client is in own room after surgery, feeling drowsy, and unaware of surroundings. Client does know parents, but does not understand all the noise of the oxygen machine, the IV, and the monitors. This is a sign of:

Sedation is a state of calm, restfulness. This client has sensory overload which generally occurs when a person is unable to process or manage the amount or intensity of sensory stimuli.

Chap 38. A client admitted to the intensive care unit complains of excessive fatigue and racing thoughts and is moderately anxious. In addition, the client is unable to follow instructions. This client has clinical signs of:

Sensory overload is characterized by fatigue or sleeplessness, racing thoughts, anxiety, irritability, and reduced task performance. A sensory deficit is impaired perception, reception, or both of one or more of the senses. Examples are deafness and blindness. In sensory deprivation, clients experience decreased attention span, drowsiness, excessive sleeping, and apathy. Sensory reception is the process of receiving stimuli or data.

Chap 49. Because a client is scheduled for a colonoscopy, the nurse will instruct the client to perform which of the following?

Small-volume enemas along with other preparations are used to prepare the client for a colonoscopy. An oil retention enema is used to soften hard stool. Return flow enemas help expel flatus. Because of the risk of loss of fluid and electrolytes, high, large-volume enemas are seldom used.

Chap 52. The nurse is admitting a new client, 80 years old, with congestive heart failure into your home health agency. The following assessment findings have been determined after meeting the client: overweight but no gain since the client left the hospital two days ago; VS: T 99.0, HR 100, R 22, BP 130/86. Foods eaten include canned soup at each meal, ham, and cheese. When completing the care plan for this client the nurse should include which of the following nursing diagnoses:

Sodium is found in high quantities in the foods noted that the client has consumed. When sodium levels increase in the body, water is retained, adding to the volume of fluid in circulation and making it harder for the body to move circulate fluids. Therefore, the excess fluid may in time impair gas exchange if levels eventually act on the lungs. Fluid volume is increasing, not decreasing, in this situation and Impaired Skin Integrity has no involvement with platelets.

Chap 48. Which of the following factors affect voiding?

Some foods and fluids can change the color of urine. For example, beets can cause urine to appear red. Antinflammatory medication to not interfere with the normal urination process. The client living alone or being physically active do not affect voiding.

Chap 30. Some nurses will prioritize their sequence of performing a health exam according to:

Some nurses will prioritize their sequence of performing a health exam according to the disease process, beginning with the body system related to the client's primary concern.

Tracheal Sunctioning

Suction when adventitious breath sounds are heard, when secretions are present at endotracheal tube, and when gurgling sounds are noted. Use aseptic technique throughout procedure Wear mask and goggles Advance catheter until resistance is felt Apply suction only when withdrawing catheter Never suction for more than 10 to 15 seconds, and pass the catheter only 3 fewer times Oxygenate with 100% O2 for 1 to 2 minutes before and after suctioning to prevent hypoxia

Chap 42. After the death of several long-term clients, which of the following actions indicates the nurse is demonstrating ineffective coping?

Taking on additional work would only serve as an additional stressor. In addition, a nurse who has not begun resolution of feelings is unlikely to be able to meet clients' emotional needs. Effective coping may include verbalizing feelings (one-on-one or in groups) or initiating distractions. Of course, the nurse may not disclose confidential information to a friend or others who would not already have this information.

Chap 34. The nurse is providing patient education about a fecal occult blood test. Which of the following statements is correct?

Taking the sample from the center of a formed stool to ensure a uniform sample is a correct technique for a fecal occult blood test. The nurse should state: "Avoid collecting specimens during your menstrual period and for 3 days afterward, and while you have bleeding hemorrhoids or blood in your urine." Either of these situations would give a false positive result to the fecal occult blood tests. Using a ballpoint pen to label the specimens with your name, address, age, and date of specimen is a correct technique for a fecal occult blood test. Avoiding contamination of the specimen with urine or toilet tissue is a correct technique for a fecal occult blood test.

Chap 48. The normal adolescent should void approximately how much urine in a 24-hour period?

The average adolescent from 14 years through adulthood should have an average of 1500 mL of urine output in a 24-hour period.

Chap 43. Proper handling of the body following death is an important intervention for the client, family, and nurse. Which of the following interventions reflects an important principle of postmortem care?

The body is to be handled with dignity at all times. After the body is cleaned and the linen freshened, the sheet is pulled to cover the patient's shoulders. Laws and policies differ regarding the nurse's ability to declare death.

Chap 42. When discussing their father's behavior during a family counseling session, a brother says to his sister, "Sure, Dad was rough, but not as bad as Grandma. Don't you remember Grandma? She was much worse. If it weren't for her, he would have been OK." The defense mechanism the brother is using is:

The brother is projecting the behavior being discussed to his grandmother. He is not rationalizing or justifying his father's behaviors by faulty logic or ascribing motives. He is not minimizing by not acknowledging his father's behavior. He is not compensating by emphasizing a more desirable trait of his father.

Chap 48. The nurse recognizes that urinary elimination changes may occur even in healthy elders because:

The capacity of the bladder may decrease with age but the muscle is weaker and can cause urine to be retained. Elders do not ignore the urge to void and may have difficulty in getting to the toilet in time. The kidney becomes less able to concentrate urine with age.

Answer: D Adduction is movement toward the body. Abduction is movement away from the body. Flexion is movement that decreases the angle of the joint, whereas extension is movement that increase the angle of the joint.

The client is being assessed for range of joint movement. The nurse asks the client to move the arm toward the body to evaluate: A) Flexion B) Extension C) Abduction D) Adduction

Chap 38. Which statement by a hospitalized client indicates a need for further orientation to time, place, person, or situation?

The client's statement "I'm tired of sitting in this train station" indicates disorientation. Listen to clients carefully to pick up signs of disorientation and not confuse them with other causes. Comparing a hospital to a busy train station is an appropriate analogy for an oriented person in the hospital. A client's statement about not remembering things well is not uncommon. Not remembering a staff member may reflect impaired memory related to organic causes, multiple caregivers, medication, or stress; yet the client asks an appropriate question to clear the memory lapse.

Answer: C Since the "up ad lib" orders are new and the client has been on bed rest, checking orthostatic blood pressure before allowing the client to ambulate is the correct answer. If no sign of orthostatic hypotension is present, then a nursing assistant could assist him to the bathroom. Giving the client a urinal is not a good choice if the client is asymptomatic when orthostatic blood pressure is checked.

The client, who has been on bed rest for 2 days, asks to get out of bed to go to the bathroom. He has new orders for "up ad lib." What action should the nurse take? A) Give him some slippers and tell him where the bathroom is located. B) Ask the nursing assistant to assist him to the bathroom. C) Obtain orthostatic blood pressure measurements. D) Tell him it is not a good idea and provide a urinal.

Chap 42. _____________ is a short-term helping process of assisting clients to work through a crisis to resolution and restore their precrisis level of functioning.

The goal of crisis intervention is to provide immediate relief for the client. Crisis intervention goes beyond kindness. It involves deliberate acts to benefit the client. Emergency management, goal clarification, and trauma interventions are not interventions that are implemented during a crisis.

Answer: D The anterior hypothalamus controls heat loss by initiating the mechanisms of sweating and vasodilation of blood vessels. Blood is redistributed to surface vessels (flushing of the skin) to promote heat loss, not heat retention. The posterior hypothalamus controls heat production by initiating the mechanisms of shivering, vasoconstriction of blood vessels, and reduction of blood flow to the skin and extremities.

The hypothalamus controls body temperature. The anterior hypothalamus controls heat loss, and the posterior hypothalamus controls heat production. What heat conservation mechanisms will the posterior hypothalamus initiate when it senses that the client's body temperature is lower than comfortable? A) Vasodilation and redistribution of blood to surface vessels B) Sweating, vasodilation, and redistribution of blood to surface vessels C) Vasoconstriction, sweating, and reduction of blood flow to extremities D) Vasoconstriction, reduction of blood flow to extremities, and shivering

Chap 42. According to Selye, what is the body's reaction when a client receives disturbing or happy news?

The initial reaction of the body is the alarm reaction, which alerts the body's defenses.

Chap 34. A client is to obtain a clean-catch urine specimen. Which statement by the client demonstrates a lack of understanding regarding the procedure?

The initial stream of urine contains bacteria from the distal urethra and urinary meatus, so it should be discarded as contaminated.

Answer: A An educator helps clients, families, and communities gain greater skills and knowledge to provide their own care. An advocate is someone who helps clients walk through the system, identifies services, and plans for accessing appropriate resources. A collaborator is an individual who engages in a combined effort with other individuals to develop a mutually acceptable plan that will achieve common goals. A case manager develops and implements a plan of care.

The local school has an increasing number of adolescent parents. The nurse works with the school district to design and teach classes about infant care, child safety, and time management. These are examples of which nursing role? A) Educator B) Advocate C) Collaborator D) Case manager

Chap 52. A client is complaining of a bounding pulse, dyspnea, and confusion. Upon further assessment, the nurse notices distended neck veins and hears moist crackles. These are clinical manifestations of:

The manifestations of fluid volume excess are full bounding pulse, dyspnea, moist crackles, confusion, and distended neck veins. In fluid volume deficit, the pulse rate is rapid and weak, neck veins are flat, and there are no evidences of confusion or moist crackles in the lungs. The signs of hyponatremia are confusion, lethargy, muscle twitching, abdominal cramps, headache, and seizures. In hypercalcemia, the client is weak and lethargic, has polyuria, and dysrhythmia.

Chap 43. A client whose child died from a motor vehicle crash states, "I know I should grieve, but I just don't have the time. My other children need me and I can't be crying." The nurse's most therapeutic response is:

The most therapeutic response is empathetic and nonjudgmental. Making the suggestion that working through one's grief is important because bereavement may have potentially devastating effects on health is part of the necessary teaching. The other responses give advice ("have to take time," "need to look at") and judge the client ("you should be grieving").

Answer: C Case management is a model of organizing care in which the case manager monitors, directs, and advises the nursing care personnel on specific care issues and the progress of a client. In team nursing, care might be provided by groups composed of registered nurses, licensed practical nurses, and possibly assistive personnel. Nursing process is used to plan the nursing care for a client. Interdisciplinary care is care provided by a team whose members come from a variety of disciplines.

The multidisciplinary care model used to move clients efficiently from admission to discharge is known as: A) Team nursing B) Nursing process C) Case management D) Interdisciplinary care

Chap 52. The following are normal values of arterial blood gases except:

The normal values of arterial blood gases are: pH7.35 to 7.45; PaO280 to 100 mm Hg; PaCO235 to 45 mm Hg; HCO3-22 to 26 mEq/L;Base excess-2 to +2 mEq/L; O2 saturation95 to 98%

Chap 34. The client is being tested for bleeding in the colon. The nurse knows that when performing a test for occult blood, three specimens are required. The specimens must be obtained:

The nurse must know that when performing a test for occult blood, three specimens are required. The specimens must be obtained from three separate, consecutive bowel movements.

Answer: C To measure pulse deficit the nurse and a colleague assess the radial and apical pulse rates simultaneously and subtract the radial from the apical pulse rate. The result is the pulse deficit. Tachycardia and bradycardia are assessed by measuring the pulse rate for 1 minute. A rate of more than 100 beats per minute is categorized as tachycardia, whereas a rate of less than 60 beats per minute constitutes bradycardia.

The nurse's documentation indicates that a client has a pulse deficit of 14 beats. The pulse deficit is measured by: A) Subtracting 60 (bradycardia) from the client's pulse rate and reporting the difference B) Subtracting the client's pulse rate from 100 (tachycardia) and reporting the difference C) Assessing the apical pulse and the radial pulse for the same minute and subtracting the difference D) Assessing the apical pulse and 30 minutes later assessing the carotid pulse and subtracting the difference

Chap 34. Before obtaining a capillary blood specimen, in addition to assessing the client's understanding of the procedure and response to previous testing, what other factors need to be determined prior to the procedure?

The other factors that need to be determined prior to obtaining a capillary blood specimen include: medications that may prolong bleeding, status of client's skin, and circulatory status. It is not necessary to assess the client's complete blood count. Taking capillary blood samples does not precipitate an insulin reaction. Factors such as psychotropic medications, cognitive impairment, and the caregiver's response have no bearing on the capillary blood specimen collection. It is important to check the equipment prior to doing the procedure. Medication history is important, but the nurse needs to focus on specific drugs such as anticoagulants.

Chap 38. A client needs to have which of the following components to experience a sensory experience:

The sensory process involves two components: reception and perception.

Chap 42. The spouse of a client is discussing the difference between anxiety and fear. Which of the following statements indicates a need for further teaching?

The source of anxiety may not be identifiable; the source of fear is identifiable. Anxiety is related to the future, that is, to an anticipated event. Fear is related to the present. Anxiety is vague, whereas fear is definite. Anxiety is the result of psychologic or emotional conflict; fear is the result of a discrete physical or psychologic entity.

Which of the following nurses is showing behavior that indicates that the nurse is providing presence in a caring relationship?

The staff nurse who stays with a client who is undergoing an unfamiliar procedure

Chap 42. A young adult comes to the clinic complaining of chest pain. Upon further assessment, the nurse knows that which of the following stressor(s) could be contributing to the client's physical pain?

There are many sources of stress. Developmental stressors occur at predictable times throughout an individual's life. The stressors that are associated with the young adult developmental stage may include: getting married, leaving home, managing a home, starting a career, continuing education, and/or rearing children.

Answer: C Theories will be tested to describe or predict client outcomes as nursing is addressed as a science and an art. Scientists will not make nursing decisions, and nursing will base client care on the practice of nursing science, which will be guided by multiple theories.

There is a contemporary move toward addressing nursing as a science or as evidenced-based practice. This suggests that: A) One theory will guide nursing practice. B) Scientists will make nursing decisions. C) Theories will be tested to describe or predict client outcomes. D) Nursing will base client care on the practice of other sciences.

Chap 30. A client asks the nurse, "What is the purpose of a physical examination?" Which response by the nurse is not correct?

These are some of the purposes of the physical examination: To obtain baseline data about the client's functional abilities; To supplement, confirm, or refute data obtained in the nursing history; To obtain data that will help establish nursing diagnoses and plans of care; To evaluate the physiologic outcomes of health care and thus the progress of a client's health problem; To make clinical judgments about a client's health status; To identify areas for health promotion and disease prevention; The nurse collects data to supplement, confirm, or refute data obtained in the nursing history, during the physical exam. The nurse is not trying to determine if the client is being dishonest.

Chap 49. An 80-year-old client is in the emergency department. The client complains of diarrhea and vomiting for the past 2 days. In assessing the client, it is noted that the client has lost 8 lb, is itchy, and has dry skin that can be tented. Which NANDA diagnosis would be most appropriate to use with this client in making a plan of care?

This client is showing signs of dehydration. Risk for Deficient Fluid Volume related to prolonged diarrhea and vomiting is the only appropriate answer.

chronic glomerulonephritis

This condition results from Acute Glomerulonephritis which has not been treated. Can cause permanent damage. Causes damage to glomerulus and loss of kidney function. Urine starts out looking like Cola, then turns back to pale yellow because kidneys are not working (filtering) anymore.

Meniere's syndrome Signs and Symptoms & Priority Nursing Intervention

Tinnitus Unilateral sesorineural hearing loss vertigo Priority - instituting safety measures

Chap 30. Which examination technique is being used when the nurse touches the client's abdomen to examine the size of the liver?

Touching signifies palpation, inspection is looking, percussion is tapping, and auscultation is listening.

During the oliguric phase of renal failure, protein should be severely restricted..Why?

Toxic metabolites that accumulate in the blood (urea and creatinine) are derived mainly from protein catabolism.

Chap 42. Which of the following is an appropriate strategy for the nurse when dealing with a client's anger?

Try to understand the meaning of the client's anger. Let clients talk about their anger. After the interaction is completed, take time to process your feelings and your responses to the client with your colleagues. Listen to the client, and act as calmly as possible.

Answer: D To assess skin turgor, a fold of skin on the back of the forearm or sternal area is grasped with the fingertips and released. When turgor is good the skin lifts easily and snaps back immediately. The skin stays pinched when turgor is poor. The hands and neck are not the best places to test turgor, because the skin is normally loose and thin in those areas.

Turgor is the skin's elasticity, which can be diminished by edema or dehydration. Which is the best place for the nurse to assess skin turgor in the older adult? A) Side of the neck B) Back of the hand C) Palm of the hand D) Over the sternal area

Chap 43. At which age do children begin to accept that they will someday die?

Until children are about 5 years old, they believe that death is reversible. Between ages 5 and 9, children know death is irreversible, but believe it can be avoided. Between 9 and 12 years of age, children recognize that they, too, will someday die. At 12 to 18 years old, children build on previous beliefs and may fear death, but often pretend not to care about it.

2

When illnesses occurs, different attitudes about it cause people to react in different ways. What do medical sociologists call this reaction to illness? 1. Health belief 2. Illness behavior 3. Health promotion 4. Illness prevention

Chap 49. At the local wellness fair, the nurse is asked to share information on having healthy bowel life. Included in this area is the topic of having a healthy defecation. The nurse should include which of the following information as an appropriate action to follow?

When peristaltic waves move the feces into the sigmoid colon and the rectum, the sensory nerves in the rectum are stimulated and the individual becomes aware of the need to defecate. The number of times a person defecates a day would be important information to have about bowel habits, but it is not related to the actual process of defecation.

1

When taking care of patients, the nurse routinely asks them if they take any vitamins or herbal supplements, encourages family to bring in music that the patient likes to help them to relax, and frequently prays with her patients if that is important to them. The nurse is practicing which model? 1. Holistic 2. Health belief 3. Transtheoretical 4. Health promotion

Answer: A Standards of care describe the competency level of nursing care as described by the ANA. The Nurse Practice Act regulates the licensing and practice of nursing; it describes the scope of practice. Accreditation allows the facility, school, or hospital to operate and be recognized in good standing according to standards set by peers. National council licensure is the standardized national examination that assess for a minimum knowledge base relevant to the client population that the nurse serves.

Which of the following assures clients that they will receive quality care from a competent nurse? A) Standards of care B) Nurse Practice Act C) Accreditation certification D) National council licensure

Answer: B, D Extended care encompasses intermediate medical, nursing, or custodial care for clients recovering from acute illness or clients with chronic illnesses or disabilities. Extended care facilities include intermediate care facilities and skilled nursing facilities.

Which of the following clients should be cared for in an extended care facility with skilled nursing? (Select all that apply.) A) Client who had a stroke, can talk, and has lost bowel and bladder control B) Severely brain injured client on a ventilator who is receiving intravenous medications C) Client with Alzheimer's disease who is abusive, combative, and a threat to self and others D) Young child who recently had a spinal cord injury and is living with quadriplegia and needs to learn a new way of life

Answer: C The prospective payment system is one of the most significant factors influencing payment for health care. The prospective payment system groups payments into diagnosis-related groups for Medicare and Medicaid clients. Managed care organizations are systems in which there is administrative control over primary health care services for a defined client population.

Which of the following was most significant in influencing competition in health care costs? A) Medicare and Medicaid B) Diagnosis-related groups C) Prospective payment system D) Managed care organizations

Answer: C The S2 (dub) sound is the second heart sound and indicates closure of the aortic and pulmonic valves. The closing of the mitral and tricuspid valves is the S1 sound.

While auscultating heart sounds, the nurse documents that S2 is best heard at the base. This sound (S2) correlates with closure of which of the following? A) Aortic and mitral valves B) Mitral and tricuspid valves C) Aortic and pulmonic valves D) Tricuspid and pulmonic valves

Answer: C Complaints of tenderness in the calf during palpation may indicate phlebitis. Other characteristics of phlebitis are swelling, warmth, redness, and sometimes a positive Homans' sign. Cyanosis, pallor, and brown pigmentation around the ankles as well as ulceration and reduced hair growth are indications of venous or arterial insufficiency, which would not cause tenderness on palpation. Venous distention may be indicative of varicosities, which also are not associated with tenderness.

While the nurse was palpating the calf muscles of the client's right leg, the client complained of tenderness. Further assessment by the nurse should include which of the following? A) Observation for reduced hair growth and ulceration B) Observation for venous distention while the client is standing C) Observation of the area for swelling, warmth, redness, and a positive Homans' sign D) Observation for cyanosis, pallor, and change in pigmentation around the ankles

Chap 52. A client taking lasix (furosemide) for congestive heart failure is seeing the primary care provider for a potassium value of 3.0. An order for oral potassium taken daily is written and discussed with the client. In addition, potassium-rich foods should be eaten. The nurse educator meets with this client and has the client identify all of the following foods as potassium-rich except:

White bread is known to help meet fiber needs for the body. Potassium is found in many fruits, vegetables, meat, and fish.

Which statement by the pt with type 2 diabetes is accurate? A) I am supposed to have a meal or snack if I drink alcohol B) I am not allowed to eat any sweets because of my diabetes C) I do not need to watch what I eat because my diabetes is not the bad kind D) The amount of fat in my diet is not important; it is just the carbohydrates that raise my blood sugar

a Rationale: Alcohol should be consumed with food to reduce the risk of hypoglycemia.

If a hypertonic IV solution is administered, the mechanism involved in equalizing the fluid concentration between the ECF and the cells is A) osmosis B) diffusion C) active transport D) facilitated diffusion

a Rationale: Osmosis is the movement of water between two compartments separated by a semipermeable membrane. Water moves through the membrane from an area of low solute concentration to an area of high solute concentration.

goodpastures syndrome

autoimmune disease characterized by the formation of antibodies to the body's own basement membrane of both kidney's and alveoli. early signs cough, SOB, hemoptysis w/hematuria, proteinuria and edema

An example of distraction to provide pain relief is? A) TENS B) music C) exercise D) biofeedback

b Rationale: Distraction involves redirection of attention away from the pain and to something else. Distraction can be achieved by engaging the patient in any activity that can hold his or her attention (e.g., watching TV or a movie, conversing, listening to music, playing a game).

A patient on the surgical unit has a history of parasomnia (sleep-walking). Which of the following is true regarding parasomnia? A) hypnotic medications reduce the risk of sleepwalking B) the patient is often unaware of the activity on awakening C) the patient should be restrained at night to prevent personal harm D) the potential for sleepwalking is reduced by exercise before sleep

b Rationale: During sleep walking, the individual does not speak and may have limited or no awareness of the event. On awakening, the individual does not remember the event

Polydypsia and polyuria related to DM are primarily due to A) the release of ketones from cells during fat metabolism B) fluid shifts resulting from the osmotic effect of hyperglycemia C) damege to the kidneys from exposure to high levels of glucose D) changes in RBC's resulting from attachment of excessive glucose to hemoglobin

b Rationale: The osmotic effect of glucose produces the manifestations of polydipsia and polyuria.

What can be used for strasbismus?

botox inj

Wheres the pulse on an infant for CPR?

brachial

heparin/lovanox

bruising will occur aroun inj. sites, apply pressure for at least 30 seconds, check for bleeding gums, stools, sheck V/S for internal bleeding with anticoagulants

foley catheter(woman)

check balloon for patency, non dom hand open labia, dom hand cleans, insert cath 2-3" until urine returns, inflate balloon, pull back, push 1 more ", document

monitor elsrely pt with IV's more frequently to make sure of no fluid overload

check lung sounds q shift for crackles

wet to dry dressings

check orders, rmv dressing/packing, observe, clean, moisten packing, cover with gauze, initial/date/time

the intent of diagnosis related groups(drg) was

contains hospital costs

enema

do not give more than 3 consecutive bouts

Placement post hip replacement?

don't sleep on operated side do not elevate > 60 do not elevate HOB > 45 maintain hip abduction by seperating hip and thighs with pillow

ammoinum chloride, guaifenesin K iodide

expectorants, loosens bronchial secretions. SE nausea/drowsiness. nursing care: increase fluid intake

current views of health include the concept of

expressing the full range of one's potentialities

What are hyperPTH sx?

fatigue, muscle weakness, renal calculi, back and joint (r/t inc Ca)

What are s/s of PE/air embolism?

feeling of doom SOB CP tachy pale/cyanotic

48 to 72

following ESWL, bright red urine may be seen initially but should diminish within ___-___ hours

septicemia

generalized infection throughout body

oil retention enema

given in small amounts bc is retained

What do you give timolol for?

glaucoma

NPH

has regular insulin in it already

What dz causes ribbon like stools?

hirschsprungs

What are common sx of Cushing dz?

hypernatremia, hypertension, hypervolemia, hyperglycemia HYPOkalemia

What is exopthalmus indicative of?

hyperthyroidism/grave's disease

ALT (Alanine aminotransferase)

id hepatocellular disease of liver 4 - 6 international units/L

AST (Aspartate aminotransferase)

id suspected hepatocellular or if with cardiac markers to evalulate CAD 0 to 35 units / L

Thyroid test invalidated

if client has undergone a radionuclide scan within 7 days before the test

What are v/s for inc ICP?

inc BP, dec HR dec RR

What are sx of a thyroid storm?

inc temp, pulse and HTN

single voided urine specimen

instruct pt to clean genitalia with soap and water and void approx 60 ml

What position for pt with heat stroke?

lie flat with legs elevated

In a laboring pt, whats the first nursing axn?

listen to FHR

foley cath(male)

lubricate "5-7" of tubing, retract foreskin, advance cath 5-7" or until urine return, inflate balloon, pull out, back 1, return foreskin

What drug is admin to prevent preterm labor?

mag sulfate

when on lasix

monitor K+

suctioning

no longer than 10-20 seconds at a time, allow pt to rest between suctioning, flush cath with NS between suctionings, 80-100 mm is normal adult suction

stool testing

no red meat, radishes, beets, etc... for 3 days prior to test

Do you give yogurt to immunosuppressed pt?

no, b/c it has live cultures

Can a 4 yr old understand time?

no, do it around a common event; mom will be home after diner

ESR (Erythrocyte sedimentation rate)

nonspecific test to detect illnesses associated with acute and chronic infections, inflammation, advanced neoplasm, tissue necrosis or infarction 0 - 30 mm/hr varies with age of client

Regular insulin

onset:15-30 mins, clear, peak:30-11/2 hr., duration:3-4 hrs., can mix with intermed.& short

Humulog

onset:15-30mins., clear, peak: 30-11/2, duration: 3-4 hrs., mix with intermed., reg, short

Respiratory Alkalosis

pH: elevated PCO2: decreased HCO3: normal Causes: Hyperventilation- blowing off CO2 - Hypocapnia

What's characteristic of glaucoma?

painful vision loss tunnel/gun/barrel/halo vision loss of PERIPHERAL VISION

edema

palpate areas for tenderness/consistency

POsitioning for detached retina?

place area of detachment in dependent position

What does CP result from?

poor oxygenation during birth

What's placement after myringotomy?

position of side of affected early postop to facilitate drainage

dysequilibrium syndrome

post dialysis cerebral edema, cause is thought to be rapid removal of BUN --> edema and increased ICP

What are gout meds?

probenecid (inc cipro axn) colchicine (tx acute phases) allopurinol

inherent in any definition or philosophy of nursing are several core concepts

promoting wellness facilitating coping preventing illness restoring health

the function of feedback in communication is to

provide the sender with information about the receiver's perception of the information

the skin is the first line of defense and protects the body by

providing an intact physical barrier secreting bactericidal substances

TB testing

read 48-72 hrs later, inject 0.1 ml, assess for induration(hard bump), measure in mm, greater than 5 mm is queestionable, 5-10 need CXR, redness indicates localized reaction not a concern, problem is induration

Glycosylated serum Albumin (fructosamine)

relfects blood glucose over 2 -3 weeks more sensitive to recent changes nondiabetic client - 1.5 - 2.7 mmol/L diabetic client - 2 - 5 mmol/L fast for 12 hrs b4

peritoneal dialysis

removal of toxic waste substances from body by placing warm dialysate into peritoneal cavity; metabolic wastes are filtered out of blood across peritoneum via osmotic pressure; used in treating renal failure and certain poisonings

a person who has continuous dry rattling sounds

rhonchi

larger to smaller

right

valium

sedative/hypnotic, a tranquilizer (trade name Valium) used to relieve anxiety and relax muscles

What is imp postop thyroidectomy?

semi-fowlers prevent neck flexion/hyperextension trach at bedside

What are the genetics of of hemophilia?

sex linked mom passes to son

metered dose inhaler

shake med, have pt breathe out, when starts to inhale press down, have the pt hold breath for 10 seconds, wait 1 min befroe next dose

s/sx of hypoglycemia

shaky, slurred speech, weak, seizure, drowsy, sweating, inability to swallow

Albuterol (Proventil)

short acting beta2 adrenergic agonist used for rapid bronchodilation for acute asthma attacks.

What does hydrourea tx?

sickle cell

What's the best way to warm an infant?

skin to skin with mom in a warm blanket

cleansing enema

soapsuds, stimulates peristalsis thorugh distention and irritation of colon and rectum, average adult is 500-1000ml

dialysate

solution made up of water and electrolytes used to create oncotic pressure and pull out metabolic waste

acute glomerulonephritis

the following are s/s of what disease process? periorbital edema, smokey/rust or cola colored urine, oliguria (<400ml per day)

stress incontinence

the inability to control the voiding of urine under physical stress such as running, sneezing, laughing, or coughing

a major reason clients in the Us are sicker when they are admitted to the hospital is that

they are unable to gain access to preventative services so their illness progresses to an acute stage before they seek help

What drugs are MAOI?

think pirates arrrrr nARdil pARnate mARplan or PArnate, NArdil, MArplan (PANAMA)

transdermal meds

topical agents designed to absorb through skin for systemic effects

fluid volume replacement

treatment for the diuretic phase

promoting UOP, lasix (rem: pt is hyperkalemic no K sparing meds)

treatment for the oliguric phase of ARF focuses on what and what medications would be most likely used

nitroglycerin

treatment of CHF associated with an acute MI, relievs anginal attacks

What is sinemet?

tx PD pt may sweat urine may turn reddish brown drowsy

Selye's theory of general adaptation states that homeostasis will be regained

unless adaptive mechanisms are overwhelmed

sudafed

vasoconstrictor (trade names Privine and Sudafed) used in nasal sprays to treat symptoms of nasal congestion and in eyedrops to treat eye irritation

What's indicative of retinal detachment?

visual floaters flashes of light curtain vision

What are sx of hypovolemia?

weak/rapid pulse, inc temp, inc RR, HYPOtension, spgrav> 1.030

A client is admitted to the trauma intensive care unit (ICU) with a gunshot wound of the neck. The client, diagnosed with a spinal cord injury at the level of C4, is tearful, constantly complains of discomfort, and requests to be suctioned. The nurse understands that the client's attention-seeking behaviors may be due to which of the following? 1. Anger and frustration. 2. Awareness of vulnerability. 3. Increased social isolation. 4. Increased sensory stimulation.

(1) is not accurate for situation (2) correct—is experiencing an increased awareness of his physical vulnerability due to his spinal cord injury; fosters increased dependency needs that are real due to his injury; is trying to determine who is consistent and trustworthy for meeting his significant physical needs (3) is not accurate for situation (4) is not accurate for situation

The nurse can best demonstrate caring to a client who has recently suffered a loss through miscarriage by:

) Sitting with the client in silence

Contemporary nursing requires that the nurse possess knowledge and skills to carry out a variety of professional roles and responsibilities. Examples include which of the following? 4

A) Autonomy and accountability B) Advocacy C) Provision of bedside care D) Health promotion and illness prevention

Identify 2 nursing interventions for the client on hemodialysis

Do not take BP or perform venipuncture on the arm with the AV shunt, fistula, or graft. Assess access site for thrill and bruit

What position do you put veins in for better perfusion?

eleVate Veins

What's the positioning for amputations?

elevated for first 24 hr followed by positioning prone

proteinuria, hematuria

what would you expect a urine sample for a pt with acute glomerulonephritis to reveal?

When to avoid salt sub?

when taking dig and k-supplements

when is a nurse NOT obligated to follow the orders of a licensed physician

when the orders would result in harm to the client

the number of clients admitted to nursing homes is steadily growing

which statement is true regarding current trends for the delivery of health care services

autosomal - dominant, autosomal recessive

which type of autosomal PKD is typically diagnosed in adult hood. Which type usually results in death of the infant

because when serum protein levels are low, the liver makes lipoproteins (cholesterol) which causes hyperlipidemia

why does the pt with Nephrotic syndrome require cholesterol lowing agents to treat hyperlipidemia?

dysequilibrium syndrome

why is installation and removal of dialysate done slowly? To prevent ________ __________

danger of bacteremia

why is prostatic message contraindicated for prostatitis?


Related study sets

Drivers Ed Seat-belt true false quiz

View Set

Paper 2 (Human Relationships): Evaluate two theories explaining Altruism in Humans

View Set

LET Gen Ed (Part 1) - PNU, QUICE, Lorimar, Eduphil, and more

View Set